You are on page 1of 195

Business Associations Outline

Chapter 1 – Introduction to Business Forms


A) The subject in general
 3 ways business can obtain money [that it needs to operate and to generate income from investors]
o (1) company can borrow money  when a company borrows money it incurs “liability” or “an obligation
to repay creditor who has loaned the money
o (2) earn money by generating profits from operating the business
o (3) company can obtain money from “equity investors” or “residual claimants” or more narrowly,
“shareholders,” “partners,” or “sole proprietors”
 Terms
o Equity investors  owners of business who provide cash to company just as creditors do
 Unlike creditors, however, who have no rights to profits of business, equity investors are
entitled to shares in any profits from the venture
 Equity investors are known as residual claimants because they are entitled residual, i.e.
everything left over after company creditors are paid
o Capital  money contributed to business organization by equity investors
o Creditors  individuals or entity’s that loan money to the company rather than contribute capital to it,
are legally entitled to the return of their principal with interest as provided in loan agreement or other
contract that describes terms in which money is loaned
 Because claims are established by “fixed contracts,” creditors are known as fixed claimants
o Assets  money either earned by operating a business or put into a business by creditors or equity
investors
 When money used to buy anything for that business (factories or computers) purchased items
also becomes assets, and it is from these assets that creditors are repaid and equity investors
receive return on their capital
o Balance sheet  document that shows assets, liabilities, and equity of a company
 Formula: assets = equities + liabilities
 If value of company assets is shown on balance sheet as less than value of companies liabilities,
firm has negative equity (bankrupt)
 Business Association categories
o (1) assets
o (2) liabilities (fixed claims)
o (3) equity (residual claims)
 If not enough assets to pay creditors  bankrupt
 If more assets than liabilities  excess is available to the equity investors for distributions or
may be kept as what are known as “retained earnings” and be reinvested in the company
 Business Organization
o Emergence of different types revolves around 2 questions
 (1) how much liability should be imposed on business owners for the debts of the business
 (2) how should business organizations be taxed
 Concerns [when forming business]
o (1) limiting Liability
 Ex) if owners of Walmart could be personally sued at any time someone slipped and fell, no one
would want to invest in Walmart
 Limited liability  owners liability is limited to amount paid for his ownership interest in
business
 If limited liability business cannot pay its debts, then creditors of the business, rather
than the owner’s, bear the loss
o (2) minimizing taxes
 (a) legitimate tax avoidance  “tax planning”
 (b) tax evasion  may lead to fraud penalties
 Incorporation 
 if business owners has NOT incorporated her business, then profits of business treated
as earnings of owner and taxed along with any other earnings that owner may have
 if business owner HAS incorporated her business, then earnings of incorporation are
not automatically attributed to the owner (known as “shareholder” in corporate
context), instead these earnings are taxed to the corporation according to the
applicable tax rates for the corporation
o if the corporation then distributes profits to shareholder in form of dividends,
corporation is subject to “double taxation”
 “double taxation”  taxed once at entity level and once at shareholder level
 (1) first tax imposed at corporate level on earnings of the business
 (2) second tax imposed when those earnings are distributed as dividends: every share
holder receiving a dividend must pay income tax on the amount she receives
o Unincorporated
 business operated as sole proprietorship makes $1million in profits for
the year
 If sole proprietorship is in 40% tax bracket, she will keep $600k for
profits and pay $400k in taxes
o Incorporated
 If sole proprietor incorporated business, becomes its sole shareholder,
corporation would have to pay its own taxes on the $1million in profits
before any of that profit could be distributed to the shareholder
 Suppose corporate tax rate was 25%, the corporation would pay $250k
in corporate income tax, leaving $750 to distribute to shareholders
 Of this $750k, $300k (40% or $750k) would go to taxes and the
shareholder would be left with remaining $450k
 “pass through taxation” earnings of business are taxed only once / business itself not taxed
as entity (level 1), instead, all business profits are “passed through” to owner’s who then pay
taxes on their income
 Sole proprietorship, partnership, limited liability subject to pass through taxation

B) Intro to Business forms


 (3) Agency costs  the risk that investors’ funds will be used for purposes other than those that further the
interests of the investors
o Include not only the money lost to outright dishonesty but also cost of mere negligence to or
inattention to investors interests, as well as incompetence
o Law of agency  exists to deal with problems of agency costs
 Forms
o (1) sole proprietorship  basis of trade for thousands of years
o (2) general partnership  association of two or more person to carry on as co-owner, a business for
profit
 Considered to be equal by default
 Agency law  partners legally both principals and agents of the partnership
 Ex) partner traveling the silk road could enter into contract and take other actions that
would bind his whole partnership back home in Siena
 GP today  can be informally created, i.e. does not require the filing of an organizational
document with the state. So, as long as two or more persons are carrying on, as co-owners, a
business for profit, a GP is created, regardless whether co-owner intended that result
 Statutes
 (1) view GP as separate legal entity whose identity is distinct from that of its owners
(“partners”) (RUPA)
 (2) view as aggregate of individual partners with no legal differentiation between
business partners (UPA)
 Characterization
 (1) Easy Exit  GP form provides partners with relatively easy exit from venture, as
partners withdrawal typically causes a buyout of the partners interest of dissolution of
the partnership itself
 (2) structural flexibility  partners can contractually arrange to run the business largely
as they see fit
 (3) restricted transferability of ownership interests  transferee of a partnership
interest can only become a partner with unanimous consent of the other partners
 (4) pass through taxation  partnership income is only taxed at partner level, rather
than “doubly taxed” at both the partnership and partner levels
 Negatives
 (1) personal liability  like sole proprietorship, law imposes personal liability on
partners for obligations of partnership. Debts of the business are viewed by the law a
debt of owner, and owners’ personal assets are at risk in lawsuits arising out of
business activities
 (2) easy exit  makes it a rather unstable business form in certain circumstances – a
situation which may be unsuitable for particular types of ventures
o (3) Corporation  legal entity distinct from its owners; could own public property in its own right, even
if corporate owner’s had no claim to corporate property; could enter into contracts, incur debts, sue and
be sued
 Statutes
 Corporation viewed as separate legal entity whose identity distinct from that of its
owners
 Corporation itself enters into contracts, barrows money, owns property, sues and
defends, and otherwise conduct business as if it were real person
 Corporation v. GP
 (a) corporation formed by consciously filing organizational document with state
 (b) shareholders ≠ have the right to participate in the management of the business
(except in extraordinary situations i.e. mergers). Instead, shareholders elect directors
who have ultimate decisions making power in the firm
 (c) shareholder in corporation typically has no ability to compel a buyout of his
ownership interest or dissolution of firm
 (d) ownership interests in corporation may be freely and fully transferred without
consent of other shareholders
 (e) income of corporation is double taxed
 (f) in addition to double taxation on income taxes, corporation (but not partnership),
also subject to state franchise taxes  taxes imposed for privilege of organization
business in a state
 (g) corporation provides shareholders with limited liability for obligations of the
business. In a lawsuit against a corporation, shareholders personal assets are not at risk;
the most shareholders can lose is the amount of his investment
 Closely v. publicly held
 Closely held  business with relatively few owner’s whose ownership interests are not
publicly traded on established market
 Publicly held  business typically has large number of owners with ownership
interest that are routinely bought and sold on a public market
Chapter 2 – Agency
A) Intro
 Issue: whether agency relationship exits and whether agent has power to bind the principal
o Ordinary agency law principals govern with business as principal and various officers/employees as
agent
 Law of agency  law of delegation, i.e. the legal principles that govern the ability of one-person (the principal)
to have another person (the agent) act on his behalf
o Ex) relationship between a sole proprietor and employees is governed by the law of agency, as is
relationship between corporation and officers
o Ex) corporation  artificial legal construct that has no physical being of its own, can act only through
agents for everything it does (i.e. writing a check, selling product, entering into merger)
 Agency concepts
o General partnership  partners are agents with apparent authority to bind the partnership for acts in
the ordinary course of the partnership business
o Even if relevant business organizational statute does not explicitly incorporate an agency concept, CL of
agency will apply unless it is clearly displaced by statutory scheme at issue

B) Creation of the Agency Relationship


 Terms (Rest. 2 Agency § 1. Agency; Principal; Agent)
o Agency  fiduciary relationship which results from the manifestation of consent by one person to
another that the other shall act on his behalf and subject to his control and consent by the other so to
act
o Agent  person acting for another. The one who is to act.
o Principal  person agent is acting for. The one for whom action is to be taken.
 [3] Elements of Agency relationship
o (1) consent by principal and agent;
o (2) actions by agent on behalf of principal; and
o (3) control by principal of agent to supervise performance
 General rule  If there is mutual assent to an arrangement including an agent acting to further the principals
interest and subject to the principals control; thus it is appropriate to make principals liable for agents actions
o (1) “consent”  consent of both parties required to form agency relationship
 Specifically, both principal and agent must consent to the agent acting on principals behalf and
subject to control of another person
 Principal  must manifest his consent to agent (can be written, oral, or implied form parties
conduct)
 Agents  must also manifest consent to principal (can be written, oral, or implied form parties
conduct)
 Ex) P asks A to complete a task pursuant to P’s instructions and A does so, agency
relationship has been created even if A didn’t expressly communicate to P his
agreement to perform the task
o (2) “on behalf of”  agent must be acting on principal’s behalf
 Agent must be acting primarily for benefit of principal rather than for benefit of agent or some
other party
 Simply acting in a way that benefits another, even when there is control, is insufficient to
establish an agency relationship
 Ex) homeowner calls electrician to install new furniture ≠ agency relationship because
electrician is not working on behalf of the woman, he sets his own price and retains
profits
o (3) “control”  agent must act subject to principals’ control, but degree of control exercised by
principal does not have to be significant
 Principal need not exercise physical control over actions of agent; agent must be subject to P’s
control over result or ultimate objectives of agency relationship
 Not focused on all the agents actions, focus is on whether P has control of result or
ultimate objective
 Control may be exercised by prescribing the agents obligation or duties before or after agent
acts, or both.
 This is why a 1099 contractor is not an agent. 1099 = what they are labeled as on the tax code.
Independent contractor. You control the ends not the means.
 Situations that look like agency [but are not].
o (1) Gratuitous bailment (bailee receives no compensation)
 If you hit someone while bailee, who is liable?
 Depends on control (principal agency relationship)  more control you have over friends use of
car more likely could find principal/agency relationship
o (2) Creditor / Debtor
 In general, no agency relationship exists
 BUT when lender finances business, it is common to impose conditions on how money can be
used and how business may run operations  conditions give degree of control
 A creditor who assumes control over debtor’s business for mutual benefit of himself and his
debtor may become principal with liability for acts and transactions of the debtor in connection
with the business
 Creditor who appreciates this risk will sign decrease level of control exercised over debtor in
effort to avoid risk of agency liability; but decrease in control = increase in default so creditor
will most likely charge more for loan to compensate heightened risk
 Notes
o (1) agency definition uses “person” to refer to both agents and principals; agency relationship is not
limited to natural persons; artificial entities such as corporations, trusts, partnerships or LL companies
ay act as a principal or agent
o (2) if legal definition of agency has been met through satisfaction of elements, agency relationship is
present regardless of whether parties intended to create

C) Liability from the agency relationship


 1) Tort liability from the agency relationship
o Issue: what liability is created when agent interacts with a third party (vicarious liability)
 General rule: P has the right to control the conduct of A with respect to matters entrusted to
the agent
 Qualifier: Within the scope of employment.
o Relationship Types (Rest. 2d Agency § 220. Definition of Servant)
 (1) Master / Servant (A able to bind P)
 Master  principal who “employs an agent to perform services in his affairs and who
controls or has right to control the physical conduct of the other in the performance of
the service
 Servant  agent so employed by master to perform the services in his affairs whose
physical conduct in performance of service is so controlled or subject to right to control
by master.
o If person subject to control of another as to the means used to achieve
particular result = servant
 (2) Independent contractor (A not able to bind P)
 Person who contracts with another to do something for him but who is not controlled
by the other nor subject to others right to control with respect to physical conduct in
performance of undertaking
o If person subject to control of another as to his results only (but not over how
to achieve those results = independent contractor
o Servant v. independent K (questions for jury)
 (1) extent of control which, by the agreement, the master may exercise over the details of the
work
 (2) whether employer or workman supplies instrumentalities, tools, and place of work for
person for whom they are doing work for
 (3) length of time for which person is employed
 (4) method of payment, whether by time or by the job
o Liability Differences
 (1) Master  liable for tort committed by a servant within the scope of his employment. Rest. 2
Agency § 228(1).
 if, but only if (a) it is of the kind is employed to perform; (b) occurs substantially within
the authorized time and space limits; (c) it is actuated at least in party by purpose of
master; and (d) if force is intentionally used by servant against another, use of force ≠
unexpectable by master.
 whether tort has occurred on job: frolic or detour to decide to impose liability
o (1) frolic  new and independent journey / P ≠ vicariously liable
o (2) detour  mere departure from assigned task / P = vicariously liable (ex –
stop to get gas)
 (2) Principal  generally not liable for torts committed by independent contractor in
connection with his work (even in course of work)
o Terms
 Vicarious liability  imposition of liability on one person for actionable conduct of another
 Respondeat superior  “let the master respond”
 Direct liability  person always responds to own torts
o Rest
 Does NOT use master/servant / independent contractor
 Uses “employee”  an agent whose principal controls or has right to control manner and
manes of agents’ performance of work and provides that “employer is subject to vicarious
liability for tort committed by employee acting in scope of employment”
 “employee agents” / “non-employee agents” (independent contractor)

 2) Contract liability form the agency relationship


o A contractual transaction between agent and third party may impose liability upon the principal, third
party, and/or agent
o Liability inquiry is often affected by the type of principal that is present at transaction
 (1) disclosed principal
 (2) partially disclose principal
 (3) undisclosed principal
o (1) Disclosed Principal  P is disclosed if, at the time of agent’s transaction, the third party has notice
that the agent is acting for principal, and has notice of P’s identity
 If 3rd party doesn’t know identity, but should be able to reasonably infer the identity of P from
info at hand  still disclosed
o (2) Partially Disclosed Principal  P is partially disclosed if, at the time of agent’s transaction, third
party has notice that the agent is or may be acting for the principal, but no notice of identity
o (3) Undisclosed Principal  P is undisclosed if, at the time of agent transaction, third party has no
notice that agent is acting for principal
 In effect, third party is dealing with agent as though the agent is the real party in interest

A) Liability of the Principal to the Third Party


 P will be liable on a contract between agent and third party when the agent acts with actual, apparent, or
inherent authority
o Even if lacks authority  P may be liable under doctrine of estoppel or ratification
o When is the principal liable to a third party? When the agent acts with authority.
 Then broaden. An agent has different types of authority – Actual, apparent, Inherent. All three
would bind the principal under contract.
 (1) Actual (express) Authority  arises from the manifestation of a principal to an agent, that agent has power
to deal with others as representative of P. Agency and Actual Authority: PA.
o If P’s words or conduct would lead a reasonable person in agents position to believe that the agent has
authority to act on P’s behalf, agent has actual authority to bind the principal. Rest. 2d Agency § 33.
o General Rule: A disclosed, or partially disclosed principal is subject to liability upon contracts made by an
agent acting within his authority if made in proper form and with the understanding that the principal is
a party. Rest. 2d Agency § 144.
o General Rule: An undisclosed principle is bound by contracts and conveyances made on his account by
an agent acting within his authority, expect that the principal is not bound . . . upon a contract which
excludes him. Rest. 2d Agency § 186.
o Express  oral or written statements, including provisions in the companies organizational documents.
o Implied  inferred from P’s previous acts or custom
 Implied actual authority = “incidental authority”  the authority to do incidental acts that are
related to a transaction that is authorized
 (2) Apparent Authority [and estoppel]  arises from the manifestations of a principal to a third party that
another person is authorized to act as agent for the principal. The other person has apparent authority and an
act by him within the scope of apparent authority binds the principal. “Cloaking” with authority.
o If P’s words or conduct would lead a reasonable person in third party position to believe that agent (or
other person) has authority to act on P’s behalf, agent has apparent authority to bind the principal
o Apparent authority commonly arises when a principal creates the impression that broad authority
exists in an agent when In fact it does not; the theory is that if a third party relies on appearance of
authority, the third party may hold the principal liable for the action of the agent
o Notes
 (1) some circumstances, the scope of an agents’ authority will be equal to scope of agents actual
authority
 Ex) if principal sends identical letters describing agents authority and its limits to both
agent and third party, actual and apparent authority are co-extensve
 (2) apparent authority can exist even in absence of principal / agent relationship
 Ex) apparent authority can arise when person falsely represents to third party that
someone else is his agent
 Apparent authority applies to actors who appear to be agents but are not, as well as to
agents who act beyond the scope of their actual authority
 (3) apparent authority is based on P’s manifestations to a third party
 Thus, apparent authority cannot be created by the mere representation by an agent or
other actor
 Not even the most convincing and persuasive person can create an agency or apparent
agency entirely on its own.
 Plead: apparent, inherent, or actual.
o Alternatively: estoppel, ratification.
 (3) Inherent authority  the power of an agent which is derived not from actual, apparent, or estoppel, but
solely from the agency relation and exists for protection of persons harmed by or dealing with a servant or
another agent
o A general agent for disclosed or partially disclosed P has inherent authority to bind the P for acts done
on his (P) account which usually accompany or are identical to transactions which agent is authorized
to conduct if, although they are forbidden by the P, the other party reasonable believes that he agent is
authorized to do so and has no notice he is not so authorized
 Something that accompanies their duties (even if forbidden)
 “Name tag” authority.
 (4) Equitable Estoppel  a person who is not otherwise liable as a party to a transaction purported to be done
on his account, is nevertheless subject to liability to persons that have changed their position because of their
belief that the transaction was entered into by or for him if:
o (a) intentionally or carelessly caused such belief
o (b) knowing of such belief and that others might change their positions because of it, he did not take
reasonable steps to notify them of facts. Rest. 2d Agency § 8B.
o Distinction
 (1) Doctrine of apparent authority  holds principal responsible for third parties belief
because of principals manifestations of authority to the third party
 Apparent authority may be created without need to establish detrimental change in
position
 (2) Doctrine of estoppel  applies when principal has not made any manifestations of
authority at all; instead, p is held responsible because p contributed to third party belief or
failed to dispel it
 Estoppel may apply when apparent authorly is unavailable
 Rule: if the claim of a party to a transaction is based solely upon estoppel, he must
prove change in position.
o Rest. 3 Agency § 2.05 cmt. c. Furniture store ex: if actions of furniture store are justifiable, Principal may
be estopped to deny the agent’s authority b/c principal could have easily instructed agent not to
purchase or informed store agent lacked authority.

 (5) Ratification  a validation of unauthorized acts. § 82. Affirmance § 83.


o Even if the agent acts without authority, the P will be liable to the third party if
 (1) the agent purports to act on the P’s behalf; AND
 (2)(a) the P affirmatively treats the agents act as authorized (express ratification); OR
 (2)(b) the P engages in conduct that is justifiable only if the P is treating the agents act
as authorized (implied ratification)
o Note: ratification will not occur unless the P, at the time of ratification is fully
aware of all material facts involved in original transaction
o Material facts  those that substantially affect the existence or extent of
obligations included in the transaction as distinguished from those which affect
values or inducement involved in it. § 91 – Knowledge of P at time of
Affirmance.
o Types
 Express ratification  commonly occurs through oral or written statements (i.e. company
resolutions)
 Implied ratification  commonly occurs when the P has knowledge of unauthorized
transaction entered into purportedly on his behalf, but P nevertheless accepts benefits of
transaction
o Ratification occurrence
 Ratification occurs as soon as P objectively manifests his acceptance of the transaction even if
the fact of ratification is not communicated to third party, agent, or any other person
 When ratification occurs  effect is to validate the contract as if it were originally authorized by
the P
 Liability is imposed through ratification up on the P in the same manner as if P had
actually authorized the contract in the first place
o Ratification ineffective
 Ratification is not effective unless it occurs before the third party has withdrawn from
transaction
 Ratification not effective if it would be unfair to the third party as a result of changed
circumstances
o Notes
 (1) function of ratification
 Confirms or validates agents right to have acted the way the agent did
 If ratified ≠ show agent acted with actual authority / eliminates claims P has against A
 (2) Restatement
 2nd rest  can be no ratification by an undisclosed principal
o In undisclosed principal situation  third party has no notice that agent is
acting for P and thus agent is not purporting to act on P’s behalf
 3rd rest  allows ratification by undisclosed principal
o “a person may ratify an act if the actor acted or purported to act as an agent on
persons behalf

B) Liability: Third Party Liability to Principal (3P  P)


 Rule: When agent makes a contract for a disclosed (‘with Ms. Bonilla’) or partially disclosed (‘with a gal’)
principal, the third party is liable to the P if the agent acted with authority (actual, apparent, inherent, or
ratification) so long as P is not excluded as a party by the form or terms of contract, existence of P is not
fraudulently concealed, and there is not set-off or similar defense against agent
o Fraudulent concealment exception  when agent contracting on behalf of an undisclosed P falsely
represents that agent is not working for a principal (i.e. agent falsely represents that he is acting solely
for himself), 2nd and 3rd Rest. both provide that third party can avoid the contract if the principal or
agent has notice that third party would not have dealt with principal
 This only comes up when agent lies and says not acting on behalf of P
 Agent must do more than simply signing name on contract
 Intentionally undisclosed.
o Substantial changes  undisclosed P cannot bind a third party to a contract if P’s role in the contract
substantially changes the third parties rights

C) Liability: Agent to the Third Party (A  3P)


 General rule [re: agent liability – ‘Wrinkle’]  if the agent purports to act on behalf of a principal but lacks
power to bind the principal, the agent is liable (even in disclosed principal situation). No actual, no inherent, no
apparent (no ratification, etc.) A is liable all by himself.
 Disclosed Principal
o Rule: If an agent contracts with a third party on behalf of a disclosed principal, the agent is not a party
to the contract and is not liable to third party [A Never liable to 3P]
 Partially Disclosed Principal
o Rule: If an agent contracts with a third party on behalf of a partially disclosed or undisclosed principal,
the agent is a party to the contract and is liable to the third party (regardless of whether P is also liable
to the third party)
 Liability
o (1) Implied Warranty  agent who purports to act on behalf of a P makes implied warranty of
authority to third party
 If agent lacks power to bind the principal, the agent is liable to the third party for breach of
implied warranty (unless explicitly conveyed he was not making such a warrant or third party
knew agent had no authority)
o (2) Misrepresentation  Agent may also be liable to third party under theory he has tortuously
misrepresented his authority
 Note
o If agent becomes party to the contract and if third party breaches contract in some manner, third party
may be liable to agent

D) Duties of Agent and Principal to Each Other


 1) Agents duties to principal
o Agency relationship = fiduciary relationship [within the scope of agency]
 A  P = fiduciary
 P  A ≠ fiduciary
o Agent duties to Principal
 (1) agents to act only as authorized by the principal
 (2) Agents owe fiduciary duties to the principal
 (a) duty of care  required to act as reasonable agent acts in similar circumstances.
o Standard of care is equal to the profession. “Battle of the experts.” Comparing
them to their profession (in regional area).
o ‘Don’t be an idiot.’
 (b) duty of loyalty  act solely for benefit of principal
o ‘Don’t be a dirtbag.’
 2) Principals duties to Agent
o Principal duties
 P must perform his contractual commitments to agent
 Must not unreasonably interfere with agents work
 General act fairly and in good faith to agent
 If agent incurs expenses or suffered other losses in carrying out principal instructions, principal
has duty to indemnify agent
o Not fiduciary in nature.

E) Termination of Agents Power


 Actual Authority  terminates when objective or relationship has been achieved, when P or A dies, if P or A
revokes it
o If relationship is based on contract, the decision to terminate may be breach of contract; nevertheless,
actual authority has ended, even though contract liability may exist for its termination (power to
terminate but not always the right)
 Apparent authority  because inference of apparent authority may be based on existence of prior actual
authority, termination of actual authority doesn’t itself eliminate apparent authority of agent
o May be necessary to give notice of termination to third parties who deal with agent or who otherwise
continue to believe the p has authorized agent to act
 Pg 13 examples
o (1) General motors corp employs an individual to serve as head designer of a new automobile with a
salary of $300k a year. The designer is a “servant” and GM is his “maters” (second restatement). The
chief executive officer of GM comes to the designer and says, “John, the board of directors likely your
sketches for the new convertible. They feel however, that it looks a little boxy and they think the
headlights are too conspicuous. Please streamline it a little more and move the headlights inot the front
fenders
 Head designer makes the changes that were requested, thereby indicating that he is a servant
o (2) A builder enters into a contract with the owner of a lot to build a house on the lot in accordance with
certain plans and specifications prepared by an architect.
 The builder is an independent contractor / he is employed merely to accomplish a specific result
and is not otherwise subject to the owner’s control
o (3) A broker enters into a contract to sell goods for a manufacturer. His arrangement involves the receipt
of a salary plus a commission on each sale, but the broker has discretion as to how to conduct his
business. He determines which cities to visit and who to contact. He uses his own automobile to visit
prospects.
 The broker is an independent contract because the manufacturer does not control the details of
how the broker conducts his day-to-day business
 Pg 14 examples
o (1) P, the owner of a successful retail operation with two stores, hires D to driver her delivery truck and
to deliver goods to her two stores. Before doing so, P checks D’s driving record and arranges for him to
go to a driving school for truck drivers. D’s record shows that he has had no accidents from 20 years and
he completes the driving school program without difficulty. Three weeks later, while driving P’s delivery
truck, D is negligent and has a serious accident, injuring T.
 P is liable to T for his injuries
 D is a servant, and P’s liability is independent of whether P exercised care in hiring D, or even
whether she knew that D was her employee at all
 P’s liability in this situation may be described as “vicarious liability” and the consequence of
“respondeat superior”
o (2) The manufacturer’s broker has an automobile accident while driving his own car to visit a prospect.
 The manufacturer is not liable for injuries to third persons arising from the accident
 The same would be true of a person injured by the builder in the above examples while working
on the owner’s house
 Because the broker and the builder are both independent contractors, the general rule of non-
liability for the principal applies
 The broker and the builder themselves would both be personally liable for the injuries that they
caused in these examples, as a person is always responsible for his own torts.
 D, the servant in delivery truck example would also be personally liable for T’s injuries, since he
too is a tortfeasor
 Pg. 15 notes
o (1) Justification for the doctrine of respondeat superior
 What has emerged as the modern justification for vicarious liability is a rule of policy, a
deliberate allocation of risk.
 The losses caused by the torts of employees, are placed upon that enterprise itself, as a required
cost of doing business
 They are placed upon employer because, having engaged in an enterprise, which will on the
basis of all past experience involve harm to others through the torts of employees, and sought
to profit by it, it is just that he, rather than the innocent injured plaintiff, should bear them; and
because he is better able to absorb them, and to distribute them, through prices, rates or
liability insurance, to the public and so to shift them to society, to the community at large
o (2) General rule: principal is not liable for torts committed by independent contractor
 The reason for distinguishing the independent contractor from the employee is that, by
definition of the relationship between a principal and an independent contractor, the principal
does not supervise the details of the independent contracts work and therefor is not in a good
position to prevent negligent performance, whereas the essence of the contractual relationship
known as employment is that the employee surrenders to the employer the right to direct the
details of his work in exchange for receiving a wage.
 Situations in which liability is imposed on a principal for the torts of an independent contractor
 Nondelegable duties (such as duties associated with abnormally dangerous activities), torts that
are authorized by the principal, and fraud or misrepresentation (in some situations) by the agent
Overview (Module 1 / Lecture #1)
 Types of Business Associations
o (1) Sole Proprietorship  Single person / formed without any government agency
 Advantage  easy to establish / begins by conducting business
 Disadvantage  owner of proprietorship is personally liable for all business obligations since no
separation between owner and business
 Debts of business = debts of owner
 Owners personal assets at risk in lawsuit arising out of business activity
o (2) General Partnership  association of 2 or more persons to carry out as co-owner’s a business for
profit
 Advantage  can be informally created; doesn’t require filing of organizational document with
state
 Disadvantage  law imposes personal liability on partners for the obligations of the partnership
 Creditor can sue every member of partnership for partnership debts
 Easy exit = unstable business form
o (3) Limited Partnership  creates limited liability for all buy partner that assumes role of ‘general
partner’
 Formation  comply with statute (more formal than GP)
 Disadvantage  even though general partner has liability, a limited partner can also become
liable if he takes part in control of business
o (4) Limited Liability Company  provides limited liability for all participants whether or not they are
active in management of business
 Advantage  flexibility in internal management (can be member managed or manager
managed)
 Eligibility for partnership taxation
o (5) Corporation  formed by following procedures in statute of MBCA and filing “articles of
incorporation”
 Advantages  provides shield from liability for shareholders
 Disadvantage  has entity status; non-favorable taxation
 3 tiers
 (1) shareholder  owners who each own part of company (interest represented by
shares)
 (2) board of directors  responsible for oversight of company affairs; usually elected by
shareholders (can be shareholders, officers, both, or neither)
 (3) officers  CEO/CFO/COO who run company to effectuate instructions of board
 Taxation
o Direct taxation  entity itself is taxed (e.g. corporation)
 Corporations have own tax schedule with regularly high taxer rates than individuals
 Subjects shareholders to double taxation (corporation taxed / shareholders taxed through
dividends)
o Pass-Through taxation  entity itself is not taxed; only profits passed through to partners are taxed
(e.g. partnership)
 often called “partnership taxation”
 contract taxation  available to GP, LP, LLC, and LLP
 Formation and Duration
o (1) partnership
 Agreement to share profits
 Withdrawal of partner dissolves partnership
o (2) limited partnership
 Filing of certificate
 By agreement or withdrawal of general partner
o (3) LLC
 Filing of certification of organization
 Perpetual
o (4) corporation
 Filing of certificate of incorporation
 Perpetual
 Management and ownership functions
o (1) sole proprietorship
 All functions unified in single owner
o (2) partnership
 All functions divided equally among partners
o (3) corporation
 Functions divided unequally among directors, officers, and shareholders
 Liability to Outsiders
o (1) Sole Proprietorship
 Unlimited liability of owner
o (2) Partnership
 Unlimited liability of partner for partnership debts.
o (3) Limited partnership
 General partner = unlimited liability
 Limited partner = not liable for partnership debts unless they assume control
o (4) LLC
 Members not liable for LLC debts
o (5) Corporation
 Shareholders not liable for debts
 Piercing the corporate wall
 Taxation as a Factor
o Basic Idea: A corporation is a separate tax-paying entity, but a partnership is not.
o Partnership: Income “flows through” to partners, who pay tax on it.
o Corporation: Business pays tax on its income, and shareholders also pay tax on dividends.
o Effect: Corporate income taxed twice.

Chapter 3 – The General Partnership


A) Intro. UPA (1914) (all adopted); RUPA (1997) (34 states and virgin islands) & RUPA (2013) (3 states & DC).
 UPA § 6: Partnership Defined.
o (1) a partnership is an association of two or more person to carry on as co-owners a business for profit
B) Formation
 1) The Definition of Partnership
o UPA § 7: Rules for Determining the Existence of a Partnership
 In determining whether partnership exists, these rules shall apply
 (4) the receipt by a person of a share of the profits of a business is prima facie evidence
that he is a partner in the business, but no such inference shall be drawn if such profits
were received in payment
o (a) as a debt by installment or otherwise
o (b) as interest on a loan, though the amount of payment vary with the profits of
the business
 Note: no intent required. Sharing profits creates presumption.
Rebuttable if profits received as payment of debt, or interest on a loan.
 Intent to do the thing that meets the legal definition of partnership.
o RUPA § 202: Formation of Partnership
 (c) In determining whether partnership exists, these rules shall apply
 (3) a person who receives a share of the profits of a business is presumed to be a
partner in the business, unless the profits were received in payment
o (i) of a debt by installment or otherwise
o (v) of interest or other charge on a loan
 Note: no intent required
o RUPA § 202, c. 1: Existence of Partnership
 Ownership involves the power of ultimate control and to state that partners are co-owners of a
business is to state that the each have power of ultimate control
 Martin v. Peyton (creditors trying to hold a lender liable as partner of bankrupt firm)
o Facts:
 The brokerage firm of Knauth, Nachod & Kuhne (KN&K) made a series of bad investments, which
resulted in the firm suffering severe financial difficulties. In order to save KN&K, one of its
partners, Hall, entered into a transaction with Peyton (defendant) and other persons (lenders)
for a loan of $2,500,000 worth of securities to KN&K. In return for the loan, the lenders were to
receive 40 percent of KN&K’s profits until the debt was repaid. The transaction was based on
three documents: an agreement, indenture, and option.
 (Control) The agreement provided that: (1) two of the lenders were appointed “trustees” who
were to be informed of transactions affecting them, paid dividends and income from those
transactions, had the power to buy and sell their loaned securities and substitute those
securities with those of equal value, but could not commingle those securities with KN&K’s
other securities, and were required to keep the securities valued at a certain level; (2) Hall was
given the power of directing the management of KN&K until the loan was repaid, and his life
was to be insured for $100,000 with the insurance policies given to the trustees as additional
collateral; (3) the trustees were to be kept informed of the important matters of KN&K’s
business, could inspect the books, and had the power to veto certain business decisions that
could affect their collateral; and (4) each KN&K member was to assign their interest in the firm
to the trustees, member could receive a loan from KN&K, the members’ draw amount was fixed,
and no other distribution of profits could be made.
 The indenture was basically a mortgage on the collateral delivered by KN&K to the trustees. The
option: (1) gave the lenders the opportunity to buy into KN&K by buying 50% or less of the
members’ interest at a listed price; (2) enabled the formation of a corporation to replace KN&K
if its members and lenders agreed; and (3) provided for the resignation of any KN&K member at
the demand of Hall.
 Martin (plaintiff), a creditor of KN&K, sued the lenders, claiming that their transaction with
KN&K, as illustrated by the agreement, indenture, and option, made them partners in that firm
and thereby liable for KN&K’s debts. KN&K went bankrupt, Plaintiff (creditor) is trying to hold
Peyton, Perkins, & Freeman liable (they have deep pockets). Partners are liable individually for
all partnership debt.
o Issue: Do agreements intended to protect the financial interests of creditors necessarily make them
partners of a debtor firm?
o Rule: In order for a creditor to be a partner in a firm, the creditor must be closely enough associated
with the firm so as to make it a co-owner carrying on the business for profit.
 Formation  A partnership is not formed unless two or more parties are closely associated so
as to be co-owners carrying on a business for profit
 When creditors have executed loan documents with a debtor firm that contains
provisions for the collection of collateral, this court must examine the extent to which
those documents associate the creditors with the business operations of the firm.
o Holding:
 (1) In this case, no partnership was formed. The agreement’s providing for appointment of two
of the lenders as trustees, for example, does not indicate a partnership.
 The trustees were in charge only of transactions affecting their collateral, and they were
prohibited from commingling the collateral with KN&K’s other securities.
 (2) Similarly, Hall’s life insurance and management power does not imply an association with
KN&K because Hall was trusted by the other lenders to keep an eye on KN&K and work to
ensure that it was run efficiently enough to return to profitability and pay back the lenders.
 (3) And, the trustees’ veto power does not indicate a partnership, as it gave them the ability
only to safeguard against bad investments concerning their collateral. The trustees had no
authority to initiate transactions on half of KN&K, nor bind the firm by their actions.
 (4) Further, the assignment of firm interest to the lenders is not indicative of a partnership,
because the intent was to protect the firm’s profits, which represented the lenders’
compensation for the loan. The indenture was basically a mortgage, containing the terms of
KN&K’s performance of the loan. The indenture did not contain any terms of partnership. The
option’s provision giving Hall the right to demand the resignation of KN&K members was
unusual, but as the intent was to protect the lenders against speculative transactions that could
render the option itself worthless, it does not show that a partnership was formed. Questions of
whether a partnership is formed between various entities are a matter of degree, to be
determined on a case-by-case basis. In this case, the loan documents do not show that a
partnership existed between the lenders and KN&K. The judgment of the trial court is affirmed.
o Notes
 If a creditor possesses too much control over the activities of the debtor, this risks a finding that
he becomes liable as a partner for debtors obligations
 Facts courts look to [in determining whether partnership exists] (not determinative
individually)
 (1) Sharing profits
o This case  lenders get 40% profits until debt paid off but, this is payment of
debt so acts as exception to rule
o Unlike true share of profits, there existed a floor and ceiling payment limit
regardless of profits of firm.
o Additionally, there was a mandatory debt payment schedule (fixed charge)
regardless of the profits.
 (2) Control
o Totality of circumstances inquiry that involves question of degree: ownership
requires control. Viewed on a spectrum. This is passive.
o Passive |-----------------------|Active
 Passive  stopping things from happening
 Active  affirmatively making things happen
 Peyton, Perkins, and Freeman (Ds) expressly declined to become partners of the firm.
 Is that relevant? No. The only thing that matters is the intent to do the things that meet
the legal definition of the partnership: “two or more person to carry on as co-owners a
business for profit” regardless of subjective intent.
o Lipien v. Malsbenden (demonstrates a partnership) P. 38, n. 3.
 Facts:
 Plaintiff entered into a written agreement with Steven Cragin, doing business as York
Motor Mart, for the construction of a Bradley Automobile. The plaintiff paid a total of
$4,450 of the $8,000 purchase price, but the Bradley was never received. After Cragin
disappeared, a suit was filed against Frederick Malsbenden on the theory that he was a
partner of Cragin in the business
 Issue: Whether a partnership existed
 Holding:
 Notwithstanding Malsbenden’s assertion that he was only a “banker,” his “total
investment” in the Bradley operation was that of a partner.
 Malsbenden had a financial interest of $85k in the Bradley portion of York Motor Mart’s
operations. He conceded that the loan carried no interest. His loan was not made in the
form of a fixed payment or payments, but was made to the business, at least in
substantial part, in the form of day-to-day purchases of Bradley kits, other parts and
equipment, and in the payment of wages. Further, the loan was not to be repaid in
fixed amounts at fixed times, but rather only upon the sale of Bradley’s automobiles.
 The evidence also showed that unlike a banker, Malsbenden had the right to participate
in control of the business and in fact did so on a day to day basis.
 Additionally, there was no indication that the arrangement would end when payment
was satisfied. This was very active control.
 Rule: Degree of control factors
 (1) active
 (2) ran all aspects of business
 (3) made day to day decision
 (4) paid employees
 (5) got repaid when goods sold (not fixed payment schedule)
 (6) no interest
 (7) loan made to business (not in fixed payments)
 **Whether a partnership has been formed is a totality of circumstances inquiry that involves question of
degree: (1) profit sharing, (2) control – are the most important factors in this inquiry. If profit sharing is not
determinative, focus on whether direct control is still present.

 2) The Partnership Agreement.


o RUPA § 103(a): Effect of Partnership agreement; nonwaivable provisions
 Except as otherwise provided in subsection (b), relations among the partners and between the
partners and the partnership are governed by the partnership agreement. To the extent the
partnership agreement does not otherwise provide, this [act] governs the relationship among
the partners and between the partners and the partnership
 Partnership agreement may be written, oral or implied among partners
 SOF  may require written agreement when real estate is contributed as partnership
property or involves terms of more than 1 year
o Note
 Almost all statutory provisions function merely as default rules that can be altered by
agreement of the partners; thus in a general partnership, partners can structured how they see
fit.

 3) Entity v. Aggregate Views


o (1) Aggregate  aggregate of partners with no separate legal status
 UPA § 29: Dissolution defined
 The dissolution of a partnership is the change in the relation of the partners caused by
any partner ceasing to be associated in the carrying on as distinguished from the
winding up of the business
o If aggregate  a partnership is dissolved whenever any partner ceased to be
associated in the carrying on of the business
o if legal entity  departure of partner would not have to affect the existence of
the partnership itself (see RUPA)
o 2) Distinct Legal Entity  partnership can be sued in its own name; can hold (real) property on its own
 RUPA § 201(a) Partnership as entity [rejects UPA].
 A partnership is an entity distinct from its partners.
o Fairway v. Title Insurance Co.
 Facts:
 A general partnership purchased a title guaranty policy. The defendant, title insurance
company attempted to avoid payment under the policy on the theory that the insured
partnership no longer existed.
 Issue:
 Whether partnership is an entity or an aggregate of partners.
 Rule:
 CL: A dissolution occurs, and a new partnership is formed whenever a partner retires or
a new partner is admitted.
 Holding:
 Conclusion accords with the aggregate theory of partnership , which, applied to this
case, recognizes Fairway Development I not as an entity in itself, but as a partnership
made up of three members. That partnership ceased when the members of the
partnership changed
 The terms of the title guaranty extended only to the named party guaranteed, that
party being fairway development I, and that fairway development II therefore has no
standing (because it no longer existed) to sue the defendant for breach of contract in
question.
 How could you avoid this situation (in UPA jurisdiction)? (1) By contract. When client sends you
the policy, make sure it covers the partnership and any successor partnership. (2) Have the
policy also cover the individual partners as well as the partnership.
o Taxes
 Taxed on pass-through basis. Profits and losses passed through to partner and each partner
must file own tax return. No double taxation.
 If partnership has losses, individual partners can use those losses to offset other income on
individual return. Has tax benefit.

 4) Partnership by Estoppel
 Smith v. Kelley (estoppel for third parties against partner that was purportedly held out as partner)
o Facts:
 Smith (plaintiff) worked at an accounting firm where Kelley and Galloway (defendants) were
partners. There was no written partnership agreement between Smith, Kelley and
Galloway, though the contract between them called Smith a partner, as did various other
forms. Kelley and Galloway testified that Smith did not contribute to the partnership’s
assets, take part in management, hire or fire employees, or sign notes for borrowed
money, that he was not liable for the partnership’s losses, and that there was no
agreement that Smith would share in a percentage of the partnership’s profits. During
Smith’s three-and-a-half years at the firm, he did not claim entitlement to a share of the
firm’s profits. After Smith left the firm, he brought suit for a partnership accounting,
claiming entitlement to a percentage of the firm’s profits.
o Issue:
 (1) Have the parties inadvertently formed a partnership, and if not, does argument of
estoppel create a partnership?
 (2) Could plaintiff (Smith) be liable to third parties?
o Rule:
 (1) A partnership may be created by estoppel, without a formal partnership agreement,
where the parties intend to and by their actions do create a partnership relationship.
 (2) RUPA § 308: Liability of Purported partner
 (a) if a person, by words or conduct, purports to be a partner, or consents to being
represented by another as a partner in a partnership or with one or more persons
not partners, the purported partner is liable to a person to whom the
representation is made (i.e. third party), if that person, relying on the
representation enters into a transaction with the actual or purported partnership
o Holding:
 (1) Here, the trial court’s finding that Kelley and Galloway did not intend for Smith to share
in a percentage of the firm’s profits is not clearly erroneous. In addition, the conduct of
Smith, Kelley, and Galloway during Smith’s three-and-a-half years at the firm shows that no
partnership was intended or created. Because Smith was not a partner in Kelley and
Galloway’s partnership, the trial court’s dismissal of Smith’s claim is affirmed.
 Estoppel doesn’t help in this case because it is meant to help third parties against a
partner that was purportedly "held out” as a partner NOT for partners themselves
against other partners
 (2) Under RUPA § 308, Smith could have been liable as a partner for the debts of the
partnership against third parties because he consented to be held out as a partner (“name-
tag” partner).
 Assume a client sues the partnership for malpractice. Key Q: Whether the client
entered into an engagement with the partnership in reliance on Smith being a partner.
o Client investigates Smith’s solvency/ involvement before deciding to do business
= it would seem client was relying on Smith’s statements that he was a partner
and believed his assets would be available.
o On the other hand, if the client would have done business with the partnership
without Smith (or it only had 2 partners) = difficult to argue credit was given on
the faith of Smith’s partnership representation.
o Analyze the definition and analyze the presumption, then argue in the alternative. “Even if…”
o **How is a partnership taxed?
 In partnerships it is a “pass through tax” so the profits pass through the partnerships to the
partners, and it is the individual partners that pay the tax on their own profits.
 As opposed to corporate taxation that has the double taxation of taxing both the corp. and the
people receiving distributions/ profits.

C) Management and Operations


 UPA § 18: Rules Determining Duties of Partners
o (c) all partners have equal rights in the management and conduct of the partnership business
o (h) any difference arising as to ordinary matters connected with the partnership business may be
decided by a majority of the partners; but no act in contravention of any agreement between the
partners may be done rightfully without the consent of all the partners
 RUPA § 401: Partner’s Rights and Duties
o (f) each partner has equal rights in the management and conduct of the partnership business
o (j) a difference arising as to a matter in the ordinary course of business of a partnership may be decided
by a majority of the partners. An act outside the ordinary course of business of a partnership and an
amendment to the partnership agreement may be undertaken only with the consent of all the partners
 Summers v. Dooley.
o Facts
 Summers (plaintiff) and Dooley (defendant) were co-partners in a trash collection business. Both
partners operated the business. The partners agreed that when one partner was unable to
work, he could hire a replacement at his own expense. Several years after the formation of the
partnership, Summers asked Dooley if he would agree to hire an additional employee. Dooley
refused, but Summers hired the worker anyway and paid him out of his own pocket. In spite of
the fact that the new worker was a good employee, Dooley would not agree to pay him out of
the partnership funds. Summers sued Dooley, seeking reimbursement for the expenses
Summers incurred in hiring the new employee.
o Issue:
 Is a partner who refused to hire an additional employee liable to a co-partner for expenses
incurred in hiring a new worker?
o Rule:
 General Rule: In a general partnership, each partner has an equal right in managing the
partnership’s business.
 Unless there is an agreement to the contrary, differences between the partners about
everyday business are to be decided by a majority of the partners.
 When a partnership consists of only two partners, one partner cannot unilaterally bind
the partnership by incurring expenses over the objection of the other.
o Holding: In this case, Summers hired the additional worker after Dooley clearly expressed his objection.
Under these circumstances, it would be unfair for Dooley to be forced to pay an expense that Summers
incurred for his own benefit, rather than for the benefit of the partnership.
o Notes:
 Q1: What could the partners have done to prevent this issue?
 Put language in the partnership agreement re decisions like this
 Q2: What vote is required to authorize an ordinary business atter in a general partnership?
 Majority
 Q3: What vote is req’d to authorize an extraordinary business matter in a general partnership?
 100%
 National Biscuit Co. v. Stroud
o Facts: Stroud (defendant) and Freeman formed a general partnership to sell groceries. The partnership
agreement did not limit either partner’s authority to conduct ordinary business on behalf of the
partnership. Several months before the partnership was dissolved, Stroud told a National Biscuit
Company (NBC) (plaintiff) official that he would not be personally liable for any bread sold to the
partnership. Freeman subsequently ordered more bread on behalf of the partnership, and NBC
delivered that bread to the partnership. Shortly thereafter, the partnership was dissolved, and Stroud
refused to pay for the bread delivered at Freeman’s behest. NBC sued the partnership and Stroud for
the price of the bread. The trial court found in favor of NBC.
o Issue
 Can one general partner restrict another partner from conducting business on behalf of a two-
person partnership?
 Whether partner has powers and authority to transact in the ordinary course of business and
bind the partnership and his co-partner
o Rule: In a general partnership with two partners, each party has the power to bind the partnership in
matters pertaining to the partnership’s business.
 Each partner has an equal right in the management and conduct of a partnership, and
differences within a partnership are decided by a majority of the partners.
 However, when there are only two partners there can be no majority, and neither
partner can prevent the other from binding the partnership in the ordinary course of
business.

o Holding: Freeman’s purchase of bread was a binding transaction, done pursuant to the partnership’s
business. Stroud, as Freeman’s sole co-partner, had no authority to negate Freeman’s purchase. The
partnership sold the bread that Freeman bought, and consequently Stroud, as well as Freedman,
benefited from that purchase.

 Case Comparison (Stroud v. Summers)


o Summers
 You can’t do anything in the ordinary course of business unless the majority says yes
o Stroud
 You can do anything in the ordinary course of business unless majority says no
o Reconciliation
 Look at the status quo (ordinary course) at the time business decisions were made
 Summers status quo  at time decision was being made, there were no employees and no
practice of hiring employees
 Majority required to change status quo
 Stroud status quo  at time decision was being made, the company already sold bread to
NBC / trying to change status quo
 Majority required to change status quo
 Authority
o UPA § 9: Apparent Authority
 (1) Every partner is an agent of the partnership for the purpose of its business, and the act of
every partner…for apparently carrying on in the usual way the business of the partnership of
which he is a member binds the partnership, unless the partner so acting has in fact no
authority to act for the partnership in the particular matter, and the person with whom he is
dealing has knowledge of the act that he has no such authority
o RUPA § 301: Apparent Authority
 (1) each partner is an agent of the partnership for the purpose of its business. An act of a
partner…for apparently carrying on in the ordinary course of the partnership business or
business of the kind carried on by the partnership binds the partnership, unless the partner had
no authority to act for the partnership in the particular matter and the person with whom the
partner was dealing knew or had received a notification that the partner lacked authority
o Issue: why do these statutes cause a problem for partnerships?
 They tell us that every partner has the ability to bind the partnership using apparent authority in
ordinary course of business
 They do NOT allow elimination of apparent authority by contract because contract cannot be
used
o Issue: How can a partnership avoid this problem?
 Letter of intent / authority.
 Partnership can provide notice of a partner’s lack of authority to creditors
 RUPA permits public statement of partnership authority but only for real estate transaction
 Inspection and Information
o UPA/RUPA  gives partners right to inspect books and records of partnership
o This is an affirmative obligation and failure to provide information constitutes breach of partners
fiduciary duty

D) Financial Rights and Obligations


 1) Partnership Accounting
o Capital account  money and property contributed by the partner +/- profits or losses by the
partnership allocated other partner (–) any distributions made to the partner
 Shows: is there value in the company, and if there is, how much of that does the partner have in
terms of equity (partners equity in partnership)
o Balance sheet  assets (–) liabilities = owner’s equity
 Shows: snapshot of companies positions at one point in time
 Because creditors always come before owners, any money that is left over after paying liabilities
belongs to partners but if liabilities exceed assets, there is negative owners’ equity
 2 ways to handle negative owners’ equity
 (1) retain and reinvest
 (2) distribute out to partners
o Income statement  revenue (–) expenses = net revenue/income
 Shows: Profit or loss for the year; paints an overview picture of how company operated for a
fiscal year. (“bottom line”)
o Statement of cashflow  trash cash (real) as it goes in and out of the business (i.e. personal checking
account)

 2) Sharing of Profits and Losses


o Profit Splitting
 UPA § 18: Profit Splitting (default rule)
 The rights and duties of the partners in relation to the partnership shall be determined,
subject to any agreement between them, but the following rules
o (a) each partner shall be repaid his contributions, whether by way of capital or
advances to the partnership property and share equally in the profits and
surplus remaining after all liabilities, including those to partners, are satisfied,
and must contribute towards the losses, whether of capital or otherwise,
sustained by the partnership accounting to his share in the profits
 RUPA § 401: Profit Splitting (default rule)
 (b) each partner is entitled to an equal share of the partnership profits and is
chargeable with share of the partnership losses in proportion to the partners share of
the profits
 Hypo
 Facts: Partner 1 contributes 70% of capital ($70k) / Partner 2 contributes 30% of capital
($30k)
o Rule: The fact that the partners contributed unequal amounts does not change
the default rules regarding profits; Contribution amounts are irrelevant to
default rules
o Holding: if partnership, after paying liabilities makes an extra $1mil, both would
get $500k as distribution
 Issue 1: is there any way to change this result?
o Holding: Partner 1 and Partner 2 can agree that the profits should be split
according to contribution amounts (must be in partnership agreement)
 Issue 2: what if partners agree that only profits will be split between contribution
amounts and agreement is silent about these losses
o Holding: because default rule of equal profit sharing has been displaced, profits
will be split according to the agreement (70/30) and as such, losses will also be
split according to this allocation
 Issue 3: what if partner 1 and partner 2 agree that losses will be split according to
contribution amounts
o Holding: default rule of losses has been displaced so losses split as agreed 70/30
allocation
o BUT default rule of equal profit sharing has not so profits would still be split
50/50
 DEFAULT RULE: losses follow profits BUT profits do not follow losses

 Kessler v. Antinora
 Facts
o Kessler (plaintiff) and Antinora (defendant) entered into a partnership
agreement to build and sell a house. The partnership agreement stated that
Kessler would provide the money and Antinora would act as the general
contractor for the project. The agreement also provided that, after the house
was sold, Kessler would be refunded his contribution plus interest, after which
Kessler would receive 60 percent of the profits of the sale and Antinora would
receive 40 percent. The partnership agreement did not mention what would
happen in the event of a loss. The house was sold at a loss. Kessler sued
Antinora to recover 40 percent of his loss on the sale.
 Issue
o Where there is no partnership agreement as to losses, and where one partner
contributes money to the venture and the other contributes services or labor, in
the event of a loss is either party liable to the other for any loss sustained?
 Rule
o In the absence of a partnership agreement, it is assumed that partners intend
to share equally in the profits and losses of the enterprise, regardless of the
amount of money they contributed to the partnership.
 Where there is no partnership agreement as to losses, and where one
partner contributes money to the venture and the other contributes
services or labor, in the event of a loss neither party is liable to the
other for any loss sustained.
 When such a partnership suffers a loss, each partner will lose his
contribution, one partner a portion of his money, and the other all of his
labor.
 Under New Jersey’s Uniform Partnership Law, each partner must share
equally in the profits of the partnership, and must contribute equally to
the losses sustained, unless an agreement between the partners states
otherwise..
 Holding
o In this case, the agreement between Kessler and Antinora is controlling, not the
statute. The agreement between them states that, after the house was sold,
Kessler would be refunded his contribution plus interest, after which Kessler
would receive 60 percent of the profits of the sale and Antinora would receive
40 percent. This agreement shows that the parties intended that Kessler would
be repaid his investment from the sale of the house, not directly from
Antinora. This court is persuaded by the reasoning of Kovacik v. Reed, 315 P.2d
314 (1957). In that case, the California Supreme Court acknowledged that, in the
absence of a partnership agreement it is assumed that partners intend to share
equally in the profits and losses of the enterprise regardless of the money they
contributed to the partnership. However, the court in Kovacikfurther held that
when only one partner contributes money, and the other partner contributes
labor and services, neither partner is liable to the other for any loss sustained.
o When such a partnership suffers a loss, each partner will lose his contribution,
one partner a portion of his money, and the other all of his labor. Here, Kessler
lost some of his money, and Antinora lost the entire value of his labor. The
written partnership agreement provided that Kessler would be repaid on his
investment out of the proceeds of the house’s sale, not out of Antinora’s assets.
In the absence of an agreement on how to apportion losses, both parties share
in the loss, one in dollars, and the other in labor. The lower court’s judgment is
reversed, and summary judgment is entered for Antinora
 Notes
o If venture had been profitable
 Assume cost $400k to construct home and the house sold for $900k
($500k surplus)
 To complete project Kessler put in $100k
 He would have to be paid first  $400k surplus to be split on 60/40
basis
 Kessler $240k
 Antinora $160k
o Re capital account
 A court may credit the capital account of the services providing partner
with the value of his services and labor
 Shifts losses away from a labor only partner

 3) Liability to 3rd parties


 Contractual Liability [of the partnership to third parties]
o General rule: partnership is liable for contract entered into on behalf of its partners with actual or
apparent authority
 (and all partners have apparent authority, but does not = “always” bound. Restrict authority
and notify others of restriction.)
o UPA § 9: Contractual Liability of the Partnership to third Parties
 (1) every partners is an agent of the partnership for the purpose of its business, and the act of
every partner…for apparently carrying on in the usual way the business of the partnership of
which he is a member binds the partnership, unless the partner so acting has in fact no
authority to act for the partnership in the particular matter, and the person with whom he is
dealing has knowledge of the fact that he has no such authority
 Partner is agent  this binds the partnership if the partners has authority
o RUPA § 301: Contractual Liability of the Partnership to third Parties
 (1) each partners is an agent of the partnership for the purpose of its business, and the act of
every partner…for apparently carrying on in the ordinary course the partnership business OR
business of the kind carried on by the partnership binds the partnership, unless the partner so
acting has in fact no authority to act for the partnership in the particular matter, and the person
with whom he is dealing has knowledge of the fact that he has no such authority
 Tort Liability [of the partnership to third parties]
o General rule: Partnership liable to third parties for any wrongful act or omission of any partner acting (1)
in the ordinary course of the business; or (2) with authority of his co-partners
o UPA § 15: Nature of Partner’s Liability
 All partners are liable
 (a) jointly and severally for everything chargeable to the partnership under section 13
and 14 [TORT]
 (b) jointly for all other debts and obligations of the partnership [CONTRACT]: but any
partner may enter into a separate obligation to perform a partnership contract
o Jointly and severally  permits plaintiff to sue one or more of the partners
without having to sue them all (or even partnership).
o Jointly  require plaintiff to join off of the partners in the litigation
o RUPA § 306(a): Partner’s Liability
 Except as otherwise provided in subsection (b) and (c) [related to actions prior to admission as a
partner and actions taken while limited liability entity] all partners are liable jointly and
severally for all obligations of the partnership unless otherwise agreed by the claimants and
provided by law. [Exception – Judgment creditors must first exhaust Partnership assets.
Seemingly “secondarily” liable].
o Force partners to sign an individual / personal guarantee. That kicks in if client wishes to sue.

 A judgement creditor is first required, with certain exceptions, to sue the partnership,
along with the partners and then exhaust partnership assets before proceeding directly
against a partners individual assets
o Typical Route (under RUPA) if you are representing a partnership and partners for the debts of the
partnership
 Roach v. Mead
o Facts
 Berentson and Mead (defendants) were partners in a law firm. Mead represented Roach
(plaintiff) on several occasions. When Roach asked Mead’s advice on investing the proceeds of a
business sale, Mead told Roach he would take the money at 15 percent interest. Roach
testified that he considered this to be legal advice, and he gave Mead $20,000. Mead did not
repay any of the money, and Roach sued Mead’s partnership for negligence. Berentson moved
for a directed verdict, arguing that he was not vicariously liable for Mead’s negligent acts
because they were outside the scope of the partnership’s business.
o Issue
 May a partner in a law firm be held vicariously liable for another partner’s failure to advise a
client of the legal consequences of loaning money to that law partner?
o Rule
 When a lawyer borrows money from a client but fails to advise the client about the legal
aspect of the loan, the lawyer’s law partners may be held vicariously liable for that negligence.
 A partner is jointly and severally liable for the tortious acts of another partner if he authorizes
those acts or if the acts are committed in the ordinary course of the partnership’s business.
 If a third party reasonably believes that the services he has requested from a partnership will be
performed in the ordinary course of the partnership’s business, the partnership as a whole is
liable for any tortious acts committed in the course of providing those services, regardless of
whether others in that profession would consider those services to be an ordinary part of such
a business.
 Whether the third party’s belief is reasonable is a question to be determined on the facts of
each particular case.
o Holding
 In this case, Roach thought Mead was giving him investment advice and that such advice was
an ordinary part of Mead’s law practice. The jury and the court of appeals found this belief to
be reasonable, and this court agrees. When a lawyer borrows money from a client, the lawyer
must advise the client to seek independent legal advice, and to secure the loan and check the
debtor’s financial status, and must advise the client of the risks of providing a usurious
interest rate. Mead’s failure to do these things constitute failures as a lawyer in advising a client,
which occurred within the scope of the legal partnership. Berentson is therefore vicariously
liable for that negligence, as Mead’s law partner. The court of appeals’ decision is affirmed.
o Notes
 Issue: Why was the extra $1500 loaned to Mead not in the ordinary course of business?
 Loan was personal so not in scope  roach was not asking what he should do with the
$1,500
 If client did not seek such advice or assistance but merely “relied” on attorney to protect him 
no establishing of attorney client relationship or legal malpractice with respect to loans
 Plaintiff paid attorney retainer to “in return for full, total, and immediate access to
attorney legal advise
 This is distinct from the money plaintiff loaned attorney

 Indemnification (security against legal liability for ones actions)
o Indemnification  obligation of a partnership to a partner (reimbursement – 100% [if it meets
definition]).
o Contribution  obligation of a partner to another partner (If partnership does not have the money, look
to partners or for $, partner contributes half, I contribute half).
o UPA/RUPA
 (1) a partnership must indemnify a partner for expenses incurred by the partner in the ordinary
course of partnership business
 (2) partners must contribute to the partnership if the partnership is unable to satisfy its
obligations (including any indemnification obligations)
 Net effect of indemnification and contribution is that, if all parties are solvent (having
an excess of liabilities / unable to cover debts), a partner who is rightfully sued for
entirety of partnership obligation will only bear his/her pro rata share of the obligation
 In ongoing partnership  indemnification payments reduces partnership profits / as a
result, each partner of a profitable venture “suffers” from indemnification in proportion
to his profit share
 On dissolution  partnerships obligation to indemnify is paid out of partnership assets
o As between partnership and outside creditors
 Individual partner has unlimited personal liability for obligations of the partnership
 Outside creditor  may collect the entirety of a partnership obligation from any
partner (under UPA) and any partner (under RUPA) if RUPA requirements have been
met
o As between partners themselves
 Each partner is only responsible for his share of the partnership obligation
 If one partner pays off partnership obligation, he is entitled to indemnification from the
partnership
 If partnership lacks funds to indemnify the partner, partners are required to contribute
according to loss share
 Summer v. Dooley
 Assume summers did have the authority to hire the employee and pay employee salary
out of pocket ($10k)
 Issue: would Summers be entitled to indemnification?
 Holding: Yes, employee compensation was made on the partners behalf in the ordinary
course of the business
 Issue: what if the partnership lacks funds sufficient to indemnify summers?
 Holding: Dooley would have to make a contribution payment of 50% of the contribution
(because only 2 partners)
o Dooley’s loss share would go to the partnership and the partnership would use
those shares to indemnify summers
 Net result  summers and Dooley would end up funding ½ of the partnership
obligation out of his own respective pocket
 Issue: what if Dooley is also solvent?
 Holding: there would be o for reimbursement purposes

E) Fiduciary Duties
 1) The Common Law
o Meinhard v. Salmon (broad fiduciary duty of honesty)
 Facts
 Salmon (defendant) executed a 20-year lease (Bristol Lease) for the Bristol Hotel which
he intended to convert into a retail building. Concurrent with his execution of the
Bristol Lease, Salmon formed a joint venture with Meinhard (plaintiff). The joint
venture’s terms provided that Meinhard would pay Salmon half the amount required
to manage and operate the property, and Salmon would pay Meinhard 40 percent of
the net profits for the first five years, and 50 percent thereafter. Both parties agreed to
bear any losses equally. The joint venture lost money during the early years, but
eventually became very profitable. During the course of the Bristol Lease another lessor
acquired rights to it. The new lessor, who also owned tracts of nearby property, wanted
to lease all of that land to someone who would raze the existing buildings and
construct new ones. When the Bristol Lease had four months remaining, the new lessor
approached Salmon about the plan. Salmon executed a 20-year lease (Midpoint Lease)
for all of new lessor’s property through Salmon’s company, the Midpoint Realty
Company. Salmon did not inform Meinhard about the transaction. Approximately one
month after the Midpoint Lease was executed, Meinhard found out about Salmon’s
Midpoint Lease, and demanded that it be held in trust as an asset of the joint venture.
Salmon refused, and Meinhard filed suit
 Issue
 Is a co-adventurer required to inform another co-adventurer of a business opportunity
that occurs as a result of participation in a joint venture?
 Whether Salmon breached his fiduciary duty as partner and if so, what is the extent of
Meinhard’s equitable interest in the new lease
 Rule
 Co-adventurers, like partners, have a fiduciary duty to each other, including sharing in
any benefits that result from the parties’ joint venture.
 As sharers in a joint venture, co-adventurers owe each other a high level of fiduciary
duty which is not honesty alone but the honor most sensitive (** Broad).
 A co-adventure who manages a joint venture’s enterprise has the strongest fiduciary
duty to other members of the joint venture.
o Partners have no obligation to remain partners under this standard
 Holding
 The Midpoint Lease was an extension of the subject matter of the Bristol Lease, in
which Meinhard had a substantial investment. Salmon was given the opportunity to
enter into the Midpoint Lease because he managed the Bristol Hotel property. Because
Salmon’s opportunity arose as a result of his status as the managing co-adventurer, he
had a duty to tell Meinhard about it. Salmon breached his fiduciary duty by keeping his
transaction from Meinhard, which prevented Meinhard from enjoying an opportunity
that arose out of their joint venture. Accordingly, the judgment of the appellate division
is affirmed, with a slight modification. This court holds that a trust attaching to the
shares of stock should be granted to Meinhard, with the parties dividing the shares
equally, but with Salmon receiving an additional share. The additional share enables
Salmon to retain control and management of the Midpoint property, which according to
the terms of the joint venture Salmon was to have for the entire length of that joint
venture
o The fact that Salmon kept quiet was the problem
 Dissent
 Salmon did not breach his fiduciary duty to Meinhard. The joint venture’s purpose was
to exploit the Bristol Lease exclusively, for a limited duration of 20 years. Salmon
fulfilled his duty to Meinhard by managing the Bristol Hotel property and distributing
Meinhard’s share of the profits during the term of the Bristol Lease. Salmon’s fiduciary
duty to Meinhard was restricted to matters pertaining to the Bristol Lease, and ended
when the Bristol Lease expired. The judgment of the lower courts should be reversed,
and a new trial ordered.
 Notes
 (1) Fiduciary duty is not affected by distrust between the partners
 (2) Meinhard suggests that a partners disclosure obligation is fiduciary in nature (i.e.
partners fiduciary duty includes a duty to disclose info to other partners in certain
circumstances
o UPA does not refer to obligation as a fiduciary ones
o RUPA  the use of “only” clearly indicates that the disclosure obligation is not
an independent fiduciary duty

 2) Statutory Developments and the role of contract


o UPA § 21: Partner Accountable as a Fiduciary
 (1) every partner must account to the partnership for any benefit, and hold as trustee for it any
profits derived by him without the consent of the other partners from any transaction
connected with the information, conduct, or liquidation of the partnership or from any use by
him of its property
 Does NOT define duty of care for partners
 Broad fiduciary duty among partners
o RUPA § 404: General Standards of Partner Conduct [substantially clearer]
 (a) the only fiduciary duties a partner owes to the partnership and the other partners are the
duty of loyalty and the duty of care set forth in subsection (b) and (c)
 (b) a partners duty of loyalty to the partnership and the other partners is limited to the
following
 (1) to account to the partnership and hold as trustee for it any property, profit, or
benefit derived by the partner in the conduct and winding up of the partnership
business or derived from a use by the partner of partnership property, including the
appropriation of a partnership opportunity
 (2) to refrain from dealing with the partnership in the conduct or winding up of the
partnership business as or on behalf of a party having an interest adverse to the
partnership; and
 (3) to refrain from competing with the partnership in the conduct of the partnership
business before the dissolution of the partnership
 (c) a partners duty to care for the partnership and the other partners in the conduct and
winding up of the partnership business is limited to refraining from engaging in grossly
negligent or reckless conduct, intentional misconduct, or a knowing violation of the law
 Note: the BJR provides a substantial amount of protection against duty of care claims
[when they are acting in good faith even if they have been negligent in their decision
making]
 (d) A partner shall discharge the duties to the partnership and the other partners under this
[Act] or under the partnership agreement and exercise any rights consistently with the
obligation of good faith and fair dealing.
 Contractual concept imposed upon the partners because of the consensual nature of
partnership
 (e) A partner does not violate a duty or obligation under this [Act]or under the partnership
agreement merely because the partner’s conduct furthers the partner’s own interest.
 Modifies 404(b)(2) such that fair conflict of interest transaction would not result in
liability.
 Excuses partners from accounting incidental benefits obtained in course of partnership
activity without detriment to partnership
 (f) A partner may lend money to and transact other business with the partnership, and as to
each loan or transaction the rights and obligations of the partner are the same as those of a
person who is not partner, subject to other applicable law.
 Partner could purchase assets of partnership at a foreclosure sale.
 *Rupa § 404(b) focus on duty of loyalty.
 (1) Account. Receiving value for partnership, protect it, and provide to partnership.
 (2) Refrain from dealing. Should not take away from partnership opportunities.
 (3) Refrain from competing. Setting up something else to take away from partnership.
 Rupa § 404(e) E.g., Renting from bank at interest rate when they were willing to lend to
partnership. Fights will be about whether the benefit extends to the partnership (or just
detriment). Incidental benefits are okay.
o A) The Duty of Loyalty (directed at partner not being dirt bag) [Dirt Bag]
 See notes above. Rupa § 404(b).
 Enea v. Superior Court (how RUPA 404 works and how may be modified by 404(e)/(f))
 Facts
o Enea (plaintiff) sued William and Claudia Daniels (defendants), his former
partners, for breaching their fiduciary duties by renting the partnership’s sole
asset, an office building, to themselves at less than fair market value.  The trial
court found that California law authorized such conduct, that the Daniels had
no duty to collect fair market value rent in the absence of a contract requiring
them to do so, and that the primary purpose of the partnership was to hold the
property for later sale as opposed to collecting market value rent.
 Issue
o May partners rent partnership property to themselves at less than the fair
market rental value?
o Whether defendants were entitle to lease partnership property to themselves
or associated entities, at less than it could yield in the market
 Rule
o Partners must carry out the business of the partnership with the loyalty and
care of a fiduciary, and may not take advantages for themselves at the
expense of the partnership.
o Partners may not rent partnership property to themselves at less than the fair
market rental value.
o Revised Uniform Partnership Act (RUPA) § 404(e), a partner does not violate his
duties of loyalty and care merely because his conduct furthers his own
interest. However, this section does not permit a partner to further his own
interest at the expense of the partnership
 Purpose of this  the excuse partners from accounting for incidental
benefits obtain in the course of partnerhsip activities without
detriment to the partnership
 Holding
o In this case, the Daniels clearly took advantages for themselves of partnership
property at the expense of the partnership, by renting partnership property at
less than fair market value, at the cost of collecting full market value rent from
an independent tenant who could have rented the property.
o Contrary to the trial court’s finding, California Corporations Code § 16404 does
not permit the Daniels to rent the partnership property to themselves at less
than fair market value. Under § 16404(e), which is identical to the Revised
Uniform Partnership Act (RUPA) § 404(e), a partner does not violate his duties
of loyalty and care merely because his conduct furthers his own interest.
However, this section does not permit a partner to further his own interest at
the expense of the partnership.
o While § 16404(e) would permit the Daniels to rent partnership property at the
same rental rate that another tenant would have paid, it does not permit them
to rent partnership property for a lower rental rate at the expense of the
partnership. The trial court also incorrectly found that the Daniels had no duty
to collect full market value rent in the absence of a contract requiring them to
do so. Partners do not simply owe those duties to one another that are set out
by contract. Rather, fiduciary duties of partners are imposed by law.
o Finally, the trial court erred in finding that the Daniels’ conduct was permissible
because the partnership’s primary purpose was to hold the property for
appreciation and later sale. The fact that the main economic benefit to be
gained from the partnership was the proceeds from an eventual sale does not
entitle partners to take other benefits for themselves that would otherwise
have gone to the partnership. The Daniels breached their fiduciary duty as
partners by renting partnership property to themselves at less than full market
value rent. The trial court’s judgment is therefore reversed, and the Daniels’
motion for summary judgment denied.
 Notes
o (1) What if defendant rented to themselves at fair market value?
 Protects partners from liability for incidental benefits not detrimental to
partnership
 RUPA 404(e)  Fair rent would not have deprived partnership of
valuable assets/not at partnership expense.
 RUPA 404(b)(2)  even fair market rent could still be problematic.
 Defendants have an interest adverse to partnership as they
are lessees generally seeking lower rent at all times while
partnership as is lessor is seeking higher rent at all times
ongoing.
 Any conflict of interest transaction, whether fair or unfair is
impermissible.
 Reconciling RUPA 404(e) and RUPA 404(b)(2).
 RUPA 404(e)  provides fairness defense if market rate is used
(only prohibited if self-dealing hurts partnership) [modifies
404(b)(2) -- Fair Conflict of Interest transaction would not result
in liability]. Excuses partners from accounting incidental
benefits obtained in course of partnership activity without
detriment to partnership.
o (2) Fiduciary duty pre-formation
 UPA  “refers to formation, conduct, and liquidation’
 Imposes this duty at formation
 RUPA  refers to “conduct and winding up’
 No duty of loyalty in connection with formation of partnership
 Pre formation subject to general contract obligation to deal in
honesty and without fraud
o B) The Duty of Care (directed at partner not doing a bad job) [Bad job]
 See notes below for definition. RUPA 404(c) –
 Requires gross negligence/recklessness
 Does not speak to ordinary negligence (i.e. negligent mismanagement not deliberate
wrongdoing)
 **Best defense of someone accuses you of negligent actions = Business Judgment Rule (creates
presumption).
 Business Judgment Rule (BJR) is in Rupa not UPA.
 E.g., suing a doctor just involves proving negligence compared to a industry standard.
Same with a lawyer. No Business judgment rule helping in their favor.
 **Very difficult to prevail because of presumption (acted with duty of care) / law says
“ordinary negligence is not enough’.
 Could make the dumbest decision of the decade, but if you can show you were informed
about it (thought about it) you are okay.

 Bane v Ferguson (fiduciary duties of former partners / UPA)
 Facts
o Bane (plaintiff) was a partner in a law firm in Illinois. The firm’s retirement plan
instrument provided that the plan and retirement benefit payments would end
if the firm dissolved without a successor entity. Bane retired and began
collecting retirement benefits. The firm merged with another firm and
eventually dissolved without a successor entity, and Bane’s retirement
benefits were terminated. In a diversity action governed by Illinois law, Bane
sued the members of the firm’s managing council (the council) (defendants),
claiming that the council negligently managed the firm in the period leading
up to the dissolution and merger. The district judge dismissed the complaint
 Issue
o Does a retired law partner (former partner) have a common law or statutory
claim against his former law firm for its acts of negligence?
 Rule
o A retired law partner does not have a common law or statutory claim against
his former law firm for its acts of negligence. A partner has fiduciary duties to
other partners, but not to former partners, since a partner’s withdrawal
terminates his relationship in the partnership.
o BJR (Business Judgment Rule) helps to guard a corporations BoD from
frivolous legal allegations about any way it conducts business. The rule states
that boards are presume to act “in good faith”—that is, within fiduciary
standards of loyalty, prudence, and care directors owe to shareholders
 Holding
o (1) Bane first argues that the council’s mismanagement of the firm violated the
Uniform Partnership Act (UPA) § 9(3)(c), which provides that a partner is not
authorized to act in a way that would make it impossible to carry on the
partnership’s ordinary business, unless authorized to do so by the other
partners.
 However, that section is inapplicable here, since its purpose is merely
to limit the liability of partners for the unauthorized acts of another
partner, not to make the partners liable to parties with whom the
partnership transacts.
o (2) Bane next argues that the council violated a fiduciary duty to him.
 To the contrary, the council had no fiduciary obligations toward Bane.
A partner has fiduciary duties to other partners, but not former
partners, since a partner’s withdrawal terminates his relationship in the
partnership.
 Even if the retirement plan’s managers were fiduciaries of Bane, the
negligence alleged is not in the management of the retirement plan,
but in the management of the firm.
 Bane does not allege that the plan’s managers failed to inform him of
his rights under the plan or mismanaged the plan’s funds.
 Further, even if the firm’s council is a fiduciary of Bane, it is shielded
from liability for negligence in operating the firm under the business
judgment rule, just as a corporation’s directors would be shielded from
shareholders’ negligence claims.
o (3) Bane also argues that the council breached its contract with him.
 However, the retirement plan instrument specifically states that the
plan would end upon the firm’s dissolution, and contains no
commitment to maintain the firm’s existence.
o (4) Lastly, Bane argues that the council violated a general tort law duty of care
owed to him.
 Illinois courts have previously found that a corporation’s dissolution
alone is not sufficient grounds for a tort action if it is motivated by
good faith judgment for the corporation’s benefit. This principle is just
as applicable to partnerships as it is to corporations. In this case, Bane
does not challenge the good faith of the council, merely its
competence. Because there is no remedy under Illinois law, the district
judge correctly dismissed the complaint, and the judgment is affirmed.
 Notes
o Issue: what if Bane is still partners and still making a claim of partnership being
dumb/negligent?
o Duty of Care: RUPA 404(c) governs
 Requires gross negligence/recklessness
 Does not speak to ordinary negligence (i.e. negligent mismanagement
not deliberate wrongdoing)
 In line with Business Judgment Rule which provides a substantial
amount of protection against duty of care claims.
 UPA does not attempt to define a duty of care for partners, which is why
we end up with CL claims and RUPA.
o C) Duty to disclose
 UPA § 20
 Partners “shall render on demand true and full information of all things affecting the
partnership to any partner”
 RUPA § 403: Partnership Rights and Duties with Respect to Information
 (c) each partner and the partnership shall furnish to a partner, and to the legal
representative of a deceased partner or partner under legal disability
o (1) Without demand, any information concerning the partnerships business and
affairs reasonably required for the proper exercise of the partners rights and
duties under the partnership agreement or this act
o (2) On demand, any other information concerning the partnership business and
affairs, except to the extend the demand or the information demanded is
unreasonable or otherwise improper under the circumstances
 Note: Meinhard suggests that partner disclosure obligation is fiduciary in nature BUT neither
RUPA/UPA refer to duty to disclose as fiduciary/it is a statutory/contractual obligation
o D) Contractual Limitations
 Issue: Whether contract can modify the obligations imposed by fiduciary duties (i.e. limiting or
eliminating duty of care)?
 **RUPA § 103: Contractual Modification of Duties
 (a) Except as otherwise provided in subsection (b), relations among the partners and
between the partners and the partnership are governed by the partnership agreement.
To the extent the partnership agreement  does not otherwise provide, this [Act]
governs relations among the partners and between the partners and the partnership.
 (b) The partnership agreement may not
o (3) eliminate the duty of loyalty under Section 404(b) or 603(b)(3), but:
 (i) the partnership agreement may identify specific types or categories
of activities that do not violate the duty of loyalty, if not manifestly
unreasonable; or
 (ii) all of the partners or a number or percentage specified in the
partnership agreement may authorize or ratify, after full disclosure of
all material facts, a specific act or transaction that otherwise would
violate the duty of loyalty;
 Duty of loyalty  Can’t be eliminated wholesale but can be
limited
o (4) unreasonably reduce the duty of care under Section 404(c) or 603(b)(3);
o (5) eliminate the obligation of good faith and fair dealing under Section 404(d),
but the partnership agreement may prescribe the standards by which the
performance of the obligation is to be measured, if the standards are not
manifestly unreasonable;
 **Default Rule (UPA & RUPA): We can alter, as long as the modifications or limitations on duty
of care / duty of loyalty do not rise to being “manifestly unreasonable.”
 Manifestly Unreasonably = found in litigation. Decided by judge.

 Singer v. Singer (partners are protected by language in partnership agreement)


o Facts: The Singer family created an oil production partnership, Josaline Production Co.,
in the late 1930s. Over the years, partnership interests were conveyed and passed down
to other family members. Andrea and Stanley Singer (defendants) are members in the
partnership, as is Joe L. Singer (plaintiff). In 1977 Josaline’s partnership agreement was
re-drafted. Paragraph 8 of the re-drafted partnership agreement states that any partner
may conduct his business as if he were not a member of the partnership, that
individual partners are free to enter into business transactions that conflict or
compete with the partnership’s business, and that other partners may not claim an
interest in such transactions. Shortly before a 1979 partnership meeting, Joe L. asked
Stanley to look into purchasing 95 acres of mineral land for the partnership. The
possible purchase was discussed at the partners’ meeting, but no decision was made.
After the meeting, Stanley and Andrea formed a general partnership realty company
and purchased the 95 acres without consulting any of the Josaline partners. Joe L.
brought suit, claiming that based on the fiduciary duties of partners Josaline was
entitled to participate in the land purchase. The district court agreed, finding that the
land purchased by Stanley and Andrea is held in constructive trust for the partnership.

o Issue: May partners compete with one another, and with the partnership, if the
partnership agreement permits such competition?
o Rule
 A partner has a fiduciary duty not to compete with other partners, or with the
partnership itself, in the absence of a partnership agreement stating otherwise.
 Where the partnership agreement permits it, partners may directly compete
with one another, and with the partnership itself.
o Holding: This court would thus agree with Joe L.’s contention that Josaline is entitled to
participate in the purchase of the 95 acres, were it not for paragraph 8 of the
partnership agreement. Paragraph 8 states that any partner may conduct his business as
if he were not a member of the partnership, and that individual partners are free to
enter into business transactions that conflict or compete with the partnership’s
business. Paragraph 8 therefore gives Stanley and Andrea a contract right to compete
with Josaline and its partners in acquiring assets, though it would not permit them to
obtain already-acquired partnership assets or to otherwise financially encumber the
partnership. Because Stanley and Andrea had a contract right to act as they did, the
district court’s judgment is reversed.

o Notes
 (1) ¶8  permitted any partner to engage in outside business transactions even
if those transactions competed with the partnership
 Absent contract modification  defendants are in breach of duty of
loyalty
 (2) Does RUPA allow contractually modifications to the duty of loyalty?
 RUPA §103(b)  modification allowed because not wholesale
elimination of duty of loyalty.
 (3) what about manifestly unreasonably language
 This is when it comes too close to wholesale elimination of duty or has
harmful effect on partnership (i.e. partnership collapses)
 3) Duties When Leaving a Partnership.
o Issue: what duties does a partner have to the partnership when the partner is contemplating leaving
the partnership to start a competing firm
 UPA § 20
 Partners “shall render on demand true and full information of all things affecting the
partnership to any partner”
 RUPA § 403: Partnership Rights and Duties with Respect to Information
 (c) each partner and the partnership shall furnish to a partner, and to the legal
representative of a deceased partner or partner under legal disability
o (2) On demand, any other information concerning the partnership business
and affairs, except to the extend the demand or the information demanded is
unreasonable or otherwise improper under the circumstances
 RUPA § 404: General Standards of Partner Conduct
 (b) a partners duty of loyalty to the partnership and the other partners is limited to the
following
o (3) to refrain from competing with the partnership in the conduct of the
partnership business before the dissolution of the partnership
 General Rule  partners may make arrangements to compete when they form a partnership
before leaving previous partnership, as long as departing partners are still carrying on roles in
correct partnership in appropriate fashion
 What Partners should not do
 (1) make misrepresentations or omission to other partners re: intention to leave
 (2) compete with the partnership while still acting as members of partnership (includes
soliciting current clients to follow the partners to the partnership
o Violates UPA § 20 and RUPA §403/404.
 **Partnership clients belong to the partnership, no matter how personal they may feel.

o Meehan v. Shaughnessy (leaving firm to form new partnership and poaching clients)
 Facts:
 Meehan and Boyle (plaintiffs), disgruntled partners in the law firm of Parker, Coulter, Daley &
White (Parker Coulter) (defendants), decided to quit that firm and form their own legal
partnership. Meehan and Boyle were subject to a Parker Coulter partnership agreement which
provided that partners leaving the firm could, for a fee, take clients who they themselves had
originated, subject to the right of the clients to remain at Parker Coulter. While still employed
at Parker Coulter, Meehan and Boyle secretly began preparing to take some clients with them.
Meehan met with a big client to discuss transferring that client’s business to the new firm.
Boyle prepared form letters on Parker Coulter letterhead addressed to a number of clients,
inviting them to become clients of the new firm. During Meehan and Boyle’s last few months at
Parker Coulter, various partners asked them if they were planning to leave. Meehan and Boyle
denied their intentions, preferring to wait until the end of the year to give Parker Coulter one
month’s notice of their resignation. Almost immediately after tendering his resignation, Boyle
sent his solicitation letters to selected Parker Coulter clients, and contacted attorneys who
could refer additional clients to the new firm. The Parker Coulter partners asked Boyle for a list
of clients he and Meehan planned to take with them, so they could inform the clients that they
could stay with Parker Coulter if they wished. Boyle waited several weeks to provide that list.
Meanwhile, Meehan and Boyle obtained authorizations from many Parker Coulter clients,
agreeing to become clients of the new firm. After leaving Parker Coulter, Meehan and Boyle
sued their former firm for compensation they claimed was unfairly withheld from them. Parker
Coulter filed a counterclaim alleging that Meehan and Boyle had breached their fiduciary duty
by unfairly acquiring consent from clients to remove cases from Parker Coulter.
 Issue
 Is it a breach of fiduciary duty for partners, while associated with a partnership, to
secretly solicit the partnership’s clients for their own gain, while denying their intentions
to other partners?
 Rule
 A partner has a fiduciary duty to provide, on demand of another partner, true and
complete information of any and all things affecting the partnership.
 Partners owe each other a fiduciary duty to act with loyalty and in good faith to each
other.
 Consequently, partners may not use their status as partners to purely benefit
themselves, particularly if their actions harm the other partners.
 Partners are allowed to make preparations before leaving a firm i.e. they are
permitted to set up a new firm while still working at partnership
o Making logistical arrangements for new firm = OK
 Holding
 Meehan and Boyle took unfair advantage of the other Parker Coulter partners by acting
in secret to solicit clients, falsely denying their plans to the other partners, and
delaying the release of the list of clients they planned to take with them until after
they had won their business.
 Also, the content of Boyle’s client letters was unduly harmful to Parker Coulter.
Pertinent ethical standards require that when attorneys planning to leave a firm solicit
clients, they must state that the clients have a choice of staying with the firm or
transferring their business to the departing attorneys’ new firm.
 Boyle did not put that information in his solicitation letters. This court finds that Meehan
and Boyle’s actions constituted a breach of their fiduciary duty to the other Parker
Coulter partners.
 Notes
 (1) Issue: What if plaintiff had spent 70% setting up new firm?
o Logistical preparation cannot detract from partners work at current firm
o Key: whether usual standard of performance is maintained
 Approaching other partners  part of logistics necessary to set up new
firm
o Breaches of Fiduciary duty
 (1) Meehan lied about decision to leave firm when directly asked him (did
not render full and true information)
 (2) plaintiff communication with existing clients was inappropriate (UPA §20
 Acting inappropriately with competing with the firm for clients prior
to leaving (RUPA §404(b)(3)
 Breach of duty because didn’t give firm fair chance to keep those
clients
 (2) what should plaintiff have done
o (1) letter 1  we are leaving firm / here is new firm information
o (2) letter 2  solicitation/once you have left and arrived at new firm when you
no longer owe fiduciary duty to partnership

F) Ownership Interests and Transferability


 1) Partnership property
o UPA § 25
 (1) A partner is co-owner with his partners of specific partnership property holding as a tenant
in partnership
 Aggregate view
 Net result  partnership, rather than partners, is effectively treated as owner of
partnership property
 (2) a partner may not assign his interest (can’t be used as a loan) in partnership property; a
partners right in partnership property is not subject to attachment (cant be used as remedy for
judgement creditor) or execution
 (a) on a claim against the partner
 (b) on the death of a partner, his right in partnership property bests in the surviving
partners and a partners right in partnership property is not subject to dower, curtesy or
allowance to widows, heirs or next of kin
o Partner has no rights to possess property for non-partnership purposes
o Partner may not assign his interest in partnership property
o Partners right in partnership property is not subject to attachment or
execution on a claim against the partner
o RUPA § 203
 Property acquired by the partnership is property of the partnership and not of the partners
individually
 Entity view
o RUPA § 501
 A partner is not a co-owner in partnership property and has no interest in partnership property
which can be transferred, either voluntarily or involuntarily
 2) Transferring a partnership interest
o UPA § 26: Nature of Partners interest in the partnership
 A partners interest in the partners is his share of the profits and surplus and the same is
personal property
o *RUPA § 101(9) Partnership Interest
 Partnership interest is all of a partners interest in the partnership including the partners
transferable interest and all management and other rights
 (1) financial right
 (2) management right

o RUPA § 502
 The only transferable interest of a partner in the partnership is the partners share of the profits
and losses of the partnership and the partners right to receive distributions. The interest is
personal property
 **Of the bundle of rights, may only transfer financial rights. This is your personal
property. Can transfer without the consent of other partners.
o *Pick your partner rule would be violated by a transfer of management rights.
o Default Rule: “Pick your Partner “Rule
 UPA § 18: Rules determining rights and duties of partners
 The rights and duties of partners in relation to the partnership shall be determined,
subject to any agreement between them, by following the rules
o (g) no person can become a member of a partnership without the consent of
ALL the members
 RUPA § 401: Partners rights and duties
 (i) a person may become a partner only with the consent of all the partners
 Policy behind rule
 A partners wrong-doing while conducting partnership business can create a partnership
obligation
 Partners have unlimited personal liability for partnership obligations. Thus, because
partners actions can create personal liability for other partners  default rule sensible
 Issue: Can a partner transfer only his financial interest (profits and distributions) but not full
partnership interest?
 Yes – can transfer “transferrable interest” i.e. financial rights aspect of partnership
interest
 Can be altered by agreement, this is just a default rule.

o Rapaoport v. 55 Perry (transferring partnership interest and claiming transferees are partners)
 Facts
 Simon, Genia, and Ury Rapoport (the Rapoports) (plaintiffs) joined with Morton, Jerome
and Burton Parnes (the Parnes) (defendants) to form a partnership. Paragraph 12 of the
partnership agreement states that partners do not have authority to assign or sell
partnership property, or to enter into an agreement that grants another person an
interest in the firm without the written consent of a majority of the partners.
Immediate family members of partners, however, could be granted an interest without
consent. Paragraph 15 provides that when a partner dies, her heir steps into her shoes
and is granted the same rights and obligations that the partner had. Without
consulting other partners, Simon and Genia Rapoport assigned 10 percent of their
partnership share to their two adult children and attempted to make them partners.
The Parnes objected. The Rapoports sued the Parnes seeking a declaratory judgment
that the partnership agreement permitted them to unilaterally add their adult children
as full partners.
 Issue
 Does the right to assign a partnership interest without the consent of the other
partners include the right to admit new members to the partnership?
 Rule
 When a partnership agreement does not address a specific issue, the matter is
governed by the Partnership Law (New York’s version of the Uniform Partnership Act).
 The Partnership Law distinguishes between an assignee and a new partner.
o An assignee is entitled to a share of the enterprise’s profits but lacks the right
to share in the management or control of the business.
 Unless otherwise agreed to, a partner may assign her interest to a third party without
consulting the other partners, but admitting a new partner with management rights
requires unanimous consent
 Holding
 In this case, the partnership agreement alters the consent required for assignment but
not for the admission of new partners. Paragraph 12 restricts a partner’s ability to
assign her interest. It states that a partner may unilaterally assign her interest only to
immediate family members. That paragraph does not grant the authority to add one’s
adult children as full partners. The inclusion of paragraph 15 shows that the drafters of
the partnership agreement recognized the distinction between assigning an interest and
granting full rights. Thus, the Rapoports may assign their interests to their children, but
they may not admit their children to the partnership without unanimous consent of all
partners. The ruling of the trial court is reversed and summary judgment is granted to
the Parnes.
 Dissent:
 Agreement is ambiguous, and should go to trial on the intent of the parties. Partnership
agreement is a contract and subject to interpretation, if drafted in an ambiguous way.
o Charging Order:
 HYPO
 Assume a partner has an individual creditor who obtains a judgment against the partner
that has nothing to do with the partnership
 Judgement creditor cannot execute against the partnership assets because the partner
does not have any ownership interest in P’s assets/that is partnership property
 Allowing the creditor to seize the judgement partners entire partnership interest could
violate the pick your partner rule
 To accommodate these interest  developed ‘charging order’ that court can impose
against a debtor partner upon the requirement of that partners creditor
o Creditors ask for this in cases where partner has no assets/creditor wants to go
after partnerships assets
 **Charging order  lien on the partners transferrable interests
o Difficult to collect even if you have the right.
 Issue:
 How does judgment creditor obtain this remedy and how will it effective work?
 RUPA § 504: Partners Transferable Interest Subject to Charging order
 (a) on application by a judgement creditor of a partner…a court having jurisdiction may
charge the transferable interest of the judgment debtor to satisfy the judgment
 (b) a charging order constitutes a lien on the judgement debtors transferable interest in
the partnership. The court may order a foreclosure of the interest subject to the
charging order at any time. The purchaser at the foreclosure sale as the rights of a
transferee
 (e) this section provides the exclusive remedy by which a judgment creditor of a
partner…may satisfy a judgment out of the judgement debtors transferable interests in
the partnership
o Diverts the debtors and partners share of profits to the judgment creditor and
creditor could eventually foreclose on the interest and attempt to dissolve the
partnership.
 Issue
 How does the partner avoid paying the creditor?
o Instead of declaring distributions at the end of the year based on surplus, the
partners could pay themselves w2 salaries and not declare any distributions
what so ever and creditor would not have access to W2 salaries.
 Only way you can get at a debtor’s partnership interest.
 Do they become partner via charging order?
o No. Merely obtains financial interest.
G) Dissociation and Dissolution .
 1) Dissolution under UPA
o UPA 3 Phases: (1) Dissolution  (2) Winding up  (3) Termination
 (1) UPA § 29: Dissolution
 The dissolution of a partnership is the change in the relation of the partners cause by an
a partner ceasing to be associated in the carry on as distinguished from winding up of
the business (when partner leaves the partnership by agreement, death, or otherwise)
o UPA takes aggregate view of partnership  as such, whenever partner leaves
or new partner joins  dissolution occurs.
o This does NOT mean partnership ends instantaneously
 (2) Winding up
 The process of ending the partnership business; typically involves the sale of the
partnership’s assets, the repayment of partnership creditors, and the distribution of
any remaining proceeds to the partners based on a setting of profits and losses
o In some cases  dissolution causes winding up of partnership
o In other cases  it does not; the change in composition of partners ≠ always
mean the business of the partners must cease and wind up
 (3) Termination
 The moment in time when the winding up of the partnership affairs is completed – it
marks the end of the partners as a going concern
o UPA, if rightful: Assets divided up 50/50 unless:
 Assets – liabilities = equity. (creditors paid the difference).
o Elements Partner Dissolution:
 **At will  a partnership where the partners have not agreed to remain partners until the
expiration of a definition term of the completion of a particular undertaking (“common hope”)
that partnership earnings would pay for necessary expenses ≠ indicate a definite term or
particular undertaking)
 Big Picture Rule: if a partnership is “at-will”, the partner can simply dissolve the
partnership by express will. That dissolution will be considered “rightful” and the
partner will have the power to compel winding up of the partnership business (which
typically mean the partner will receive his share of the sale of the partnership assets).
o Rightful: At will / Express will means you divide up assets.
 Alternatively, the partner can use his right to wind up as leverage to motivate a buyout
of his interest by agreement of the remaining partners who prefer to continue the
business (UPA § 31(1)(b), 38(1)).
 Term  a partnership where the partners have agreed, explicitly or implicitly, to remain
partners for a definite term of until the completion of a particular undertaking
 Big Picture Rule: If a partnership is for a “term”, the partner can still dissolve by express
will, although that wrongful dissolution has some negative consequences. The
partnership will either be wound up (in some cases the wrongfully dissolving partner
will typically receive his share of the sale of the partnership assets), or the remaining
parties will purchase the wrongfully dissolving partners interest in the business
(exclusive of goodwill and offset by any damages) (UPA §31(2), 38(2))
 Note: when a partner advances a sum of money with the understanding that the
amount contributed was to be loan to partnership and repaid as soon as feasible from
profits of loan, the partnership is for the term reasonably required to repay the loan
o Partners may impliedly agree to continue in business until certain sum of
money is earned or, 1 or more partners recoup their investment, or until certain
debts are paid, or until certain property could be disposed of in favorable terms
 implied agreement found support in evidence
 Rightful  when a partner decides to withdraw from an “at-will” partnership
 Wrongful  when a partner decides to withdraw from a “term” partnership before the end of
the term or the completion of the particular undertaking

o UPA
 Regardless of whether a partnership is at will or for a term, a partner can always exit the
partnership and cash out the value of his interest
 but even though ever partner has the power doesn’t necessary mean that he has the right to
dissolve the partnership
 partner can dissolve by express will (power) but such an act of dissolution may not be rightful
(right) as it depends on large part on the at will or term characterization of the partnership
o UPA § 31: Causes of Dissolution
 Dissolution is caused
Rightful  (1) without violation of the agreement between partners
o (a) By the termination of the definite term of particular undertaking specified in
the agreement
o (b) by the express will of any partner when no definite term or particular
undertaking is specified
 Contested area of law  determined on case by case basis
 This can be difficult analysis because partnership can be formed orally /
thus, sorting out intent of parties (term or at will) has cause significant
amount of litigation
o (c) by the express will of all the partners who have not assigned their interest or
suffered them to be charged for their separate debts, either before or after the
termination of any specified term of particular undertaking
o (d) by the expulsion of any partner from the business bona fide in accordance
with such power conferred by the agreement between partners
 Contemplates that a partnership agreement will include an actual right
to expel partners for reasons set forth in the agreement; when such a
right is present, UPA § 31 provides by default for continuation of the
business with a cash payment to expelled partner less damages
Wrongful  (2) in contravention of the agreement between partners, where the circumstances do
not permit a dissolution under any other provision of this section, by the express will of
any partner at the time
o UPA § 40: Rules for Distributions
 **In settling accounts between partners after dissolution, the following rules shall be observed
 (1) non partner creditors are paid
 (2) partner creditors are paid
 (3) partner capital contributions are returned
 (4) surplus split between the partners in proportion to their profit of shares
 If profits from the partnership assets are insufficient to pay the partnership liabilities 
partners must contribute in proportion to their cost shares
 If dissolving is rightful and in the absence of an agreement to the contrary  UPA provides each
partner, with the right to compel the winding up of the business through the sale of the assets
and the surplus proceeds distributed to each partner in cash as opposed to in kind
 **Partner can use threat of a wind up to get paid / buy out. (Creditors > Owners, always).
 Way around forced liquidation is through drafting in partnership agreement
 “upon any withdraw of a partner, the business of the partnership will not be liquidated;
instead, the remaining partners will continue the business and will buy out the interests
of the withdrawing partner on some predetermined basis”
o UPA § 38: Rights of Partner to Application of Partnership Property (Default Rule)
 [absent an agreement]
 (1) when dissolution is caused in any way, except in contravention of the partnership
agreement, each partner, as against his co-partners and all persons claiming through them in
respect of their interests in the partnership, unless otherwise agreed, may have the partnership
property applied to discharge its liabilities, and the surplus applied to pay in cash the net
amount towing to the respective partners
 When no “bad guys” involved  at will liquidation
 Language appears to require the sale of partnership assets so that the liabilities can be
paid and the remainder distributed with real cash
 Gives partner right to compel liquidation through a typical sale with cash proceeds
o Collins v. Lewis (UPA governed)
 Facts: Collins (plaintiff) and Lewis (defendant) each owned 50% interest in a partnership
formed to own and operate a cafeteria. Their partnership agreement provided that Collins
would provide funds to build and open the cafeteria, while Lewis would oversee the
construction of the cafeteria and manage it once it opened for business. Lewis guaranteed
repayment to Collins at a minimum rate of $30,000 plus interest the first year, and $60,000
plus interest annually thereafter. Collins initially advanced $300,000, based on Lewis’s initial
estimate of the cost to build and open the cafeteria. After a substantial delay in completing the
cafeteria and increases in expenses, the initial cost had increased to $600,000. Collins
expressed his displeasure about the cost increase, but he advanced the entire amount. Soon
after the cafeteria opened, Collins discovered that the expenses far exceeded the receipts.
Collins demanded that Lewis immediately make the cafeteria profitable, or he would cut off
additional funding. Lewis accused Collins of unauthorized interference in the management of
the business, while Collins charged that Lewis had mismanaged the building and opening of the
cafeteria. Collins also made serious threats during the first year of the cafeteria’s operation,
forcing Lewis to lose his interest in the business. Lewis tried, but failed, to find financing to buy
out Collins. Collins subsequently filed suit, seeking dissolution of the partnership. The trial court
denied dissolution, based on the jury’s findings that: (1) there was not a reasonable expectation
of profit if Lewis continued managing the cafeteria; (2) but for Collins’s conduct which decreased
the earnings during the first year, there would be a reasonable expectation of profit; and (3)
Lewis was competent to manage the cafeteria.

 Issue: Can a partner legally force dissolution of a partnership when but for that partner’s
actions, the other partner could have performed his or her required duties?
 Rule
 In an action in equity, a court will force the dissolution of a partnership when, for
example, a partner has breached his or her fiduciary duty to the partnership or the
other partners.
 When a partner has performed his or her obligations and has not otherwise harmed
the partnership, another partner cannot force dissolution through the courts.
o Term P dissolution  terminating is breach of partnership agreement and
impacts partners payout and whether the partnership can be dissolved
o At will dissolution  partners could dissolve without liability
 Holding
 Collins asks this court to dissolve his partnership with Lewis, complaining that he should
not be forced to continue in a partnership which the jury found has no reasonable
expectation of profit. Collins’s complaint ignores the other jury findings that Lewis was
competent to manage the cafeteria, and that but for Collins’s actions there would
have been a reasonable expectation of profit from that enterprise. Lewis’s duties under
the partnership agreement were to oversee the construction, manage the cafeteria,
guaranteeing repayment to Collins at the stipulated minimum rate. The jury found that
Lewis could have performed his duties had not Collins interfered the way he did. Under
these circumstances, the court finds that Collins has no right to have this court force
dissolution of the partnership. Collins has the power to dissolve the partnership
without the intervention of the courts, but he may thereby be liable for breach of the
partnership agreement. The decision of the trial court is affirmed.
o Notes
 Why is Collins seeking to dissolve the partnership?
 (1) Trying to get leverage against Lewis in effort co compel Lewis to operate the
cafeteria as Collins wished
 (2) Collins desired dissolution because if the partnership business were sold as part of
winding up procedure, he might get his $600k worth of loans and capital back; he may
be losing faith in the business and wants to be paid right now before things go further
south
 Would Collins’ rights have been different had this been at will
 Yes, because he could dissolve under §31(1)(b)
 He could cause general partnership to wind up, split surplus after paying off liabilities
and would not have to worry about breach of contract claim

o Dreifuerst v. Dreifeurst (how a GP splits up profits and liabilities when a partner dissolves the GP/ in kind
distribution)
 Facts
 The Dreifuerst brothers formed a partnership for the purpose of operating two feed
mills. The partnership is not governed by a written partnership agreement. All but one
of the brothers (plaintiffs) elected to dissolve the partnership, and brought an action
for dissolution and wind-up of the partnership after serving the remaining brother
(defendant) with notice. The defendant asked the court to permit a sale pursuant to
Uniform Partnership Act (UPA) § 38(1), at which the partners could bid on the property,
thus allowing the plaintiffs to continue running the business and the defendant’s
partnership equity to be satisfied in cash. The trial court denied the defendant’s
request and instead divided the partnership assets in kind, granting the physical assets
of one of the feed mills to the plaintiffs, and the physical assets of the other feed mill to
the defendant. Defendant appeals the trial court’s in-kind division of the partnership’s
assets.
 Issue
 Can a partner force a sale of partnership assets upon dissolution and wind-up of the
partnership, in the absence of a written agreement to the contrary?
 Rule
 Yes. In the absence of a written agreement to the contrary, a partner may force a sale
of partnership assets upon dissolution and wind-up of the partnership.
 Under the UPA §§ 29 and 30, when a partner ceases to be associated in the carrying on
of the partnership’s business, the partnership is dissolved, but continues on until the
winding-up of the partnership is complete.
 UPA § 38(1) states that, unless otherwise agreed, upon lawful dissolution of a
partnership any partner may have partnership property applied to any debt owed to a
creditor, and may be paid any surplus amount owed to him in cash.
 A distribution-in-kind  is a payment made in the form of securities or other property
rather than in cash. ... It can also refer to the transfer of an asset to a beneficiary over
the option of liquidating the position and transferring the cash.
 A partnership is at will if there is no partnership agreement
 Holding
 Because the partnership here was a partnership at will, the plaintiffs could rightfully
dissolve the partnership without the defendant’s consent. The parties also agree that
the defendant has not violated a partnership agreement, as there was no such
agreement. The partnership here was therefore rightfully dissolved.
 Upon lawful dissolution, each partner has the right to have the partnership’s assets
liquidated and his share paid to him in cash. A court may order in-kind distribution
only if the partnership agreement permits it, or if all the partners have agreed to it.
There was no such agreement here.
 In Rinke v. Rinke, 48 N.W.2d 201 (1951), the court permitted in-kind distribution absent
all of the partners’ agreement, but in that case there were no creditors to be paid; no
one other than the partners would have been interested in the business’s assets; and
the in-kind distribution was fair to all of the partners.
 In contrast, in this case there was no showing that there are no creditors. Such a
showing is important because in-kind distributions could affect creditors’ rights, since
the partnership’s assets as a whole may be worth more to creditors than the assets
after being divided up among the partners. Nor was there any showing that no one
other than the partners would be interested in the business’s assets. If others are
interested in the assets, a sale is the best method for establishing the market value of
the assets, thus ensuring that each partner will receive his fair share of those assets,
whereas a sale only between partners would not involve any competition and would not
establish a fair value for the assets. Because the facts of Rinke do not apply to this case,
an in-kind distribution here was not proper. In addition, however, insofar
as Rinke creates an exception to the rule of UPA § 38 that a partner is entitled to
payment in cash of his partnership assets upon dissolution and wind-up, this court
declines to adopt it. UPA § 38 does not permit in-kind distribution under any
circumstances, unless all of the partners agree. The trial court’s judgment is reversed,
and the matter is remanded for further proceedings.
 Notes
 What each party wants
o Plaintiff/partners  want in kind distribution of assets
o Defendant  wanted sale of assets and wanted cash (regular liquidation)
 Why is defendant brother opposing in kind distribution?
o (1) economic matter  defendant may think that assets of partnership as a
whole may be worth more than if assets are divided up and being sold peacemil
which is often times very true
o (2) defendant may have though that all of the partners would be better off if the
partnership assets were sold together as an operating going concern business
o (3) objection to in kind distribution because sale is better way to establish the
true fair market value of the partnership assets because bidding competition it
creates and market would create fair market value
o (4) defendant may believe he was awarded the asset of lower value / got
screwed
 Why do plaintiffs want in kind distribution over liquidation
o (1) plaintiffs want to stay in the business without the defendant and plaintiff
may not have had the financing or ability to obtain financing to purchase entire
business at liquidation sale
 Do plaintiffs have right to dissolve and seek winding up?
o Yes – the partnership was at will and under § 31, a partner can rightfully dissolve
an at will partnership by express will
 Continuation agreement  an agreement among the partners that, in the event of
dissolution, the business of the partnership can be continued without the necessity of
liquidation

o Bohatch v. Butler & Binion


 Facts
 Colette Bohatch (plaintiff) was a partner in the Washington, D.C. office of the law firm
Butler & Binion. The office had two other attorneys, McDonald and Powers, both
partners. Almost all of the D.C. office’s work was done for Pennzoil. In 1990, based on
her review of billing reports and time records, Bohatch began to suspect that
McDonald was overbilling Pennzoil. She reported her concerns to Powers, and to other
partners and members of Butler & Binion’s management committee. The management
committee investigated the allegations. Pennzoil’s in-house counsel told committee
members that Pennzoil believed McDonald’s bills to be reasonable. The management
committee determined that there was no basis for Bohatch’s allegation that McDonald
was overbilling. In January 1991 Butler & Binion denied Bohatch her year-end
partnership distribution, and reduced her tentative distribution share for 1991 to zero.
It continued to pay Bohatch her monthly draw until June 1991. On October 21, 1991 the
firm voted to expel Bohatch from the partnership. Bohatch brought claims against
Butler & Binion and its partners (the firm) (defendants) for breach of fiduciary duty and
breach of contract. The jury found that the firm breached the partnership agreement
and its fiduciary duty, and awarded damages. The court of appeals found that the firm’s
only duty to Bohatch was not to expel her in bad faith, that is, for self-gain. Finding no
evidence that the firm had expelled her for self-gain, the court of appeals found that
Bohatch could not recover on the breach of fiduciary duty claim. The court of appeals
also found that the firm breached the partnership agreement by reducing Bohatch’s
tentative distribution for 1991 to zero without notice, and for terminating her monthly
draw three months before she left the firm.
 Issue
 Does a partnership have a duty not to expel a partner for reporting suspected
overbilling by another partner?
 whether the fiduciary relationship between partners creates an exception to the at-will
nature of partnerships, specifically, whether a partnership has a duty not to expel a
partner for reporting another partner’s overbilling
 Rule
 A partnership does not owe a duty to a partner not to expel him or her for reporting
suspected overbilling by another partner.
 A partnership relationship is fiduciary, and imposes a duty of loyalty, good faith,
fairness, and honesty between partners in matters concerning the partnership.
 However, partnerships are by nature at-will, that is, partners do not have an
obligation to remain partners.
 Other courts have held that a partnership may expel a partner for purely business
reasons, to protect relationships within the firm and with clients, or to resolve a
fundamental schism in the partnership.

 Holding
 This court rejects the argument of Bohatch and several legal scholars that public policy
imposes a limited duty on partnerships to retain a whistleblower partner in order to
promote compliance with the rules of professional conduct. A partnership only exists
because its partners choose to place their personal confidence and trust in one another.
A partner’s accusation that another partner is overbilling could profoundly affect this
personal confidence and trust and lead to an irreparable schism between partners.
Thus, a partner may be expelled for accusing another partner of overbilling without
subjecting the partnership to a tort claim. This holding does not, however, eliminate
the ethical duties of lawyers to report suspected overbilling.
 In this case, the firm did not owe a duty to Bohatch not to expel her for reporting
another partner’s overbilling. As to Bohatch’s breach of contract claim, the court of
appeals did not err in finding the firm liable for breaching the partnership agreement.
The agreement guarantees a monthly draw at a set rate, and permits the firm to
reduce Bohatch’s tentative distribution only after giving her proper notice. The firm
does not dispute that it failed to give Bohatch proper notice that it was reducing her
tentative distribution. The court of appeals’ judgment is affirmed.
 Notes
 Could plaintiff have claimed that her expulsion constituted a wrongful dissolution
under UPA 31(1)(d)
o Dissolution caused by expulsion not wrongful under UPA if it is bona fide i.e.
good faith (not for self gain or for some other illegal reason)
 RUPA?
o RUPA §601(3)/(4)  does not include bona fide phrase, presumably because §
404(d) requires partners to exercise their rights including right to expel in good
faith

 2) Dissociation and Dissolution under RUPA


o RUPA Differences
 First
 Under UPA ‘dissolution’ effectively means a change in the aggregate of the partners
 Thus, under UPA, any partner who leaves or joins the partnership causes “dissolution”
 Under RUPA, partners may leave the partnership without necessarily causing
“dissolution”
 As such: because RUPA conceives of the partnership as an entity, the entity can
continue even if the composition of its members changes
 Second
 RUPA introduces the new term due to highlighted change above
 “dissociation” effectively means “separation” or “withdrawal”
o “cessation of partners status”
o Dissociation  commencement of winding up process
 A partner that has dissociated, in other words, has separated or withdrawn from the
partnership
 Partners dissociation often leads merely to a buyout while leaving the partnership entity
and his business fully intact
o RUPA “Winding Up”
 Selling of the partnership assets / paying debts / and distribution net balance, if any to the
partners in cash according to their interests
 When winding up of business is completed, partnership is terminated
o Events that result in dissociation of partners
 RUPA § 601
o Death, withdrawal, bankruptcy, or expulsion
 RUPA § 602
o Rightful  when a partner decides to withdraw from an at will partnership
o Wrongful  when a partner decides to withdraw form a term partnership
before the end of the term or before completion of the particular undertaking
o When partner dissociates from partnership [what happens]
 2 statutory avenues implicated
 (1) Partnership is dissolved and partnership is wound up (liquidation)
 (2) partnership continued in existence with the dissociated partner becoming entitled to
a buyout of his partnership interest
 How do we know which avenue is implicated?
 Focus on the event of dissolution and whether it is rightful or wrongful
o At will  any partner who dissociates by his express will may compel
dissolution and winding up
o Term  if one partner dissociates wrongfully, dissolution and winding up of
the partnership are required only if within 90 days after dissociation, ½ of the
remaining partners agree to wind up the partnership
 What happens when partners dissolution does not result in dissociation and winding up
 If this is the case, partner is entitled to a buyout of his partnership interests
 If dissociation is wrongful, any damages will be offset against the buyout price
 Buyout payment may be deferred until conclusion of the term or undertaking
o Saint Alphonsus v. MRI (“if” in partnership agreement / wants to rightfully dissociate and receive
economic interest in the partnership via a buyout)
 Facts
 Saint Alphonsus Diversified Care, Inc. (St. Alphonsus) (plaintiff) and three other
organizations formed a general partnership named MRI Associates, LLP (MRIA)
(defendant). Under the written partnership agreement, any partner could withdraw
from the partnership at any time if any of four specified conditions or events were
present or occurred. St. Alphonsus dissociated from MRIA and then sued MRIA for a
determination of the amount it was entitled to receive for its interest in MRIA. MRIA
counterclaimed for wrongful dissociation
 Issue
 Is a dissociation wrongful if it is in breach of an implied provision of the partnership
agreement?
 Rule
 Under § 602(b)(1) of RUPA, a dissociation is wrongful only if it breaches an express,
not implied, provision of the partnership agreement.
 “Express” means something that is explicit and not left to inference.
o This must be a clear breach from the face of the agreement
 Holding
 Here, the partnership agreement stated that any partner could withdraw from the
partnership at any time if certain events occurred. The word “if” could be interpreted,
as the district court interpreted it, to mean that a partner could withdraw “on the
condition that” one of the listed events occurred and that a partner could not withdraw
if the listed events did not occur. However, this provision could also be interpreted to
mean that a partner was allowed withdraw from the partnership “in the event that”
any of the listed events occur.
 The agreement does not state that a partner “shall not” withdraw except under the
specified circumstances or that a partner may “only” withdraw under those
circumstances. While it may be possible to infer that the listed events were the
exclusive circumstances under which a partner could rightfully withdraw, this is not an
express provision.
o Could be non-exhaustive; or
o Could be exhaustive
 Notes
 Why does determination of wrongful dissociation matter in this case?
o Wrongful dissociation = damages against breaching party
 Courts
o Some follow strictly  holds that the statute requires the partnerships assets to be sold so that each
partner can receive his share in cash
o Some follow equities of the case 
 Reasoning: this could cause a fire sale where assets are being sold at less than fair market
value / in a non-distressed situation, the forced liquidation would seem to be detrimental to all
partners
 Could also be the case where you have one very wealthy partner who wants to squeeze other
partners out so that the wealthy partner dissolves, forces a winding up and sale and then buys
all the partnership assets at a cheap liquidation sale
o How can we avoid these problems?
 In partnership agreement, agree you don’t want to force liquidation and you want to wind up
the partnership in a different manner (i.e. in kind distribution or buyout)
 Big Picture
o GP form of business is characterized by easy exit
 RUPA does NOT change this notion  partner can always get out of a partnership
 At will  partner can dissolve by express will then wind up the business and walk away
with money
 Term  partner cannot compel winding up (left over partners could decide to wind up)
but dissociation will entitle him to a buyout / payment of buyout price may not occur
until end of partnership term and the partners wrongfully dissociation may subject him
to damages leaving him with nothing when it comes to a buyout

Chapter 5 – Formation of A Closely Held Corporation – Module 3 / Lecture 1


 Characteristics of closely held corporation
o Few shareholders
o Stock not publicly traded
o Shareholders usually engaged in management
o Often founded by friends/family whose expectation is the same as those entering into partnerships
o No market for stocks, thus no liquidation
o Hard to get out of
o More exposure for liability for individual owners of the corporations debt because of concept of
piercing the corporate veil
 Characteristics of publicly held corporation
o Investors are owners but only in a tangible sense / no effective voice in management / do not expect to
work for company
o Investor view investment as liquid  they hope to make money by selling stock on the public market for
more than the spent to buy and perhaps receiving dividends in the interim
o No exposure for individual liability in public corporation because no piercing veil in public corporations
 Characteristics both corporations share
o (1) Statutes requirement (Corporation can be formed only by satisfying the requirements set forth by
state statues)
 (1) incorporations filed with the appropriate state officer (usually secretary of state) 
document called “articles of incorporation”
 (2) corporation must appoint registered agent on whom process may be served if corporation is
sued
 (3) pay fee to state
o (2) formalities
 Operating a business as a corporation imposes corporate formalities including
 Filing annual reports
 Holding meetings of shareholders and directors
 Paying taxes
 Formalities are disadvantage over some business structures that can be formed and maintained
simply by selling X
o (3) corporation as an entity
 Corporation is considered an entity legally distinct from the persons who own and run it
o (4) Liability
 Because has true entity status, corporation itself is liable for its contracts, torts and debts
 People who manage/own are generally not liable for what the corporation does
o (5) Shareholders/Stockholders
 They are corporation “owners
 Corporation sells shares of stock (units of ownership)
 Shareholder ≠ manage  they elect directors to manage
 Shareholders  own
 Directors  manage
 Shareholders ≠ personally liable
 Managers may have personal liability depending on the actions that they take
o (6) Transferability
 Much easier to transfer stock if it is publicly traded because you can do so online
 No analysis of ‘term’ or ‘at will’
 Exit strategy for publicly traded  simply sell your stock

A) Where to Incorporate
 Formation
o (1) choose state which to incorporate
 Laws of that state will govern internal affairs
 Our focus  MBCA
 Delaware  offers great flexibility
o For small corporation the cost of incorporation there is not worth the benefits; for a local company, a
local state may make more sense; also would not want to subject yourself to a suit in Delaware

B) How to incorporate
 Terms
o Articles of incorporation
 Most basic document
 Governed by statute and must be filed with the state
 Corporate entity cannot exist without it
o Bylaws
 Generally describe how the corporation is going to be run
 Not generally required by statute, though they are critical
o Minutes
 Corporate requirement to have annual meetings
 “minutes” are the records of what happens during those meetings
 Should generally be minutes for regular meetings, and there should be initial set of minutes for
first meeting of board, upon formation of corporation
 MBCA: General Requirements to form corporation (have to look to particular state to see what is required)
o Articles must be filed with secretary of state §1.20(b) & (i)
o Articles must be executed and signed §1.20(f) & (g)
o articles must be accompanied with any payment required §2.02(a)
o Articles must set forth a corporate name for the corporation §2.02(a)
o Articles must set forth the number of shares the corporation is authorized to issue §2.02(a)
o Articles must set forth the street address of the corporations initial registered office and the name of its
initial registered agent at that office §2.20(a); and
o articles must set forth the name and address of each incorporator §2.02(a)
 MBCA § 4.01 Corporate Name
o (a) corporate name
 (1) must contain the word “corporation” “incorporated “company” or “limited” or the
abbreviation “corp” “inc.” “co” or “LTD” or words or abbreviations of like import in any language
o (b) except as authorized by subsection (c) and (d), a corporate name must be distinguishable upon the
records of the secretary of state from
 (1) the corporate name of a corporation incorporated or authorized to transact business in this
state
 (2) a corporate name reserved or registered under section 4.02 or 4.03
 Cant steal other names  misleading
 Purposes and Powers
o Purpose  refers to the reason that a corporation is formed and the type of business it will transact
 Today  almost all statutes permit formation of a corporation to engage in any lawful business
without being specific as to the scope of the business
o Powers  refers to the things a corporation may do to help accomplish its purpose
 Corporation has ‘power’  to sue and be sued, buy and sell property, borrow money, etc
 19th century  courts imposed restrictions on corporate powers
 Today  restrictions have been rejected (old restrictions still creep into business realm)
o MBCA § 3.01(a): Purposes (bad idea to restrict purpose to something narrow)
 Every corporation incorporated under this act has the purpose of engaging in any lawful
business unless a more limited purpose is set forth in the articles of incorporation
 Note:
o MBCA § 3.02 General Powers
 Unless its articles of incorporation otherwise provide, every corporation has a perpetual
duration and succession in its corporate name and has the same powers as an individual to do
all the things necessary or convenient to carry out its business and affairs
 Board of Directors
o MBCA § 8.01 Requirement for and duties of board or directors
 (a) except as provided in section 7.32, each corporation must have a board of directors
o MBCA § 8.03
 (a) a board of directors must consist of one or more individuals, with the number specified in or
fixed in accordance with the articles of incorporation or bylaws
 Shareholders
o Shareholders and capital requirement (how much money they have to put into the business)  no
requirement in statute

C) The Decline of the doctrine of ultra vires


 Ultra vires unauthorized, beyond the scope of power allowed or granted
o Corporation cannot act beyond the scope of its purpose and power.
o ** Do not limit purpose. Simply state purpose is to “engage in any lawful business.”
 Issue: how does this arise?
o Narrow purpose clause in articles of incorporation
o Because corporation lacks power under certain state statutes to engage in conduct
 Issue: What happens when corporation acts ultra vires / beyond its powers?
o If put limited purpose and engage in business outside of that purpose (i.e. oil and gas  clothing), if
corporation contracts with clothing store
 Old view  contract is void / corporation has no power to enter into business contract
 Transition  case law moved to voidable contract giving either party the option to void the
agreement if neither side had performed, if one party performed, no voiding would be allowed
(unjust enrichment)
 Modern view  ultra vires contracts are valid and enforceable and generally cannot be
challenged
 MBCA § 3.04 Ultra Vires
o (a) except as provided in subsection (b), the validity of corporation action may not be challenged on the
ground that the corporation lacks or lacked power to act
o (b) a corporations power to act may be challenged
 (1) in a proceeding by a shareholder as against the corporation to enjoin the act
 “I invested in your company to do X and you’re doing Y”
 (2) in a proceeding by the corporation, directly, derivatively, or through a receiver, trustee, or
other legal representative, against an incumbent or former director, officer, employee, or agent
of the corporation; or
 If corporation loses money on Y, corporation, through shareholders can sue directors and
officers
 (3) in a proceeding by the attorney general under section 14.03
 Opening liquor store without license
o (c) in a shareholder proceeding , to enjoin an unauthorized corporate act, the court may enjoin or set
aside the act, if equitable and if all affected person are parties to the proceeding, and may award
damages for loss (other than anticipated profits) suffered by the corporation or another party because
of enjoining the unauthorized act
 711 Kings v. FIM Marine – ultra vires challenges limited to 3 situations  cannot be invoked by employee
o Facts
 711 Kings Highway Corp. (711 Corp.) (plaintiff) leased property to F.I.M.’s Marine Repair Serv.,
Inc. (Marine Repair) (defendant) through a written lease agreement. The lease provides that
the property is to be used as a movie theatre. Marine Repair’s certificate of incorporation
provides that its purposes are restricted to marine activities such as marine repairs. 711 Corp.
brought a complaint asking for the lease to be declared invalid or rescinded, as the operation of
a movie theatre falls outside the scope of powers conferred to Marine Repair under its
incorporating document. According to 711 Corp., the lease has not been performed, and
remains merely executory. Marine Repair moves for dismissal of the complaint for failure to
state a claim.
o Issue
 May a plaintiff bring a cause of action to invalidate a corporation’s act based on the fact that the
corporation lacked the power to do that act, where state law prohibits the use of the ultra
vires doctrine?
o Rule
 No.
 Where state law prohibits the use of the  ultra vires  doctrine, a plaintiff may not bring a cause
of action to invalidate a corporation’s act based on the fact that the corporation lacked the
power to do that act.
 Under the ultra vires doctrine  a plaintiff may challenge a corporation’s act on the ground
that the act is beyond the scope of powers granted to the corporation under its incorporating
document.
o Holding
 However, section 203 of the New York Business Corporation Law  provides that a
corporation’s act cannot be declared invalid based on the fact that it lacked the power to do
such an act, except in an action brought by a shareholder to enjoin a corporate act, by a
corporation against a former director, or by the Attorney General in a special proceeding.
 None of those exceptions apply in this case. Under section 203, 711 Corp.’s argument that
Marine Repair could not act outside the scope of powers conferred to it in its incorporating
document is without merit. 711 Corp.’s argument that section 203 only applies when ultra
vires is raised as a defense also fails. To the contrary, the exceptions in section 203
permitting ultra vires to be used in support of certain causes of action show that, outside of
these exceptions, ultra vires cannot be used either in support of or as a defense to a cause of
action. Finally, this court reject’s 711 Corp.’s argument that the ultra vires doctrine applies to
executory contracts. Under section 203 ultra vires may not be invoked even if the contract is
executory. Marine Repair’s motion for dismissal is therefore granted for failure to state a claim.
 Sullivan v. Hammer
o Facts
 Occidental Petroleum Corporation (defendant) informed its stockholders that it planned to
provide millions of dollars of funding to the Armand Hammer Museum and the Armand
Hammer Foundation. Dr. Armand Hammer is Occidental’s founder and Chairman of the Board.
Stockholders (plaintiffs) brought class and derivative claims, asserting that Occidental’s
proposed actions constitute a gift and a waste of corporate assets, and that Hammer
breached his duty of loyalty by causing Occidental to take these actions for his own personal
benefit. The parties now submit a settlement agreement for the court’s approval.
o Issue
 May a corporation make a charitable gift, absent any showing of bad faith or negligence by the
directors
o Rule
 A corporation may make a charitable gift, absent any showing of bad faith or negligence by the
directors.
 whether a corporation may make a gift to a charity depends on the gift’s reasonableness.
(BJR provides “reasonableness”
 Under the business judgment rule, directors of a corporation are presumed to have acted on an
informed basis, in good faith, and with an honest belief that their actions were in the company’s
best interests.
 This presumption can be overturned only by showing that a majority of the directors
expected to receive a personal financial benefit, lacked independence, were grossly
negligent in failing to inform themselves, or that the action was so irrational that it
could not have been a reasonable exercise of the directors’ business judgment.
 The role of the court in reviewing a proposed settlement agreement is to decide whether it is
fair and reasonable under the facts and legal circumstances of the case.
 In doing so, the court must consider, among other things, the validity of the claims, and the
amount of compromise compared to the amount and collectability of the judgment.
o Holding
 In this case, it is unlikely that the stockholders would prevail on their claims, as they would
likely be barred by the business judgment rule. The business judgment rule creates a
presumption that, in making business decisions, the directors of a corporation acted on an
informed basis, in good faith, and with an honest belief that their actions were in the company’s
best interests. This presumption can be overturned only by showing that a majority of the
directors expected to receive a personal financial benefit, lacked independence, were grossly
negligent in failing to inform themselves, or that the action was so irrational that it could not
have been a reasonable exercise of the directors’ business judgment.The stockholders having
failed to make such a showing, it is highly likely that the court would find that the directors are
entitled to a presumption that their actions were valid.
 In addition, whether a corporation may make a gift to a charity depends on the gift’s
reasonableness. Here, it is clear that the Museum is a charity, and that the proposed gift
appears reasonable, making it unlikely that the stockholders would prevail on this claim. As for
the consideration to be received by the stockholders under the agreement, in the court’s
business judgment it is adequate to support the settlement. While the value of the benefits
being provided to the stockholders under the settlement, such as requiring the Museum
building to carry the Occidental name, is speculative, Occidental will receive good will from the
charitable gifts and will be able to use the Museum building to promote its business purposes.
The consideration to be received by the stockholders under the settlement is therefore
adequate when compared to the weakness of their claims. The settlement is approved.

D) Premature Commencement of business. Module 3 / Lecture 2


 1) Promoters
o Promoter (founder/organizer of enterprise) entrepreneurs who have ideas who put new businesses
together
 Doing out of self-interest but in pursuing their own interest the provide an important service
which is getting that corporation ball rolling
 Getting ball rolling  getting outsiders to invest in the company or banks or other parties to
lend to them
o Promoter Duties
 (1) Fiduciary duty
 Promoter owes significant fiduciary duties to other participants in the venture
o Exacting good faith in their intracompany activities and demanding adherence
to a high standard of honesty and frankness
o Including fully advising corporation and members and person who it was
anticipated to become members of any interests...
 (2) person who sets in motion machinery that brings about the incorporation and organization
of the corporation
 (3) brings together persons interested in the enterprise to be conducted by the corporation
 (4) aids in inducing person to become members of corporation
 (5) procuring from their membership fees to carry out purposes set forth in articles of
incorporation
o Post incorporation asking for investments and loans
 Makes representations/commitments/contracts that the corporation is responsible and has a
legal obligation for
o Pre incorporation contract liability
 General rule: [when promoter enters into pre incorporation contract] promoter will be
personally liable on his contract unless the other party agreed to look to some other person for
payment
 Can avoid by clear drafting  expressly state promoter is not liable or his is liable only
until the corporation is formed then adopts the contract
 Rule: promoter will always be assumed to be personally liable unless there is evidence that the
other party agreed to look elsewhere for payment
 Burden is on promoter and any ambiguities will work against that promoter
 Note: asking for these does not mean it becomes a corporation
o “to be formed” corporation liability
 Fact that it comes into existence ≠ make the corporation liable
 Those in charge of corporation have opportunity to accept or reject the contract, unless adopts
no liability (explicitly or implied through conduct)
 If corporation accepts benefits of contract it cannot be permitted to then shun liability
o Issue: if corporation does adopt contract, does that mean promoter is relieved of liability
 No, promoter still liable on the contract until there is a novation
 Novation  until the third party agrees to have the corporation replace the promoter under
the contract
o Liability for pre-incorporation transactions

o Stanley v. Boss
 Facts
 Stanley J. How & Associates, Inc. (How) (plaintiff) entered into a contract under which it
agreed to provide architecture services for the owner of a building. As originally
drafted, the contract stated that Boss Hotels Company Inc. would be known as the
“Owner” in the contract, and listed Boss Hotels Co. Inc. on the signature line of the
contract. When presented to Edwin Boss (defendant) for signature, he erased the
words “Boss Hotels Co. Inc.” on the signature line and replaced it with the words “By:
Edwin A. Boss, agent for a Minnesota corporation to be formed who will be the
obligor.” Boss then formed an Iowa corporation to handle the project. This corporation
sent partial payments to How, but the project was later abandoned after a substantial
amount of the architectural work had been completed. How sued Boss to collect the
remainder of its fee.
 Issue
 Is a promoter personally liable on a contract even if he believed he was acting on
behalf of a projected corporation?
 Rule
 A promoter will be personally liable on his contract, even if he believed he was acting
on behalf of a projected corporation, unless the other party agreed to take payment
from some person or fund other than the promoter.
 Under the Restatement (Second) of Agency, Section 326, Comment b, when a promoter
makes an agreement on behalf of a corporation that has not yet been formed, the
intent of the parties can be represented by one of four alternatives: (1) a revocable offer
is being made to the nonexistent corporation that will result in a contract if the
corporation is formed and accepts the offer; (2) an irrevocable offer is being made for a
limited time, which offer is kept open by the promoter’s promise to organize the
corporation and try to cause it to accept the offer; (3) the promoter is liable, but that
liability terminated if the corporation is formed and manifests its willingness to become
a party to the contract; or (4) the promoter remains liable even if the corporation
becomes a party, either primarily or as surety for the corporation’s performance.
 Holding
 The contract here does not fall under the third category, because the parties have not
argued that there would be a future novation. The only issue then is whether the
contract was an offer to the nonexistent corporation, or was an agreement that Boss
was the present obligor.
 Here, the contract was signed “Edwin A. Boss, agent for a Minnesota corporation to be
formed who will be the obligor.” This language does not indicate who the present
obligor was at the time of signing. However, while ambiguous, these words are not
enough to overcome the rule that a person signing for a nonexistent corporation is
ordinarily personally liable.
 In addition, the testimony tended to support a conclusion that the parties intended Boss
to be the present obligor. Boss is therefore liable to How unless the contract can be
construed to mean that How agreed to look solely to the new corporation for payment
and that Boss did not have any duty to How to form the corporation and give it the
opportunity to pay the liability. While Boss argues that the contract should be
construed to mean that How agreed to look solely to the new corporation for payment,
because How accepted partial payments from the new corporation, the court finds that
How did not waive its rights, and no waiver of rights was pleaded. Judgment is entered
for How to recover the remainder of its fee from Boss.
 Notes
 Rule: a promoter is personally liable on a per-incorporation contract
o Exception  if the third party who contracted with the promoter knew
corporation was not in existence at the time of the contract but nevertheless
agreed, expressly or impliedly, to look solely to the corp for performance
 Rule: a corporation, once formed, is liable on a pre incorporation contract only if it
adopts the contract
o Adoption  can be express or implied (accepting benefits of K) /makes a corp a
party to the contract from the moment of adoption
o Ratification  technically a corporation cannot ratify a promoters contract
because it requires a principal to be in existence at the time of contract
formation / instead it retroactively validates a contract from the moment the
contract was formed
 Rule: promoter liability after adoption by the corporation does not, absent novation, end
promoters liability to third parties

 2) Defective Incorporation
o De jure corporation/true corporation  one that conforms to the statute and has compiled to make it
a true corporation
o Defective incorporation
 Issue: whether an individual promoter – who believes he/she has formed a true corporation –
can be held liable when the corporation was never really formed and now a contract is
breached or a tort is committed by the defective corporation
 Whether individual promoter are personally liable
 MBCA § 2.04: Liability for pre-incorporation transactions
 All person purporting to act as or on behalf of a corporation, knowing there was no
incorporation under this act, are jointly and severally liable for all liabilities created
while so acting
o Most cases, not aware so no personal liability.
o *see also de facto corporation and corporation by estoppel
o Without protection of statue parties can look to 2 CL doctrines for protection (alternative bases for
recognizing corporate status). Defenses.
 (1) de facto corporation
 (2) corporation by estoppel
o (1) de facto corporation  one which has been defectively incorporated but is still given some effect
 (a) must be statute under which incorporation would have been permitted
 (b) must be an exercise of corporate powers – proprietors are acting as through corporation
exits
 (c) promoters made a good faith attempt to form the corporation (unaware that they didn’t
form a true corporation and came close to forming one)
 Defense. Unaware of the failure to form a de jure or true corporation.
 Came awfully close but did not form it.
o (2) corporation by estoppel  no corporation but due to something in the dealing between the parties
concerned, one or the other is precluded form denying the existence of the corporation
 Only applies in contract cases
 Typically, because third party treated the business as a corporation and relied upon corporate
assets and credit
 Defense. May be able to assert that Plaintiff is estopped from holding them personally
liable, because she treated business as a corporation and relied upon corporate assets
and credit.
o If court refuses to apply de facto corporation or corporation by estoppel, it must then determine which
of the proprietors is liable
 Multiple proprietors might have formed a partnership and will thus face liability as partners
 Courts may look to active participation in the business and hold that those who actually called
the shots in the business to bear liability
o when doctrines are recognized, defacto and estoppel have been considered alternative bases for
recognizing corporate status
 (1) extent the corporation complied with incorporation statute
 (2) extent the parties have dealt with the business as a corporation

o Robertson v. Levy
 Facts
 Robertson (plaintiff) and Levy (defendant) entered into an agreement, under which Levy
was to form Penn Ave. Record Shack, Inc. (Penn Ave.) to purchase Robertson’s
business. Levy submitted articles of incorporation for Penn Ave., but they were rejected.
Robertson assigned his lease to Levy as president of Penn Ave., and Levy began to
operate the business under the same name. Robertson executed a bill of sale disposing
of his business assets to Penn Ave. in return for a note providing for installment
payments, signed “Penn Ave. Record Shack, Inc. by Eugene M. Levy, President.” The
certificate of incorporation was later issued to Penn Ave., and one payment was made
on the note. Penn Ave. ceased doing business and is now without assets. Robertson
sued Levy for the balance due on the note.
 Issue
 If a defendant acts as a corporation before the certificate of incorporation is issued, can
he be held personally liable?
 Rule
 A defendant who acts as a corporation before the certificate of incorporation is issued
can be held personally liable.
 MBCA  a corporation only exists after the certificate of incorporation is issued, and
individuals who act as a corporation before then are jointly and severally liable.
 Earlier courts distinguished between: (1) de jure corporations, formed according to
mandatory statutory conditions; (2) de facto corporations, which were recognized
despite being defectively incorporated, where there was an attempt to organize under a
valid corporation law and a good faith claim to be doing business as a corporation; and
(3) corporations by estoppel, which were recognized for equitable reasons, and which
estopped the parties from denying the existence of the corporation.
 MBCA §§ 50 and 139 did away with de facto corporations and corporations by estoppel.
Under MBCA § 50, a corporation exists only after the certificate of incorporation is
issued, before which time there is no corporation, whether de jure, de facto, or by
estoppel.
 Under MBCA § 139, individuals who act as a corporation before the certificate of
incorporation has been issued are jointly and severally liable. It does not matter
whether a third party believed he was dealing with, or intended to deal with, a
corporation. Before the certificate of incorporation is issued, the individuals are liable,
not the corporation
 Holding
 In this case, Penn Ave. was not a corporation when the contract was signed, when the
lease was assigned, when the bill of sale was executed, or when Levy began doing
business under the Penn Ave. name. Levy is personally liable because he assumed to
act as a corporation before the certificate of incorporation had been issued.
 Robertson is not estopped from denying the existence of the corporation based on his
acceptance of one payment on the note after the certificate of incorporation had been
issued. A person who is liable under MBCA § 139 for acting without authority is not
relieved of liability when the corporation later comes into existence.

o Frontier refining company v. Kunkel’s – Seem to avoid liability because they are creditors/can rebut a
showing of partnership of paying a debt.
 Facts
 Clifford Kunkel (promoter) approached Fairfield and Beach (defendants) about financing
the lease and operation of a filling station and truck stop owned by Frontier Refining
Company (plaintiff). Fairfield and Beach agreed to finance and purchase equipment for
the venture, on the understanding that it would be Kunkel’s responsibility to manage
the business and ensure that it was incorporated prior to taking any action. Kunkel
never attempted to incorporate the business. Kunkel signed various agreements with
Frontier, including a lease under his individual name, doing business as Kunkel’s Inc.
Fairfield and Beach did not see these agreements or discuss them with Kunkel before
they were signed. Without Fairfield or Beach’s knowledge, Kunkel took over the filling
station and started doing business. Frontier sold gasoline to the station, billing it to
Clifford Kunkel doing business as Kunkel’s Inc. When the business failed to pay for
several thousand dollars’ worth of gasoline, Frontier brought suit against Fairfield and
Beach (defendants), as members of the Kunkel’s Inc. partnership, to recover the
balance due. Fairfield and Beach denied the existence of a partnership, and claimed that
Kunkel operated Kunkel’s Inc. as an individual. The trial court found that Kunkel’s, Inc.
was not a partnership, found for Fairfield and Beach, and dismissed the action. Frontier
now appeals.
 Issue
 Are two people personally liable for a business’ debt if they did not hold themselves
out as a corporation, and if the creditor knew that a corporation had not been formed
and proceeded to conduct business with another party as an individual?
 Rule
 No. People are not personally liable for a business’ debt if they do not hold themselves
out as a corporation.
 The general rule is that when two or more people hold themselves out to be a
corporation when there is no such corporation, they will be liable individually as
partners for its debts.
 A Wyoming statute also provides that a person who acts as a corporation without
authority to do so shall be jointly and severally liable for debts incurred as a result.
 Holding
 Here, there is sufficient evidence to support the trial court’s conclusion that Fairfield
and Beach were not partners in the venture, and thus not individually liable on the
debt.
 First, the trial court could have inferred from the evidence that Fairfield and Beach did
not authorize Kunkel to represent to Frontier that they were in a corporation known
as Kunkel’s, Inc., or to enter into contracts with Frontier under that name, and could
thus have concluded that Fairfield and Beach did not hold themselves out as a
corporation.
 Second, the trial court could have inferred that the debt was not incurred in the name
of the pretended corporation, as Frontier knew that a corporation had not yet been
formed, but proceeded to conduct its business with Kunkel as an individual.
 Finally, Frontier’s argument in this case is inconsistent with its position in a companion
case, in which Frontier brought an action of replevin to recover possession of the
business’s equipment, based on a mortgage obtained from Kunkel. The mortgage was
signed by Kunkel and stated that it was not executed on behalf of a pretended or
existing corporation. The court in that case found the mortgage to be valid, and entered
judgment for Frontier. Implicit in that judgment is the finding that Frontier’s debtor,
and the owner of the property that was pledged to secure that debt, was Kunkel as an
individual. Frontier cannot now disavow that judgment in favor of a finding that Fairfield
and Beach are personally liable. The trial court’s judgment is affirmed.

Chapter 6 – Disregard of the Corporate Entity


A) The common law doctrine of piercing the corporate veil (how liability can arise when the corporation has been
property formed)
 Piercing the corporate veil  the judicial act of imposing personal liability on otherwise immune corporate
officers, directors, and/or shareholders for the corporations wrongful acts
o Not restricted to corporations; can be applied to any limited liability (LLP / LLC)
 HYPO
o Form A corporation but run it like a sole proprietorship by not adhering to any corporate formalities
o Creditor sues the corporation but finds out the corporation is insolvent
o You are suing the corporate form to shield yourself but you are otherwise not using the corporate form
legitimately


o Piercing the veil  taking that claim that claimant has against the corporation and going after owner as
if there is one economic unit because owner did not treat corporation in way it should
 Basic Terms
o Capital
 Money or assets invested, or available for investment, in a business
 The total assets of a business, especially those that help generate profits
 The total amount or value of a corporations stock (corporate equity)
o Working capital
 Current assets (cash, inventory, and accounts receivable) less current liabilities
 Working capital measures liquidity and the ability to discharge short term obligations
 CA/CL = Working capital ratio
o Capitalization
 The total amount of long term financing used by a business, including stocks, bonds, retained
earnings, and other funds
o Undercapitalization
 The financial condition of a firm that does not have enough capital to carry on in its business
 Company lacks significant resources to cover prospective risks
 Assessed based upon likely economic needs in a specific line of business
 (1) [Particularly relevant in] tort cases (insurance for tort cases factor in undercapitalization)
 Tort cases  all torts are non-consensual / no one contracts to get hurt
 Doesn’t seem fair to set up a corporation to avoid personal liability and then not put any
funds in the business for anticipated issues that might come up
 (2) Contract cases
 Contracts are consensual
 A creditor lending money has the ability to inspect the corporate books and records and
require a verified financial statement to see how heavily the corporation is and whether
it has efficient working capital ration to meet its debt payment
 Consensual creditor also has ability to get a person guarantee, a co-signer or secure
debt through corporate assets
 Issue: what state law applies when piercing the corporate veil?
o Based on state of incorporation

 Bartle V. Home Owners – CL piercing the corporate veil.


o Facts
 Home Owners Co-op (defendant) was organized to provide low-cost housing to its members,
mostly veterans. Westerlea Builders, Inc. was organized as a wholly owned subsidiary of Home
Owners, to construct housing. When Westerlea began to have financial difficulties, its creditors
took over construction responsibilities pursuant to an extension agreement. Westerlea was
later adjudicated bankrupt. Bartle (plaintiff), Westerlea’s trustee in bankruptcy, brought suit
against Home Owners, claiming it is liable for Westerlea’s contract debts. The trial court found
that during the period when creditors extended credit, Home Owners and Westerlea maintained
outward indications of being two separate corporations. The trial court also found that the
creditors were not misled, that there was no fraud, and that Home Owners’ actions had not
injured Westerlea’s creditors by depleting assets or otherwise. Finally, the trial court held that
the creditors could not dispute the existence of separate corporate entities because of the
extension agreement. The Appellate Division affirmed. Bartle appealed, arguing that the lower
courts erred in refusing to pierce the corporate veil of Westerlea.
o Issue
 May a plaintiff pierce the corporate veil of a subsidiary to reach the parent corporation, where
there has been no fraud, misrepresentation, or illegality
o Rule
 A plaintiff may not pierce the corporate veil where there has been no fraud, misrepresentation,
or illegality.
 A plaintiff may pierce the corporate veil of a corporation and reach its shareholders to prevent
fraud or to achieve equity.
o Holding
 Here, there has been no fraud, misrepresentation, or illegality. Incorporation is permitted
specifically to escape personal liability. Home Owners’ placement of its construction operations
in a separate corporate entity is within the limits of public policy. The judgment is affirmed.
 No fraud
 Creditors were always aware Westerlea was a separate entity
 Creditors should have protected themselves
 DeWitt Truck v. W. Ray
o Facts
 W. Ray Flemming Fruit Co. (the corporation) (defendant) acted as an agent for growers of farm
products, selling the growers’ products on their behalf. Under its agreement with the growers,
the corporation gave the growers the full sale price, less the corporation’s sales commission,
and the transportation costs for transporting the products from the growers to the buyers,
which was to be given to DeWitt Truck Brokers (plaintiff). Flemming, the corporation’s
president, testified that he owned approximately 90 percent of the corporation’s stock, and
was unclear as to who the other stockholders, officers, and directors were. There were never
any directors’ or stockholders’ meetings. Flemming was the only stockholder or officer to ever
receive a salary or dividend from the corporation, or had any say in its operation. Flemming
received $15,000 to $25,000 a year at a time when the corporation was not showing a profit and
had no working capital, and which payments were not authorized by the board of directors.
Flemming does not dispute that the corporation had no capital reserves. DeWitt brought suit,
seeking to impose personal liability on the corporation’s president. The district court pierced
the corporate veil and imposed individual liability on Flemming. Flemming now appeals.
o Issue
 Where a single individual owns a substantial portion of a corporation’s stock, and where there
are other factors in support of piercing the corporate veil, may the stockholder be held
individually liable?
o Rule
 Where a single individual owns a substantial portion of a corporation’s stock, and where there
are other factors in support of piercing the corporate veil, the stockholder may be held
individually liable.
 A corporation is presumed to be separate and distinct from its officers and stockholders, and
the debts of the corporation are not those of the individual stockholders.
 However, courts will not recognize the corporate entity if doing so would extend incorporation
beyond its legitimate purposes and would cause injustice or inequity.
 In such cases courts will pierce the corporate veil and treat the corporation and its stockholders
as identical.
 Piercing the corporate veil should be done reluctantly and cautiously, and the burden for doing
so is on the party asserting the claim.
 The question of whether to disregard the corporate entity is one of fact, to be determined on a
case by case basis.
 When substantial ownership of a corporation’s stock lies in a single individual, and there are
other factors in support, courts often apply the “alter ego” or “instrumentality” doctrine to
pierce the corporate veil and make the individual stockholder liable.
 Factors to consider in applying this doctrine are whether:
 the corporation was grossly undercapitalized for its purposes;
o i.e. dividends weren’t paid
 corporate formalities were not followed;
 dividends were not paid;
 the debtor corporation is insolvent;
o meaningless
 the dominant stockholder siphoned corporate funds;
 the other officers or directors are non-functioning;
 there are no corporate records;
 and the corporation exists merely as a façade for the operations of the dominant
stockholder.
o Legal conclusion ≠ fact
 Mainly  falling to run the business by the book
o Need showing of bad people with intent to rob corporations and creditors
 The decision to disregard the corporate entity cannot rest on a single factor, and there must be
an element of injustice or fundamental unfairness. personally liable for DeWitt’s charges. The
district court’s judgment is therefore affirmed.
o Holding
 The district court’s decision to disregard the corporate entity in this case was not clearly
erroneous. Flemming testified that
 he owned approximately 90 percent of the corporation’s stock,
 and was unclear as to who the other stockholders, officers, and directors were.
 There were never any directors’ or stockholders’ meetings.
 The corporation was operated in a purely personal manner.
 Flemming was the only stockholder or officer to ever receive a salary or dividend from
the corporation, or had any say in its operation.
 The corporation’s existence was for Flemming’s exclusive benefit.
 He received $15,000 to $25,000 a year at a time when the corporation was not showing
a profit and had no working capital, and those payments were not authorized by the
board of directors.
 The corporation was also clearly undercapitalized. It lacked capital to pay any dividends.
 Flemming does not dispute that the corporation had no capital reserves, and it appears
that the corporation’s only operating funds were its sales commissions and the
transportation charges, which the corporation told the growers it was withholding from
the full sale price to pay to DeWitt.
 Equity and fundamental justice support disregarding the corporate entity and holding Flemming.

 Baatz v. Arrow Bar – Plaintiff trying to pierce the veil and impose liability on the owners re: tort CoA.
o Facts
 Edmond and LaVella Neuroth formed the Arrow Bar, Inc. The corporation purchased the Arrow
Bar business for $155,000 with a $5,000 down payment, and the Neuroths executed a
promissory note personally guaranteeing the remaining $150,000. The corporation obtained
financing for $145,000 toward the purchase agreement, and the Neuroths again personally
guaranteed the corporate debt. Peggy and Kenny Baatz (Baatz) (plaintiffs) were injured in an
automobile accident when the motorcycle they were driving was struck by a vehicle driven by
Roland McBride. McBride was uninsured, and Baatz brought suit against Arrow Bar and the
Neuroths (defendants). The complaint alleges that the Arrow Bar negligently served alcohol to
McBride when he was already intoxicated, prior to the accident. The trial court entered
summary judgment dismissing the Neuroths as individual defendants, and Baatz now appeals.
o Issue
 May the court pierce the corporate veil and hold shareholders individually liable where there is
no evidence that recognition of the corporation would produce injustices and inequitable
consequences?
o Rule
 No. Where there is no evidence that recognition of a corporate entity would produce injustices
and inequitable consequences, the court may not pierce the corporate veil and hold individual
shareholders personally liable.
 A corporation is considered a separate legal entity, unless continued recognition as a separate
entity would produce injustices and inequitable consequences, in which case a court may
pierce the corporate veil and hold shareholders individually liable.
 Factors that may permit a court to pierce the corporate veil are: (1) fraudulent representations
by directors; (2) undercapitalization; (3) failure to observe corporate formalities; (4) absence of
corporate records; (5) corporate payment of individual obligations; or (6) use of the corporation
to promote fraud, injustice, or illegalities.
o Holding
 Baatz first argues that the Neuroths should be personally liable because they personally
guaranteed corporate obligations.
 However, a personal loan guarantee is a contract and does not affect tort liability.
 In addition, personally guaranteeing a corporate obligation is the opposite of factor (5)
listed above, and as such supports recognition of the corporate entity.
 Baatz next argues that the corporate veil should be pierced because the corporation is merely
an alter ego of the Neuroths.
 Baatz fails to show that the Neuroths were treating the corporation as an
instrumentality for conducting personal business, and the evidence indicates that they
treated the corporation as separate from their personal affairs.
 Baatz also argues that the corporate veil should be pierced because the corporation is
undercapitalized, as it was funded with only $5,000 of borrowed capital.
 Normally, questions of individual liability resulting from undercapitalization should not
be addressed until corporate liability has been determined.
 Moreover, there is no evidence that the corporation’s capital was inadequate for its
operations.
 Finally, Baatz argues that the corporate veil should be pierced because the corporation failed to
observe corporate formalities, noting that Arrow Bar’s signs and advertising did not indicate
that it was a corporation.
 To the contrary, the corporation has complied with statutory requirements.
 Even if it had failed to do so, however, the occasional failure to follow all the required
forms for conducting corporate activities does not justify disregarding the corporate
entity, especially where the claimed defect and the resulting harm are unrelated. There
is no evidence that the Neuroths personally served McBride on the day of the accident,
or any evidence that the Neuroths’ treatment of the corporation would produce the
injustices and inequitable consequences required to pierce the corporate veil. The trial
court’s dismissal of the Neuroths as individual defendants is therefore affirmed
o Dissent
 Thinks corporation should have had insurance to deal with issues like these
o Notes
 (1) Considerations
 Fraud
 Undercapitalization (particularly relevant in tort cases)
o You should have sufficient money to run the business on day to day basis OR
there must exist enough money to cover unforeseen contingencies (tort
obligations)
 Corporate formalities
 Absence of corporate records
 Payment by corporation of individual
 (2)Tort v. contract
 Unlike tort creditors, contract creditors can deline to deal with the business unless they
are satisfied that eht entity is credit worthy
 If creditor perceive risk, she can decldine to deal with the corporation, require personal
guarantees from investors or others, or charge significantly high rates of interest to
compensate for risk
o Tort ≠ voluntary dealing
o Contract = voluntary dealing

 Radaszewski v. Telecom Corp. – Parent subsidiary context / only has personal jurisdiction if can pierce the veil /
piercing the veil to impose upon a parent corporation liability incurred by subsidiary.
o Facts
 Radaszewski (plaintiff) was injured in an automobile accident when the motorcycle he was
driving was struck by a truck driven by an employee of Contrux, Inc. Contrux, Inc. is a wholly
owned subsidiary of Telecom Corporation (defendant). When Telecom formed Contrux, it
contributed loans, not equity, and did not pay for all of the stock that was issued. Telecom did
provide Contrux with $1 million in basic liability coverage, and $10 million in excess coverage.
Contrux’s excess liability insurance carrier became insolvent two years after Radaszewski’s
accident. Telecom argued that the district court lacked personal jurisdiction over it. The
question of jurisdiction turned on whether Radaszewski could pierce the corporate veil and
hold Telecom liable for the conduct of its subsidiary, Contrux. Telecom argued that the
corporate veil could not be pierced on the basis of undercapitalization, because of the insurance
it had provided to Contrux. The district court rejected Telecom’s argument that insurance could
determine a subsidiary’s financial responsibility. The district court nonetheless held that it
lacked jurisdiction over Telecom on other grounds.

o Issue
 May a plaintiff pierce the corporate veil to bring a parent corporation into a case against a
subsidiary where the subsidiary was undercapitalized in a traditional accounting sense, but
was provided with more than adequate liability insurance?
o Rule
 A plaintiff may not pierce the corporate veil and bring a parent corporation into a case against a
subsidiary if the subsidiary was undercapitalized in a traditional accounting sense, but was
provided with more than adequate liability insurance.
 A person injured by a corporation or its employees may generally recover only from the assets
of the employee or the employer corporation, and not from the shareholders of the corporation
or its parent corporation.
 As found in Collet v. American National Stores, Inc., 708 S.W.2d 273 (Mo.App. 1986), to pierce
the corporate veil and make corporate shareholders liable a plaintiff must show:
 (1) complete domination and control over the finances, policy, and business of the
corporation, so that the corporation at the time of the transaction had no separate
mind, will, or existence of its own;
 (2) the control was used by the defendant to commit fraud, to violate a legal duty, or to
act dishonestly or unjustly in violation of the plaintiff’s legal rights; and
o Undercapitalizing a subsidiary satisfies the second element of the Collet test,
since creating a business and operating it without sufficient funds to be able to
pay bills or satisfy judgments against it implies a deliberate or reckless disregard
of the rights of others.]
o Having insurance negates this element
 (3) the control and breach of duty proximately cause the plaintiff’s injury.
o Holding
 In this case, the district court found that Contrux was undercapitalized according to generally
accepted accounting principles. Telecom contributed loans, not equity, and did not pay for all of
the stock that was issued. However, Telecom argues that Contrux was financially responsible
because it was provided with $11 million in liability insurance to pay judgments such as the
one now sought by Radaszewski. The district court rejected the argument that insurance could
determine a subsidiary’s financial responsibility. This court disagrees.
 The policy behind the second element of the Collet test is to ensure financial responsibility.
Insurance meets this policy just as well as other forms of capitalization. The purpose of the
limited liability doctrine is to protect a parent corporation when a subsidiary becomes insolvent.
This doctrine would be destroyed if a parent corporation could be held liable for errors in
business judgment. Something more than an error in business judgment is required
under  Collet. The district court’s dismissal of the complaint for lack of jurisdiction is therefore
affirmed, but modified to be with prejudice.

 Fletcher v. Atex, inc. – Delaware / what states law should apply to piercing claim  law of state of
incorporation / cash management system.
o Facts
 Atex, Inc. (defendant) is a Delaware corporation wholly owned by Eastman Kodak, Inc. (Kodak)
(defendant). Atex produces keyboards. The plaintiffs sued Atex and Kodak, arguing that Atex’s
keyboards caused repetitive stress injuries. The individuals claiming to have been harmed by
the keyboards wish to hold Kodak directly liable for the injuries on the theory that Kodak
abused the corporate form and Atex was simply its alter ego. Atex participated in a cash
management system in which its funds were pooled with other Kodak subsidiaries and
managed by Kodak. Kodak’s approval was required for Atex’s major decisions or expenditures.
There was a small degree of overlap between the boards of Kodak and Atex. Some Kodak
corporate documents referred to a merger between Kodak and Atex, and referred to Atex as a
division of Kodak. Finally, Atex assigned its CEO’s mortgage to Kodak as part of a transaction
with a third party. With regard to the mortgage assignment and corporate business generally,
Atex observed all the corporate formalities. =
o Issue
 May a shareholder be held liable for corporate debt if the shareholder and corporation did not
operate as a single entity, or if their actions did not create injustice or unfairness?
o Rule
 Shareholders are shielded from liability for general corporate debt except in extraordinary
circumstances.
 One basis for piercing the corporate veil is the alter ago theory, which applies when the
corporation is used as an alter ego or instrumentality of its owner.
 Under Delaware law  to recover under the alter ego theory, the plaintiff must show that the
corporation and shareholder functioned as a single economic entity, AND that their actions
caused injustice or unfairness.
 (1) show fraud
 (2) show alter ego
 A variety of factors should be considered in determining whether the two acted as a single
entity, including
 whether the corporation was adequately capitalized,
 whether corporate formalities were observed,
 and whether the corporation functioned simply as a façade for the owner.
o Holding
 Here, application of the alter ego theory is not justified. Cash management systems are
common where a corporation owns numerous subsidiary corporations. Absent evidence that
the system functioned as a complete commingling of funds, courts have declined to find alter
ego theory liability on the basis of a cash management system. No such evidence exists here.
 It is also quite common for controlling shareholders to hold veto power on major decisions.
Courts have found that this level of oversight does not show that the two corporations
functioned as a single economic entity.
 The limited degree of board membership overlap between Atex and Kodak, the loose language
in some Kodak documents, and the assignment of the mortgage to Kodak are likewise
insufficient in this case, since corporate formalities were assiduously followed.
 Finally, even if the above actions were evidence of a single economic entity, the plaintiffs’ claims
would still fail, because they fail to demonstrate injustice or unfairness. The decision of the trial
court granting summary judgment to Kodak is therefore affirmed.

B) Reverse Piercing
 Reverse piercing
o Creditor of a shareholder seeks to impose liability on the corporation
o permitting a creditor to access an entity’s (corporations) assets in satisfaction of an owner’s liability
 Cargill inc v. Hedge
o Facts
 Sam and Annette Hedge (defendants) bought a 160-acre farm, which they placed in a Minnesota
family farm corporation, Hedge Farms, Inc. Annette was the sole shareholder of Hedge Farm,
Inc., and the Hedges and their daughters were the corporation’s only officers. Sam purchased
farm supplies from Cargill, Inc. (plaintiff), which was unaware that the corporation existed.
When Sam failed to pay for the supplies, judgment was entered against Sam Hedge and Hedge
Farms, Inc.
 Cargill was the successful bidder at an execution sale on the farm.
 The trial court enjoined the proceedings, allowed Annette Hedge to join the suit as an
intervenor, and found that the Hedges had a right to exempt 80 acres from the execution, as
their homestead.
 The court of appeals affirmed, finding that Annette had an equitable interest in the corporate
property as the sole shareholder of Hedge Farms, Inc., and that her equitable interest, along
with the Hedges’ occupancy of the farm, satisfied the requirements under the homestead
statute. The court of appeals implied that it would be willing to pierce the corporate veil to
reach the same result. The Supreme Court of Minnesota granted Cargill’s petition for review.
o Issue
 May a court disregard the corporate entity and treat corporate assets as the personal assets of
the shareholders or officers where doing so would further the purposes of the homestead
exemption, and where the corporation is an alter ego for the shareholders or officers?
o Rule
 A court may disregard the corporate entity and treat corporate assets as the personal assets of
the shareholders or officers where there are strong policy reasons for doing so, and where the
corporation is an alter ego for the shareholders or officers.
 Under Minnesota’s constitution and statutes  a debtor has a right to exempt the house he
owns and occupies, along with the land it is on, from sale or seizure.
 By statute, 80 acres may be exempted in rural areas.
 Because a corporation has no need for a dwelling, it is not entitled to a homestead
exemption, and if such an exemption is granted in this case it would need to apply
personally to the Hedges.
o Statute only applies if farm is owned by individual, not entity
o Holding
 The court of appeals held that Annette had an equitable interest in the corporate property as
the sole shareholder of Hedge Farms, Inc., and that her equitable interest, along with the
Hedges’ occupancy of the farm, allowed them to personally assert a homestead exemption in
corporate property.
 This court declines to adopt this equitable interest theory, as it is ill-suited for determining
creditors’ rights and the relationship between shareholder and corporation.
 Instead, this court applies a reverse piercing of the corporate veil, which may be used where
there are strong policy reasons for disregarding the corporate entity.
 In determining whether to reverse pierce, the court must also look at the extent to which the
corporation is an alter ego of the individual, and whether others would be harmed by a pierce
of the corporate veil.
 For example, in Roepke v. Western National Mutual Insurance Co., 302 N.W.2d 350 (1981), this
court disregarded the corporate entity in order to further the purposes of the No-Fault Act,
finding that it would be unfair to deprive a business owner personally of no-fault coverage
where he was the president and sole stockholder of the corporation and where the company
vehicles in question were used solely as family vehicles.
 In this case, the policy reasons in support of a reverse pierce, that is, furthering the purposes of
the homestead exemption, are even stronger than those in Roepke.
 In addition, Hedge Farm, Inc. was an alter ego for the Hedges. They operated the farm as their
own, had no lease with the corporation, paid no rent, and used the farmhouse as their home.
Annette Hedge was the sole stockholder, and Sam and Annette Hedge and their daughters were
the corporation’s only officers. The court will therefore disregard the corporate entity of Hedge
Farm Inc. and treat the farm as if owned by Sam and Annette Hedge. The debtor Sam Hedge,
as co-owner of the farm, is entitled to claim 80 acres of the farm as a homestead exemption,
and the execution sale of those 80 acres is therefore void.
o Notes
 (1) case law
 Employee of subsidiary sue parent corporation
 Under workers comp law, employee cannot sue employer
 Parent corporation was not the employees employer but argue it should be seen as
employer and be immune from suit under reverse piercing
 This case  didn’t allow parent corporation to disregard the subsidiary
 As such, employee was able to sue
 Pepper v. Litton
o Facts
 Litton (defendant) is the sole shareholder of the Dixie Splint Coal Company (Dixie Splint).
Pepper (plaintiff) sued Dixie Splint for royalties due under a lease. While the case was pending,
Litton sought and obtained a judgment against Dixie Splint for claims of back salary. Pepper
then obtained a judgment. Litton executed on his judgment and purchased the corporation’s
assets in an execution sale. Litton then caused Dixie Splint to file for bankruptcy. The
bankruptcy trustee brought suit in state court to have Litton’s judgment and the execution sale
set aside, but was unsuccessful. Litton then filed a claim in bankruptcy court for the remainder
of his judgment that was not satisfied by the execution sale. The bankruptcy court disallowed
Litton’s entire claim and ordered that the bankruptcy trustee should recover all of the assets
that Litton purchased at the execution sale. The court of appeals reversed.
o Issue
 May a court disregard the corporate entity and disallow or subordinate an officer or
stockholders claim against a corporation for equitable purposes?
o Rule
 Yes. A court may, for equitable purposes, disregard the corporate entity and disallow or
subordinate an officer or stockholder’s claim against a corporation.
 Proceedings in bankruptcy courts are inherently proceedings in equity, and such courts have
the power to decide the claims of a corporation’s officers, directors, and stockholder’s during
that corporation’s bankruptcy.
 An officer or stockholder’s claim against a corporation is sometimes required to be disallowed or
subordinated for equitable purposes. Occasionally this is done by disregarding the corporate
entity.
 Directors and stockholders are fiduciaries. Their dealings with the corporation must be
subjected to rigorous scrutiny, and the burden is on them to prove good faith in their
transactions and inherent fairness to the corporation and to those interested in the
corporation.
 If, under all of the circumstances, the transaction does not carry the earmarks of an arm’s
length transaction, it will be set aside for equitable purposes
o Holding
 Here, the district court was clearly correct in disallowing or subordinating Litton’s salary claim.
Litton did not seek to enforce his claims until his corporation ran into financial difficulties, and
then used his claim to impair another creditor’s rights. Upon enforcing his claim, Litton
acquired the corporation’s assets not for cash or something else of value to a creditor, but
rather for bookkeeping entries that represented Litton’s own estimate of his services. Because
the evidence here shows a planned and fraudulent scheme, the bankruptcy court was required
to grant equitable relief and disallow the Litton claim. The court of appeals’ judgment is
therefore reversed, and the bankruptcy court affirmed.
o Notes
 Deep rock doctrine
 Rather than allowing Litton to profit as creditor of his own corporation, court adopts
“deep rock doctrine”
 Doctrine  claims of insiders who now are creditors can be subordinated or
disallowed to claims of outside creditors when equity demands
o Way we can ignore separate existence of corporation and its insiders i.e. when
elements of the veil piercing are present
o Puts shareholder last in line to paying of creditors
o Demonstrates courts broad equitable power to subordinate debts of creditors
guilty of some misconduct

C) Successor Liability
 Issue; what happens to corporate debts and obligations known and unknown when a business fundamentally
changes
o (1) company ceases operations entirely (asset acquisition)
o (2) company merging into another
 (1) Merger (entity intact)
o The absorption of one organization (esp. corporation) that ceases to exist, into another that retains its
own name and identity and acquires the assets and liabilities of the former

 (2) Asset Acquisition (entity not intact; shell)


o Acquisition of a corporation by purchasing all of its assets directly from the corporation itself, rather
than purchasing shares from its shareholders
 Wants to acquire all the good and none of the bad
 Buys the assets but doesn’t merge / leave company as a shell
 Creditors focus on successor company to try to get paid
 General Rule: successor companies are not liable for debts or liability’s it takes over by way of asset acquisition
o *see exceptions list
 Nissen Corp v. Miller
o Facts
 Frederick Brandt (plaintiff) bought a treadmill designed, manufactured, and marketed by
American Tredex Corporation (Tredex). Nissen Corporation (defendant) entered into an
agreement with Tredex to purchase its assets, as well as some of its obligations and liabilities.
The purchase agreement expressly excluded liability for injuries resulting from products
previously sold by Tredex. Under the contract, Tredex would continue to operate for five years.
Brandt was injured by the treadmill. Over a year later, Tredex was administratively dissolved. A
year after that, Brandt sued Tredex and Nissen..
o Issue
 Does a successor corporation acquire its predecessor’s liabilities and debts based solely on its
continuation of the predecessor’s business operations or enterprise?
o Rule
 A successor corporation does not acquire its predecessor’s liabilities and debts simply because it
continues the predecessor’s business operations or enterprise.
 The traditional rule is that a successor corporation does not acquire its predecessor’s liabilities
and debts unless: (1) there is an agreement to assume liabilities; (2) the transaction is merely a
consolidation or merger; (3) the successor is merely a continuation or reincarnation of the
predecessor; or (4) the transaction was fraudulent, made in bad faith, or made without
sufficient consideration.
o Holding
 The parties do not argue that this case falls within any of the four traditional exceptions to the
rule of nonliability of corporate successors. Instead, Brandt argues that in products liability cases
this court should recognize a fifth “continuity of enterprise” exception to a successor’s
nonliability, which would apply where the successor continues the predecessor’s business
operations or enterprise.
 Nissen argues that the traditional rule and its four exceptions balance creditors’ and successors’
rights by promoting alienability of corporate assets while protecting consumers and creditors
from unjust corporate transactions.
 Brandt argues that the traditional rule was developed to protect creditors and shareholders in
the corporate context and does not apply to products liability cases.
 Brandt argues that under strict products liability, some entity must be held responsible when a
consumer suffers an injury. This court has previously held that under the theory of strict liability,
proof that the product was defective at the time it left the seller’s control implies fault by the
seller, sufficient to impose liability for injuries caused by the product.
 However, a corporate successor is not a seller, and it would be unfair to impose liability on a
party that has no connection with the acts that caused the injury. The Third Circuit has
previously recognized the unfairness of the continuity of enterprise theory, noting that a
successor that decides not to manufacture one of its predecessor’s products because it is too
dangerous may nonetheless be liable for claims on that product under the continuity of
enterprise theory. The Restatement (Second) of Torts, § 402A, on which Maryland’s strict tort
liability is based, also does not impose liability on successor corporations.
 Brandt also argues that Nissen benefited from the goodwill of Tredex, and that it would
therefore be unfair to escape the burden of paying Tredex’s tort liabilities. However, this
ignored the fact that Nissen will lose this goodwill if Tredex’s products do cause injuries.
 In addition, although Brandt argues that major corporations should not be allowed to purchase
only the benefits of a predecessor and not its attendant liabilities, the continuity of enterprise
theory would also impose liability on small businesses that are not in a position to spread risk or
to insure against it. Brandt also argues that he was not made aware of changes in the corporate
structure that would affect consumer protection against Tredex’s products. However, this court
does not believe that consumers retain products based on their ability to sue a particular
corporate entity if a problem with the product arises. Finally, while Brandt argues that Nissen
should be liable because it maintained a network for servicing Tredex products, Nissen should
not be penalized for assuming some, but not all, of Tredex’s commitments.
 This court therefore rejects the continuity of enterprise theory of successor corporate liability,
and in products liability cases adopts the traditional rule of successor nonliability and its four
traditional exceptions.

Chapter 7 – Financial Matters and the Corporation


A) Debt and Equity Capital
 Debt/Equity
o Businesses need money (capital) to function
o 3 ways [businesses can obtain money]
 (1) borrow it (debt)
 (2) allow investors to buy ownership interest in company (equity) (selling shares)
 (3) retaining earnings from the corporation through sales and service
 Debt
o Disadvantage
 Risky because it must always be repaid and often time we secure assets when we get debt so a
creditor or bank can come in and take collateral because we are utilizing the debt
o Advantage
 Tax benefit because corporation can deduct interest payments on debt but cannot deduct
interest on distributions to shareholders; so deducting interest reduces income which reduces
taxes
 Helps with leverage; you are putting our own money in and because of that you could possibly
gain
 Ex) if you barrow $100k at 5% interest but that $100k made you 10% in gains/you
experienced a 5% benefit from barrowing and you never had to use own money to do it
 Equity
o Disadvantage
 Risky for investors because company might go under and when it does the rule always is – debt
is first in line in terms of getting paid
o Advantage
 Equity has much bigger payday if the company does well
 * Creditors always come before investors.
o *First in line to get paid = still unsecured.
o *But accounts receivable (perfected security interest) = can still demand.

B) Types of Equity Securities


 1) Shares generally
o Equity capital  reflected via ownership of shares (stock)
o Shares  the units into which the proprietary interest in a corporation are divided (MBCA §1.40(22))
 Ownership units
o Authorized stock  the maximum shares a corporation can issues which is put in the articles of
incorporation
 Cannot issue more shares than is authorized by articles
 No statutory limit on number of shares that can be authorized by a corporation and no
requirement that all authorized shares actually be issued
 Number authorized should be enough for future efforts to raise capital
o Issued stock  number of shares the corporation actually sells
o Publicly traded stock  “issuing” v. “buying”/”selling”
 When investor buys stock, she is buying it from another member of the trading public; the
company from which you are buying is NOT issuing it  thus, the company is not making
money, the person who sold it is
 Only the initial sell by the corporation, before it was public was the issuance

 2) common and preferred shares


o every state permits the corporation to establish different classes or levels of stock
 common v. preferred  do not have to have both / have to have common
 if they do have both, distinction is important because of distributions
o distribution/dividends  payments by the corporation to shareholder out of surplus
 preferred stock holders get paid first before any other shares
o (1) Common stock
 A class of stock generally entitling the holder to
 (1) vote on corporate matters (each share carries one vote)
 (2) receive dividends after other claims
 (3) share in assets upon liquidation
 Non-financial rights
 (1) right to inspect corporate book
 (2) right to bring derivative action
 (3) right to financial info
 [Your slice of the Pie]
o (2) Preferred Stock [Pay first]
 A class of stock generally giving its holder a preferential claim to dividends and to corporate
assets upon liquidation but usually carrying no voting rights

o Terms
 Series of stock  different classes of stock (series A / series B)
 Options  corporation can give options to employees as an incentive
 Option gives holder right to purchase the stock at the option price
 She does not become a shareholder however until she exercises the option and pays
purchase price
 At that point, the corporation issues the stock to her
 Puts and calls  particular kind of option
 Put  option to sell stock at a set price
 Call  option to buy stock at set price

 3) special contractual rights of publicly traded preferred shares


o Types of preferred stock
 (1) cumulative  if dividends are not paid 1 year, they carry over to the next year and the
common stock cannot get a dividend until the preferred shares are paid the full amount
 Preferred stock with a provision that stipulates a rate of return and stipulates that if any
dividend payment have been missed in the past, the dividends owed must be paid out
to the cumulative preferred shareholders first; this is before other classes of preferred
stock and common stock can receive dividend payment
 Most desired type of stock because allows those dividends to carry over each year
 (2) non-cumulative  each year you hit the reset button and there is no carry over effect
 If you don’t get paid, you don’t get paid
 Distribution is always tied to the year insolvency of the corporation
 (3) partially cumulative  cumulative to the extent that there are earnings in a given year (that
do not get paid out as dividends), but if the whole dividend cannot be paid, the excess does not
carry forward
 (4) participating preferred  these stockholders get an additional dividend when the common
shares get paid specified amount
 Holder right to receive dividends are paid to preferred shareholders as well as additional
dividend with common stock holders
o (1) get paid first with preferred
o (2) get paid again with common
 (5) classes of preferred  the corporation can create different classes of preferred stock that
will take before other classes
 Class A preferred / class B Preferred
o Dividends: Purely ownership related. Purely discretionary. Paid out of surplus.
o Vs. Salary: Nothing to do with ownership. Steady basis. W2 Employees.

 4) classes of common shares


o MBCA § 6.01: authorizes creation of classes of common shares by appropriate provision in the articles
of incorporation.
 May vary in terms of management, financial, or voting rights.
 Often designated by alphabetical reference, e.g. “Class A common shares.”
 Or description, e.g., “Nonvoting common stock.”
 Classes of common shares are widely used as planning devices in closely held corporations.

C) Issuance of shares: herein of subscriptions, par value, and watered stock


 1) Share subscriptions and agreements to purchase securities
o Stock Subscriptions
 General Rule: corporations can issue stock through subscriptions. [True contract]
 Stock subscription  [way capital is raised is through] promises to buy a certain amount of
stock
 Subscription  written offer to buy stock from the corporation and subscription
becomes subscription agreement once accepted by board of directors
 Subscription agreement  contract binds outside investors to buy shares
 Subscriber  does not become a shareholder until she has paid the issuance price in
full / failure to pay in full is a default.
 Traditional: Capital up front for stock subscription.
 Modern: Can be services. Any proper consideration for stock.
 MBCA § 6.20: Subscription for Shares Before Incorporation
 (a) a subscription for shares entered into before incorporation is irrevocable for six
months unless the subscription agreement precludes a longer or shorter period or all
the subscribers agree on allocation
 (d) if a subscriber defaults in payment of money or property under a subscription
agreement entered into before incorporation, the corporation may collect the amount
owed as any other debt

 2) authorization and issuance of common shares under the MBCA


o MBCA § 6.01; Authorized Shares
 The articles of incorporation must authorize
 (1) one or more classes or series of shares that together have unlimited voting rights;
and
 (2) one more classes or series of shares (which may be the same class or classes as those
with voting rights) that together are entitled to receive the net assets (total assets –
totally liabilities) of the corporation upon dissolution (i.e. distributions)
o Issuance of Shares
 Issue: can a corporation issue stock for something other than money (services, tangible
property, future promises to pay
 Historical view  no
 Modern view  MBCA
 MBCA § 6.21: Issuance of Shares
 (a) the powers granted in this section to the board of directors may be reserved to the
shareholders by the articles of incorporation
 (b) the board of directors may authorize shares to be issued for consideration
consisting of any tangible or intangible property or benefit to the corporation,
including cash, promissory notes, services performed, contracts for services performed,
or other securities of the corporation
 (c) before the corporation issues shares, the BoD must determine that the
consideration received or to be received for shares to be issued is adequate. That
determination by the BoD is conclusive insofar as the adequacy of consideration for the
issuance of shares relates to whether the shares are validly issued, fully paid, and
nonassessable

 3) par value and stated capital


o Par Value
 Issue: assuming the issuance is made to investors for valid consideration] whether the issuance
was made for a proper amount of consideration
 I.e. what is the corporation charging to buy a piece of the corporate pie
 Par  minimum price at which the stock must be issued
 Set arbitrarily and has no correlation with the actual value of the issuance price of the
stock
 No par  no minimum issuance price
 BoD sets issuance price at its discretion depending on value and demand
 HYPO
 XYZ corporation issues 1,000 shares of $2 par stock
 It must receive at least $2k for this issuance (because par is minimum issuance price)
 If it receives less than $2k, there will be liability for “watered stock”
 Because par value does not connote “maximum”, the corporation may sell this stock for
more than $2k
 The board will set the actual issuance price when it authorities the issuance
o Par stock  supposed to protect creditors
 If XYZ issues 10k shares of $2 par stock for for $50k, $20k must be allocated to stated
capital (10k x $2), while the excess par $30,000 goes into capital surplus which can be
used to pay out dividends
o 10,000 shares
o -par value: $2
o -price of stock: $5
o (50,000 divided by 10,000)
o $20,000 has to go to stated capital because it’s the par value
 Stated capital  creditor cushion in which creditors could be paid for the corporations
debts
 This “protection” became illusory
 Law has never required par to be a particular amount so corporations have always been
able to set par at nominal amount and when these rules came out, corporations started
to issue penny stock
 Penny stock  setting a low par gives board greater flexibility in setting actual issuance
price and avoid potential for lawsuit for watered stock
 This also makes it so the stated capital fund is very small, offering no protection to
creditors
 HYPO
 If XYZ issues 10k shares of $.01 par stock, $100 must be allocated to stated capital (10k x
$.01)  offering creditors virtually no protection
o Insolvency limitations
 Most states have moved away from the stated capital requirement and instead impose
insolvency limitations on distributions to shareholders
 Statute  it is illegal to pay shareholders distributions from the company if the company is
insolvent or if the distribution will render it insolvent (cant pay shareholders unless out of
surplus and only way you can have surplus is if assets > liabilities).
 Being “insolvent” isn’t a bad thing. It just means you have more liabilities than assets.
 ** Assets – Liabilities = Owners Equity.
o Watered Stock
 Concept can still creep up in jurisdiction where corporation is still required to state a par value
or where incorporations place par value amount in articles of incorporation
 Watered stock  stock that is issued for less than par value or for ineligible type of
consideration
 Why do we care?
 Parties/creditors want to know how safe a corporation is/how safe loan or investment is
 HYPO
 Board of XYZ corporation authorizes the issuance of 10,000 shares of $2 par stock to
shareholder for $16k; what’s the problem
 Under the par rules, the incorporation should have received $20,000. It actually
received $16,000. Thus, there is $4k of water
 The corporation can sue to recover the amount OR if the corporation is insolvent,
creditors can sue to recover the $4,000
o The directors who approved issuance are jointly and severally liable
o Under CL  purchaser liable even if they didn’t actually know about the par
value because it is in the articles and on stock ticket (charged with notice)
o Today  majority of states hold that shareholder is liable if she/he pays less for
the stock than the consideration set by the board when it authorized the
issuance
 Imposes liability for issuance price and not par value
o Hanewald v. Bryans Inc
 Facts
 Keith and Joan Bryan incorporated Bryan’s, Inc. to act as a retail clothing store. The
company’s articles of incorporation authorized it to issue 100 shares of common stock
with a par value of $1,000 per share. Bryan’s, Inc. issued 50 shares to Keith, and 50
shares to Joan, but did not receive any payment for the stock that was issued. Bryan’s
Inc. purchased a dry goods store from Hanewald (plaintiff), for $55,000 in cash and a
promissory note for $5,000. The $55,000 payment was made from a bank loan that was
personally guaranteed by Keith and Joan Bryan. Bryan’s Inc. also leased the store
building from Hanewald for five years at $600 per month. Bryan’s Inc. operated for four
months, and was later dissolved. Bryan’s Inc. paid off the $55,000 bank loan, a $10,000
personal loan from the Bryans, and its other creditors. It did not pay the $5,000
promissory note to Hanewald, and attempted to avoid the remainder of its lease.
Hanewald sued Bryan’s Inc. and the Bryans (defendants) for breach of the lease and the
promissory note, seeking to hold the Bryans personally liable.
 Hanewald = creditor and leaseholder.
 Issue
 Whether Appellees as the corporation’s sole shareholders shall be personally liable for
the debt incurred by Bryan’s Inc. to Appellant.
 Rule
 Generally, shareholders are not liable for corporate debts beyond the capital they have
contributed to the corporation.
 Holding
 Yes. Appellees are personally liable to the corporation’s creditors to the extent they
failed to pay for the stock issued to them.
 Bryan’s, Inc. was authorized to issue 100 shares of stock at $1,000 per share. The three
Appellees each received 50 shares for no consideration. This makes them personally
liable for the corporation’s debt. The loan was a debt not an asset of the corporation
and was repaid before the business was abandoned. Therefore the loan cannot be
considered a capital contribution.
 4) Eligible and Ineligible consideration
o True or false:
 The board of directors may authorize shares to be issued for consideration consisting of services
to be performed.
 True.
 5) Par value in modern practice
o

D) Debt Financing
 Terms
o Debenture  a debt secured only by the debtors earning power, not by a lien on any specific asset
o Bond  an obligation secured by a lien or mortgage on corporate property
o Bond  often used loosely to refer to either a debenture or a bond
o Unsecured debt  debt secured only by the debtors earning power/no assets/no collateral
o Secured debt  you are promising to pay a bank money and you are taking a security interest in your
assets so if you don’t pay they can repossess

 1) The concept of leverage


o 2 basic points
 (1) if interest rates are low, it actually makes more sense to barrow if you believe you will get a
greater return on your investment than the interest rate you are paying
 More return than spending on interest earnings / share will be higher because you have
avoided dilution
 (2) interest payment on a loan are tax deductible from the corporations earnings, but
distributions and dividends are not
o When raising capital
 Should consider taking out a loan as well (as investing for example) and not capitalize your
future company with just investments
 More investment  less control of company
 Depends on whether the company can generate a return in excess of what the bank/lender is
going to charge
 Note: loan is unforgiving so if you default on payment you can be pushed out of business
 Missing a dividend or distribution does not carry those same consequences because not
a credit obligation but more stock you issue the less control you have

 2) Tax treatment of debt


o Because interest payments made by the company to its creditors are deductible, shareholders in a
corporation may want to claim that an investment is a loan and not an equity interest (so can be tax
deductible)
 This lets them avoid double taxation problem and guarantees some return on their money

o HYPO 1
 As a business, you have paid $10k in year 1 on interest payments off of barrowed money
 This reduces the bottom line  If you have gross income of $100k but you’re paid interest of
$10k, you’re only taxed at $90k
o HYPO 2
 Now assume, instead of paying interest, you’re paid a dividend to owners
 $10k a year on a dividend payment means you are still taxed at $100k and then the shareholder
that received dividends also has to pay taxes on their own
o Case Law
 Court recognized that a shareholder that lends money may not be as demanding about
repayment terms but it still has to be a loan with real world requirements and expectations
o Debt v. Equity


 Straight Debt: Congress safe harbor provision for Straight Debt.
 (loans not equity)  if requirements met, shareholder can claim his loan is debt
 A written unconditional promise to pay a sum certain where:
 (A) interest rates and payments not contingent on profits
 (B) no direct or indirect convertibility of the debt into stock is available; and
 (C) creditor is eligible shareholder

 3) debt as a planning device

o
o Case law - Advantages of using split structure in planning our corporate capital setup.
 Obre agreed to contribute 65k (cash & equipment) in the above manner.
 Nelson agreed to contribute 10k structured in voting common stock.
 Business failed and creditors wanted Obre’s $35k debt contribution to be treated as equity
 Creditors wanted this because shareholders only have a residual claim  only get what is left
over after other debts paid
 If equity  creditors get paid first before Obre gets his money back
 There is more money in the creditor pot if he leaves and is treated as purely an equity owner
 Holding: rejected based on the fact that:
 (1) the corporation had more equity than debt when it started
 (2) the other creditors knew or should have known about this arrangement and can’t
complain now that they don’t like it.
 HYPO
 Why not simply have Obre put in $65k for 50% common stock and nelson $10,000 for
the other 50%
 This could have adverse tax consequences for Nelson as he would be viewed as having
gotten a huge discount; in fact he could be taxed for the phantom $55k extra that he
received
 And is Obre going to like having to split assets equally with nelson if they liquidate?
o No
 HYPO
 Why not give them equal amounts of common stock?
 $10k each but let Obre invest the rest, $55k as preferred stock?
 This isn’t as attractive as splitting some in debt because there are no tax benefits to the
corporation which in turn help common stock because it creates more surplus to
distribute
 Also, as a shareholder, even a preferred shareholder, he takes after other creditors if the
business fails
 HYPO
 Why doesn’t he put the entire remainder of the $55k into debt?
 The risk that this may be viewed as equity instead of debt
 He has now made the corporation start out with more debt than capital
 Also, if the company enters bankruptcy this could maybe trigger that deep rock doctrine
 he could be viewed as using his position to favor his interests and get his claim
equitably subordinated under deep rock doctrine.

E) Planning the capital structure for the closely held corporation


F) Public offerings
 Going public  selling company stock on public markets
o Issued stock before going public  private placements
o Now, it might want to do Initial public offering (IPO).
 Expensive and time-consuming registration process.
 Prospectus filed and approved by the SEC.
 Can be criminal and civil liability for misstatements in the prospectus.
o Pros and Cons:

G) Issuance of shares by a going concern: preemptive rights and dilution


 Dilution
o General Rule: board determines the price at which the corporation will issue stock; they have
substantial discretion in doing so
o HYPO
 Articles of YZ corporation authorize it to issue 2,000 shares. Suppose the business wants an
initial capitalization of $100k. Y and Z—the individuals engaged in the business—agree that each
will invest $50k and each will own ½ of the stock
 1 share @ $50,000 issuance price = $50,000
 10 shares @ $5,000 issuance price = $50,000
 100 shares @ $500 issuance price = $50,000
 YZ corp  100 shares to Y, Z @ $500/share = $50,000+$50,000=$100k
 YZ corp  100 shares to A @ $50/share = $5,000
 Y, Z, and A are now 1/3 owners
 Y paid $50k for his 1/3
 Z paid $50k for his 1/3
 A paid $5k for his 1/3
 Example of dilution  the value of Y and Z’s stock has been diluted buy this low-price issuance
to A; Y and Z are out by 1/3 for someone who paid 1/10 of price.
o Explain to client that their percent (e.g., 51%) is not “locked” unless  Preemptive Right.

 Stokes v. Continental Trust


o Facts
 P has 221 total Shares. Total price on 1/29/1902: 221 * 309.69 = $68,441.49.
 Blair & Co: Offers 5,000 new shares at $450/ share.
 P wants to purchase 221 additional shares for $100 each (par value, what he originally
purchased his 100 shares for).
 Sale goes through: (1) Shares go up to $550/share. (2) By 1904: $700/share.
 P brought suit to compel D to issue to him at par 221 shares of an increase made in its capital
stock or, in the alternative, damages.

o Issue
 Whether Appellant had the legal right to subscribe for and take the same number of share of
the new stock that he held of the old.
o Rule:
 A stockholder has an inherent right to a proportionate share of new stock issued for money only
and not to purchase property for the purposes of the corporation or to effect a consolidation,
and while he can waive that right, he cannot be deprived of it without his consent except when
the stock is issued at a fixed price not less than par and he is given the right to take it at that
price in proportion to his holding, or in some other equitable way that will enable him to protect
his interest by acting on his own judgment and using his own resources.

o Holding
 Yes. The right claimed by Appellant is a right of property belonging to him as a stockholder and
he could not be deprived of it by the joint action of the other stock holders, directors, and
officers of the corporation.

 Preemptive rights
o Preemptive right  right of existing shareholders to maintain her % of ownership by buying stock
whenever there is new issuance of stock for money
 If she chooses not to purchase, her ownership will be diluted but value of stock should not
because corporations must issue stock for fair market value
 If the corporation does not issue stock for fair market value, if they sell it watered down, party
that purchased the stock will have to return it or pay its true value and disgruntled shareholders
will have a CoA against directors for breach of fiduciary duties
o MBCA § 6.30: Shareholders preemptive rights (opt-in)
 (a) the shareholders of a corporation do not have a preemptive right to acquire the corporations
unissued shares except to the extent the article of incorporation so provide
 (b) a statement including the articles of incorporation that “the corporation elects to have
preemptive rights” (or words of similar import) means that the following principles apply except
to the extend the article of incorporation expressly provide
 (1) the shareholder of the corporation have a preemptive right, granted on uniform
terms and conditions prescribed by the board of directors to provide fair and reasonable
opportunity to exercise the right, to acquire proportional amounts of the corporations
unissued shares upon the decision of the board of directors to issue them
 (2) a shareholder may waive his preemptive right. A waiver evidenced by writing is
irrevocable even though it is not supported by consideration
o This is permissive ≠ mandatory  this right exists if the articles provide for them
(i.e. if company opts in)
o Protects investors
o Opt Out (minority)
 “she shareholders of corporation have preemptive right to acquire the corporations unissued
stock except to the extent that the article of incorporation provides otherwise
 Preemptive rights apply unless the articles take them away.
 Company must opt out if they do not want preemptive rights
 Protects the corporation
o Preemptive rights are cumbersome and harmful to the flexibility of the company
to grow and manage its equity.
o **Preemptive right = Right to maintain ownership. Expect fair market value.
 If it is not (FMV), you have a cause of action.
 Most parties do not exercise this right (opt out).
 Dilution is natural. But watered-down stock is bad, that is what creates the cause of action.
o **How does the pie grow? Allow to issue more than you actually do (later sell more for higher
amounts).
 Or amend the bylaws / operating agreement to allow more parties to invest.
 Katzowitz v. Sidler (NY, 1969) – Unfair dilution because selling for less than Market Value.
o Facts
 Katzowitz (plaintiff) and Sidler and Lasker (defendants) are the stockholders, and make up the
board of directors, for Sulburn Holding Corp. In 1961 Sulburn owed each of the three
stockholders $2,500. Instead of collecting this debt, Sidler and Lasker wanted to loan it to
another of their corporations. At a board of directors meeting, Sidler and Lasker proposed
issuing some of the remaining capital stock to the directors at $100 per share in lieu of the
$2,500 owed to each of them.. Sulburn then dissolved.
 Certificate of auth. authorized it to issue 1,000 shares of no par value stock for $100 / share.
 First issuance: 5 shares to Sidler, Lasker, and Katzowitz.
 Second issuance: 25 shares to Sidler (30) and Lasker (3) (Katzowitz didn’t purchase = 5).
 Changed ownership makeup from 33% each to 46% (D), 46% (D) and 7% (P).
 At time of liquidation: surplus that was distributed was $630/ share.
 P got $3,147.59. D got $18,885.52.
 No issue here because P had an equal opportunity to buy and chose not to.
 Issue is about how shares were valued when Sidler and Lasker issued additional shares.
 **Book value was $1,800/ share = $45,000.00 per each (90k total). Sold for 1/18 th of
book value.
 Katzowitz brought suit to recover an equal interest in Sulburn’s assets, less the $5,000 that
Sidler and Lasker paid for their additional shares. The trial court found that Katzowitz had failed
to exercise his preemptive right, and waived his right to purchase the new stock or object to its
sale to Sidler and Lasker. The appellate court affirmed, finding that a disparity between the book
value and the offering price for shares of stock is not in itself sufficient to prove fraud.
o Issue
 Whether under all the circumstances, the additional offering of securities should be condemned
o Rule
 When the issuing price is shown to be markedly below book value in a close corporation and
when the remaining shareholders-directors benefit from the issuance, a case for judicial relief
has been established. In that instance the corporation’s directors must show that the issuing
price falls within some range that can be justified on the basis of valid business reasons.
o Holding
 Yes. The additional offering of securities should be condemned because the directors have not
established a valid business reason for issuing at that price.
 HYPO (dilution/preemptive right)
o X owns 1,000 shares of XYZ corp. There are 4,000 shares outstanding, so X owns 25% of the company
(1,000/4,000). Board says we need more money, so lets issue an additional 1,000 shares. If that happens
and X doesn’t buy 250 shares (25% of the 1,000) of the newly issued shares, what happens?
o X’s percentage interest goes from 25% to 20% upon the sale (1,000/5,0000). But if X has a pre-emptive
right, he could maintain his current percentage by buying his current percentage – i.e. 25% of 1,000
which is an additional 250

H) Distributions by a closely held corporation


 When corporation is profitable, it faces choice of what to do with the money:
o (1) retain earnings (put into business to acquire assets)
o (2) pay money to shareholders (called distribution/dividend)
 Issue: whether there exists a right to dividends
o Ordinarily, why may a court not want to touch the issue of dividends? (Ford v. Dodge)
 BJR  unless directors acting in bad faith or grossly negligent, we don’t want to second guess
decisions
 Ford testimony of “doing social good” = bad faith thus overcomes BJR

 Dodge v. Ford Motor – Whether or not there is a right to a dividend.


o Facts
 Defendant corporation was the dominant manufacturer of cars when this case was initiated. At
one point, the cars were sold for $900, but the price was slowly lowered to $440 – and finally,
Defendant lowered the price to $360. The head of Defendant corporation, Henry Ford, admitted
that the price negatively impacted short-term profits, but Ford defends his decision altruistically,
saying that his ambition is to spread the benefits of the industrialized society with as many
people as possible. Further, he contends that he has paid out substantial dividends to the
shareholders ensuring that they have made a considerable profit, and should be happy with
whatever return they get from this point forward. Instead of using the money to pay dividends,
Ford decided to put the money into expanding the corporation.
o Issue
 Whether Plaintiff shareholders can force Defendant to increase the cost of the product and limit
the money invested into expansion in order to pay out a larger dividend.
o Rule
 Courts should not interfere in the management of directors unless it is clearly made to appear
that they are guilty of fraud or misappropriation of the corporate funds, or refuse to declare a
dividend when the corporation has a surplus of net profits which it can, without detriment to its
business, divide among its stockholders, and when a refusal to do so would amount to such an
abuse of discretion as would constitute a fraud.
o Holding
 Plaintiffs are entitled to a more equitable-sized dividend, but the court will not interfere with
Defendant’s business judgments regarding the price set on the manufactured products or the
decision to expand the business. The purpose of the corporation is to make money for the
shareholders, and Defendant is arbitrarily withholding money that could go to the shareholders.
 Court orders dividends to be paid. (1) Amt of surplus was huge. (2) Ford sealed own fate with
testimony (to do social good). Ford failed to even offer evidence of real business justification,
other than to just retain money.
 Notably, Ford did not deny himself a large salary for his position with the company in order to
achieve his ambitions. However, the court will not question whether the company is better off
with a higher price per vehicle, or if the expansion is wise, because those decisions are covered
under the business judgment rule.
 ** Exception to the rule.

 Redemptions/Repurchases
o If the corporation requires a proportional part of each shareholders stock, the result is the equivalent of
a dividend with each shareholder getting a pro rata distribution
o HYPO
 Issue: what if the corporation redeems (or has each shareholder sell back) 10 shares @
$1k/share; has the ownership changed at all?

 The 30 shares now owned by the corporation ≠ assets  it has no value right now;
might at some point be issued to new investors
 Effect of transaction  corporation is poorer by amount of consideration paid for the
shares ($10,000) but shareholders are that much richer ($10,000) so, it is like a dividend
o This is fine and normal especially if a corporation began with only a certain
amount of shares and they are all issued but it wants additional investors to join
o Problems occur when it is not proportional and not offered to all the
shareholders.
o Result is to increase the proportional ownership of those shareholder with their
stock that is not being re-purchased.
 Issue: what if corporation only repurchases Olivia’s 100 shares for 100k?
 Olivias ‘dividend’ not shared with everyone so it is a disproportionate dividend.
o O gets money (100k).
o H/C get more interest in the corporation (50/50 owners).
 Issue: Why would H/C be upset?
 (1) even though he is now a 50/50 owner, $100k left the company to buy Olivia out
 (2) maybe H/C wanted same return, but he wasn’t given the option to sell back to the
corporation and because company is not public he may be holding liquid stock
 Issue: how did CL deal with this issue
 Equal opportunity doctrine  in closely held corporations, a corporation that is
offering to buy back or redeem 1 shareholders stock, it must make the same
proportional offer to others
o Can get around this by operating agreement/bylaws (drafting).
I) Legal restrictions on distributions
 1) Model Business Corporation Act
o MBCA § 6.40: Distributions to shareholders
 (a) a BoD may authorize and the corporation may make distributions to its shareholders subject
to restrictions by the articles incorporation and the limitation in subsection (c)
 (c) no distribution may be made if, after giving it effect
 (1) the corporation would not be able to pay its debts as they become due in the usual
course of business; or
o Equity solvency test  looks to whether the corporation is insolvent in the
equity sense / whether the distribution makes it so the corporation can no
longer meet its obligations as they become due.
 (2) the corporations total assets would be less than the sum of its total liabilities plus
(unless articles permit otherwise) the amount that would be needed, if the corporation
were to be dissolved at the time of the distribution, to satisfy the preferential rights
upon dissolution of shareholders whose preferential rights are superior to those
receiving the distribution
o Balance sheet test  simply looks at the company’s balance sheet, it aims to
see that, if on the balance sheet, assets < liabilities then we cannot issue
distributions.
 Simple formula to recognize: Assets – liabilities = owners’ equity (how
you are getting distributions and dividends)
 Subsection (c) creates a statutory legal limit on when a distribution may be paid.
 If the distribution fails either of the two then no distributions may be made. Distributions made
despite failing = illegal, creates CoA.
 2) Non-Model Act Statutes
o Other states consider other tests.
 **Debt is not always bad. (Why being “debt-free” is not the best).
o(1) A business wants to make its money work for the business. If you can borrow low and invest high.
o(2) Encumbered assets (e.g., bank having a lien on a car) protects them from aggressive creditors taking
those assets (e.g., on tort actions). Protects those assets.
 Allow bank to take security interest (security agreement) in your assets.
o (3) Completely debt free = you have spent it all.
 Given up money to be debt free.
 Not a lot for exciting projects, and not having enough to pay salaries.
o (4) Tax treatment. Interest payments are deductible.
 Subtract from the bottom line = reduces taxes.
 Common for a Shareholder to not only invest and take interest, but also be a lender (creditor) to same corp.
o No capital gains when you are paid back (as creditor)  not income = not affecting the bottom line.
o Not affecting bottom line = tax benefit of being creditor while being shareholder.
o *Taxed on your dividends as a Shareholder.
 *Also being a creditor = will not be taxed on paying back with interest (not capital gains).

Chapter 8 – Management and control of the corporation


A) The traditional roles of shareholders and directors
 Traditional Model
o Shareholders  run the corporation but don’t manage it / elect BoD
 Ultimate owners
 Passive  no management because not elected as officer or BoD
o BoD  manages the corporation / makes policy decisions for the business / elects’ officers
 Act as body/group (along with shareholders)
o Officers  implement board policy choices
 do not operate in groups
 agents of the corporation  officers can bind the corporation as long as they have requisite
agency authority. President, vice president, etc.
 Duties
o Shareholder duties
 (1) elect and remove directors by vote
 Removing for cause
 Rights to change makeup depending on voting power you have
 (2) in most states, directors adopt bylaws / majority of shareholders have power to amend
bylaws
 (3) ability to vote on a make fundamental changes (mergers, amendment to articles)
 Publicly held corporation Shareholders never participate on a day-to-day basis
o Directors duties
 (1) Governing body of corporation
 (2) Elected by shareholder to establish corporate policy
 (3) Appoint executive officers
 (4) Make major and business and financial decisions
 Initial BoD named in the articles and after that, shareholders will elect directors 1/yr
@ annual meeting
o Officers duties
 Appointed by the board (NOT shareholders)
 Officers have actual and apparent authority to bind the corporation
 Corp Formal Requirements
o (1) must have board meetings with notice w quorum and voting requirement
o (2) shareholder meeting with similar requirement
 Formal requirements inconsistent with managerial flexibility of a partnership
 Today – those who structure closely held corporations can structure management however they
can never abolish BoD and seize management themselves
 McQuade v. Stoneham - Case Brief
o Facts:
 In 1919, Plaintiff and Defendant John McGraw each purchased 70 shares of NEC stock from the
majority 1,306 shares that Stoneham owned. NEC was the company that owned the New York
Giants. At the time of purchase, the parties agreed to do everything in their power to keep
Stoneham as president, McGraw as vice-president and Plaintiff as treasurer. Plaintiff and
Stoneham had a number of conflicts and in 1928, the 7-member board of directors of NEC voted
in a new treasurer (McGraw and Stoneham abstained from the vote). Plaintiff was not removed
for any misconduct or ineptitude, but rather for his conflicts with Stoneham. Plaintiff brought
this action to be reinstated as treasurer, and he cited the agreement that he entered with
McGraw and Stoneham that provided for each of them to use their “best endeavors” to keep
each other in their respective positions.
o Issue:
 Whether the shareholder agreement between Plaintiff and Defendants to use their best efforts
to keep each of the parties in their respective positions is valid.
o Rule:
 Shareholders can not form an agreement to control the decisions traditionally vested in the
judgment of the directors of a company.
o Holding:
 The Court of Appeals of New York held that the shareholder contract to keep the parties in their
positions within NEC was invalid as a matter of public policy. Shareholders should not be able to
usurp director decision-making, and directors should be beholden to the corporation and not
the shareholders. Stoneham may have exercised bad faith, but the director’s intentions are
irrelevant because the court does not want to put directors in a position wherein they would
have to defend future decisions. Creditors and minority shareholders could be hurt. Directors
must have unfettered discretion to do what is best for the corporation. Should not be abdicated
by some side agreement.

 Galler v. Galler - Case Brief


o Facts:
 Plaintiff’s late husband and his brother, Isadore Galler, owned all but 12 shares of a close
corporation, Galler Drug. The brothers, in an effort to provide for their families if something
were to happen to either brother, entered a shareholder agreement that would guarantee
that their spouses would be elected to the board and that each would have equal
representation on the board. The agreement also provided an annual payout to the spouses.
There was no set expiration date of the agreement provisions. After Plaintiff’s spouse’s death,
Defendants tried to destroy all copies of the agreement. Plaintiff sued to review the agreement
in order to enforce the provisions therein. Defendant argued that the shareholder agreement
was unenforceable because it violated state statutes that render invalid shareholder
agreements that seek to control management decisions.
o Issue:
 Whether the shareholder agreement between the majority shareholding brothers was invalid
per statute or public policy.
o Rule:
 A shareholder agreement, particularly in closed corporations, that controls the voting for board
members and the members’ management decisions, should nevertheless be enforced as long as
the agreement is not fraudulent or harmful to the public.
o Holding:
 The agreement was valid and Plaintiff should be entitled to specific performance and money
that was owed under the agreement. Galler Drug was a closely held corporation, and therefore
subject to different circumstances than a shareholder of a large corporation. The court cited a
number of prior cases, including Dodge v. Clark, to support the premise that because this
agreement did not harm the public and was fair to the parties of the agreement, there is no
offense to any public policy concerns.
 Zion v. Kurtz Case Brief
o Facts:
 The appellant held all of the Class A stock in Group while appellee owned all of the Class B stock.
The appellant and the appellee executed a shareholders’ agreement, before appellant bought
shares in Group, that no decisions would be made by Group without the consent of the
holders of the Class A stock. The appellee promised in the agreement that he would take all
steps necessary to fulfill the purpose of the shareholder agreement.
 Group was not formally registered as a close corporation and the articles of incorporation did
not mention the shareholder agreement. The appellee approved two agreements without the
consent of the appellant. The appellant brought suit to cancel the two agreements on grounds
that they violated the shareholders’ agreement.
o Issue:
 Whether a shareholder agreement that gives management power to shareholders is
enforceable even though the corporation has not met all of the formal statutory requirements.
o Rule:
 An agreement between shareholders of a Delaware corporation that requires the minority
shareholder’s consent before conducting business is enforceable, even though the corporation
has not met all of the formal statutory requirements.
o Holding:
 Yes. A shareholder agreement that gives shareholders management powers is enforceable even
though corporate formalities have not been followed, because stockholders may manage close
corporations. The agreement should not be unenforceable solely because the corporation had
not completed the steps to become a close corporation. The appellant promised to take all
reasonable steps to carry out the purposes of the shareholder agreement which included giving
the shareholders management power. Form over substance issue.

 Matter of Auer v. Dressel Case Brief


o Facts:
 The Stockholders of Hoe & Co. Inc. (P) sought a special stockholder’s meeting after more than
50 percent of the required voting class compelled the President (D) to call for one. The
President (D) refused claiming he didn’t have the information to verify authenticity of the
required number. He also claimed the meeting was improper, and did not follow four stated
purposes for meetings. (1) a purpose for reinstating the former president, (2) a purpose to
amend the by laws so that vacancy on the board be filled up only by those stockholders that
director represented, (3) to vote upon charges against four specific directors, (4) to amend the
by laws regarding the number of directors needed for a quorum. The court ordered the
President (D) to call the meeting, and he subsequently appealed.
o Issue:
 Whether corporate management must call a special stockholder’s meeting when enough voting
shares back such a vote and if the purpose of the meeting is proper?
o Rule:
 Corporate management must call a special stockholder’s meeting when enough voting shares
back such a vote and if the purpose of the meeting is proper.
o Holding:
 Yes. Corporate management must call a special stockholder’s meeting when enough voting
shares back such a vote and if the purpose of the meeting is proper. In this case, there is no
reason why the stockholders could not show their approval of the corporation’s former
president. For purpose (2), stockholders who already have the power to elect directors also
inherently have the power to remove them with cause. In addition, stockholders of one class
can influence the vacancy of another director-less class when that director only represents one
particular class. Stockholders have an important right to have such meetings which would be of
little practical value if corporate management can ignore their requests, forcing lengthy and
costly legal proceedings when knowledge of stockholdings and rights are already known by the
corporate officers. Affirmed.

B) Shareholder voting and agreements


 MBCA § 7.32: Shareholder Agreements
o (a) an agreement among the shareholders of a corporation that complies with this section is effective
among the shareholders and the corporation even though it is inconsistent with one or more other
provisions of this act in that it
 (1) eliminates the board of directors or restrict the discretion or power of the board of directors
 (3) establishes who shall be directors or officer of the corporation, or their terms of officer or
manner of selection or removal
 (8) otherwise governs the exercise of the corporate powers or the management of the business
and affairs of the corporation the relationship among the shareholder, the directors and the
corporation, or among any of them, and is not contrary to public policy
 Ways in which shareholders may usurp board of directors power including eliminating
or restricting power
o (b) an agreement authorized by this section shall be
 (1) set forth (a) in the articles of incorporation or bylaws and approved by all person who are
shareholders at the time of the agreement or (b) in a written agreement that is signed by all
person who are shareholders at the time the agreement is made known to the corporation
 (2) valid for 10 years unless the agreement provides otherwise
 Permits these restricts as long as you put people on notice that this is how corporation
will operate
 Shareholders do not have to do any of this (restrict board power) / they can retain a
board structure but can also divide up management
 MBCA § 7.07 Record Date
o (1) the bylaws may fix or provide the manner of fixing the record date for one or more voting groups in
order to determine the shareholders entitled to notice of a shareholder meeting, to demand a special
meeting, vote, or to take any other action. If the bylaws do not fix or provide for fixing a record date, the
board of directors of the corporation may fix a future date as the record date
o (b) a record date fixed under this section may not be more than 70 days before the meeting or action
requiring a determination of shareholders
 Interim period allows corporation to give proper notice of meeting, prepare voting list, establish
who is entitled to vote, permits management to solicit votes informally
o Record Date (registering shareholder of record date)  if registered shareholder, on the record date,
you are entitled to notice (when/where/why) / entitled to vote on matters / right to receive declared
dividends
 If record owner transferred stock  It should get recorded so new shareholder receives notice
and can vote
 In large publicly held corporations  usually done by brokerage
o Issue: who gets to vote?
 Person shown as the owner of the stock in the corporate records
 Record date  date picked that decides who gets to vote on a certain matter
 How do we pick/
 MBCA 7.07
o Shareholders (how they act)
 Generally individual shareholders do not have owner to do anything on behalf of the
corporation
 Publicly Held vs. Closely Held – p. 306.
o In a publicly held, ownership and management are usually vested in different persons or groups.
o Closely held: majority shareholders are likely to be the CEO and possess the ability to name or remove
directors virtually at will.
o Quorum Requirement
 Can also act through written consent
 Modern rules allow shareholders to do a unanimous written consent on certain matters so that
the don’t have to waste time having a physical meeting
 2 types meetings
 Annual meeting  elect directors/requirements by corporation
 Special meeting  any other meeting usually to vote on something important that the
company is going to do
 [both types of meetings require] written notice to all shareholders entitled to a vote
 Statutes set this time but most say notice must be given out not fewer than 10 and not
more than 60 days before
 Notice must state time/place/purpose of meeting
 Statement of purpose limits the actions that shareholders can take at meetings
 Failure to give proper notice renders voidable any action that was taken at the meeting
unless any shareholder not notified waives the defect
o True whether or not the shareholders vote would have affected the outcome
or not
 Quorum (=majority of the shares entitled to vote)
o MBCA § 7.25: Quorum and Voting requirements for voting groups
 (a) shares entitled to vote as a sperate voting group may take action on a matter at a meeting
only if a quorum of those shares exists with respect to that matter. Unless the articles of
incorporation or this act provide otherwise a majority of the votes entitled to be case on the
matter by the voting group can constitute a quorum of what voting group for action on that
matter
 (c) if a quorum exits, action on a matter (other than the election of directors) by a voting group
is approved if the votes cast within the voting groups favoring the action exceed the votes cast
opposing the action, unless the article’s of incorporation or this act require a greater number of
affirmative votes
 Determination of a quorum focuses on the number of shares represented, not the
number of shareholders
 Unless article provides otherwise, each outstanding share is entitled to 1 vote and a
quorum consists of a majority of outstanding voting shares i.e. 51% of outstanding
shares
o HYPO
 A corporation has 10K outstanding shares and 500 shareholders
 Issue: what do we need for a quorum
 5,001 shares represented at the meeting  the amount of shareholders is irrelevant;
strength is measured by ownership not people when it comes to voting (person with
160 shares has 2x as many votes as person with 50 shares)
 Note: note unusual for the articles of incorporation to require greater quorum to take
action on important matters
o Ex) corpo could require that a “super majority” such as 2/3 of shares be
represented to constitute a quorum
 Issue: Assuming voting requirement is met, what vote is require to pass a resolution; i.e. to
approve act by shareholder?
 Vote required y shareholders to take action usually depends on what issue is being
considered
o Quorum and voting requirements for voting groups
 Pay attention to verbiage. ROUTINE matter, majority of shares present.
 Quorum should not change.
 Shares present out of 50. 26 = quorum.
 Electing directors = plurality (majority)
 Corporation has 20,000 outstanding voting shares, all of which are represented at a
meeting to elect directors. Four candidates are nominated for one seat on a board.
 A – 6,000 votes (wins even though only received 30% of vote)
 B – 5,000 votes
 C – 4,800 votes
 D – 4,200 votes
 Routine matters  majority of shares present
 Corporation has 20,000 outstanding shares. 12,000 are at a meeting (we have quorum).
Of those 12,000, 6,001 vote to pass a measure and 5,999 vote not to.
 That action can now go forward since a majority of the present shares voted yes
 Removing directors = majority of the shareholders entitled to vote
 Corporation has 20,000 outstanding shares. We would need 10,001 to vote yes, since
that would be a majority of the vote
 Fundamental changes = super majority of shares entitled to vote (i.e. 2/3)
 Corporation has 20,000 outstanding shares. We would need 12,334 shares to approve
the change
 Can contract (draft around) around all the default rules.
 The key is communication with the client.
 Communication up front.
 Basic Concepts
o Proxy: one who is authorized to act as a substitute for another. In corporate law, a person who is
authorized to vote another’s stock shares
o Proxy contest: a struggle between two corporate factions to obtain the votes of uncommitted
shareholder
 Note: a proxy contest usually occurs when a group of dissident shareholders mounts a battle
against the corporations managers
 Shareholders can appoint an agent to cast their vote
 Supposed to be put into writing
 Must be signed by record shareholder
 Needs to be language offering another to vote on the shares

 Voting Concepts *Only applies when voting for Directors*


o Straight (noncumulative) voting: a corporate voting system in which a shareholder in board elections
can vote his/her total shares for each candidate.
 The result is that a majority shareholder will elect the entire board of directors
 This is the most simply voting scheme
 Each director position filled in a different election within a meeting
 Get to vote all shares for each election
o Cumulative voting: a system in which each voter may allocate his/her shares among the candidates.
 This voting helps a minority elect at least one representative
 It is common in shareholder elections
 Does not apply when shareholders are voting on other matters (approving fund. change).
o Explain to a client the benefits / detriments of each .
 Duty to explain the difference between straight and cumulative.

o HYPO
 Olivia = 60 shares
 Brady = 40 shares
 Olivia wants to per herself, A, B, C, and D on the BoD (5 total directors)
 Brady wants to put himself, E, F, G, and H on the board (5 total directors)

Here – If Olivia does not think about her votes, she only gets 2 of the 5 total spots.
Here – Olivia can sneak in and steal a seat that Brady could have won, had he allocated his 200 to himself and E.

 Salgo v. Matthews Case Brief


o Facts:
 There was a proxy contest at General Electrodynamic’s Corporation and an Inspection
Supervisor was appointed. The insolvent Pioneer Casualty Company held a large block of
General Electrodynamics Corporation’s stock. The Texas trial court appointed a receiver and
ordered him to surrender the proxy to Pioneer Casualty Company, instructing him to deliver the
proxy to Appellee. The inspector of elections invalidated the proxies and votes prompting
Appellee to file suit.
o Issue:
 Whether the court’s order was proper and must be recognized by the company.
o Rule:
 An election inspector has discretion to determine the validity of proxies.
o Holding:
 No. Injunctive Relief was improper since an election inspector’s discretion to determine the
validity of proxies is not subject to judicial control.
 Humphrys v. Winous Co Case Brief
o Facts: Director 1  Year 1. Director 2  Year 2. Director 3  Year 3.
 Appellant has the minimum number of three directors and their terms of office are for three
years, one to be elected each year. This classification of three directors into three classes each
containing one director effectively divests the minority shareholders of the ability to elect one
member of the board through cumulative voting. Appellees brought suit claiming that this
classification restricted the right their right to vote cumulatively as specifically guaranteed by
Section 1701.58 and therefore the classification was invalid.
o Issue:
 Whether the classification of three directors into three classes each containing one director is
invalid because it restricts the right to vote cumulatively.
o Rule:
 Minority shareholders are granted only the right of cumulative voting but are not necessarily
guaranteed the effectiveness of the exercise of that right to elect minority representation on the
board of directors.
o Holding:
 No. This classification is not invalid because Section 1701.58 grants a right that may not be
restricted or qualified rather than one ensuring minority representation on the board of
directors.
 MBCA § 8.06: Staggered Terms for Directors
o The articles of incorporation may provide for staggering the terms of directors by diving the total
number of directors into two or three groups, with each group containing 1/2 or 1/3 of the total, as near
as may be
 Note; corporations do NOT have to offer cumulative voting as a right
 MBCA § 7.28: Voting by Directors; Cumulative voting
o (a) unless otherwise provided in the articles of incorporation, directors are elected by a plurality of the
votes cast by the shares entitled to vote in the election at a meeting at which a quorum is present
o (b) shareholders do not have a right to cumulate their votes for directors unless the articles of
incorporation so provide
 Opt-in  corporations use straight voting in all cases UNLESS they opt out
 MBCA § 8.08 Removal of Directors by Shareholders
o (c) if cumulative voting is authorized, a director may not be removed if the number of votes sufficient
to elect him under cumulative voting is voted against is removal. If cumulative voting is not authorized,
a director may be removed only if the number of votes cast to remove him exceeds the number of votes
cast not to remove him
 Minority votes that get director elected in first place are enough to keep director in place 
backstop to general voting scheme

 Methods for combining voting power/management


o Issue: why would we want to combine voting in a document?
 Shareholders may want to pool voting power to get results they want
 Can combine voting power in two ways
 (1) voting trust  effective but cumbersome.
 (2) voting agreement  easy but not effective in some states.
o (1) Voting Trust  a trust used to hold shares of voting stock in a coronation and empowering the
trustee to exercise the right to vote.
 In a voting trust: Shares are transferred.
 Note: the trust acts as custodian of the share but is not a shareholder
 MBCA § 7.30: Voting Trust
 Follow 4 requirements:
 (1) Written trust: must be written trust agreement (controlling how shares are used)
 (2) Has to be transfer or state in document that legal title of shares are being
transferred to voting trustee
 (3) transfer of legal title has to be recorded with corporation / corporation must receive
written notice and must be recorded that trustee is in place that is holding votes of
certain group that wants to vote in certain way
 (4) original shareholders have to receive trust certificates that shows that they have
given away right to vote to trustee in certain way and that they retain all shareholder
rights except for voting
o (2) Voting agreement  shareholder simply agree to vote in a certain way
 Requirements
 (1) must be written
 (2) singed by shareholder
 Really just a contract between shareholders. Court will not force a party to vote her shares in
accordance with an agreement.
 MBCA § 7.31: voting Agreements
 (a) two or more shareholders may provide for the manner in which they will vote their
shares by signing an agreement for that purpose. A voting agreement created under
this section is not subject to the problems of section 7.30
 (b) a voting agreement created under this section is specifically enforceable
 Issue: why are we concerned with shareholder forming agreements
 Concern that they will end up acting not in best interest of the corporation
 Separating votes from ownership interests can do this
 Issue: why would shareholders want this arrangement
 Permits minority shareholders to ban together for greater protection
 Gives minority shareholders predictability

 MBCA § 6.27: Restriction on Transfer of Shares and other Securities


o (a) the articles, bylaws, an agreement among shareholders, or an agreement between shareholders and
the corporation may impose restrictions on the transfer or registration of transfer of shares of the
corporation
 Generally, ownership interest is freely transferrable (opposite than partnership)
 If A sells interest to B, B becomes a shareholder  can vote at meetings, propose candidates for
elections, inspect books, bring derivative suit
o (b) a restriction on the transfer or registration of transfer of shares is valid and enforceable against the
holder or a transferee of the holder if the restriction is authorized by this section and its existence is
noted conspicuously on the front or back of the certificate or is contained in the information statement
required by section 6.26(b). Unless so noted, a restriction is not enforceable against a person without
knowledge of the restriction.
 Adds ppl trying to claim they are a BFP/because on stock certificate it would have notice.
 A restriction is NOT enforceable against a person without knowledge of the restriction.
 Publicly Held vs. Closely Held Corporation .
o In a publicly held corporation, can simply sell the stock.
o Closely-held: Free transferability of stock runs the risk of disrupting the relationship among original
proprietors. less of a market. “Not” a market for it.
 Still a risk that a shareholder may go out and try to sell.
 Default rule: Pick your partner rule.
 X, Y, and Z form XYZ closely-held corp. Each owns 100 of 300 issued shares of stock & is a
director.
 If X sells to A, A will become a full-fledged shareholder. Attend meetings, propose candidates,
and bring derivative suits.
o Issue: how do you avoid transferability problems (ways can get around pick your partner rule in closely
held corp)
 Put stock transfer restriction in bylaws or articles of incorporation
 Stock transferability restrictions upheld as long as they are reasonable (i.e. right of first refusal,
buy-sell arrangement)
 Right of First Refusal: Opportunity to meet the best price the shareholder has been able to
obtain from outsiders.
 How to Avoid this problem: Put a stock transfer restriction. Will be upheld, so long as they are
reasonable. May see a typical right of first refusal (chance to meet the best legitimate price).
 Sometimes: complete restriction on type of class. Ways to get around the pick your partner
rule that we do not have in closely-held corporations.
o Share Transfer restrictions
 Reasons why restrictions aren’t generally used in closely held corporation
 (1) close corporation owners may be unsophisticated
 (2) close corp owners are often relatives or close friends, or business associates in
context
 (3) investors and entrepreneurs are afraid to introduce the topic of protective
agreement; and
 (4) the difficult in foreseeing future circumstances
 Ringling Bros. Barnum & Bailey v. Ringling Case Brief
o Facts: 1. Haley (315), 2. Ringling (315) – Enter into pooling agreement. 3.North (370).
 Ringling (P) and Haley (D) entered a stock pooling agreement by which they agreed to always
vote their shares together. They made this agreement because it would give them enough
voting strength to elect five of the seven directors. In 1946, Ringling (P) and Haley (D) agreed to
elect themselves and family as directors, but could not agree on a suitable fifth director. There
was arbitration, and the arbiter, Loos, found that Ringling’s proposed director, Dunn, should be
Haley’s (D) vote. Haley (D) refused, and abstained from the fifth vote. Without enforcement of
Loos’ decision, one of North’s candidates would instead become fifth director. Ringling (P)
brought suit for enforcement, while Haley (D) contended that their voting agreement was illegal.
The court found that the agreement was valid, and a new election must be held upholding the
voting agreement. Haley (D) appealed.
o Issue:
 Is it lawful for groups of shareholders to contract the way they vote with each other in the
future? What was the remedy?
o Rule:
 It is lawful for groups of shareholders to contract the way they vote with each other.
o Holding:
 Yes. It is lawful for groups of shareholders to contract the way they vote with each other. The
agreement in question didn’t give Loos power of enforcement. His decision is only enforceable
by parties. The agreement was also not illegal, the law does not intervene with agreements
between shareholders binding how they vote with one another. Pooling agreements have
previously been found valid and distinguished from voting trust. The provision for an arbiter was
clearly intended to break deadlocks. This agreement did not allow parties to take unlawful
advantage of one another or other persons. For remedy, the only valid votes should be the ones
made by Ringling (P) and North (votes cast by breaching party should be ignored = only 6 people
were elected) Though it leaves one board vacancy, the board can fill the vacancy on its own at
the next meeting which is close by anyway. As modified, affirmed.
 Allows the agreement, but problem with remedy (should have allowed specific performance).

 Brown v. McLanahan Case Brief


o Facts: Happened through plan of reorganization. Overall goal: get all creditors to approve bankr plan.
 In 1935, debentures and preferred stock of the corporation were issued to lenders during the
corporation’s bankruptcy. Only the preferred and common stock (issued to previous common
stockholders and unsecured creditors) had voting power during this reorganization. Plan
established a 10-year voting trust, all stock issued to 8 voting trustees in exchange for voting
trust certificates to those parties entitled to shares. Voting rights were to revert at termination
of trust. At the time of the reorganization, debenture holders and preferred stockholders were
the same, later transfers of the preferred stock made them into two separate classes. The
trustees (D) held substantial amounts of debentures and were also a majority of the
corporation’s directors after having voted themselves in. Near End of trust: afraid of losing
power with the termination of the trust, the trustees (D) voted to amend the corporate charter
(without notice). The amendment granted debentures voting power and diverted control from
the preferred shareholders. A holder (P) representing 500 preferred shares brought forth a class
action to dismiss the amendment.
o Issue:
 May trustees under a corporate voting trust agreement exercise their voting rights to the
detriment of the actual owners of the shares held in trust?
o Rule:
 Under corporate voting trust agreements, trustees may not exercise power to the detriment of
the actual owner of the voting share, nor may the trustees of different classes favor one class
over the expense of another class.
o Holding:
 No. Under corporate voting trust agreements, trustees may not exercise power to the detriment
of the actual owner of the voting share, nor may the trustees of different classes favor one class
over the expense of another class. The voting strength attaching to shares of stock is equivalent
to a property right as any element of dominion possessed by owners of realty through this
rationale, a trustee should be allowed to divert strength of the voting share. Here, the trustees
(D) diluted the voting power of preferred shares toward their own debentures in order to
maintain control of the corporation. This action was beyond their authority, the amendment is
void. Reversed and remanded. *Calls it Breach of fiduciary duty. Improper to take power away
that was granted to them.

 Leherman v. Cohen Case Brief


o Facts: Cohen (AC)  2 directors. Lehrman (AL)  2 directors. Danzansky  West (AD)  1 director.
 Giant Food Inc., (Giant), is controlled by the Lehrman and Cohen families each owning equal
voting stock designated as Class AC (held by the Cohen family) and Class AL (held by the
Lehrmans). Cumulative Voting: Each class is entitled to elect two members of Giant’s four-
member board of directors. A dispute arose within the Lehrman family and to end the dispute,
an arrangement was made permitting Plaintiff to acquire all of the Class AL stock. In addition,
the arrangement established, and the stockholders unanimously ratified, a fifth directorship to
resolve a dead lock which would have continued if the equal division of voting power between
the AL and AC stock were to continue. A third class of stock was created called AD stock that had
the power to elect one director but was not entitled to dividend or liquidation rights except
repayment of par value.
 By resolution of the board of directors, the third class of stock was issued to Joseph B.
Danzansky who elected himself as Giant’s fifth director. In 1964 the holders of AC and AD stock
voted together against the holders of AL stock to elect Danzansky president of Giant replacing
Defendant who had been president of Giant since its incorporation. Danzansky then resigned as
director and elected in his place, Millard F. West, Jr., a former AL director. Plaintiff then brought
this suit.
o Issue:
 Whether the Class AD stock arrangement is illegal as a voting trust. Does it deprive AL or AC of
their voting rights?
 In creating a new class of voting stock, does it dilute and separate the voting rights, which
normally remain vested in the stockholders who made that class, from other elements in the
ownership of said stock?
o Rule:
 The test for a de-facto voting trust is whether (1) the voting rights of the stock are separate from
the other attributes of ownership; (2) the voting rights granted are intended to be irrevocable
for a definite period of time; and (3) the principal purpose of the grant of voting rights is to
acquire voting control of the corporation.
o Holding:
 No. AD’s stock does not separate the voting rights of the two other stocks from the other
elements of ownership. The other classes each retained complete control over their stock. Even
with voting power dispersed, Lehrman’s (P) assertion that the percentage of reduction created a
voting trust is still wrong. The first test of a voting trust is the separation element, which has not
happened here [why does this matter? disrupts normal efficient operation of corporations]. The
10 year period contention was not met here since it was never a trust. Lehrman (P) still claims
that even if not a voting trust, the AD stock is still illegal because it had only voting rights and no
participating ownership interest which goes against Delaware law. This is not a requirement in
Delaware law, see Statute 218, and nonvoting stock is directly authorized in Statute 151 (a). The
usually purpose of voting trust statutes is to avoid the plotting of stockholders seeking to
acquire voting control to the determinant of other stockholders. There is nothing wrong with
the intent of the AD agreement to break deadlocks. Affirmed.

 Ling and Co v. Trinity Sav Case Brief


o Facts: Bowman pledges stock as collateral to Trinity. Trinity wants to foreclose on stock to compel Ling
to transfer.
 Bruce Bowman (defendant) owed money on a promissory note to Trinity Savings and Loan
Association (plaintiff). Bowman pledged a certificate for 1,500 shares of common stock in Ling &
Company, Inc. (defendant) to secure payment on the note. Ling was formerly a brokerage house
member of the New York Stock Exchange. Trinity brought suit against Bowman to collect on the
balance owed on the promissory note, and to foreclose on the stock certificate pledged as
security. Ling objected to the foreclosure and public sale of the stock based on restrictions on
transferring stock set out in Ling’s articles of incorporation. The articles require a stockholder to
obtain written approval from the New York Stock Exchange, and to give the corporation and
other stockholders an opportunity to purchase, prior to selling or encumbering the stock. No
offer of sale was made to other stockholders, and Bowman’s pledge of stock was not approved
by the New York Stock Exchange. Bowman’s stock certificate refers on its face to restrictions
that are described on its reverse side. On the certificate’s reverse side, it refers to the provisions
in the articles of incorporation that restrict any transfer or encumbrance without first offering
an option to purchase the shares to the corporation and other stockholders. Trinity claimed that
the restrictions on the transfer of stock contained in Ling’s articles are invalid. The trial court
granted summary judgment for Trinity, foreclosed the security interest in the stock, and ordered
the stock sold. The court of appeals affirmed, holding that the restrictions included in Ling’s
articles were invalid because there was no conspicuous notice of them on the stock certificate,
and because the restrictions on stock transfer were unreasonable.
o Issue:
 May corporations place limitations upon the transfer of their stock?
o Rule:
 Corporations may impose restrictions upon the transfer of their stock so long as they are
reasonable restrictions. (1) Conspicuous, and (2) Reasonable.
o Holding:
 Yes. Corporations may impose restrictions upon the transfer of their stock so long as they are
reasonable restrictions. As the court of appeals noted, these restrictions must be laid out in the
charter, and on the stock certificate themselves, conspicuous and present. Ling (D) did not place
reasonable notice of this, but this deficiency does not entitle Trinity (P) to summary judgment
sans conclusive proof of the fact that Trinity (P) lacked actual notice of the limitations on
transfers. The lower court found unreasonably that Ling’s (D) restrictions were unsound. The
provisions requiring right of first refusal and NYSE approval are not unreasonable. The court of
appeals was in error for finding that the right of first refusal violated a statute of buy and sell
agreements only permitting such a limit if there were 20 or fewer shareholders. Ling’s (D)
restriction (D) pertained to options, inapplicable to buy and sell agreements. Since the lower
courts lack valid reasons to invalidate Ling’s (D) transfer restrictions, the order granting
summary judgment for Trinity (P) is reversed and the case remanded.

C) Action by Officers
 MBCA § 8.01: Req for and duties of BoD
o (a) except as provided in section 7.32, each corporation must have a BoD
o (b) all corporate powers shall be exercised by or under the authority of, and the business and affairs of
the corporation managed by and under the direction of, its BoD, subject to any limitation set forth in the
articles of incorporation or in an agreement authorized under section 7.32
 Need 1 or more directors/starts with incorporators picking them and shareholders electing each
year
o In the Matter of Drive-In Development Corp Case Brief
 Facts:
 The parent company of Drive-in (D), Tastee Freez Industries, Inc. (D) wanted to borrow
money the National Boulevard Bank of Chicago (the Bank) (P). The Bank (P) agreed on
the condition that Drive-in (D) guarantee the loan. Maranz, as chairman of Drive-in (D)
(and Dick as secretary attested to it) was willing to execute a guaranty, but the Bank (P)
also requested a copy of resolution from D board of Director proving they authorized
Maranz to sign on behalf of Drive-in (D). The resolution never appeared in Drive-in’s (D)
books (meeting minutes) leading to doubt whether the Drive-in (D) board ever adopted
it. **Corp resolution is how board takes an act at a meeting / and is proof of it. Must
have (1) meeting that satisfies quorum requirements (maj. of directors present), or (2)
unanimous agreement in writing to pass a resolution. P called on D to pay up under
guaranty, D argues not authorized = guaranty is null and void The referee dismissed the
claim, alleging that Maranz lacked binding authority to make the agreement. From
judgment confirming the referee’s decision, the Bank (P) appealed.
 Issue:
 Whether it bound the company without director approval.
 Rule:
 If made while in the scope of his authority, statements of corporate officers and binding
upon the corporations.
 Holding:
 No, so long as the officer is authorized to do so by the board. This is not a part of the
ordinary course of business to enter into guarantees of payment.
 BUT Here: Yes. If made while in the scope of his authority, statements of corporate
officers and binding upon the corporations. It is the secretary’s duty to maintain records,
and it was within Dick’s authority to certify adoption of the resolution. The Bank (P) has
no further duty of inquiry once Dick stated that Maranz had authority. Reasonable for P
to rely on Dick’s (secretary’s) representations that this did occur / was approved. All that
is required is copy of resolution certified by secretary. Thereafter: Drive-in (D) is
estopped from denying liability towards the loan. Drive-in (D) argues, inasmuch the
Bank’s officer was also a director of Tastee Freez, the Bank (P) is responsible for its own
knowledge of the resolution’s adoption. The simple answer is that the director’s contact
with Tastee Freez would not put him on notice of the resolution’s non-adoption. The
bankruptcy referee erred by dismissing the Bank’s (P) claim. Reversed in part and
affirmed in part.

 MBCA § 8.41: Duties of Officers


o Each officer has the authority and shall perform the duties set forth in the bylaws, or, to the extent
consistent with the bylaws, the duties prescribed by the BoD or by direction of an officer authorized by
the BoD to prescribe the duties of the officers.

o Lee v Jenkins Bros Case Brief


 Facts: Lee joins Jenkins brothers. Crane sells plant to Jenkins.
 In 1919 the Crane Company sold one of its plants to Respondent. Respondent sought to
hire Petitioner, the business manager of the Crane Company. Yardley, the President of
Respondent Corporation and a substantial stockholder, met with Petitioner to persuade
him to join the company. Also present at the meeting was a vice president and his wife.
Petitioner was the only surviving witness to the conversation. He testified that he was
offered a pension and promised that at the age of 60 he would receive the pension not
to exceed $1500 regardless of his status with the company. This agreement was not
recorded in writing. Petitioner accepted the position and eventually became vice
president, general manager, and director. After twenty-five years with the company, he
was discharged at the age of 55. Petitioner who had started out making $4,000 per year
was making $33,000 plus a 10% annual bonus.
 Issue:
 Whether the president had the authority to make this pension promise to induce this
employee to come work for the company?
 Whether apparent authority exists as a question of fact?
 Rule:
 Generally: President has both actual and inherent authority. Companies work through
agents. Would not make sense to try to get the board to approve every action.
 But: A president only has apparent authority to bind his company by acts arising in the
usual and regular course of business but not for contracts of an “extraordinary” nature.
 Holding:
 Whether this contract is extraordinary? Whether apparent authority exists is a question
of fact. Court leans towards no. Pension is a normal fringe benefit and does not
necessarily unduly restrict board. BUT this is an issue of fact, not law. Reasonable men
could differ as to whether or not Yardley had apparent authority to make the contract.
Chapter 20: Fiduciary Duties
 General rule: those who manage the business are fiduciaries to the business
o Officer/directors = fiduciaries
o Shareholders ≠ fiduciaries
 Fiduciary duties  owed to the corporation; it is a corporation that would have standing to bring a claim against
officers or directors for harm caused to the business
o Such a claim  derivative suit
 Basic Concepts
o Fiduciary  A person who is required to act for the benefit of another person on all matters within the
scope of their relationship; one who owes to another the duties of good faith, trust, confidence, and
candor; one who must exercise a high standard of care in managing another’s money or property
 Duty of loyalty  a person duty not to engage in self-dealing or otherwise use his or her
position to further personal interests rather than those of the beneficiary
 Obligation of good faith is subset of duty of loyalty
 Cannot engage in self-dealing and put personal interests ahead of the corporation and
entering into conflicting transactions
 Duty of care  a duty owed by the directors and officers to exercise property care in managing
the corporation
 Make informed decisions in best interest of corporation
o BJR – Busines Judgment Rule
 A rebuttable presumption that, in making business decisions not involving a direct self interest,
or self dealing, corporate directors act on an informed basis, in good faith, and in the honest
belief that their actions are in the corporations best interest
 The rule shields directors and officer from liability for unprofitable or harmful corporate
transactions if the transactions were made in good faith, with due care, and within the director
or officers capacity
 Creates a barrier for suits to be brought against management for breach of duty of care
 Standard for application of BJR
 To be entitled to protection, decision has to be made on basis of informed decision and
not involve gross negligence
 Rebutting BJR
 Shareholder would have to allege and prove that directors acted in bad faith without
requisite diligence, and without requisite disinterestment
 Issue: How does the BJR affect willingness of well qualified persons to serve as corporate
directors
 Goal  should encourage service since rebukes risk of litigation costs by facilitating
early dismissal of fiduciary duty cases
 No one would want to serve as a director if no protection in place for making poor
decision
 Duty of care (how to balance with business judgement rule)
o Governs directors conduct in managing all corporate affairs
o Governs directors response and abilities in authorizing a specific business transaction
o Judicial review of board action is governed by duty of care but its confined by BRJ
 MBCA § 8.30: Standards of conduct for directors
o (1) each member of the board of directors, when discharging the duties of a director shall ace (1) in
good faith, and (2) in a manner the director reasonably believes to be in the best interest of the
corporation
o The members of the BoD in a committee of the board, when becoming informed in connection with
their decision-making function or devoting attention to their oversight function, shall discharge their
duties with care that a person in a like position would reasonably believe approbative under similar
circumstances
 BoD (2 Sets of Tasks)
o (1) causes corporation to take acts such as declaring distributions, recommending fundamental changes
to shareholders, issuing stocks, hiring and firing officers
o (2) oversees things other people do (i.e. officers since officers making day to day decisions
 Directors breach duty of care in two ways
o (1) non-feasance  not doing anything
o (2) malfeasance  decisions that have affected the company
 (1) nonfeasance
o Failure to act when duty to act existed
 Ex) director inattentive or disengaged is guilty of non-feasance
 Frances v. Us (can’t be a passive board member of corporation and be surprised when sued for
duty of care)
 Insurance business run by father and son
 Widow elected to board / she knows nothing about business or insurance and attends 0
meetings
 Sons siphoned money from the corporate / took from widow
 Company goes bankrupt / trustee sues widow for breach of duty of care
 Holds: $10m judgement against her estate because duty of care required her to acquire
some understanding of company / to keep informed and monitor business activities
 She breached duty of care by failing to exercise the diligence, care and skin for someone
in a like position may use
 Causation must always be proven / hard to show
 (2) Misfeasance
o BoD caused the corporation to taken an act /board made decisions and decision turned out to be bad
for the corporation
o Usually involves something important: pursing new product, doing advertising campaign, exposing
company to some regulator issue
 Causation in these cases much clearer  “but for the corporations decision the corporation
would not have been harmed”
 Objectively reasonable standard  directors failed to use degree of care, diligence, and skill
that someone in their position would reasonably have used
 Shlensky v. Wrigley Case Brief
o Facts:
 Defendant is the director of the Chicago National League Ball Club, which is the company that
owns the Chicago Cubs. Although every other major league team had installed lights, Defendant
did not install them for the Cubs because he was concerned that night baseball would be
detrimental to the surrounding neighborhood. Plaintiff argued that the team was losing money,
and that the other Chicago team, the White Sox, had higher attendance during the weekdays
because they played at night. Therefore, reasoned Plaintiff, the Cubs would draw more people
with weekday night games. Plaintiff asserts that Defendant’s first concern should be with the
shareholders rather than the neighborhood.
o Issue:
 Whether the court should overrule decisions made by Defendant absent a showing of fraud,
illegality or a conflict of interest.
o Rule:
 A court will not interfere with an honest business judgment absent a showing of fraud, illegality
or conflict of interest.
o Holding:
 The court will not overturn Defendant’s decision to not install lights at the ballpark. The court
cited some reasons why the light installation could be detrimental, such as lowering the
property value of the park itself, a lack of proof on behalf of Plaintiff that financing would be
available for lights and would be certain to be offset by increasing revenues. The court cites
precedent that asserts that business decisions should not be disturbed just because a defendant
can make a reasonable case that the policy chosen by the company may not be the wisest policy
available.
 Discussion: The court notes that they were not necessarily agreeing with Wrigley’s position in
refusing to install lights, but that there was some reasons for doing so. The court stated that
there was no requirement to show all three factors of illegality, fraud or a conflict of interest,
but there was not evidence of any of the above in this case.

 Smith v. Van Gorkom Case Brief


o Facts:
 Trans Union had large investment tax credits (ITCs) coupled with accelerated depreciation
deductions with no offsetting taxable income. Their short term solution was to acquire
companies that would offset the ITCs, but the Chief Financial Officer, Donald Romans, suggested
that Trans Union should undergo a leveraged buyout to an entity that could offset the ITCs. The
suggestion came without any substantial research, but Romans thought that a $50-60 share
price (on stock currently valued at a high of $39½) would be acceptable. Van Gorkom did not
demonstrate any interest in the suggestion, but shortly thereafter pursued the idea with a
takeover specialist, Jay Pritzker. With only Romans’ unresearched numbers at his disposal, Van
Gorkom set up an agreement with Pritzker to sell Pritzker Trans Union shares at $55 per share.
Van Gorkom also agreed to sell Pritzker one million shares of Trans Union at $39 per share if
Pritzker was outbid. Van Gorkom also agreed not to solicit other bids and agreed not to provide
proprietary information to other bidders. Van Gorkom only included a couple people in the
negotiations with Pritzker, and most of the senior management and the Board of Directors
found out about the deal on the day they had to vote to approve the deal. Van Gorkom did not
distribute any information at the voting, so the Board had only the word of Van Gorkom, the
word of the President of Trans Union (who was privy to the earlier discussions with Pritzker),
advice from an attorney who suggested that the Board might be sued if they voted against the
merger, and vague advice from Romans who told them that the $55 was in the beginning end of
the range he calculated. Van Gorkom did not disclose how he came to the $55 amount. On this
advice, the Board approved the merger, and it was also later approved by shareholders.
o Issue:
 Whether the business judgment by the Board to approve the merger was an informed decision.
o Rule:
 Under the business judgment rule, a business judgment is presumed to be an informed
judgment, but the judgment will not be shielded under the rule if the decision was unadvised.
o Holding:
 The Delaware Supreme Court held the business judgment to be gross negligence, which is the
standard for determining whether the judgment was informed. The Board has a duty to give an
informed decision on an important decision such as a merger and can not escape the
responsibility by claiming that the shareholders also approved the merger. The directors are
protected if they relied in good faith on reports submitted by officers, but there was no report
that would qualify as a report under the statute. The directors can not rely upon the share price
as it contrasted with the market value. And because the Board did not disclose a lack of
valuation information to the shareholders, the Board breached their fiduciary duty to disclose all
germane facts.
 Exculpatory clauses
o Permitted to indemnify directors for violations of duty of care
o Issue: why do we have these?
o MBCA § 2.01: Limiting liability of director
 (b) the articles of incorporation may set forth
 (4) a provision limiting the liability of a director to the corporation or its shareholders for
money damages for any action taken or any failure to take an action, as a director, except
liability for (a) the amount of financial benefit received by a director to which he is not entitled
(b) an intentional infliction of harm on the corporation or shareholders (c) a violation of section
8.33; or (d) an intentional violation of criminal law
 Opt in statute

o In re Caremark Case Brief


 Facts:
 Defendant corporation, Caremark International, Inc., provides health care services and
products to patients who are often referred to them by a physician. Since the business is
reliant on referrals, there is a temptation by companies such as Caremark to
compensate physicians. A federal law, the Anti-Referral Payments Law (“ARPL”) is in
place to prevent such a system, and in 1991 the Department of Health and Human
Services began investigating potential ARPL violations. The Department of Justice joined
the investigation soon thereafter, and by 1992 Caremark instituted several new policies
and procedures in attempt to find any internal wrongdoings. But in 1994, Caremark was
indicted for violating the ARPL. Plaintiffs initiated this suit that year, alleging that the
Board of Directors did not exercise the appropriate attention to this problem
 Issue:
 Whether the Board exercised an appropriate level of attention to the possibility of ARPL
violations.
 Rule:
 Directors are potentially liable for a breach of duty to exercise appropriate attention if
they knew or should have known that employees were violating the law, declined to
make a good faith effort to prevent the violation, and the lack of action was the
proximate cause of damages.
 Holding:
 There was no evidence that the directors knew that there were ARPL violations, and
there was no systemic or sustained failure to exercise oversight. However, the terms of
the settlement merely required Caremark to institute policies to further assist in
monitoring for violations. Therefore the settlement was approved.
 Discussion: A breach of duty to exercise appropriate attention, as the court notes, is
more difficult for Plaintiffs to prove than a breach of the duty of loyalty. Most decisions
that would come under this duty will resemble many decisions shielded by the business
judgment rule.

o Strone v. Ritter (oversight and monitoring liability) Case Brief


 Facts:
 AmSouth Bancorporation (AmSouth) and a subsidiary paid $40 million in fines and $10
million in civil penalties, which arose from bank employees’ failure to file particular
reports required by the federal Bank Secrecy Act (BSA) and other federal anti-money-
laundering (AML) regulations. Evidence was shown that the corporation dedicated
considerable resources to its BSA/AML compliance program, put various procedures and
systems in place and in an effort to ensure compliance, and that these procedures and
systems permitted the Defendant to regularly monitor the corporation’s compliance
with BSA/AML regulations and requirements. On a regular basis the board (Defendant)
received reports and training in these BSA/AML compliance systems and enacted
written policies and procedures to ensure BSA/AML compliance. The shareholders
(Plaintiff) of AmSouth brought a derivative action against the corporation’s directors
(Defendant) based on these events, claiming they breached their oversight duties,
before making demand on the board. They contended that demand was excused
because the Defendant faced a good chance of liability of personal liability that would
render them incapable of exercising independent and disinterested judgment in
response to a demand request. AmSouth’s certificate of incorporation contained a
provision that would exculpate its directors for breaches of their duty of care, provided
they acted in good faith. The Chancery Court held that the Plaintiff failed to sufficiently
plead that demand would have been futile, finding that the Defendant had not been
alerted by any red flags that violations of law were occurring. Review was granted by
the state’s highest court.
 Issue:
 When specified facts do not create a reasonable doubt that the directors of a
corporation acted in good faith in exercising their oversight responsibilities, will a
derivative suit be dismissed for failure to make demand?
 Rule:
 When specified facts do not create a reasonable doubt that the directors of a
corporation acted in good faith in exercising their oversight responsibilities, a derivative
suit will be dismissed for failure to make demand.
 Holding:
 Yes. When specified facts do not create a reasonable doubt that the directors of a
corporation acted in good faith in exercising their oversight responsibilities, a derivative
suit will be dismissed for failure to make demand. Where a business decision was not
involved, as in this case, the standard to determine demand futility is whether the
particularized factual allegations create a reasonable doubt that, as of the time the
complaint was filed, the directors could have exercised their independent and
disinterested business judgment in response to a demand. The Plaintiffs’ attempt to
satisfy this standard by claiming that the Defendant faces a substantial likelihood of
personal liability, and therefore causes them to be interested in the outcome. This
argument, however, must take into account the certificate of incorporation’s
exculpatory clause, which can exculpate Defendant from a breach of the duty of care,
but not a breach of their duty of loyalty or a breach that is not in good faith. The failure
to act in good faith is a condition to finding a breach of the fiduciary duty of loyalty and
imposing fiduciary liability. [A] failure to act in good faith is not conduct that results,
ipso facto, in the direct imposition of fiduciary liability. Failing to act in good faith may
result in liability because the requirement to act in good faith is a condition of the duty
of loyalty. Therefore, because a showing of bad faith conduct is required to establish
director oversight liability, the fiduciary duty violated by that conduct is the duty of
loyalty. Second, and as a corollary, the duty to act in good faith does not establish an
independent fiduciary duty that stands on the same footing as the duty of care and
loyalty. Failure to act in good faith only indirectly gives rise to liability. Also, as a
corollary, the fiduciary duty of loyalty is not limited to financial or similar conflicts of
interest, but also includes cases where a director has failed to act in good faith.
Oversight liability itself is determined by the standards articulated in the Caremark case
(In re Caremark Int’l Deriv. Litig., 698 A.2d 959 (Del. Ch. 1996)). The bases to oversight
liability occurs where (1) the directors utterly fail to implement any reporting or
information system or controls, or (2) having put in place such a system or controls,
consciously fail to monitor or oversee its operations, thereby disabling themselves from
being informed of risks or problems requiring their attention. In either case, liability
requires a showing that the directors knew that they were not carrying out their
fiduciary obligations. When these standards are applied to the facts pleaded by Plaintiff,
it becomes clear that Defendant did not fail to act in good faith. The facts reveal that
Defendant had established a reasonable information and reporting system and had set
up various departments and committees to oversee AmSouth’s compliance with federal
banking regulations. This system also enabled the board to periodically monitor such
compliance. While it is clear with hindsight that the organization’s internal controls
were inadequate there were also no red flags to notify the board of any wrongdoing.
Defendant took the steps they needed to ensure that a reasonable information and
reporting system was in place. Therefore, although there ultimately may have been
failures by employees to report deficiencies to the board, there is no basis for an
oversight claim seeking to hold the directors personally liable for such failures by the
employees.
 Notes
o Derivative suit  shareholders sues on the corporations behalf to remedy or prevent a wrong done to
the corporation by a third party, which is typically an officer or director
 (1) Bringing a derivative suit can only happen when shareholders can show that a demand has
been made on directors to first pursue corporate claim and if they wrongfully refuse to do so,
one can bring on corporations behalf
 (2) demand can be considered “excused” because directors incapable of making impartial
decision re: whether to institute such litigation (b/c they face independent liability)
 Doubt the board can exercise independent judgment and distinction between BJR when
deciding if corporation sues directors
 Issue: when can board exercise independent judgment and disinterested BJ about
bringing CoA on behalf of corporation?
o (1) when they don’t face personal liability
o (2) if no personal liability  plaintiff must make demand to sue on behalf of
corporation first
 Steps (for derivative action)
 (1) shareholder must make demand on board prior to pursing action against corporation
i.e. “make a demand”
o Unless demand is excused (i.e. b/c board is not independent)
 (2) is board independent?
o Board independent if protected by exculpatory provisions
o Board not independent if not protected by provisions
 (2) when do exculpatory provisions apply?
o Duty of care v. duty of loyalty

o Direct action  a shareholders sues based on a cause of action arising from his or her ownership of
stock—i.e. individual injury
 Duty of care and BJR takeaway
o Rebuttable presumption exists that in making business decision, directors of corporation act on
informed basis, in good faith, and in honest belief that their actions are taken in best interest of
company
o Decision making
 To be protected by BJR in decision making, decision must be an informed one, to determine
whether was informed, courts uses gross negligence standard
 Even if BJR fail, most corporations provide indemnification and majority of states provide
exculpatory statutes
o Oversight/monitoring  duty of good faith falls under duty of loyalty
 Directors liable if (1) they fail to implement any reporting, info system, or controls; or (2) having
implemented such a system, consciously failed to monitor or oversee operations

Chapter 9 – Managemetn and control of Publicly held corporation


A) Corporate Governance in the Twenty first century
 1) Business failures and corporate scandals
o
 2) The response to the business failures and corporate scandals
o
 3) Sarbanes Oxley Act
o
 4) Analysis of the provision of the Sarbanes Oxley act
o
 5) the financial crisis of 2007-2010
B) Social Responsibility or the lack thereof
C) Shareholders
 1) In general
 2) the growth of institutional investors
D) Directors
E) Proxy Regulation and Disclosure Requirements
 1) scope of regulation
 2) proxy forms, proxy statements, and annual reports
o In the matter of caterpillar inc. Case Brief
 Facts:
 Caterpillar Brasil, S.A. (CBSA) was a subsidiary of Caterpillar Inc. (defendant). In 1989
CBSA accounted for 23 percent of Caterpillar’s net profits. Several items contributed to
CBSA’s unusually high profits in 1989, including currency translation gains, export
subsidies, interest income, and Brazilian tax loss carryovers. By February 1990, two
weeks before Caterpillar filed its 1989 Form 10-K with the Securities and Exchange
Commission (SEC) (plaintiff), management was aware that CBSA’s future performance
was difficult to predict, and that there were uncertainties as to whether CBSA could
repeat its 1989 earnings in 1990. At a board meeting in February 1990 the board of
directors was told that CBSA’s impact on Caterpillar’s overall 1990 forecast was so
significant that management felt it necessary to explain it to the directors in detail. In
March 1990 a new administration took office in Brazil and instituted sweeping economic
changes. In April 1990, at another board meeting, management discussed with the
directors the likely negative effects of these changes on CBSA’s sales and profits, and
noted that profits in Brazil would be substantially lower than in 1989. After reviewing its
April and May 1990 results, Caterpillar concluded that CBSA would suffer significant
losses in 1990. The Management’s Discussion and Analysis of Financial Conditions and
Results of Operations (MD & A) sections of Caterpillar’s 1989 Form 10-K and its Form 10-
Q for the first quarter of 1990 did not disclose the extent to which CBSA affected
Caterpillar’s results in 1989, or the risk of lower earnings for CBSA in 1990 and the
negative impact that would have on Caterpillar’s overall results in 1990. Administrative
proceedings were instituted within the SEC to determine whether Caterpillar failed to
comply with the MD & A requirements of section 13(a) and Rules 13a-1 and 13a-13 of
the Securities Exchange Act.
 (V2) Caterpillar, Inc. (Defendant) had a wholly owned subsidiary, Caterpillar Brasil, S.A.
(CBSA), which accounted for approximately one quarter of Defendant’s net profits.
Prior to Defendant’s filing of the reports required by § 13(a) of the Securities Exchange
Act of 1934, management recognized that the future performance of CBSA would be
especially hard to predict. Brazil was volatile, and the impact was significant enough to
reduce Defendant’s projected results for 1990. There was, however, nothing in the
reports of the Management Discussion and Analysis (MD & A) to suggest the
disproportionate impact of CBSA’s profits on Defendant’s overall profitability or that
adequately noted the uncertainty of management regarding CBSA’s performance for
1990.
 Issue:
 Is disclosure required when it is likely that a known trend, demand, commitment, event,
or uncertainty is likely to occur, and where such a determination cannot be made, must
an objective evaluation be made regarding the consequences of such an occurrence
assuming that it will occur?
 Rule:
 Disclosure is required when it is likely that a known trend, demand, commitment, event,
or uncertainty is likely to occur, and where such a determination cannot be made, an
objective evaluation must be made regarding the consequences of such an occurrence
assuming that it will occur.
 Holding:
 Yes. Disclosure is required when it is likely that a known trend, demand, commitment,
event, or uncertainty is likely to occur, and where such a determination cannot be
made, an objective evaluation must be made regarding the consequences of such an
occurrence assuming that it will occur. Defendant’s disclosure should have discussed
the impact of the expected changes in CSA on the overall results of Defendant’s
operations. In addition, its annual and quarterly reports should have discussed the
uncertainties of future CBSA operations and the possible risk of Defendant’s having
considerably lower earnings as a result and, to the extent reasonably practicable,
quantified the impact posed by the risk. Defendant failed to do so.

Module 6 / Lecture 2 / The duty of Loyalty – Self Dealing; Executive Compensation


 Duty of Care – A Duty owed by directors and officers to exercise proper care in managing the corporation’s
affairs.
o Informed decision making
o Why is the potential liability of breaching the duty of care almost 0?
 Business Judgment Rule (BJR): Presumption in the law that directors and officers have complied.
 Would have to show intentional bad acts, or intentional gross conduct. “Negligence” is never
enough.
 Want to encourage people to serve in these positions.
 Do not have to be right, but damn well better be informed. Does not have to be the
right one, but cannot show you flipped a coin.
 Duty of Loyalty
o Any time a director of officer participates in a transaction involving a conflict of interest (a transaction in
which the director or officer may me motivated not only by the interests of the corporation but also by
self-interest)  the duty of loyalty is implicated
o General rule  Duty of loyalty requires that director serve the interests of the corporation over their
own self interests
 Because of concerns that directors have acted in their own self interests in these transaction,
courts scrutinize cases raised duty of loyalty issue much more carefully than those of duty of
care concerns and because of this directors and officers don’t usually get the benefit of the
business judgment rule protection when their duty of loyalty is called into question
 Breach of duty of care  lazy and dumb
 Breach of duty of loyalty  greedy and self interest
o MCBA § 8.30: Standards of Conduct for Directors (duty of loyalty)
 (a) each member of the board of directors, when discharging the duties of a director, shall act:
(1) in good faith, and (2) in a manner the director reasonably believes to be in the best interests
of the corporation
o 3 classic fact patterns that implicate the duty of loyalty
 (1) competing ventures
 (2) self-dealing and director transaction (executive compensation)
 (3) usurpation of business or of corporate opportunities
o (1) Competing ventures
 A fiduciary of a company cannot be engaged in another business that directly competes that
with company, doing so breaches the duty of loyalty because operating a competitor cannot
qualify as an act taken in a manner she reasonably believes to be in the best interest of the
company to whom she owes the fiduciary duty
 It is okay
 to have more than one business venture as long as it doesn’t compete with the business
to whom you owe your duty
 to compete directly if disinterested directors’ consent
 to leave your business to compete but you have to be aware of non-competes you may
have singed
 to prepare to compete before leaving (forming a corporation, leasing office space) but
can’t solicit your companies clients to go to your business when you are still working for
that company or use you company time and resources as you’re setting up another
business that is going to compete when you leave
 Verify there is not a non-compete agreement in the employment / operating
agreement. Non-compete or non-solicit (generalized covenants).
o (2) self-dealing or interest director transactions
 Self-dealing  transaction entered into by the corporation in which one or more of its directors
has an interest on the other side
 Ex)
 assume x is a director of XYZ Inc.
 X also owns supply corp
 If XYZ Inc. enters into a contract with supply corporation there is self-dealing and we
have an interested director transaction because x is on both sides of the deal
 The problem  X, in her role as director of XYZ Inc, has an incentive to have XYZ
overpay because doing so lines her pockets as owner of supply corp
 Old common law rule  allowed XYZ to VOID self-dealing contracts at will even if just one
shareholder objected
 This is no longer valid – a fiduciary can give her corporation good that it cannot get
elsewhere (i.e. loan at a low interest rate that isn’t secured)
 Newer case law/statutes  interested director transaction will not be set aside because of
self-dealing if the director shows one of three things
 (1) transaction is approved in good faith by disinterested directors
o You remove yourself from the board from
 (2) approved in good faith by disinterested shareholder
o Put to shareholder vote and as long as they’re disinterested and it’s the
majority and done in good faith
 (3) the transaction was objectively fair to the corporation when entered
o It might not have been approved by disinterested directors or shareholders
but on its face form outside view, what happened was objectively fair and
should not be set aside
 Marciano v. Nakash
 Rule
o Self-dealing (delaware)  (a) between corporation and one or more of its
directors; (b) a business or organization in which one of its directors also serves
as a director; (c) or a business organization in which one of its directors has a
financial interest
 Application
o Self-dealing present  Loan transactions were between Nakash’s individually
and the corporation of which they were directors
o Note: it is still self-dealing even if the loan benefitted the corporation and was
received
 Result - If the claim is allowed
o Assets – liabilities = owners’ equity
o If this is valid, the Nakash’s first get 2.5 million dollars as a creditor and then
they get to share on a 50/50 basis whatever is left with the Marcianos (which is
likely not going to be anything)
 Result – if the claim is disallowed
o The two families share in all the remaining assets because this would just be
sharing equity / equity comes last in a situation of liquidation
 § 144 interest directors; quorum (see from slide)
o (1) This can be approved by disinterested directors if the material facts are
disclosed to the board and the disinterested directors authorize, approve or
ratify the transaction by a majority acting in good faith
 Do not have to prove substantive fairness
 Procedural so protected by BJR
o (2) Disinterested shareholders approve it – material facts are disclosed – they
can approve or ratify the transaction in good faith
 Do not have to prove substantive fairness
 Procedural so protected by BJR
o (3) General fairness test – interested directors can show that the deal was
objectively or substantively fair to the corporation focusing on fair price, fair
dealing
 No BJR protection, the interested director must show intrinsic or
objective fairness
 Test
o 1. Are statutory provisions (1) or (2) satisfied?
 If no  cloud of suspicion not removed and burden switches to
interested director to who intrinsic fairness
 If yes  cloud of suspicion removed and decision is protected by BJR
 Issue: are these statutory provisions exclusive?
o No  if you clear the conflict of interest hurdle through section 144 the
decision will still be subject to a challenge based on the duty of care albeit with
the powerful protection of the BJR
o If section 144 is not met the burden switches to the director to show fairness
 MBC approach
o Any transaction to which the director is a party or in which she has a known
material financial interest or in which a related person is a party or has a
material financial interest
o We remove this self-dealing taint  1-3
 In re El Paso Corp
 Self-dealing  full disclosure and hiring Morgan Stanley
o This didn’t remove the conflict
 Plaintiffs have shown there is a likelihood of irreparable injury
o Judge denies injunction and lets it go to a shareholder vote for fairness
o Why  courts don’t want to get involved in company decisions (BJR)
o (3) Self-Dealing and Executive Compensation
 Board of directors  responsible for setting executive compensation including directors own
compensation
 Inherent conflict when directors set own compensation and courts occasionally asked to
second guess such compensation
 Issue: have directors breached duty of loyalty by overcompensating themselves?
 Director compensation is treated like other forms of self dealing and must be shown to
be a fair figure, excessive pay is like stealing or wasting corporate assets which is clearly
breach of duty of loyalty
o Most courts use waste standard
 In order to overcome a compensation decision  you have to show that no reasonable
businessperson would have made the same decision / the waste standard will supplant
the BJR and provide the shareholders with the remedy
o Waste is a way of attacking a transaction and rebutting the business judgment
rule presumption
o But any rational business purpose will excuse a claim of waste
 Judged under duty of care  give CEO big salary because there is no self-dealing (the
CEO was not involved in making the salary) / as long as person getting offered money
not in board room / BJR in our favor
o BJR can be rebutted through waste doctrine
o Plaintiff has to rebut the presumption of BJR
 Judged under duty of loyalty  when person receiving salary is involved in setting said
salary / self interested
 Heller v. Boylan
 Facts
o Plaintiff sue saying salary bore no relation to the value of the services rendered
and were therefore akin to a gift
o By giving the sums away, they argued, the majority shareholders committed
waste
 Duty of care v. duty of loyalty
o The establishment of executive compensation for officers and directors often
times raise question of self-dealing which is duty of loyalty because we are
talking about all these directors sitting together as insiders setting their salaries
o Even if director whose comp is being set abstains from voting, there is a mutual
back scratch problem
 Salary Plan
o Court finds the salary plan reasonable when adopted
o Note: we are only looking for some rational business purpose for the salary
 Standard to be used in these situations
o Whether or not the resulting compensation is so large as amount to spoliation
or waste of corporate property
o This goes back to BJR  did board make informed decision when they set these
salaries? If so, the court wont second guess their judgment
o Essential same test as Marciano test
 If you clear the conflict of interest hurdle by disinterested voting
directors, the compensation package will still be subject to a challenge
based on the duty of care (with the powerful protection of the business
judgment rule)
 If the conflict of interest is not clear, i.e. the director receiving the
compensation set and voted on it, it will be on the director to show
substantive fairness and that there Is no waste of corporate property
i.e. prove your worth if you’re not disinterested on an objective basis
 Wilderman v. Wilderman
 Case different from previous cases
o Closely held corporation – 2 members
o The salary was fixed by Wilderman and only Wilderman not by a disinterested
board
 Duty of loyalty case
o A director or officers decision to give himself the corporations money implicates
the duty of loyalty because he is putting himself above the corporation through
self dealing
 Standard court focuses on
o Whether the salary was reasonable
o The burden is on the defendant because the decision to give the defendant the
salary was never sanitized by disinterested decision makers
 Holding
o Court says defendants compensation is not reasonable and it determined this
though how much others were paid, expert testimony, how IRS views it, how
much services worth
o Court is willing to second guess the reasonableness of the compensation
because he was not sanitized, he unilaterally set this price, there no biz
judgment rule benefit here / and the burden is on him to establish intrinsic or
objective fairness to the corporation
o Placing the burden on CEO’s and other executive to show salaries are fair when
the plaintiff challenges the salaries have shown that an exec has set his or own
salary is a simple application of the basic principal that in conflict of interest
transactions, that have not been vetted by independent directors or
shareholders, insiders have the burden of establishing the entire intrinsic
fairness of the transaction
o When a defendant fails to establish intrinsic fairness, the remedy will be a
disgorgement of the salary so it is a CoA that implicates directors personally
 Brehm v. Eisner
 Disney board decisions the shareholders are challenging  boards initial approval of the
Ovitz employment agreement and the boards subsequent decision to grant Ovitz a non-
fault termination of the agreement
 (1) Approval of employment contract
o Board violating duties in approving contract
 (1) Boards process was flawed (duty of care breach)
 (2) Substance of the contract was deficient (it amounted to waste /
overcomes BJR presumption)
o Ovitz not involved in decision  disinterested
 BJR governs
 No
 (2) Non fault termination clause
o Crystal didn’t quantify
o SC  neither crystal nor the board analyzed the economical substance of the
contract at all but this is harmless when it comes to suing the directors
personally because the board is entitled to rely on good faith on crystals expert
opinion and crystal isn’t the one being sued here, the board is
o Section 141(E)  members of BoD are fully protected in relying in good faith on
the statements presented by any person as to matters the members reasonably
believes are within that experts competence
o Model Business Corp act  no allegation of bad faith reliance
o Issue: Does this mean that the board can just advocate their responsibilities to
someone else and always escape liability if they dub them an expert and then
rely on them?
 Test  No  in order to survive a motion to dismiss the plaintiffs must
allege particularized facts which allege (a) directors did not rely on the
expert (b) their reliance was not in good faith; or (c) the directors did
not reasonably believe that the advice was within the experts area of
competence or (d) the expert was not selected with reasonable care or
(e) the cost to the corporation for the exercise of this compensation
agreement was material and reasonably available and so obvious that
the boards failure to consider it was grossly negligence despite any
advice received or not received from the expert (f) decision was so
unconscionably as to constitute waste or fraud
 Amended complaint (failure to set forth specific facts)
o Crystal was not hired for orbitz contract – improper reliance
o Eisner unilaterally bargained with Ovitz
o The board knew hiring the new president was important
o Neither board nor committee reviewed the contract
o Very little time to vote to discuss matter
o They approved general terms and Eisner approved the details that didn’t match
which were never put in front of board
o No outside expert consultant
 Holding
o Re process  duty of care not breached and protected by BJR
 They knew enough info (value of option ) / they were reasonably
informed in making this decision
o Re bad faith  bad faith not protected by BJF
 Carved out as exception to protection for directors
 Good faith  assumed under duty of loyalty and breaches of duty of
loyalty are not protected by the BJR or exculpatory provisions
 Court defines bad faith
 Fiduciary conduct motivated by actual intent to do harm
 Fiduciary action taken solely by reason of gross negligence
(without malevolent intent not applicable)
 **Intentional dereliction of duty (conscious disregard for ones
responsibility)
o Turning to allegation of waste, why did the shareholders not succeed on claim
or corporate waste/
 Courts will not second guess the decisions of corporation unless they
are wasteful
 Test for waste  exchange so one sided that no businessperson of
ordinary sound judgment would conclude that the corporate has
received adequate consideration
 Paying junior 1mil a year
o Paying severance was abiding my contract which cannot be viewed as wasteful,
the creation of the contract could not be wasteful bc provisions has a rational
business purpose which was to induce Ovitz to leave Disney
 Takeaway
o When setting exec compensation, the court focuses on the process / as long as
the parties were reasonably informed there is no self dealing compensation will
not be disturbed unless it is wasteful
o Waste doctrine applies to rebut all BJR situations
 Test for waste  exchange so one sided that no businessperson of
ordinary sound judgment would conclude that the corporate has
received adequate consideration
o If you can show that, the waste standard will supplant the BJR and supply the
Shareholders with the remedy
o Courts reluctant to second guess decision concerning exec compensation
o Role of good faith comes up  duty of loyalty
o Directors do not initially have benefit of BJR for duty of loyalty claims, they
cannot be exculpated by duty of loyalty claims and they cannot be insured or
indemnified for duty of loyalty claims
o Lack of good faith requires intent to do bad things to the corporation / hard
standard when fighting over exec compensation

Module 6 / Lecture 3 / The duty of Loyalty – Conflicts; Corporate Opportunity; Disclosure


 Controlling shareholder conflicts of interest
o Controlling shareholder  one who owns (1) outright majority of the voting stock or (2) a substantial
minority of the stock, with the remaining shares so dispersed that she is able to control the majority of
management
 Duty imposed upon someone because they manage the business (directors and officers)  they
owe duties of care and loyalty
 A controlling shareholder does not have an official management position in the corporation, but
it does have significant control because of its substantial ownership
 Re parent subsidiary relationship
 The controlling shareholder of one corporation is often another corporation / this is true
every time there is a parent and a subsidiary relationship because the parent owns a
controlling block of stock in the subsidiary
 When the subsidiary is wholly owned by the parent there are no fiduciary problems
 But when the subsidiary has some minority shareholders, the parent must be careful to
not breach the fiduciary duty in its dealings with the minority
o Sinclair Oil v. Levien
 Plaintiff argument  Sinclair (parent) caused Sinven (subsidiary) to pay huge dividends for the
purpose of funneling money to Sinclair and as a result Sinven lacked the funds to develop its
own business
 Sinclair owned 975 of Sinven stock
 Court says  Sinclair owes a fiduciary duty to Sinven and its other (minority) owners
 Because of Sinclair’s domination on the board and its substantial ownership / it is as if
Sinclair were a real person on the board of directors
 Two standards for reviewing Sinclair’s acts
 (1) business judgment rule
 (2) intrinsic fairness test
o Court will employ the BJR unless there is self-dealing / courts will not second
guess the board unless there is a showing of gross overreaching or fraud or
gross negligence and it will not be disturbed if there is any rational business
purpose
o In cases where there is self-dealing, the burden should be on the defendant to
prove intrinsic fairness which is an objective analysis
 Self-dealing in the parent subsidiary context Test
 When a parent is on both sides of the transaction with its subsidiary / Self-dealing
occurs when the parent by virtue of its domination of the subsidiarity causes the
subsidiary to act in such a way that the parent receives something from the subsidiary
or and a detriment to the minority of the subsidiary
o (1) Show parent company is dominating subsidiary
o (2) Party is causing subsidiary to act in a way in that the parent receives
something in a determinant to the minority
o (3) If they do that we can show that there is self dealing or a breach of duty of
loyalty
 Test
o Is there self-dealing?
 Yes  intrinsic fairness  BOP on D to prove objective fairness
 No  BJR  BOP on P to show fraud or gross overreach
 Was there self-dealing in the dividends of 108 million dollars?
 No – the minority shareholders received the same pro rata dividend that Sinclair did /
because the parent got no more that the minority shareholders, there no self-dealing or
detriment to or exclusion of the minority
o BJR standard
o Sinclair wins on claim
 Was there self-dealing in usurpation of corporate opportunities?
 Rejected application of intrinsic fairness test
 Sinclair had received nothing from Sinven to the exclusion of its minority shareholders
so there was no self dealing
 None of the potential opportunities was in Venezuela so they did and it did not
constitute a corporate opportunity
 The court applies BJR and Sinclair wins on claim
 Breach of contract btw Sinven and international
 Sinclair is also 100% owner of international
 There was self dealing under the intrinsic fairness
 Sinclair caused Sinven because of its domination and control to give up its right to
charge late payments / it forgave international of its breach even though it didn’t
purchase the fixed amount of crude and refined products from Sinven under the
contract and Sinclair benefitted as a result
 The minority shareholder of Sinven didn’t benefit because money didn’t come in – it
hurt the companies accounts receivable
 Standard adopts intrinsic fairness – Sinclair didn’t carry its burden
 Court allows this claim to go forward
o Wineburger v.
 Involuntary elimination of minority shareholders
 CoA asserted by wine  breach of fiduciary duty to the minority shareholders for failure to
disclose a conflict of interest an provide relevant info
 Issue: whether signal as majority shareholder breached its fiduciary duty to the minority by
withholding info from non-signal UOP directors about the share price and the conflict
 SC  held that the vote was not informed, signal breached its fiduciary duty and that the
minority shareholders were entitled to a greater value
o Con v. MFW Worldwide
 Controlling stock holder of MFW  owed 43 percent of stock of MFW and wanted to acquire
the remaining shares through a private type of merger
 In anticipation of litigation over the transaction that it knew would follow its merger
proposal, MNF said it would only go through with the merger if an independent
committee of directors of MFW approved the deal – an informed uncoerced majority of
the target shareholders DID approve the deal
 Why is this self dealing the acquisition is by one company of the remainder of a
company that it already controls and there is danger that the minority shareholders of
the target company may be disadvantaged by the acquisition of their interests for
inadequate consideration
 Traditional standard of review in these sorts of conflicted transactions is the entire fairness
standard
 Standard requires that the controlling shareholder bear the burden of proving that the
transaction was entirely fair to the minority shareholders of the company be acquired
 New law (Delaware SC)
 Holds that the defendant friendly business judgment rule standard should be used to
review these sorts of merger transactions where the merger is conditioned on the
approval of an independent adequality empowered special committee of the target
company board that fulfills its duty of care and on an uncoerced vote of a majority of
the minority stock holders
 Justification for using entire fairness standard in conflicted transactions particularly
acquisitions of controlled companies  because the protection of a disinterested board
and stock holder approval do not exist in such cases  the protection of a disinterested
board does not generally exist because the controlling corp that wants to acquire its
controlled corp also controls the board of the target company  thus, shareholder
approval is not a protection because the controlling shareholder can cause a merger to
be approved by voting its own shares in favor of the merger
 This can be overcome  where the controlling shareholders relinquishes decision
making authority over approval of the merger transaction by empowering an
independent committee of a target company to reject the transaction and subjecting
the deal to the approval of the majority of the target minority shareholders

 Corporate Opportunity
o Address a director taking for herself something that belongs to the corporation and precisely something
that should have been offered to the corporation so it can decide whether to pursue it
 By seizing the opportunity, herself, the director usurps a corporate or business opportunity
o 4 approaches to corporate opportunity (on exam  analyze each test)
 (1) interest or expectancy test
 (2) line of business test
 (3) fairness test
 (4) combined or dual test
o NE Harbor v. Harris
 Facts
 Harris tells board she bought the tract – title in her name and the club would be
protected
 Board did not take action – estoppel?
 She hears another tract is for sale – she doesn’t tell board members she is seeking to
buy the tract
 She then announces she has purchased the tract and had no present plans to develop
property
 Board did nothing
 The then purchased adjoining property that connected the two tracts
 After learning harris was starting to develop the tracts – the board brought suit to stop
her and bring damages that she usurped corporate opportunities that belonged to the
golf club
 (1) Line of business test
 Whether the project falls within the firms line of business, focus on what the firm could
reasonably be expected to adapt itself now or in the reasonable future including its
financial ability
 Problem w test  what’s really in the line of business? She wasn’t buying another golf
course, and do we even know if they’re in the business of developing / financial ability
element of this test is problematic because it creates an incentive for officers to keep
the company in an insolvent state so it cant take such an opportunity
 (2) Fairness test
 Determines the existence of a corporate opportunity by applying ethical standards or
what is fair and equitable under the circumstances / what is fair to the corp
 Problem w test  too vague / no guidance to officer or director on how to act and what
is really fair
 (3) Dual Test
 The line of business plus fairness
 Problem  just layers the problem / still have a hard time knowing what is within line of
business and what does it even mean to be fair
 (4) ALI Approach (court adopts)
 (1) what is a corporate opportunity?
o Something you became aware of through (a) your position, (b) through
corporate information (where you ought to offer it to the company) or (c) or
within the line of business
 (2) if corporate opportunity, inside can take if:
o (a) the director first offered the corporate opportunity to the corporation and
makes disclose concerning the conflict; and
o (b) the corporate opportunity is rejected and EITHER the rejection of the
opportunity is fair for the corporation OR the opportunity is rejected by
disinterested directors in a manner that satisfies BJR OR the rejection is
authorized or ratified by disinterested shareholders and the rejection is not
equivalent to waste of corporate assets
o (c) can show it is objectively fair for the corporation to take
 Steps
 (1) Determine whether it is a corporate opportunity (using 4 tests)
 (2) if it is then must disclose,
 (3) then explain it’s ok by disinterested board or shareholders or fairness of the
rejection
 Application
 Whether corporate opportunity
o Yes – tract G  she heard of it in her CEO capacity
o Tract S  hard to say / not info on how she became aware of it but maybe this
does fit within the line of business
 Harris loses because she didn’t make any disclosure regarding the conflict and the
opportunity as Harris did not present the opportunity, case closed, violated duty of
loyalty and must offer the property unless she can prove it wasn’t corp opportunity in
the first place

 Disclosure
o Malone v. Brincat
 Facts
 Suit filed alleging that directors of Mercury breached duty of discloser to shareholders
and that KPMG (other defendant) aided and abetted them
 Violation shareholders complaining of  argue that directors were releasing
information to shareholders of inflated financial information which was overstating the
companies worth and equity
o Damages  company lost virtually all of its value / shareholders suffer
 Issue: do directors have a duty other than SEC reporting duties, to disclose this financial info, if
no demand is ever made by shareholders/
 Rule: the board only has a duty to disclose information when it seeks shareholder action or
when there has been a proper demand for the information
 Issue: As there was no duty to disclose, did that mean that the directors could release false or
inaccurate information?
 No – that could affect the entire market place
 Rule: when directors choose to report company information, or are required to because of law
action or demand, they have a duty of honesty / when directors convey this information, they
are all required to be informed and honest
 Duty of honest is party of duty of loyalty (subset just like duty of good faith) /
Exculpatory clauses cannot protect this
 Duty of care  just directors decision making and process of making decision
 If breach of duty of loyalty or care is shown  not necessarily every director is liable, there is a
presumption that all directors present at the board meeting concurred with the action taken at
the meeting unless their dissent or obtention was entered into in the corporate books or
records
 Director who feels as if board is doing something wrong can insulate herself from
liability by making a record of her objection / even a consenting director can be
protected if he relied in good faith on a report or opinion of a competent person or
expert

Chapter 12 – Shareholder Derivative Lawsuits


 Shareholders  owners of the corporation which gives them several rights including the right to bring a
shareholder derivative suit
 Derivative actions  designed to solve a policy dilemma
o Reason for the dilemma  directors are supposed to make all major management decisions including
whether to initiate and pursue litigation but fiduciary cases will name one or more directors as
defendants usually for breach of the duty of care or loyalty so that influences the directors decision as to
what is really in the best interest of the corporation (because who wants to sue themselves?)
 Shareholder Derivative action  allows a shareholder to initiate and manage the litigation on behalf of the
corporation rather than entrusting it to interested and conflicted directors
o Strike suits  shareholders have brought, as a result of a derivative action, a corporate action simply
brought to settlement for the lawsuits nuisance value and often times are meritless;
 because of this the corporate law responded to this problem by having several requirements
that are set up like class actions in order to bring a derivative lawsuit
 Derivative v. Direct suit
o In a derivative suit  a shareholder sues on the corporation’s behalf to remedy or prevent a wrong
done to the corporation by a third party, which is usually an officer or director; Several procedural
hurdles that the shareholder must satisfy
 How to determine if derivative  simply ask could the corporation have brought this suit?
 If so  it probably a derivative suit because the plaintiff is vindicating the corporation’s claims
 (1) who suffered alleged harm  corporation or stockholder?
o One specific shareholder or the entire corporation as a whole?
 (2) who would receive the benefit of the recovery or the remedy  corporation or
stockholder?
 Example 1
 S is a shareholder of ABC corporation sues a third party because the third party
breached its contract with ABC corporation
 This is derivative because ABC corporation could sue the third party and recover the
money
 Example 2
 Board of directors of ABC corporation approves a purchase of land without any
evaluation or survey and the property turns out to be worthless
 This is breach of duty of care and is a derivative suit  duty of care and loyalty are owed
to the corporation so the corporation is the proper plaintiff and the shareholder may
bring a derivative suit against the directors to recover from them personally from the
losses suffered by the corporation
 Derivative suit will almost always be brought against a third party or against individuals who
have breached a duty to the corporation
 The suit seeks to impose personal liability on such defendants
 These cases would have been brought by the corporation itself had the board decided
to have the entity sued
o In a direct action  a shareholder sues based on a cause of action arising from his or her ownership of
stock—i.e. individual injury; No hoops to jump through for the plaintiff
 Example 1
 S a shareholder of ABC corporation sues ABC corp because it issues stock without
honoring her preemptive rights
 This is direct suit  ABC corporation could not bring this suit because ABC has not been
harmed / the harm is directly to the shareholder
 Example 2
 Suits to allow the shareholder to inspect books and records
 To honor dividend preference
 To force a dissolution
 Securities fraud case for being misled into buying stock that was undervalued or
misrepresented
o Know the difference for an exam. Ask yourself: could the corporation have brought this lawsuit?
 Judgment and fees in a successful derivative suit
o If successful  the recovery goes to the corporation NOT the plaintiff (i.e. the shareholder who is
merely doing this out of altruism)
 Plaintiff can recover litigation fees and costs
 Closely held corporation Derivative Suit
o is rare to see a derivative suit in a small, closely held corporation since a judgment in favor of the
corporation just goes right back into the hands of the owners
 These suits will usually be treated as direct
 The corporation is also a litigant in the suit and must be joined as a party
 The corporation will be listed as a nominal defendant  because the corporation did not
actually sue and the law has always been reluctant to force someone to litigate as an involuntary
plaintiff
 Prerequisites for a Derivative Suit – Procedural requirements – Surviving a motion to dismiss (FRCP 12(b)(6)).
o Contemporaneous ownership:
 Person brining the suit actually has to own the stock when the claim arose, not when the suit is
filed. See, e.g., a disgruntled employee would lack standing.
 Reason  we don’t want shareholders selling lawsuits after a derivative claim becomes obvious
 We do not want shareholders selling their lawsuits. Claims (litigation) purchasing.
o Adequate representation:
 The plaintiff must demonstrate that she will adequately represent the interests of the
corporation not herself because the result of litigation will bind the corporation
 Proper motivation / stake in the case, as the result will bind the company.
 A lot like certifying a class action. Must show continued harm / ownership.
 To show adequacy of representation  the plaintiff must show an ongoing interest in the
corporation (i.e. that the plaintiff will continue to own stock when the case is brought and
throughout the litigation
 If you sell your stock as a plaintiff during litigation you will lose standing to sue and the
case can be dismissed
o Security or bond or expenses:
 In some of these, plaintiff will be required to post a bond as a condition for bringing suit.
 Plaintiffs have used process to extort money. Only some states.
 if the suit turns out to be meritless the harmed parties can draw from that bond
o Demand:
 We focused on / will focus on. Suit is a major business decision.
 Big corporate decision  for board of directors = you send this to the Board.
 1Statutes require shareholder  to make a written demand that the corporation bring suit and
because these big corporate decisions are made by the BoD, the demand must be made on BoD
 Should not be able to proceed without giving the corporation a chance to pursue the
complaint. This demand must state: what the claim is and against whom it exists.
 Outline your causes of action, defenses, maybe even attach a complaint draft.
 Issue: do you always have to make this demand?
 Traditional approach (Delaware)  requires a pre-suit demand on the board BUT the
demand may be excused if the plaintiff can show that making a demand on the board
would be futile / because there is director bias
o Ex) action against sitting directors for awarding themselves all excessive bonus’s
 here you would be telling the directors to sue themselves / doing so would
be futile (i.e. it wouldn’t make sense to tell people who are already going to not
sue themselves to sue themselves)
o Demand excused (futile)
 Plaintiff alleged with particularity facts that create a reasonable doubt
 (1) That the current BoD is disinterested or independent; or
 (2) That the challenged transaction would be protected by the
BJR
o If demand is made
 (1) board could agree and have the corporation bring the suit (no more
shareholder involvement and it wouldn’t be derivative)
 (2) board could reject the demand and the shareholder might give up
now or sue and allege that the decision not to sue was a mistake
 Problem  decision not to sue is protected by BJR and only way
to overcome BJR is to show the decision not to sue was tainted
by some conflict (but if this were true, that there was a conflict,
the demand would have been excused in the first place, so now
you’ve admitted there is no conflict cause you made the
demand)
 Under Delaware law  the shareholder will always lose in this
demand rejected scenario because making the demand on
directors constitutions an admission that the board was
disinterested i.e. not conflicted
 This means the board’s decision is protected by BJR
o Court will look at Special Litigation Committee.
 Determines whether decision is assessed based on:
 if the committee is independent and disinterested  BJR
o Neutrality.
 if the committee’s independence is challenged  reasonable,
intrinsic fairness.
o What facts did they actually gather to make an
informed decision?
o If demand isn’t made
 The board or directors will file a motion to dismiss on the ground that
the plaintiff should have made the demand
 On this motion the court faces one issue: would a demand on the board
have been futile?
 Most cases decision will depend on whether a majority of the
directors is tainted
 If court determines demand was not futile, that it should have been
made  the suit will be dismissed
 If it determines the demand was excused  the derivative suit
continues
o If you make the demand and it’s rejected = BJR.
 If you don’t make the demand because you believe it would not have a
purpose = they will analyze the
 Universal approach (Model business corporation act) 
o Pre-suit demand on the board always has to happen whether it is futile or not
o The shareholder has to make that demand and then wait 90 days unless the
board rejects the demand or waiting those 90 days would cause irreparable
damage to the corporations.
o MBCA approach will still appoint a committee.
 Traditional (Delaware) is just a little different. Giving options, such as do
you want to make a demand, or do you believe it’s excused.
 Here, never focusing on demand being excused.
 Make the demand, and then wait.
 Similar analysis on neutrality / fairness.
 Special Litigation Committee
 Assume that a derivative suit is pending, and a corporation wants it dismissed because it
believes it is not in the best interest of the corporation.
o It will file a motion to dismiss and that motion will assert that the case should be
dismissed because it is going to be bad publicity, cost the corporation money, or
even devalue the stock
o This determination not to sue must be made by disinterested independent
people and not by those that are defendants in the suit because of course, the
potential defendants will want it dismissed if they face liability
o Thus  corporations set up special litigation committee
 Appointed to review derivative cases and make a disinterested
untainted decision as to whether the case is in the company’s best
interest
o If they conclude it is not in best interest of the company  they will move to
dismiss  and what a judge will do depends on the type of claim asserted by
the plaintiff
 derivative suit against outside third party (counterparty to contract) the
judge will give a lot of deference to the special litigation committee
because no director or officer is being implicated as a defendant
 derivative suit against inside director or officer, the judge will not give a
lot of deference to the special litigation committee because judges
believe there is still some mutual back scratching at issue
 note: this has changed a bit.
 Gall v. Exxon Corp
o Facts
 In 1975 Exxon Corporation’s board of directors established a special committee to determine
what action to take on allegations that Exxon paid $59 million in corporate funds as bribes or
political payments to Italian political parties and others (breach of duty of loyalty and bad faith).
The special committee investigated the matter, and uncovered millions of dollars of secret
payments and political contributions. The special committee unanimously determined that it
would be contrary to the interests of Exxon and its shareholders to maintain legal actions
against any present or former Exxon director, and authorized Exxon officers to seek dismissal of
shareholder derivative actions that had been instituted. In the current action, brought by
shareholder Gall (plaintiff), Exxon (defendant) has moved for summary judgment dismissal of
the complaint based on the special committee’s determination.
 Believed there would be more harm than recovery to the corp in filing the derivative
suit
 Board of directors  responsibility to bring lawsuits on behalf of the corporation when
the corporation has been harmed because corporations function through their boards
o Disinterested decisions about whether the corp should sue its directors or
officers is also governed by the business judgement rule and this makes it hard
to compel directors to sue on behalf on behalf of the corporation
o Issue
 Whether the special committee, acting as Exxon’s BoD and in the sound exercise of their
business judgement, may determine that a suit against any present or former director or officer
would be contrary to the best interests of the corporation
 Is a corporate committee’s decision not to bring a cause of action always protected by the
business judgment rule, if the plaintiff in a shareholder derivative action calls into question the
committee’s independence and good faith?
o Rule
 General rule: decision made by special committee is protected by the BJR
 Exceptions apply
 A corporate special litigation committee’s decision NOT to bring a cause of action might NOT
be protected by the business judgment rule if the plaintiff in a shareholder derivative action
alleges that the committee did not act independently or in good faith.
 Under the business judgment rule  courts will not interfere with directors’ decisions on
whether to enforce a cause of action held by the corporation absent allegations of fraud,
collusion, self-interest, dishonesty, or other misconduct amounting to a breach of trust, or
allegations that the directors’ business judgment was grossly unsound.
o Holding
 Gall alleges that members of the special committee may have been involved in the wrongdoing
that is the basis for this action, or interested in the transactions in a way that would impair their
business judgment, thus calling into question whether the special committee acted
independently and in good faith.
 In light of this allegation, summary judgment dismissal is premature. Instead, Gall should be
permitted to conduct discovery into the independence and good faith of the special committee
(show that they were not disinterested). Intent, motivation, and good faith are all issues that
cannot be determined on summary judgment.
 Exxon’s motion for summary judgment is therefore denied without prejudice, though it may be
renewed after discovery has been conducted.
 Zapata Corp v. Maldonado
o Facts
 William Maldonado (plaintiff), a shareholder in Zapata Corporation (defendant), brought a
derivative action on behalf of Zapata against 10 of Zapata's officers and directors, alleging
breach of fiduciary duty. Maldonado had not made a prior demand that the board bring the
action and instead argued that demand was futile, because all of the board members were
named defendants who allegedly took part in the challenged transactions. After two new
outside directors were added to the board, the board as a whole appointed only those two new
directors to an investigation committee charged with investigating Maldonado’s claims. The
committee found that it was in Zapata’s best interest that Maldonado's derivative suit be
dismissed.
o Issue
 In reviewing an independent committee's motion to dismiss a shareholder's derivative lawsuit
on the basis that maintaining the lawsuit is not in the corporation's best interest, must the
chancery court (1) first analyze the committee's independence and good faith and the bases
supporting the committee's conclusions, and then, (2) if the court is satisfied that the committee
was independent and acting in good faith, apply its own independent business judgment to
determine whether the motion should be granted?
 Whether the committee has the power to cause the case to be dismissed, and whether that
decision will be upheld
o Rule
 If a shareholder asserts that a corporation's best interests are served by maintaining a properly
instituted derivative suit, but a board committee asserts that the same suit should be dismissed
because it is not in the corporation's best interests, the reviewing court is faced with a problem.
 (1) On the one hand are the interests of the individual shareholder, for whom the
derivative suit is an important tool for guaranteeing good corporate governance.
 (2) On the other hand are the interests of the corporation and its body of shareholders,
for whom derivative suits may be a hassle and a needless expense.
 Some courts  have held simply that the business-judgment rule allows a board of directors to
terminate a derivative suit based on a vote by a disinterested committee, and that this decision
is thus insulated from judicial review.
 However, the business-judgment rule alone is insufficient to analyze the decision that
the action should be dismissed.
 Instead, the determination about whether the lawsuit should be maintained rests in the
discretion of the Court of Chancery.
 Accordingly, after the independent committee conducts a thorough investigation and
determines that a lawsuit should be dismissed, the committee must cause the corporation to file
a motion to dismiss in the chancery court.
 In reviewing the motion,
 (1) the court must first analyze the committee's independence and good faith and the
bases supporting the committee's conclusions.
 (2) If the court is satisfied that the committee was independent and acting in good faith,
the court must then apply the court's own independent business judgment to determine
whether the motion should be granted.
 Rule: In reviewing an independent committee's motion to dismiss a shareholder's derivative
lawsuit on the basis that maintaining the lawsuit is not in the corporation's best interest, the
chancery court must (1) analyze the committee's independence and good faith and the bases
supporting the committee's conclusions; and (2) if satisfied that the committee was
independent and acting in good faith, apply the court's own independent business judgment
to determine whether the motion should be granted.
o Test
 Was a demand made?
 Yes  was the demand honored
o Yes  lawsuit goes forward
o No  decision governed by BJR—i.e. you lose unless you overcome BJR
 When you asked the board to sue you are already admitting that the
board was disinterested (because If it wasn’t the demand would have
been excused bc would have been futile)
 No  you lose unless demand is excused
o If demand excused  corporation will still respond with SLC and question
becomes  Will the court honor the SLC’s recommendation not to move
forward with the litigation?
 Court will investigate independence and good faith of committee
 If independence/good faith, courts will decide in its own independent
business judgment whether the decision of the committee should be
upheld
o Application
 In this case, demand was excused, and Maldonado properly instituted the derivative lawsuit.
The independent investigation committee has the power to seek the termination of the lawsuit;
however, the ultimate determination rests in the chancery court's discretion after the
committee causes the corporation to make a motion to dismiss.
 Aronson v. Lewis (delaware)
o Facts
 Harry Lewis (plaintiff) owned stock in Meyers Parking System, Inc., a Delaware corporation. Leo
Fink was a director of Meyers and owned 47 percent of its outstanding stock. Meyers’s directors
approved a lucrative employment agreement for Fink and made interest-free loans to him.
Lewis brought suit against Meyers and its directors (defendants), alleging that the transactions
were only approved because Fink had personally selected the directors of Meyers, and that the
transactions had no business purpose and were a waste of corporate assets.
 The complaint stated that no demand was made on the board of directors to address the
alleged wrongs because (1) the directors participated in the wrongdoings; (2) Fink selected the
directors and thus controlled the board; and (3) litigation brought by the directors would require
them to sue themselves, precluding effective prosecution.
 i.e. demand was futile because directors not disinterested
 Meyers and its directors filed a motion to dismiss based on Lewis’s failure to make a demand or
to demonstrate that such a demand would be futile.
o Issue
 May a court dismiss a shareholder’s derivative action if the shareholder has failed to make a
demand on the board or allege facts sufficient to demonstrate that such a demand would be
futile?
 When is a stockholder’s demand upon a board of directors, to redress an alleged wrong to the
corporation, excused as futile prior to the filing of a derivative suit
 Whether the demand is futile and therefore excused
o Rule
 BJR  Absent an abuse of discretion, courts will presume that directors, in making a business
decision, acted on an informed basis, in good faith, and with the honest belief that the action
was in the company’s best interests.
 Although this business-judgment rule applies to director action, it may also apply to a
conscious decision to refrain from acting.
 business-judgment rule applies to a board’s decision to refuse a shareholder’s demand
to bring suit, or, if the demand requirement is excused due to the futility of making such
a demand, to a board’s decision to terminate a derivative action brought by
shareholders. 
 “futility’ test  whether, under the facts alleged, there is a reasonable doubt as to whether
 (1) the directors making the decision were disinterested and independent, or
 (2) the transaction at issue was otherwise the product of valid business judgment.
o If there is a reasonable doubt as to either factor, the demand requirement will
be excused.
o To show that demand would be futile, a plaintiff alleging domination and
control over the directors must allege facts demonstrating that the directors are
beholden to the controlling person, either through personal or other
relationships.
 Rule: A court may dismiss a shareholder’s derivative action if the shareholder has failed to
make a demand on the board or allege facts sufficient to demonstrate that such a demand
would be futile.
o Holding
 (1) In this case, Lewis’s allegation that Fink personally selected the directors, even given his 47
percent stock ownership, is not sufficient to overcome the presumption of board independence.
 The complaint therefore fails to render demand futile on the charge of lack of
independence.
 (2) Lewis also alleges that because any suit would be against the directors themselves as the
parties who approved the wasteful transactions, a demand for suit would have been futile
because the directors would have been motivated to avoid personal liability.
 Not only does the complaint fail to show that the transactions were wasteful, but
directors have broad corporate power to fix officer compensation and make loans.
 The complaint fails to create a reasonable doubt that the board’s actions are protected
by the business-judgment rule and thus fails to allege facts sufficient to render
demand futile on the charge of corporate waste.
 (3) Finally, Lewis alleges that demand would be futile because if a suit were approved, the
directors would need to bring the suit against themselves, which would prevent effective
prosecution.
 Without any facts to allege a lack of independence or a failure to properly exercise
business judgment, this argument is insufficient.
 Lewis has failed to allege particularized facts indicating that the Meyers directors were
interested, lacked independence, or acted contrary to the company’s best interests, and has
therefore failed to raise a reasonable doubt as to the business-judgment rule’s applicability.
 Because Lewis has thus failed to show that a demand on the board would have been futile, the
Court of Chancery’s denial of the motion to dismiss is reversed, and the matter is remanded with
leave for Lewis to amend the complaint.
o Notes
 (1) if you are plaintiff and you make the demand, you are stuck with the decision because it is
governed by the BJR because that demand is a tacit acceptance of the lack of demand futility
(you cant plead demand excused if you make a demand)
 (2) if you don’t make a demand  you have high burden of showing that the demand would be
futile
 Because these are often at the motion to dismiss stage, no discovery has even been
conducted so how do you come up with particularized facts to argue a decision was
wrongful
 You have to exercise rights you have as a shareholder to inspect the corporate books
and records which is still a difficult thing to do OR ask the judge for limited discover to
respond to the motion
 Virtually every demand is refused  what does that say about the decision makers and those
that are placed on the SLC?
 There has to be some bias going on simply because of the statistics
 Some research suggests that a lot of these cases lack merit and because every demand
is refused, demand is rarely even made in Delaware
 Gordon v. Goodyear (Idaho)
o Facts
 Navigant had a compensation committee charter that established the duties of its executive
compensation committee. The charter required the committee to consider several factors when
determining compensation for Navigant’s executives. From January 2006 to December 2010,
Navigant’s stock price dropped from $21 per share to $9.20 per share. Navigant’s board of
directors issued a proxy statement recommending pay increases or bonuses to Navigant’s
executive officers for 2010. The proxy statement stated that the compensation vote was not
binding on the board. Navigant shareholders voted against approving the recommended
compensation. The board nevertheless approved the recommended compensation.
 William Goodyear (defendant) was on the board of directors, and he was the only member of
the board that was also a Navigant executive officer. Natalie Gordon (plaintiff), a Navigant
shareholder, filed a shareholder derivative action claiming that Navigant’s executive
compensation for 2010 was excessive. Gordon did not make a demand on the board that it
reverse its compensation decision prior to filing suit. Gordon claimed that demand was futile.
Goodyear filed a motion to dismiss based on the lack of demand.
o Issue
 Excessive executive compensation
o Rule
 Demand on a board of directors is futile if the plaintiff-shareholder establishes reasonable doubt
that (1) the directors were disinterested and independent, and (2) the challenged transaction
was otherwise the product of a valid exercise of business judgment
  If such reasonable doubt is not established, the shareholder must make a demand on
the board of directors prior to filing a shareholder derivative suit.
 The shareholder-plaintiff must allege facts with particularity to make the required
showing of demand futility. Otherwise, the shareholder fails to overcome the
presumption of the business judgment rule, which protects board members’ authority
to make reasoned business decisions. 
o Holding
 In this case, Gordon has not established a reasonable doubt that the Navigant directors were
disinterested and independent, or that the challenged transaction was the product of valid
business judgment. As a result, demand on the board was not futile. Thus, the complaint gets
dismissed
 (1) First, only one of the eight directors, Goodyear, benefitted from the approved compensation
packages.
 This is not sufficient to carry Gordon’s burden of demonstrating reasonable doubt that
the entire board was disinterested and independent, particularly because Gordon did
not claim that Goodyear was on the compensation committee or held any particular
power over the other board members.
 A MAJORITY is not interested and conflicted
 (2) Second, Gordon failed to rebut the presumption of the business judgment rule.
 The shareholders’ disapproval of the compensation is not sufficient to make this
showing, because the shareholder vote was expressly not binding.
 Further, while Gordon relies on a purported lack of link between Navigant’s
performance and its executive compensation, this is only one factor that the
compensation committee was required to consider. The charter required the
compensation committee to consider a number of factors in determining executive
compensation.
 (3) board would be sued if they approved the lawsuit – doesn’t that make them conflicted
 No  there would have to be a showing to the court of a substantial likelihood of
liability
o Notes
 How can plaintiff prove that the board isn’t disinterested or independent?
 Show that the majority of the board is on both side of the bad transaction or that they
received a unique personal benefit by way of the bad transaction
 How can plaintiff prove that the challenged transaction is not protected by BJR
 (1) Plaintiff can show that the board was not informed when they made the executive
compensation decision
 (2) Show that the compensation amounts to true waste under objective theory
 (3) That they acted in bad faith which would implicate the duty of loyalty
 In Re Oracle Corp. Derivative Litigation (delaware)
o Facts
 Shareholders of Oracle Corp. (defendant) filed a complaint alleging that four members of
Oracle’s board of directors, Ellison, Henley, Lucas and Boskin (all defendants) had engaged in
insider trading by selling stock in January 2001, with the knowledge that on March 1, Oracle
would release an earnings report showing that it failed to meet its sales projections. Oracle’s
board formed a litigation committee composed of individuals, neither of whom had been on the
board at the time of the alleged insider trading.
 However, both of the litigation committee members were business professors at Stanford
University, an institution which had received sizable donations from the accused insider traders.
Boskin was also a professor at Stanford, and Lucas served on a board at Stanford. The litigation
committee found that Ellison, Henley, Lucas and Boskin had not engaged in insider trading, and
that it would thus not benefit Oracle for the litigation to continue. The litigation committee
stated its findings in a report that detailed its thorough investigation. The report detailed the
ways in which the committee was independent, but failed to mention the fact that the accused
traders were major donors to Stanford. Based on the report, Oracle moved to have the
litigation dismissed.
o Issue
 Can two professors who are a part of the SLC be independent when analyzing the actions of
individuals who are major donors to the university that employs the professors?
 Whether the members of the special litigation committee were independent from oracle and
the director defendants?
o Rule
 A member of the board of directors cannot be independent if he or she cannot analyze a
problem objectively, with only the best interests of the corporation in mind.
 It is human nature that a person cannot be completely objective when he has to
evaluate a person with whom he works. The same is true when someone has to
evaluate a benefactor.
o Holding
 In this case, the members of the litigation committee worked with professors at the same
university and in the same department as Boskin, one of the accused. Moreover, Boskin, Ellison,
Henley and Lucas were all major donors to the business school that employed both members of
the committee.
 The committee members argued that they did not know about the directors’ relationships with
Stanford, and that was the reason they failed to mention the potential source of bias in their
report. As the committee members did not know that the directors were Stanford donors, the
committee’s conclusions could not have been tainted by bias.
 However, the committee’s claim that it did not know about the directors’ relationship with
Stanford is not a credible claim, considering the prominence of the Oracle directors in the
Stanford community, and considering how Boskin and Lucas have actual roles in and around
Stanford’s Business school. It is no slight against the members of the committee to say that they
might not be impartial in this circumstance, and in fact it seems likely from their report that they
have reached the right conclusion, and that it would be better for Oracle if this litigation ended.
 However, given the likely inability of the litigation committee to be independent in their
judgment, their decision cannot be called an independent informed judgment, and because of
that, Oracle’s motion to dismiss the litigation is denied.
o Test / Delaware / Traditional Approach
 (1) was demand made?
 Yes  was it refused?
o Yes decision reviewed under BJR (making demand is admittance that the case
is not demand excused because disinterested and independent)
o No  corporation proceeds with suit
 No  was the demand excused because (a) not disinterested or independent; or (b)
transaction not protected by BJR
o Yes  Analyze SLC’s decision to dismiss
 Motion to dismiss granted  if SLC (a) independent; (b) SLC acted in
good faith; and (c) courts independent judgement is satisfied
 Motion to dismiss denied  if (a) no independence; (b) no good faith;
or (c) lacks business justification from courts view
o No  Case dismissed
o Notes
 Does the Sandford connection actually put money in the pockets of the special litigation
committee?
 No
 if the special litigation committee actually voted to allow the insider trading suit to go forward,
do you think they would lose their jobs at Stanford?
 No (they were tenured)
 What makes this case lack the necessary independence to rule in favor of the special litigations
committee’s motion to dismiss
 Implicating a colleague and former professor as well as possible influence knowing that
you would have implicated a major donor to your home institution created an
independence problem which means in additional to a pecuniary interests in the
outcome of the case, we can also consider social policies when considering the
impartiality of a special litigation committee

 Cuker v. Mikalauskas (PA decides not to follow Delaware standard)


o Facts
 PECO is a publicly regulated Pennsylvania utility. A group of minority shareholders demanded
that PECO authorize litigation against some of its directors and officers, based on allegations
that they had damaged PECO by mismanaging the collection of overdue accounts.
 Another group of minority shareholders (plaintiffs) filed a complaint against PECO directors and
officers, which made the same allegations of wrongdoing.
 PECO’s board created a special litigation committee to investigate the allegations raised in the
demand for litigation and the complaint. Only members of the PECO board who were not
named in the demand for litigation or in the complaint acted in creating the special committee,
which consisted of three outside directors not employed by PECO.
 The special committee concluded that there was no evidence of bad faith, self-dealing,
concealment, or breach of the duty of loyalty by the directors and officers, and that they
exercised sound business judgment and acted in the company’s best interests. The
nondefendant members of the PECO board voted to reject the demand for litigation and
terminate the action started by the complaint. 
o Issue
 Does a corporation’s board have the power to terminate derivative suits brought by minority
shareholders?
o Rule
 ALI principles for dismissal of derivative litigation require a
 (1) universal demand (you have to make the demand)
 (2) a sliding scale of judicial review but a very high level of deference to the business
judgment rule
 This court finds that the business judgment rule applies in Pennsylvania, and that the rule
permits a Pennsylvania corporation’s board to terminate derivative suits brought by minority
shareholders based on the validity of the board’s decision to terminate the litigation in
accordance with appropriate standards
 If a board’s decision not to bring suit or to terminate existing suits was made according
to appropriate standards, a reviewing court should dismiss the derivative action
before litigation on the merits.
 If there was no fraud or self-dealing, the decisions were within the scope of the
directors’ authority, the directors exercised reasonable diligence, and the directors
honestly and rationally believed their decisions were in the company’s best interests,
the business judgment rule will apply to prevent judicial intervention with the board’s
decision.
 BJR  in determining whether to apply the business judgment rule, courts must look to the
circumstances surrounding the board’s decision, including whether: the board or its special
committee was disinterested; assisted by counsel; prepared a written report; independent;
conducted adequate investigations; and rationally believed its decision was in the
corporation’s best interests.
 American Law Institute Principles of Corporate Governance  advance the policies of the
business judgment rule, and will provide specific guidelines for the trial court to address
shareholder derivative actions such as this one.
o Notes
 Decides not to follow Delaware complex rules and instead adopts regulations imposed by the
ALI
 MBCA § 7.42: Demand (TX)
o No shareholder may commence a derivative proceeding until:
 (1) a written demand has been made upon the corporation to take suitable action; and
 (2) 90 days have expired from the date the demand was made unless the shareholders has
earlier been notified that he demand has been rejected by the corporation or unless irreparable
injury to the corporation would result by waiting or the expiration of the 90 day period
 MBCA § 7.44: Dismissal
o (a) a derivative proceeding shall be dismissed by the court on motion by the corporation if one of the
groups specified in [subsection (b) or (e)] has determined in good faith, after conducting a reasonably
inquiry upon which its conclusions are based, that the maintenance of the derivative proceeding is not
in the best interest of the corporation
 Dismiss (a) if appropriate group determines (b) good faith (c) after conducting reasonable
iniquity and (d) comes to the conclusion that the proceeding not in best interest of corporation
o Unlike Delaware law  MBCA does NOT permit the court to take an independent investigation and to
use its own business judgment to determine whether the case should be dismissed
 Focuses wholly on the procedural components of independence and reasonable investigation
 MBCA § 7.44: Dismissal (appropriate group)
o (b) unless a panel is appointed pursuant to subsection (e), the determination in subsection (a) shall be
made by
 (1) a majority vote of independent directors present at a meeting of the board of directions if
independent directors constitute a quorum; or
 (2) a majority vote of a committee consisting of two or more qualified directors (without a
material interest) appointed by majority vote of qualified directors present at a meeting of the
board of directors, regardless of whether such qualified directors constitute a quorum
 This means  independent directors or independent committee made the decision or
the recommendation to dismiss
 MBCA § 7.44: Dismissal
o (c) If a derivative proceeding is commenced after a determination has been made rejecting a demand by
a shareholder, the complaint shall allege with particularity facts establishing either (1) that a majority
of the board of directors did not consist of qualified directors at the time the determination was made
or (2) that the requirements of subsection (a) have not been met
o (d) if a majority of the board of directors consisted of qualified directors at the time the determination
was made, the plaintiff shall have the burden of proving that the requirements of subsection (a) have
not been met; if not, the corporation shall have the burden of proving that the requirements of
subsection (a) have been met
 This means  if the plaintiff filed the derivative suit AFTER the board rejected the demand
 (c) requires that she allege in detailed facts showing either that a majority of the board
was tainted or that the underlying requirements of (a) were not met
 (d) allocates the burden of proof  if when the board rejected the demand a majority
of the board was qualified,  plaintiff must prove that the underlying requirements of
(a) were not met
o If a majority of the board was tainted and therefore unqualified  the
corporation just demonstrate that the underlying requirements (a) were
satisfied
 Test
o Derivative suits may not be discontinued or settled without the courts approval
 In addition, most statutes provide that if a court determines that a proposed discontinuance or
settlement will substantially affect the interests of the shareholders the court would have to
direct that notice be given to shareholders effective
 Most derivative suits are settled  in reviewing the proposed settlements courts will usually
consider the size of the potential recover v. the size of the proposed settlement, the possibility
of success of litigation, the financial position of the defendants and the reasonable ness o the
proposed fee to be paid to the plaintiffs lawyer
 Reasoning for giving notice to shareholders
 To solicit their input on the proposed dismissal or settlement
 Shareholders will likely show up and voice an objection
 Director / Officer Liability
o Dangerous  can expose one to a lot of litigation and potential liability
o Expenses alone of getting a lawyer can be crippling, and as a result many people s9mply refuse to serve
in corporate management positions unless they are protected from exposure
o 3 general layers of protection directors/officers will look for
 (1) exculpatory or exonerating provisions in the articles of incorporation
 (2) liability insurance
 (3) indemnification provisions
o (3) Indemnification
 The corporation reimburses its directors or officers for expenses and attorneys fees that were
incurred because that person was sued
 Issues are handled by statute in each state but there are generally three categories dealing with
indemnification
 (1) when indemnification is required
o Where the director was wholly successful on the merits / she won
 (2) when indemnification is prohibited
o When the director is held liable to the corporation / the director was found to
have breached a duty
 (3) when indemnification is permitted
o No one has won or lost and the case has settled and there is some discretion for
that indemnification
o (2) exculpation clauses
 All states have passed clauses permitting the corporation to provide in its articles that directors
will not be liable for damages in certain circumstances
 Generally, these statutes provide that the articles may include a provision eliminating or limiting
the liability of a director to the corporation or to its shareholder for money damages for any
action taken or any failure to take an action as a director
 no one would serve as a director without these clauses
 in every state – the power to exculpate from liability is not unlimited  in most states these
clauses won’t apply to cases involving a receipt of improper financial benefits, intentional
infliction of harm on the corp or shareholders, unlawful distributions, or the intentional violation
of a criminal law
 basic point  these exculpatory clauses protect directors from duty of care issues but
usually not duty of loyalty
o if you breached duty of care  you can probably look to article for protection
o if you have breached duty of loyalty  you will not be protected from these
exculpatory clauses
o (1) director and officer liability insurance
 Purchased from third party insurance companies
 All based on contract and vary from policy to policy but almost all of them have a strict exclusion
where coverage will not apply to intentional bad acts which often times are duty of loyalty
claims
 Mostly will cover duty of care issues and general negligence on the part of the director/officer

Chapter 13 – Dissension in the Closely Held Corporation


A) Deadlock
a. Closely held v. publicly held
i. Closely held  few shareholders, no market for stock, substantial shareholder participation
1. Often its common to have shareholders who also are the officers who also are directors and
employees
2. This makes closely held corporations function more like a partnership
3. The shareholder has to be careful in what hat they’re wearing in making decisions (fiduciary,
individual shareholder decision)
ii. Publicly traded  shareholders don’t participate on day to day basis
b. Dissention
i. Deadlock between the members and oppression of a minority member
c. Deadlock  ways they arise
i. There is a 50-50 split on share ownership with straight voting
ii. High quorum and voting requirements for shareholders meetings so that a minority has an effective
veto power
iii. High quorum and voting requirements at the board of directors
iv. Where there are even number of directors, and each faction gets half of the votes
d. Why not always prevent deadlock?
i. Usually rise from a failure to allocate control in advance, sometimes business lawyers provide
opportunities for deadlock
ii. Some business relationships should not proceed unless there is unanimous consent to the action
1. There may be some issues that are so important that you don’t want the corporation to
move forward without a least a super majority vote
e. Consequences deadlock
i. Spectrum  sever to lenient
1. Extreme  dissolution through filing articles of dissolution with secretary of state)
2. They can be judicially dissolved by going to a judge and saying this company can’t function
any longer
f. Ways to address deadlock in advance
i. Mandatory mediation clause
ii. Prevent number of shares from being able to create a deadlock
iii. Always have odd number of directors
 Gearing v. Kelly
o Facts
 Radium Chemical Company’s bylaws require a board of four directors, a majority of whom
constitute a quorum. In 1955 the board included Mrs. Meacham, Kelly Sr., Kelly Jr., and
Margaret Lee. In 1961 a board of directors’ meeting was called, at which Kelly Sr., Kelly Jr.
(defendants), and Margaret Lee were present. Margaret Lee offered her resignation, which was
accepted. Mrs. Meacham avoided the meeting to prevent a quorum from assembling. Following
Margaret’s resignation at the meeting, the two Kellys elected Kelly Sr.’s son-in-law Julian
Hemphill to replace Margaret on the board (without a quorum present). Mrs. Gearing (plaintiff),
a stockholder and Mrs. Meacham’s mother, brought suit, along with other stockholders, seeking
to have the election set aside and a new election ordered. 
 4 directors  quorum would constitute 3 (majority)
o If she doesn’t attend there would never be a quorum for voting purposes
o Issue
 May a director, and stockholders associated with that director, request a new corporate election
based on a lack of quorum caused by the director’s refusal to attend the stockholders’ meeting?
o Rule
 A court may order a new election for corporate directors as justice may require.
 A director, or stockholders associated with that director, may not request a new corporate
election based on a lack of quorum, when the lack of quorum was caused by the director’s
refusal to attend the stockholders’ meeting.
 the stockholders may not complain of an irregularity that they have caused
o Holding
 Mrs. Meacham had notice of the board of directors’ meeting, and avoided it specifically to
prevent a quorum from assembling and voting on a new director. Allowing Mrs. Gearing, as a
stockholder directly associated with Mrs. Meacham, to challenge board actions impacted by
Mrs. Meacham’s refusal to attend board meetings, would permit the director and stockholder to
frustrate board action until their demands were met. In addition, the new election requested as
relief would not benefit the stockholders, since Mrs. Meacham would be required, futilely, to
attend the new election. Under these circumstances Mrs. Meacham’s refusal to attend the
board meeting bars the stockholders from seeking a new election under the court’s equitable
powers. The Appellate Division’s decision is affirmed.
o Notes
 Meacham’s willful refusal to attend the meeting deprived her of her right to object to the
selection of the new director regardless if the quorum requirements were met
 Waiver or estoppel of judgement against her
o Dissent
 Following the resignation of Margaret Lee at the 1961 board of directors’ meeting, only Kelly Sr.
and Kelly Jr. were present to cast votes for Julian Hemphill. The company’s bylaws require a
majority of the board’s four directors to be present to constitute a quorum. Two board
members were insufficient to elect a new director, and the election is therefore void and must
be set aside.
 New election should be ordered regardless if the same result would be produced
o Majority v. Dissent
 Majority  there is no law imposing a duty on a shareholder or director to attend such a
meeting like this
 If the majority is correct  we are now saying that a director has a statutory duty to
hand the corporation over to her shareholder rivals
 Some argue this cannot be correct  your stake in the business is a property interest
and you should have the right to fully protect that interest in any legal manner
 What is wrong with NOT doing what the majority did (attend the meeting) 
corporation is deadlocked
 In re Radom & Neidorff, Inc
o Facts
 Radom & Neidorff, Inc. was successfully operated by David Radom (plaintiff) and his brother-in-
law Henry Neidorff, who were the corporation’s sole and equal stockholders. Upon Henry’s
death in 1950, his shares passed to Radom’s sister, Anna Neidorff (Neidorff) (defendant). Several
months after Henry’s death Radom petitioned the court for dissolution of the corporation under
section 103 of New York’s General Corporation Law. Radom’s petition acknowledged that the
corporation was solvent and successful, but alleged that, since Henry’s death, Neidorff has
refused to cooperate with Radom and had refused to sign Radom’s salary checks, and that
election of directors at the stockholders’ meeting had been impossible due to disagreements
between them. 
o Issue
 May the court deny a petition for corporate dissolution if competing interests are not
preventing efficient management or impeding the corporation’s purpose?
o Rule
 Courts have the discretion to order a corporate dissolution where competing interests are
preventing efficient management and impeding the corporation’s purpose, if dissolution
would benefit stockholders and not injure the public.
 §103  In case of deadlock, if an even number of directors are equally divided respecting
management of affairs or if the votes of the stock holders are divided, the holders of ½ of the
stock entitled to vote at the election of the directors may file a verified complaint for judicial
dissolution
 Note: this provision would never work for a minority shareholder because it says you
have to have ½ of stock entitled to vote
o Holding
 Radom’s petition acknowledged that the corporation is solvent and successful. The appellate
court noted in its decision that the corporation’s activities have not been paralyzed, but that
instead its profits have increased, and its assets have tripled during the three years that this case
has been pending. While there is feuding and distrust between the two equal stockholders,
there is no stalemate on corporate policies. It is also undisputed that the corporation is
flourishing, that dissolution is not necessary for the corporation or for its stockholders, and that,
while Radom is in a difficult situation, the only cognizable grievance he can bring before the
court is his claim for nonpayment of salary, which is an insufficient ground for corporate
dissolution. In this case, competing interests are not preventing efficient management or
impeding the corporation’s purpose. Under these circumstances, the appellate court had the
discretion to dismiss Radom’s petition for dissolution. 
o Notes
 Construed the statute to give the court discretion and the court here doesn’t think that
dissolution is in the best interest of all involved because it is still functioning, and it is still making
money
 The order WOULD be granted for judicial dissolution ONLY when the competing interest are so
discordant as to prevent efficient management and the object of its corporate existence cannot
be maintained
 Prime inquiry  whether judicially imposed death would be beneficial to the
stockholders and NOT injurious to the public
o Dissent
 The General Corporation Law permits a petition for corporate dissolution if the stockholders’
votes are so divided that they cannot elect a board of directors, or if there are an even number
of directors who are hopelessly deadlocked. This is the case here. It is undisputed that Radom
and Neidorff have been unable to elect a board of directors. The court could also find, on the
facts alleged, that the stockholders are hopelessly deadlocked as to the corporation’s
management, and that the corporation cannot continue to function effectively or profitably
under these conditions. As such, the court should have allowed a hearing on whether
dissolution was an appropriate remedy.
 Under the circumstances, there does not appear to be an adequate alternative remedy.
 Radom could continue on as president and manager of the corporation without
compensation and at odds with Neidorff
 he could quit the enterprise and risk a loss greater than the loss involved in terminating
the business; or
 he could start a competing business, exposing himself to suit from the corporation.
 Buy out the other director
 An orderly dissolution appears to be the most reasonable course .
 In addition, while Neidorff and the majority rely on the fact that the corporation is currently
solvent and profitable, the only issue under section 103 of the General Corporation Law is
whether there is a deadlock in the corporation’s management. Whether the business is
profitable is irrelevant. Radom should not have to wait until the corporation starts operating at
a loss to have his petition considered. The appellate court’s judgment should be reversed, and
the trial court’s affirmed.
 MBCA §14.30: Grounds for Judicial Dissolution
o The [relevant adjudicator body] may (intended to be discretionary) dissolve a corporation
o The grounds for involuntary dissolution:
 Board deadlock: directors cannot agree; shareholders cannot break the impasse; and the
business is suffering (§ 14.03(2)(i))
 Shareholder deadlock: shareholders have been unable to elect new directors (§ 14.03(2)(iii))
 Misconduct: those in control have engaged in “illegal, oppressive, or fraudulent conduct” (§
14.03(2)(ii))
 Waste: corporate assets are being “misapplied, or wasted” (§ 14.03(2)(iv))
 What do courts do when deadlock?
B) Oppression
a. General rule: shareholders do not owe fiduciary duties to one another / directors and officers owe these
duties
i. Some courts and legislatures recognize that without some duties in place, minority shareholders in
close corps could be significantly harmed (oppression)
b. Oppression scenario
i. Majority shareholder terminates the minority shareholders participation in management and then
refuses to declare dividends, so the stockholder is stuck with no position in the company, no salary,
no dividends, and they’re basically frozen out
c. Hypo
i. XYZ form XYZ corp, each owns 100 shares, each is director on three-person board, they each have
job with the corporation with nice salaries and the corp is paying regularly healthy dividends
ii. What if YZ become close and end up not liking X  this means X now loses on every decision 2-1
iii. YZ can even fire X  assume they do so
iv. Now YZ also choose to bump up their own salaries and stop declaring dividends since they have the
business judgment right to do so
v. X now gets 0 out of the company despite the fact that she had a 1/3 interest
vi. We would say this situation is a classic shareholder oppression/freezeout
d. Change in courts
i. Start considering, because of this situation, the imposition of partner-type fiduciary duties in the
closely held corporation since those are more akin to partnership
 Donahue v. Rodd Electrotype Co. (CL case recognizing partner-like duties)
o Facts
 Harry Rodd (defendant) served for many years as the president and general manager of Rodd
Electrotype Co. In 1955, Harry Rodd held 200 of Rodd Electrotype Co.’s 250 shares, representing
an 80 percent stake in the company. Joseph Donahue owned the remaining 50 shares, which
passed on his death to his wife Euphemia Donahue (plaintiff) and his son. By the end of the
1960s, Harry Rodd had ceded the management of the corporation to his sons Charles and
Frederick Rodd (defendants), and Harry wished to dispose of his shares. He gave most of them
to his children as gifts. Additionally, Charles and Frederick – who controlled the board – caused
the corporation to purchase 45 of Harry’s shares for $800 per share. When the Donahues
learned of the purchase, they offered to sell their shares to the corporation on the same terms
given to Harry. The board rejected the offer. Euphemia Donahue sued Harry, Charles, and
Frederick Rodd, as well as the third member of the board, for breaching fiduciary duties owed
to her as a minority shareholder because the company failed to afford her an equal
opportunity to sell her shares to the corporation. She asked the court to rescind the
corporation’s purchase of Harry Rodd’s stock.
 Defendant argument  this stock purchase was within the powers of the corporation, its
protected by the BJR and there is no right to an equal opportunity in stock purchases
o Issue
 Do shareholders in a close corporation owe one another heightened duties of care and loyalty?
o Rule
 CL  Stockholders in close corporations owe one another strict duties of care and loyalty,
substantially the same to the duties owed among partners in a partnership.
 “the utmost good faith and loyalty” / stock holders in closely held corporations must
discharge their management and stockholder responsibilities in conformity of this
strict good faith standard
 Close corporations  are corporations which consist of a small number of shareholders, have
no market for their shares, and are substantially controlled by majority shareholders.
 Close corporations present a unique opportunity for controlling shareholders to exploit
minority shareholders.
 In a large, publicly traded corporation  a dissatisfied shareholder may simply sell her shares.
 Because there is little or no market for shares in a close corporation, this option is not
available to oppressed minority shareholders in a close corporation.
 In partnerships  a disaffected partner can terminate the partnership at any time and
withdraw his capital; shareholders lack this recourse.
 Since close corporations resemble partnerships in many respects, analogous duties are
owed.
 Shareholders in a close corporation  must exercise strict good faith and loyalty in their
dealings with one another. Controlling shareholders may not use their authority to obtain
special advantages from the corporation at the expense of the minority shareholders. If
controlling shareholders cause the corporation to repurchase their shares of stock, they have
obtained a benefit from the corporation. Therefore, they may not do so unless they make the
same terms available to all shareholders.
o Holding
 Rodd Electrotype Co. is clearly a close corporation. There are only a few shareholders, there is
no general market for the stock, and most of the shareholders exercise control over the
operation of the company. Thus, the strict duties of good faith and loyalty apply. Because the
Rodd family caused the corporation to repurchase some of their shares, they were required to
offer the same terms to the Donahues. The judgment of the appellate court is therefore
reversed. The case is remanded to the trial court, and the Donahues are entitled either to have
their shares purchased by the corporation at $800 per share, or to compel Harry Rodd to repay
the amount he received for his stock to the corporation.
 Problem with holding  significant judicial overreaching / the BJR should protect these
decisions and closely held corporations are not partnerships so why should we treat
them the same
 The Equal Opportunity doctrine (CL) (equal treatment of shareholders in a closely held corp)
o Where a closely-held corporation repurchases the shares of a dominant shareholder, the majority’s
fiduciary duty may be satisfied only by allowing the minority to participate in the repurchase on the
same terms
o Since closely held corporations are similar to partnerships, shareholders should owe the same fiduciary
duties to one another that general partners owe to one another
 Wilkes v. Springside Nursing Home (modifies donahue holding and sets up the fiduciary duty doctrine that
prevails today in a lot of jurisdictions)
o Facts
 Wilkes (plaintiff), Riche, Quinn, and Connor were the four directors of the Springside Nursing
Home, Inc. (Springside) (defendant), each owning equal shares and having equal power within
the corporation. Eventually the relationship between Wilkes and the other three directors
(defendants) soured. When Springside became profitable, the defendants voted to pay out
salaries to themselves, but did not include Wilkes in the group to whom salary would be paid.
Then, at an annual meeting, Wilkes was not reelected as director and was informed that he
was no longer wanted in the management group of Springside. Over the course of these
events, Wilkes faithfully and diligently carried on his duties to the corporation. Wilkes brought
suit against the defendants for breach of their fiduciary duty owed to him. 
o Issue
 Are majority shareholders in a close corporation liable for breach of a fiduciary duty to a
minority shareholder if they remove him from office and cut off his salary without any showing
of misconduct?
o Rule
 Majority shareholders in a close corporation owe minority shareholders a strict duty of the
utmost good faith and loyalty, unless a legitimate business purpose can be demonstrated to
justify a breach of that duty.
 Where no legitimate business purpose can be shown, the majority shareholders are
liable for their breach of that duty
o Holding
 In this case, there has been no showing of misconduct or poor performance in Wilkes’s role as
director. It was merely a “personal desire” of the defendants to remove Wilkes from office and
deny him salary. The defendants’ actions constitute an unlawful corporate freeze out and
because they did not show any legitimate business purpose for the freezing out of Wilkes,
they are liable for breach of their fiduciary duty to him. The lower court is reversed and the
case is remanded for a final determination on damages.
o Notes
 Retreat from Donahue  recognizes that directors in a business, especially when a majority
vote is present, they do need a large measure of discretion with respect to some of their
decisions:
 Declaring/withholding dividends
 Setting salaries of office
 Dismissing directors
 Hiring/firing corporate employees
 Broad equal opportunity rule (Donahue interpretation)  would make it next to impossible to
operate a business
o Test
 (1) plaintiff must show that the controlling shareholders treated her oppressively
 Need real evidence of this / not hearsay or self-serving declarations
 (2) if she does, the burden shifts to the defendants to show a legitimate business reason for
the action
 Firing for cause, necessary downsizing, etc.
 (3) if the defendants make such a showing, the plaintiff may still win if she shows that the
legitimate purpose could have been met by a less restrictive alternative
 If finances are bad, instead of firing me, why don’t we all take a 10% hit, or reduce
dividends we’re going to pay
 Tells us  unequal treatment is OK and permissible with a legitimate business purpose and no
less harmful alternative
 Donahue  that shareholders in closely held corporations owe a fiduciary duty to one another
is still intact (strict standard of utmost good faith and loyalty) but the modified doctrinal
framework eliminates the equal opportunity rule so long as there is a legit business purpose for
the treatment (no less harmful alternative)
 Donahue result under Wilkes approach
o (2) There WAS a legitimate business purpose – the desire to facilitate the
retirement of a manager who is no longer making productive contributions to
the firm
o (3) The less harmful to the minority analysis, however, may be problematic 
maybe there could have been a termination and removal but kept his stock in
the company (that could remove the unproductive manager)
 Note: first thing that was asked  what was their [directors] expectation when making the
company?
 Important for courts to say, if you invested in this company, knowing that you were
going to be an employee and an officer and that you were going to have management
control, this was your baby to begin with // this is different than someone coming in as
passive investor with no expectations in the business to then say that they are being
oppressed in certain situations
 Merola v. Exergen Corp (indicates that minority investors in close held corps are not simply as a result of their
shareholder status automatically entitled to employment and other non-traditional shareholder benefits)
o Facts
 Exergen Corp. (defendant) is a close corporation that has been controlled since its inception by
its president and founder, Francesco Pompei (defendant). Steven Merola (plaintiff) began
working as a part time employee (no investment) and then became full-time at Exergen in 1982,
with the understanding that he would have the opportunity to become a major shareholder of
the company (he was a full-time employee, officer and shareholder). During the years 1982 and
1983, Merola purchased over 5,000 shares of Exergen stock, first at $2.25 a share and then at $5
per share. He was later terminated without cause, though this was permissible under his
contract. He sold his stock back to the company in 1991 for $17 per share. Merola sued Exergen
and Pompei, arguing that by terminating him, Pompei breached the duty owed to Merola as a
minority shareholder by terminating the employment without cause.
o Issue
 Does the termination of the employment of a minority shareholder in a close corporation
automatically constitute a breach of the fiduciary duty owed by controlling shareholders to
minority shareholders?
o Rule
 Rule: Termination of the employment of a minority shareholder in a close corporation does
not automatically give rise to a claim for breach of fiduciary duty owed to the minority
shareholder.
 Courts may conclude that terminating a minority shareholder’s employment constitutes
a “freeze-out,” but close corporations retain some discretion to hire and fire
shareholders for proper business purposes.
 In some cases, employment and stock ownership are intricately intertwined, such that
the salary paid constitutes the return on investment.
 Where the vast majority of a corporation’s earnings are paid out to shareholders as
salaries, a termination of a shareholder without cause is suspect.
o i.e. if salary is contingent on stock purchases = suspect
 However, where shareholders’ salaries are independent from their equity investment,
termination may be acceptable.
o Holding
 In this case, Merola received independent value for his equity investment. He ultimately sold his
shares at a price more than triple what he initially paid. His salary was never made contingent
on stock purchases. His termination was therefore within the discretion of the controlling
shareholder and did not constitute a breach of fiduciary duty. The ruling of the appellate court
in favor of Merola is reversed.
 Because plaintiff was unable to estalishe that shareholder status and employment were
LINKED
 As a result, plaintiff did NOT have an investment interest in this job from the get-go
 The job was not understood to be a part of his return on investment
 When an employee has no investment interest in his job, the job is subject to the
employment SOMETHING doctrine
o Notes
 To obtain protection for non-traditional shareholder benefits (employment, salary, etc) an
obvious connection between the investment in the company and the benefit of
employment/salary is required
 The shareholder has to establish that in his company, the benefit was reasonably
understood to be an entitlement that one received as a result of investing in the venture
 In re Kemp & Beatley Inc
o Facts
 Note; Many jurisdictions have an involuntary dissolution statute allowing shareholders to
petition for dissolution on the grounds of oppressive conduct by those in control
 Dissin and Gardstein (plaintiffs) held a combined 20.33 percent of the stock of Kemp & Beatley,
Inc. (defendant), a New York close corporation. Both were long-time employees of Kemp &
Beatley and held significant management positions. During their employment they and the
other shareholders received either dividends or extra compensation in proportion to their
stock holdings each year. Dissin resigned in 1979, and Gardstein was terminated in 1980. After
Dissin and Gardstein left, the company began to make its annual distributions on the basis of
service rendered to the corporation, rather than stock ownership (i.e. no more distributions,
just large salaries). As a result, Dissin and Gardstein no longer received annual distributions.
Dissin and Gardstein petitioned the court for dissolution of the company, arguing that the
controlling shareholders had frozen them out via fraudulent and oppressive conduct.
 Why would dissolution help them  allow them to liquidate the investment and allow
them to deploy their capitol elsewhere
o Issue
 Is forced dissolution of a corporation an appropriate remedy for minority shareholders when
the controlling shareholders have engaged in conduct that substantially defeats the minority
shareholders' reasonable expectations?
o Rule
 Rule: If majority shareholders take actions that substantially defeat the reasonable
expectations of minority shareholders, they have engaged in oppressive conduct, and the
court may order forced dissolution of the corporation.
 The New York Business Corporation Law  provides that holders of at least 20 percent of a
close corporation’s stock may petition the court to compel dissolution.
 The remedy should be granted  if the court finds that controlling shareholders have engaged
in illegal, fraudulent, or oppressive conduct or when the majority misappropriates corporate
assets
 Conduct is oppressive  when majority conduct substantially defeats the expectations of
minority shareholders, provided those expectations were reasonable and were an important
factor (central) behind the minority shareholder’s decision to invest in the company.
 Dissolution  should NOT be granted if there is an adequate, alternative remedy, or if majority
shareholders are willing to buy out the complaining shareholders at fair value. 
o Holding
 In this case, adequate evidence was presented to support the trial court’s conclusion that the
conduct was oppressive. Having found that Kemp & Beatley had a policy of providing bonuses
on the basis of stock ownership, and further that the policy was changed after Dissin and
Gardstein left the company, the court reasonably concluded that the change was an
oppressive act intended to deprive Dissin and Gardstein of the value of their stock. Because no
feasible alternative to dissolution was found, the trial court’s ruling was proper. The appellate
court's substantive determination is affirmed, but the judgment is modified to allow Kemp &
Beatley shareholders 30 days from the date of this order to exercise the option to purchase
Dissin and Gardstein's shares.
 Nixon v. Blackwell (Delaware  no dissolution for oppression statute)
o Facts
 E.C. Barton & Co. (Corporation) (defendant) was a closely-held Delaware corporation. Upon
founder E.C. Barton’s death, all of the Class A voting stock passed to employees, and only Class B
non-voting stock passed to Barton’s family. The Corporation offered to repurchase Class B stock
through a series of self-tender offers. The Corporation also set up an Employee Stock
Ownership Plan (ESOP) that allowed employees to cash out or receive Class B stock on
termination or retirement. The Corporation had a right to repurchase Class A stock on an
employee’s death or retirement. The Corporation also entered agreements with top executives
giving it the right to convert their Class A shares to Class B should they leave their roles, so
new Class A shares could be issued to their replacements. Further, the Corporation took out
life insurance policies on those executives benefiting the company. The Board resolved to use
employee life-insurance benefits to repurchase Class B shares from employee estates.
 Since closely held corporation  all shareholders have liquidity problem in realizing the
cash value of their shares (no market for them)
 Buyout option available to employee shareholders but that is not available to the
inactive family shareholders (partial liquidity for the employee shareholders)
 Under Donahue’s equal opportunity doctrine  this would not have been allowed/all
shareholders subject to equal treatment
 Fourteen Class B stockholders (plaintiffs) sued the Corporation and the directors (defendants) in
the Court of Chancery alleging that the defendants (1) tried to force minority stakeholders to
sell by paying only minimal dividends, (2) breached fiduciary duties by approving undue
compensation, and (3) breached fiduciary duties by discriminating against non-employee
stockholders. 
o Issue
 Under Delaware law, do corporate directors owe a fiduciary duty of equal treatment to all
shareholders?
o Rule
 Rule: Under Delaware law, corporate directors owe a fiduciary duty of fair, but not necessarily
equal, treatment to all shareholders.
 Rejects partnership analogy / rejects any right to equal liquidity and protection
 In corporate law  the entire-fairness standard applies to interested transactions, or
transactions in which one party is on both sides of the deal.
 This departure from the business-judgment rule is justified because business judgment
implies a business decision made by a disinterested, independent corporate
decisionmaker.
 Case law makes clear that stockholders do not have to be treated equally under all
circumstances. There is a fiduciary duty of fair treatment, but that does not require
equal treatment. Delaware law creates no such requirement.
 Further, there are no special judicial safeguards available for minority shareholders of
closely-held corporations. The creation of these types of safeguards would be contrary
to standard corporate practice. A corporation must opt in to the special provisions of
Delaware law aimed at statutory "close corporations," which the Corporation has not
done
o Holding
  In this case, the defendants have proven entire fairness. The ESOP and insurance policies are
both examples of standard corporate behavior. The Corporation’s policies were aimed at
maintaining employee control and management of the business. The Corporation made multiple
repurchase offers to Class B shareholders, and Class B shareholders benefited from the
increased share value. There was no requirement to afford minority shareholders equal
liquidity. Class B shareholders were not entitled to equal treatment.
 Gallager v. Lambert (courts assessment of whether a breach of fiduciary duty or frustration of reasonable
expectations has occurred, will be heavily influenced by contracts entered into between the parties
o Facts
 Gallagher (plaintiff) was an at-will employee, officer, and director of Eastdil Realty (defendant), a
close corporation. In the summer of 1984, Gallagher purchased an 8.5 percent interest in Eastdil
in response to an offer Eastdil made to all its executive employees. The purchase was subject to
a buy-back provision, which stated that upon “voluntary resignation or other termination” of
employment prior to January 31, 1985, Gallagher would be required to sell the stock back to
the corporation at book value. After January 31, 1985, the buy-back price would be keyed to
Eastdil’s earnings. On January 10, 1985, Eastdil terminated Gallagher. Gallagher demanded that
his shares be repurchased according to the post-January 31 terms. Eastdil refused. The book
value of Gallagher’s holdings was $89,000, while the new valuation would have been around
$3,000,000. Gallagher sued Eastdil, alleging breach of the fiduciary duty of good faith and fair
dealing
o Issue
 May close corporations enforce stock buy-back agreements to the detriment of minority
shareholders without breaching the fiduciary duty owed to minority shareholders?
o Rule
 Rule: Close corporations may enforce stock buy-back agreements (used as employment
incentives) to the detriment of minority shareholders without breaching the fiduciary duty
owed to minority shareholders.
o Holding
 In this case, the terms of the agreement are clear. Gallagher knew or should have known that if
his employment terminated prior to January 31, 1985, his stock would be repurchased at book
value. His employment in fact terminated prior to that date, entitling him to book value only,
regardless of Eastdil’s motives. Since there is no basis for the court to upset the plain language
of the agreement, Gallagher’s complaint must be dismissed on summary judgment
 A minority shareholder that agrees that her shares may be repurchased upon
termination acquires no protection against at-will discharge from her status as a
shareholder
 You can contract around oppression / contracts define the scope of the relevant
fiduciary duties and supplies certainty of obligations for each side / they should not
simply be undone by an allegation of unfairness because this would destroy the purpose
which is to provide a formula at which to value stock in the future
o Effect of holding
 A shareholder in a closely held corporation may be the beneficiary of fiduciary duties that
protect both her inters as an investor and as employee
 BUT the shareholder surrenders such special fiduciary protection by agreeing to have her shares
repurchased upon the termination of her employment for any reason
 Remedies: When a shareholder is successful in proving oppressive conduct or deadlock
o Death penalty  dissolution.
o Buy-out  purchase the shares from the shareholder petitioning for dissolution.
o Appoint custodian or receiver to take over.
o Push the company into involuntary bankruptcy.
o Alter the bylaws or resolutions upon a vote of majority shareholders.
o Force dividends or paying of damages.
o Equitable relieve court would deem appropriate.
o Involuntary bankruptcy

 Davis v. Sheerin (courts have often been willing to create remedies on their own as part of equitable authority
even without explicit statutory authorization)
o Facts
 Sheerin (plaintiff) and Davis (defendant) incorporated a business, in which they served as
directors and officers. Davis owned 55 percent of the corporation’s stock, and Sheerin owned 45
percent. In 1985 Davis denied Sheerin the right to inspect the corporate books unless he could
produce his stock certificate, claiming Sheerin had gifted his 45 percent interest in the business
to Davis in the 1960s. Sheerin sued Davis, claiming oppressive conduct and a breach of fiduciary
duties Davis owed to himself and the corporation. 
o Issue
 Is a buy-out an appropriate remedy for oppressive acts in the absence of a specific remedy in a
statue precluding that remedy?
o Rule
 Rule: Courts may order a buy-out of shares as a remedy for oppressive acts, as an alternative
to corporate liquidation, even in the absence of express statutory or contractual authority
when there is oppressive conduct
 This court finds that  Texas courts may order a buy-out where appropriate, if less harsh
remedies are not adequate to protect the parties’ rights.
 A buy-out is most commonly ordered  where there has been oppressive conduct, especially
where the majority in a closely held corporation has attempted to squeeze out the minority.
 Oppressive conduct does not require a showing of fraud, illegality, mismanagement,
wasting of assets, or deadlock. Courts have found that oppressive conduct arises when
the majority’s conduct defeats the reasonable expectations of the minority shareholder
that were central to his decision to join the venture. Other courts have alternatively
defined oppressive conduct as: burdensome, harsh and wrongful conduct; a lack of
probity and fair dealing in the affairs of a company to the prejudice of some of its
members; or a visible departure from the standards of fair dealing, and a violation of fair
play on which every shareholder who entrusts his money to a company is entitled to
rely.
o Holding
 This court finds that conspiring to deprive a shareholder of his stock in a corporation is
oppressive conduct. Davis’s conduct would substantially defeat, and in fact totally destroy, any
reasonable expectations Sheerin may have had when deciding to join the venture. The jury’s
finding of a conspiracy to deprive Sheerin of his interest in the corporation, along with the fact
that Sheerin would be denied any voice in the corporation, is sufficient to support the trial
court’s conclusion that there had been oppressive conduct. Davis’s oppressive conduct indicates
a desire to gain total control of the corporation, which is exactly what a buy-out will achieve. 
 Buy out appropriate based on SC holding that courts could order liquidation under their
general equity powers in the absence of statutory authority, saying that a court could
order less harsh remedies under those same equity powers
 In this case a buyout was appropriate because less harsh remedies were not available
 Legislation to correct oppression problem (clear up caselaw)
o Majority of states  provide for involuntary dissolution of close corporations upon illegal, oppressive,
or fraudulent behavior
o Thus, if you are an oppressed minority interest holder in closely held corp, file a complaint and move
for dissolution or any other remedies available per the statute.
o MBCA §14.30: Grounds for Judicial Dissolution
 The grounds for involuntary dissolution (discretionary):
 Board deadlock: directors cannot agree; shareholders cannot break the impasse; and
the business is suffering (§ 14.03(2)(i))
 Shareholder deadlock: shareholders have been unable to elect new directors (§
14.03(2)(iii))
 Misconduct: those in control have engaged in “illegal, oppressive, or fraudulent
conduct” (§ 14.03(2)(ii))
 Waste: corporate assets are being “misapplied, or wasted” (§ 14.03(2)(iv))
 Once we can establish one of these grounds for saying that there is oppression and want a
judicial dissolution  there are other ways of dealing with this…
o MBCA §14.34: Election to Purchase in Lieu of Dissolution
 (a) in a proceeding under section 14.30(2) to dissolve a corporation, the corporation may elect
or, if it fails to elect, one or more shareholders may elect to purchase all shares owned by the
petition shareholder at the fair value of the shares. A election pursuant to this section shall be
irrevocable unless the court determines that it is equitable to set aside or modify the election
 There are ways to save it without being pushed into dissolution
o MBCA §14.32 Receivership or custodianship
 (a) a court in a judicial proceeding brought to dissolve a corporation may appoint one or more
receivers to wind up and liquidate, or one or more custodians to manage the business and
affairs of the corporation. The court shall hold a blank, after notifying all parties to the
proceeding and any interested person designated by the court, before appointing a receiver or
custodia. The court appointing a receiver or custodian has exclusive jurisdiction over the
corporation and all of its property wherever located.
 Even if you meet standards for judicial dissolution, we’re going to give the judge
discretion to fashion a better remedy
Chapter 14 – Transactions in Share: Securities Fraud and Sales of Control; Rule 10b-5
A) The development of a federal Remedy: Rule 10b-5
 Common Law Fraud Review
o Hypo 1
 Homer and Ned are neighbors who own adjoining grazing land
 Homer drills a well and strikes a huge oil deposit, but Ned doesn’t know it
 Issue 1: Can homer quietly buy out Ned’s land as grazing land?
 Holding: Yes, as long as Homer makes no false representation or commits any other sort of
fraud, and has no contractual fiduciary or other legal equitable duties to Ned (which he doesn’t),
there is no problem with making this purchase. Of course, this transaction is real estate, it is
NOT securities.
 Issue: Does it make sense to hold securities transactions to a higher legal standard than
transactions in other sorts of assets (real estate).
 Holding: it appears so. At least, congress thought so. We now have laws that don’t apply to
transactions for real estate or the sale of goods, but we do with securities. Congress thought it
was easier to deceive and manipulate investors when it came to actual public shares.
 Issue 2: If so, what are the two primary things that fraud is meant to stop?
 Holding: Generally, two infractions that CL fraud is supposed to try to stop: (1) lying/deception
(we can consider this a commission because its active); (2) omitting/failure to state a material
fact if there is a relationship of trust or confidence (omission  intentional inaction)
 Omissions  have to analyze relationships of trust and confidence (a relationship that
indicates that the two parties are not dealing as strangers and at arm’s length but rather
are associates or someone cooperating individuals. It may be based on personal
friendship or prior transactions in which one party relied on the other to treat him or
her fairly.
o Hypo 2
 Burns is the CEO and Chair of the Springfield Nuclear Powerplant, and he is aware that the
earnings report for that year is going to be great
 He delays sending out the report until he can buy a bunch of stock from a current shareholder,
Smithers
 Issue 1: is it fair to permit him to use this knowledge for his personal benefit?
 Holding: Probably not.
 Issue 2: Should a duty be imposed on him for that reason?
 Holding: Yes. Burns is the director and he is dealing with a current shareholder. Wouldn’t
Smithers believe that Burns would represent his interests since he is the director of the
corporation? Isn’t this a special relationship maybe different from Homer’s and Ned’s? This
could get Burns in trouble because he is omitting something when he has that special
relationship. This will probably be considered CL fraud.
o Hypo 3
 Now consider the converse situation
 Assume that Burns sells shares from his personal portfolio shortly before he releases bad news
to a member of the general public who was not previously a shareholder.
 Issue: Can a director be said to owe a duty to the public in general?
 Holding: This transaction seems equally unfair but there is no way, under te CL of fraud (like in
last situation) to say that a director owes a duty to disclose bad news to everyone in the world
with whom he plans to trade. The CL simply does not have a legal principal or a remedy that
would cover the situation. SO, unlike the Burns case, if you didn’t have a special type or statute
or regulation to cover this, it would be really hard for some stranger in the general public to go
after Burns in this case. That is why we now can introduce rule 10(b)5.
 SEA Section 10: Manipulative and Deceptive Devices (Congress Rule to the SEC / very broad command)
o It has be unlawful for any person, directly or indirectly, by the use of any means or instrumentality of
interstate commerce or of the mails, or of any facility of any national securities exchange—
 (B) to use or employ in connection with the purchase or sale of any security registered on a
national securities exchange or a security not so registered, any manipulative or deceptive
device or contrivance in contravention of such rules and regulations as the commission may
prescribe as necessary or appropriate in the public interest or for the protection of investors
 Rule 10b-5: Employment of Manipulative and Deceptive Devices
o It shall be unlawful for any person, directly or indirectly, by the use of any means or instrumentality of
interstate commerce, or of the mails or of any facility of any national security exchange
 (a) to employ any device, scheme, or artifice to defraud,
 (b) To make any untrue statement of material fact or to omit to sate a material fact necessary
in order to make the statements made, in light of the circumstances under which they were
made, not misleading, or
 (c) to engage in any act, practice, or course of business which operates or would operate as a
fraud or deceit upon any person,
o In connection with the purchase or sale of any security
o Rationale/Designed to accomplish
 Designed to make things open and honest with respect to disclosures
 we are not concerned with corporate governance or enforcing fiduciary duties to shareholders,
we are concerned with the accuracy of disclosure in the purchase or sale of ANY security
whatsoever
o Kardon v. National Gypsum Co. (helps interpret some of the broad language of 10b-5)
 Facts
 Two families who owned two businesses
 The Slavin’s made arrangements to sell one of the businesses for $1.5 million
 Prior to selling the Slavin’s bought out the Kardon’s for $500k without mentioning the
intended sale
 Issue 1
 whether there is a private CoA for violations under rule 10b-5?
 Holding:
 Yes. The SEC or a private buyer or seller or securities can sue in federal court.
 Issue 2
 Whether rule 10b-5 is limited to corporations that are registered under the securities
and exchange act / is this only for public corporations or large corporations?
 Holding
 No. it is applicable to purely local, closely held corporations with even just a few
shareholders so long as the mails or facilities of interstate commerce are used in
connection with the purchase or sale of a security
 This really broadened what was once perceived as only a public CoA for publicly traded
corporations can now be in even the smallest corporations (including a small
partnership or an LLC)
 Interstate Commerce
o SEA § 3(a)(17) (Broad standard)
 The term “interstate commerce” means trade, commerce, transportation, or communication
among the several states, or between any foreign country and any state, or between any state
and any place or ship outside thereof. The term also includes intrastate use of (A) any facility of
a national securities exchange or of a telephone or other interstate means of communication; or
(B) any other interstate instrumentality
 Anything that could potentially cross state lines
o Hypo
 Assume that Moe’s tavern has only two shareholders, Moe and Barney, both living in Springfield
 The corporation’s business is entirely local
 Moe (who is also the director and CEO of the corporation) calls up Barney on the telephone and
offers to buy his shares.
 Issue 1: is Rule 10b-5 applicable to that transaction?
 Holding: Yes – Moe used a phone. If Barney feels he was misled or defrauded or that Moe failed
to disclose material information, he could bring a lawsuit in federal court under rule 10b-5
 Issue 2: What about new means of communication (internet / text / email)
 Holding: Yes – text messages would cross into that interstate commerce category
 Issue 3: Is a nexus established if Moe simply mails a check in payment for the stock purchased?
 Holding: Yes – the code uses the word ‘mail’ as being an instrumentality
 Issue 4: What if Moe makes the offer in person over a beer at the bar and the check is hand
delivered to Barney a few minutes later?
 Holding: Probably not - that would not then implicate it because we do not get back to that
situation of truly moving things around through interstate commerce. This is an in-person
situation and maybe fraud would have to be the remedy but likely not a 10b-5 CoA.
o Result of these various rules
 Rule 10b-5 is not limited to insider trading (which was probably the impotence) but is also used
in many garden varied private security fraud cases
 Consider facts of Kardon case  two families owned two businesses and the Slavins made
arrangements to sell one of the businesses for 1.5mil. Prior to selling, the Slavin’s bought out the
Kardon’s without mentioning the intended sell. The omitted a material fact in connection with
the purchase or sale of an entire business which again, we can look at as a security.
o [Anlyze first with client – how did you hear about this? Anything over the mail, telephone, text
message, implicates interstate commerce. Unless this was in a bar, not across state lines…]

o Blue Chip Stamps v. Manor Drug Stores


 Facts
 Blue Chip Stamps (defendant) was required by a United States district court to make a
tender offer for the minority shares of the corporation in 1968. Two years after the
tender offer, Manor Drug Stores (plaintiff) brought suit against Blue Chip Stamps,
alleging that Blue Chip Stamps had misrepresented its financial health in an attempt to
discourage the minority shareholders from taking part in the tender offer, so that it
could later purchase the same shares at a lower market price. However, Manor Drug
Stores did not purchase or sell any Blue Chip Stamps during the time period in which the
misrepresentations were made
o Argued that the plaintiffs failed to purchase securities when offered to them
because a prospectus was overly pessimistic
 Issue
 (1) Is an overly pessimistic prospectus a violation of rule 10b-5
o i.e. does the plaintiff, in a rule 10b-5 case have to be a purchaser or an actual
seller of shares or can it be a potential or seller?
 (2) In order to have standing under § 10(b) of the Exchange Act, must one have been
injured after purchasing or selling a security?
 Holding
 Rule 10b-5  the court held that only the persons who buy or sell securities can be rule
10b-5 plaintiff, NOT those who have spotted something fishy in the market or have
spotted deception in the market.
 Rule
 In order to have standing under § 10(b) of the Exchange Act, one must have been
injured after purchasing or selling a security.
o Ernst & Ernst v. Hochfelder (negligence/aiding and abetting)
 Facts
 Ernst & Ernst (defendant) is an accounting firm, which had conducted audits for First
Securities Company of Chicago (First Securities) for over twenty years. From 1942 until
1966, Leston Nay, the president of First Securities, obtained funds from investors
(Hochfelder) (plaintiffs), which he immediately transferred to his own personal account.
In 1968, Nay committed suicide, leaving a note detailing the fraud he had committed.
Hochfelder then brought suit against Ernst & Ernst, claiming that it was guilty of
securities fraud under § 10(b) by not diligently conducting its audits of First Securities.
o Involves liability of an accounting firm for failing to discover a serious fraud
perpetrated by the president of first securities, Nay.
o Complaint  Ernst & Ernst should have learned of Nay’s personal mail rule
which in effect required all envelopes addressed to him personally to be
delivered to him unopened, whether he was there or not.
o Why should this awaken suspension  a mail rule of this nature is a serious
matter in any organization handling large sums of money. There should be a
bureaucratic system of handing correspondence if funds are to be accounted for
entity. The possibility that a single person will handle exclusively certain
correspondence dealing with money is very dangerous.
o Bank / institutions  exists a thing called ‘the 2-week vacation rule’ and that’s
a rule enforced by many banks that every employee must take a two week
vacation each year and may not be physically present in the bank during this
period. The theory is that any person engaging in fraudulent conduct and
covering his conduct by entering fictitious transactions will be able to cover the
transaction by book entries during that two week period and the likelihood that
the fraud will be discovered is increased. Refusal by a person to take a vacation
is therefore of itself highly suspicious.
o We know  the accounting firm did not profit personally from the fraud so the
real issue becomes…
 Issue
 (1) Whether an accounting firm can be liable under rule 12(b)(5) for negligence?
 (2) May one be found guilty of securities fraud under § 10(b) of the Exchange act if an
intent to defraud did not exist?
 Holding
 Scienter is an essential element of Rule 10b-5 claims.
 Note: SC later held, in central bank of Denver, that private aiding and abetting claims are
not covered by section 10b
 Rule
 Scienter  a mental state in which one has knowledge that one’s action, statement,
etc., is wrong, deceptive or illegal: often used as a standard of guilt
o Issue: Is scienter or intent explicit in rule 10b-5
o Holding: It appears no. The court appears to infer it from the language of this
rule. This rule is not speaking to actual intent.
o Issue: what about reckless disregard of the truth, can that constitute the
scienter language?
o Holding: this has been addressed by a large number of lower courts (hasn’t
gone to SC) and they are virtually unanimous that reckless disregard is the
equivalent of scienter but that ordinary negligence is not.
o Santa Fe Indus v. Green
 Facts
 Santa Fe Industries, Inc. (Santa Fe) (defendant) controlled 95 percent of Kirby Lumber
Corp.’s (Kirby) stock. Santa Fe sought to own 100 percent of the stock, so it filed for a
merger under the Delaware short-form merger statute, which allows a parent
corporation owning at least 90 percent of the subsidiary to merge with the subsidiary
and make a cash payment to the minority stockholders. The statute does not require
approval of the minority stockholders, only notice within 10 days of the merger. The
statute provides that if the minority stockholders are unhappy with the merger, they
can petition the Delaware Court of Chancery to attempt to obtain fair value for their
shares. Santa Fe followed the statute and completed the short-form merger, giving the
minority stockholders of Kirby $150 per share, a value based on an appraisal by Morgan
Stanley. The minority stockholders (plaintiffs), rather than petitioning the state court as
called for by the statute, brought suit in federal court seeking to set aside the merger on
the grounds that the appraisal was fraudulent in violation of SEC Rule 10b-5.  
o The plaintiff sought to challenge, under rule 10b-5, a cash out merger that
eliminated minority shareholders on allegedly, unfair terms.
 Issue
 (1) Was the plaintiff’s complaint that the transaction was deceptively presented or that
there was an absence of full disclosure?
 (2) If not, must a fraudulent transaction under § 10(b) of the Securities Exchange Act of
1934 involve conduct that is manipulative or deceptive?
o Whether an inherently unfair transaction could be considered actionably under
rule 10b-5?
 Holding
 (1) No. There was full disclosure
 (2) The plaintiffs were complaining that the transaction at issue constituted a breach of
fiduciary duty and that as such, 10b-5 was somehow violated. The SC said that although
they were dealing with the purchase or sale or a security, false statements or deceptive
or material non-disclosure is always an essential element of ruel 10b-5. This might be a
crappy or unfair transaction but there was no allegation or proof of misrepresentations
or failure to disclose. What they should have done, if they were harmed by the directors
is bring a regular state court action and sue for the breach of duty of care. Just because
you have a duty of care or duty of loyalty issue that comes up with respect to the selling
of shares, we can’t always assume that we can bring a 10b-5 CoA as a shareholders. We
still have to be that buyer/seller and we always have to show that allegation of
misrepresentation or failure to disclose.
 Rule
 A fraudulent transaction under § 10(b) of the Securities Exchange Act of 1934 must
involve conduct that is manipulative or deceptive.
 Basic Elements of 10b-5 Claims
o (1) Interstate commerce: mail, telephone, national exchange, etc
o (2) Bad acts
 Misrepresentation of material information
 Failure to disclose material inside information
 Tipping: passing along inside information for a wrongful purpose (tipper must pass along insider
information and benefit from it / the tippee must trade on the tip and know that the tipper
breached the duty to their company)
o (3) Purchase/sale of security: bad act must be in connection with purchase/sale of security
 Security  very broad; includes: partnership interest, LLC interest, bonds, stock, oil interest,
mineral interest, public stock, etc (never a tangible item unless that tangible item is relating to
the interest i.e. purchasing a fleet of vehicles as part of contribution into partnership)
o (4) Material fact: Bad act must involve a material fact – one that reasonable investor would consider
important when making an investment decision
 *Analyze the materiality of what is being said.
o (5) Scienter: D has intent to deceive, manipulate, or defraud (negligence not enough, but reckless
disregard probably enough).
 Intentional failure to disclose. (Ernst & Ernst v. Hochfelder – Scienter was required as an
element - negligence not enough, but reckless disregard probably enough).
o (6) Reliance: Presumed in cases of nondisclosure or misrepresentation
 D could argue  plaintiff never even relied on this information, I have proof that he/she was
going to buy or sell without ever receiving this type of information
o (7) Damages: Generally, difference between price paid and FMV

 In Re Enron Corporation securities, Derivative & ERISA Litigation (secondary actor liability)
o Facts
 Purchasers of Enron Corporation’s public securities (plaintiffs) brought a class action against
Citigroup, Vinson & Elkins, Arthur Anderson, and other entities (defendants), alleging that they
made false statements or failed to disclose adverse facts while selling the Enron securities, or
participated in a scheme that defrauded or deceived the securities purchasers. The complaint
alleges that the defendants engaged in an enormous Ponzi scheme involving the creation of
Enron-controlled entities and the sale of unwanted Enron assets to these entities in non-arms-
length transactions to obscure debt and create sham profits on Enron’s books, in an effort to
induce new investments and maintain the scheme. The complaint alleges that Enron’s
accountants, outside law firms, and banks, who worked closely with Enron, greatly benefitted
from this scheme, causing them to support Enron’s misstatements about its financial condition,
and thereby deceiving investors and the public. Specifically, the complaint alleges that Citigroup
participated in a repeated pattern of disguised loans to Enron totaling $2.4 billion at double the
normal interest rates, which provided Citigroup with $70 million a year for its role in the Ponzi
scheme; that the law firm Vinson & Elkins wrote and approved Enron’s SEC filings and public
statements, and helped create the Enron-controlled entities, in exchange for lucrative fees; and
that the accounting and auditing firm Arthur Anderson engaged in a pattern of fraudulent audits
of Enron, and gave Enron a clean audit opinion despite having intimate knowledge of the details
of Enron’s alleged fraudulent activity.

o Issue
 May secondary actors like lawyers, accountants, banks, and underwriters be liable under SEC
Rule 10b-5 as a primary violator, for creating a misrepresentation on which an investor relies?


o Rule
 Rule: A secondary actor may be held liable under SEC Rule 10b-5 as a primary violator if he
creates a misrepresentation on which an investor relies, provided he acted with the requisite
scienter.
 SEC Rule 10b-5, promulgated pursuant to section 10(b) of the Securities Exchange Act of 1934,
makes it unlawful to employ a scheme to defraud, make an untrue statement of material fact or
omit a material fact necessary to make a statement not misleading, or to engage in an act or
course of business that would operate as a fraud or deceit on a person in connection with the
purchase or sale of a security.
 In Central Bank of Denver  the Supreme Court held that the anti-fraud provisions of section
10(b) and SEC Rule 10b-5 do not proscribe aiding another party who commits a section 10(b)
violation, and instead prohibit the actual making of a material misstatement or omission or
commission of a manipulative act.
 However, the Court in Central Bank also concluded that, assuming all of the
requirements for primary liability under Rule 10b-5 are met, secondary actors like
lawyers, accountants, banks, and underwriters may be liable under Rule 10b-5 for
employing a manipulative device or making a material misstatement or omission on
which a purchaser or seller of a security relies.
 This court adopts the SEC’s construction of section 10(b) and Rule 10b-5 to mean that a
secondary party may be held liable as a primary violator if he creates a misrepresentation on
which an investor relies, including writing misrepresentations that are included in a document
to be given to investors, if the secondary party acted with scienter.
o Holding
 In examining the defendants’ motions to dismiss in this case, this court must determine whether
the complaint adequately alleges a primary violation of section 10(b) by each defendant and
raises a strong inference of the required scienter. The complaint must allege that each
defendant knowingly or with reckless disregard contravened legitimate and professionally
acceptable activities in performing material acts to defraud the public.
 In this case the scienter pleading requirement is satisfied, in part, by allegations against all of the
defendants generally of a regular pattern of conduct involving the creation of unlawful Enron-
controlled entities and the sale of unwanted Enron assets to these entities in non-arms-length
transactions, in an effort to obscure debt and create sham profits on Enron’s books, resulting in
large profits for the defendants; a common motive of monetary gain, including the receipt of
extraordinary fees and interest rates that increased with the image of corporate success that the
defendants fraudulently created; and extensive relationships with Enron, suggesting
opportunities to learn about and take an active role in the decision-making about Enron’s
secretly controlled entities.
 The complaint also adequately alleges a primary violation of section 10(b) and Rule 10b-5 and
raises a strong inference of the required scienter as to each of the defendants individually.
 (1) The complaint alleges that Citigroup participated in a repeated pattern of disguised
loans to Enron totaling $2.4 billion at double the normal interest rates, providing
Citigroup with $70 million a year for its role in the scheme.
 (2) The complaint also alleges that the law firm Vinson & Elkins wrote and approved
Enron’s SEC filings and public statements, and helped create the Enron-controlled
entities, in exchange for lucrative fees. If Vinson & Elkins had remained silent in public,
the attorney-client relationship, and the traditional rule barring non-client suits against
lawyers, would protect the firm from liability. However, the complaint alleges that
Vinson & Elkins did not remain silent, and instead frequently made public
misrepresentations about Enron’s financial situation. Having chosen to make public
statements, Vinson & Elkins had a duty to ensure those statements were accurate and
truthful.
 (3) Finally, the complaint alleges that Arthur Anderson engaged in a pattern of
fraudulent audits of Enron, and gave Enron a clean audit opinion despite having intimate
knowledge of the details of Enron’s alleged fraudulent activity. The complaint therefore
adequately alleges a primary violation of section 10(b) by each defendant and raises a
strong inference of the required scienter.
o Notes
 On complex motions to dismiss, the court here in Texas concluded that claims based on scienter
have been plead here, (1) against Citigroup and several other banks as primary violators of rule
10b-5, (2) against the law firm of Vincent and Elkins on the theory that it participated in an
ongoing Ponzi scheme and deliberately, and with severe recklessness directed public statements
that were designed to keep the Ponzi scheme alive, and (3) against Arthur Anderson, the
accounting firm, on the theory that it was intimately privy to the smallest details of Enron’s
alleged fraudulent activity and its fraudulent audits.
 The refusal of the judge to dismiss most of the defendants in this case promises to help shape
the debate concerning secondary actor liability. The opinions handling of the motion not to
dismiss departs from the traditional view that secondary actors are almost always, at best,
aiders and abettors who cannot be held personally liable. This case strengthens the likelihood
that some secondary actors including lawyers, may be found liable in similar cases.
 Rule: if you’re so closely associated with the fraudster there is a chance, that instead of the
secondary aider and abettor claim, this could really have a profound affect with individual and
personally liable in these situations including law firms and accounting firms.
Chapter 18 – The Limited Partnership
A) Introduction
 Issue: if a client wants to form a partnership, what should you worry about as a lawyer with this set-up?
o Personal liability
 Issue: Can you form a partnership or invest in one where you avoid personal liability and where you don’t have
any managerial duties or undertakings?
o Yes – through a limited partnership
 Limited Partnership  Involve limited partners and general partners who fill different roles and have different
responsibilities
o LP  is a creature of statute (like a corporation) comprised of at least one general partner (GP) and at
least one limited partner
 GP = unlimited liability of the obligations of the firm; generally control of the firm
 Limited Partner = no liability for the debts of the venture beyond the loss of his investment;
often does not exercise control
o Issue: what if you have a client come in, you form a limited partnership and your client puts in 45%,
another puts in 45%, and the last puts in 10% and is also designated as the GP
 That 10% owner, designated as the GP runs the limited partnership as the general partner and is
the only one who is personally liable despite being a minority owner with only a 10% interest
 5 Aspects of the Limited Partnership
o (1) unlike a general partnership, a limited partnership requires a statutory filing for creation;
o (2) general partners have unlimited personal liability for the obligations of the limited partnership, while
limited partners have limited liability
o (3) limited partnerships provide their owners with significant contractual freedom and flexibility
o (4) limited partnership are subject to pass-through tax treatment; and
o (5) General partners typically (though not always) have the right to manage the venture while the
limited partners serve primarily as passive investors
 Applicable Statutes
o (1) The first uniform act on limited partnership was the 1916 Uniform Limited Partnership Act (First
ULPA)
o (2) ULPA was revised in 1976 and 1985 to the Revised Uniform Partnership Act (RUPA) (TX and majority)
 Does have “linkage”
o (3) There is a more modern statute enacted in 2001 called the Uniform Limited Partnership Act (ULPA) –
standalone statute with no “linkage” (~16 states)
o (4) There is a 2013 New ULPA (ULPA 2013) (~3 states)
o (5) Because most states (including TX) follow RULPA, and 16 follow ULPA 2001, these will be our focus
and for the purposes of the exam these are the two on which I will test
 2001 ULPA (no ‘linkage”)
o Means  RULA is linked to all of the general partnership statutes that we discuss / so, depending on the
state, a lot of the times to answer some questions with respect to the LP, we should look to UPA or we
would look to RUPA to fill in some of the gaps and to clarify
o ULPA 2001  it expressly disallows linkage i.e. we don’t ever look to UPA or RUPA for gaps or
clarification
 Why do we need to know about LP?
o For many years, the limited partnership stood alone as the only business form that provided the best of
both worlds
 The corporate trait  limited liability
 The partnership trait  pass through taxation and structured flexibility
o With the birth of the LLC, however  most business owners choose the LLC option NOT the limited
partnership option
o Note(s):
 the relatively long history of use of limited partnerships in this country has produced a level of
comfort among attorneys and business owners with that form
 also, that same history of use has generated a significant amount of common law precedence’s
that makes the limited partnership structure more predictable than new business structures
 also, the legal framework of business forms like the LLC is derived in large part from limited
partnership law and limited partnerships are still popular in certain specialized areas (estate
planning / real estate / venture capital / oil and gas)
 LP and the law of LP is likely to remain very relevant for many years

B) Formation
 Filing
o Corporation (to file)  articles of incorporation
o LP 
 (1) need certificate of limited partnership
 State: name of GP and name of actual LP
 (2) nonpublic document governing the operation of the venture
 Corporation  this is the bylaws
 LP  call the partnership agreement
 Formation
o RULPA  LP is formed at the time of the filing of the certificate / that is done in the officer of the
secretary of state as long as there has been substantial compliance with the requirements of RULPA
o Questions;
 (1) Issue: Why don’t we have to file to form general partnership but have file to form limited
partnership?
 Holding: Whenever a state grants the privilege of limited liability, it requires public filing (notice
issue) / at a minimum, the filing serves to alert the world that the owners of a particular
business entity have limited liability for the ventures obligation / because partners have
unlimited liability for the obligations of a general partnership and because the general
partnership is the default form of business, we have a state filing
 (2) Issue: Whether we have to have the written agreement that will govern the partnership
(partnership agreement)?
 Holding: No – but we want one so that we don’t have any fights between partners (who is
limited partner / who is managing partner / what are the roles of each)
 (3) Issue: what if you defectively form the limited partnership with improper filing (file in wrong
place or mix up GP with LP or treat as GP with full liability)?
 Holding: Depends on the facts of the transaction at hand and who is trying to enforce the
liability. Generally, there is some protection.
 RULPA § 304: Person erroneously believing himself a limited partner
o (a) Except as provided in subsection (b), a person who makes a contribution to a
business enterprise and erroneously but in good faith believes that he has
become a limited partner in the enterprise is not a general partner in the
enterprise and is not bud by its obligations by reason of making the
contribution, receiving distributions form the enterprise, exercising any rights of
a limited partner, if, on ascertaining the mistake, he
 (1) cause an appropriate certificate of limited partnership or a
certificate of amendment to be executed and filed; or
 (2) withdraws from future equity participation in the enterprise by
executing and filing in the office of the Secretary of State a certificate
decaling withdrawal under this section
o Takeaway: this tells us this idea of a substantial compliance rule and a reliance
component for asserting liability on a “supposed’ general partner, (more so than
in corporation law or in the corporate context) (i.e. here is some wiggle room if
the limited partnership is defectively formed) / focuses on a good faith belief
when it comes to the infected
o (b) a person who makes a contribution of the kind described in subsection (a) is
liable as a general partner to any third party who transacts business with the
enterprise
 (i) before the person withdraws and an appropriate certificate is filed to
show withdrawal; or
 (ii) before an appropriate certificate is filed to show that he is not a
general partner,
o but in either case only if the third party actually believed in good faith that the
person was a general partner at the time of the transaction
o Takeaway: even on these situations we’re talking about third parties actually
believing in good faith that the person was a general partner at the time of the
transaction / see

C) Management and Operation


 RUPA § 403(a)
o General partners in an LP have the same rights and powers as general partners in the general
partnership (unless partnership agreement states otherwise)
o This includes: the ability to participate in management, the ability to bid the partnership via apparent
authority, and the ability to vote
o BUT
 These rights and powers are default rules
 I.e. the partnership agreement can restrict, limit, or alter them
 Issue: could the limited partners also manage the limited partnership?
o *technically* Yes – nothing in the statute that forbids it BUT if the limited partners manage then we do
have a problem of it not qualifying for limited partnership status
o So if you have limited partners who are told that their limited partners but then you are also giving them
some management authority, we run into a problem
o Statute
 RULPA does not grant or deny management rights to limited partners
 BUT – under RULPA § 303, limited partners who participate in the control of the business risk
liability for some or all of the obligations of the venture
o Note: if you’re going to select a general partner and generally select a limited partner, you also have to
carefully select what those roles are going to be and you really should be telling limited partners that
they are simply passive, the more that they move into an active role the more they look like a general
partner so the more chance there is that unlimited personal liability

D) Agency
 Hypo
o Your client is a 10% owner and is general partner
o Issue: Can the remaining partners, that hold 90%, do they have power to still bind the limited
partnership with respect to third parties?
 Note: they don’t have actual authority because they are limited partners. They do not have
voting rights. One way around this is to contract around this and give them voting power.
However, they can never vote on the day-to-day stuff because this will appear to give actual
control. Would have to be things outside the course of business.
o Issue: What would have to be argued by someone (third party) who is transacting with these limited
partners?
o Holding: We would have to argue apparent authority and if they have that, that could potentially bind
the limited partnership
o Issue: could a limited partner with no actual authority bind a limited partnership to transactions in the
ordinary course?
o Holding (unsettled area):
 RUPA is silent
 Some cases hold no / other don’t / grey area in the law
 because if anyone were to pull up the documents filed w secretary of state they would
see that the limited partners are those that they might be transacting with so they
would automatically be on notice that they don’t have real authority
 others say if they’re holding themselves out then there still is some sort of apparent
authority going on
 Voting
o Issue: If you select a 10% owner as the general partner, can the other limited partners vote on matters
that are before the limited partnership?
o Holding: No – we gave the general partner the authority to make decisions, not the limited partners so
our percentage interest does not matter /
 no default voting rights under RULPA (unless can point to something extraordinary or expressly
set forth in the partnership agreement)
o Issue: are there any instances where maybe the limited partners would be able to vote?
o Holding: Yes
 (1) RULPA § 302: Voting (pursuant to partnership agreement) - strict
 Subject to section 303, the partnership agreement may grant to all or a specified group
of the limited partners the right to vote (on a per-capita or other basis) upon any
matter
 (2) You additionally may have courts that say an extraordinary matter (bankruptcy or removal
of general partner) would be a case you could have limited partners vote as well
 (3) ULPA
 Does provide limited partners with default voting rights which can be expanded or
limited by agreement.
 Unless by agreement. If you want voting power, we need it expressly set forth.
RULPA § 302. Voting.
Subject to Section 303, The Partnership Agreement may grant to all (or a specified group) of the LimParts the right to vote (on
a per capita or other basis) upon any matter.
- Generally, does not = control such that liability LimPart is triggered
ULPA (2001) – Grants LimParts a right to vote on extraordinary matters only unless modified by Agreement
Voting rights differences:
o RULPA  you don’t get to vote if you’re a limited partner (strict) UNLESS, it is expressly set
forth in the partnership agreement
o ULPA  provides limited partners with default voting rights

E) Financial Rights and Obligations


 Profit Splitting
o Hypo
 Limited partnership that you are forming for your client
 Lets assume we have capital contribution of $1 million / we have 45% owner / another 45%
owner / and a 10% owner
 Issue: how do they now split the profits?
 Under general partnership  UPA/RUPA – default was equal (i.e profits were split
equally among all partners
o Equal sharing makes sense because all partners have unlimited liability
 RULPA  unless otherwise agreed in a written partnership agreement, the profits,
losses and distributions of a limited partnership shall be allocated “on the basis of the
value…of the contributions made by each partner to the extent they have been received
by the partnership and have not been returned
o You can contract around all this in your partnership agreement
 Holding: if the limited partnership had a million in profits or surplus, then upon distribution, the
45% owner would get $450k, 10% owner would get $100k (not the $333k each party would get
under RUPA/UPA and Gen. Part.) This is their pro rata distribution.
Profits and Losses
 Under UPA and RUPA, profits are split equally among all Partners.
 Under RULPA, unless otherwise agreed in a written Partnership Agreement, the profits, losses, and distributions of a
limited Partnership shall be allocated "on the basis of the value of the contributions made by each Partner to the
extent they have been received by the Partnership and have not been returned."

F) Limited Liability
 Issue: What if the 10% general partner of the limited partnership defrauds or misleads a creditor, can the
creditor go after everyone else individually?
 Holding: No – an essential feature of a limited partnership is the limited liability to limited partners. Limited
partners have no liability for the debts of the venture beyond the loss of what they put in (i.e. their investments)
 Issue: is it possible to lose this limited liability protection?
 Holding: Yes – a limited partner can lose their limited liability protection if they participate in the control of the
business.
o The issue of control creates a lot of litigation because it’s such a fact intensive inquiry
 Issue: how does a limited partner exert control?
 Holding: depends mostly on the governing statute.
o Section 7 of 1916 ULPA
 “a limited partner shall not become liable as a general partner unless, in addition to the exercise
of his rights and powers as a limited partner he takes part in the control of the business”
 Very broad  lead it up to the jury
o RULPA (1985) § 303(a): Limited Partner “Holding Themselves Out”
 Retained the control rule and further provided, “However, if the limited partner participates in
the control of the business, he is liable only to persons who transact business with the limited
partnership reasonably believing, based upon the limited partners conduct, that the limited
partner is a general partner
 Not that anyone that participates in control is liable / it’s only if that sort of controlling
activity leads others to rely on the belief that you are a general partner
 RULPA also expanded the safe harbor list of protected limited partner activities
o ULPA (2001) § 303
 Completes the pro-limited partner evolution by wholly eliminating the control rule stating, “a
limited partner is not personally liable, directly or indirectly, by way of contribution or
otherwise, for an obligation of the limited partnership solely by reason of being a limited
partner, even if the limited partner participates in the management and control of the limited
partnership”
 You could be in an ULPA state where control is not an issue, or you could be in RULPA where
you do have to analyze control.

o Gateway potato v. GB investment (fleshes out the evolution of the ‘control’ rule and highlights some of
the differences between the statutory versions)
 Facts
 Sunworth Packing Limited Partnership (SPLP) consisted of one general partner,
Sunworth Corporation, and one limited partner, G.B. Investment Co. (defendants).
Gateway Potato Sales (Gateway) (plaintiff) sold potato seeds to SPLP and was never
paid. Gateway had been wary of the transaction but agreed to it after Sunworth
Corporation’s president informed Gateway that they were in partnership with G.B.
Investment, a respected institution, and that G.B. Investment (limited partner) had
approved the transaction. Gateway never interacted directly with G.B. Investment.
Nevertheless, Gateway was aware that G.B. Investment was controlling the operations
of the partnership and believed it to be a general partner, based on statements made
by Sunworth Corporation's president. When SPLP failed to pay the amount owed,
Gateway sued SPLP, Sunworth Corporation, and G.B. Investment. G.B. Investment
moved for summary judgment on the grounds that, as a limited partner, it was not
personally liable for partnership debts. Evidence at trial suggested that G.B. Investment
participated at least to some degree in the control of the partnership.
o Gateway had no contact with the limited partner prior to entering into the
transaction
 Issue
 May a limited partner be held personally liable for a partnership debt without having
any personal dealings with the creditor?
 Rule
 General Rule  a limited partner is shielded from personal liability on partnership
debts.
 RULPA (1976) § 303(a)  (as adopted in Arizona) a limited partner may face personal
liability if she is also a general partner or if she exercises a degree of control over the
business that is substantially the same as that of a general partner.
o Does not require contact between the creditor and the limited partner
 Updates to the RULPA  (enacted in other states) expose a limited partner to personal
liability only where the limited partner has transacted business directly with the
creditor and created the impression that the limited partner is a general partner
o Arizona has not adopted these changes.
 In Arizona  when a limited partner exercises control in a manner substantially the
same as that of a general partner, she may be liable to creditors of the partnership
even if she had no personal dealings with them. 
o (1) substantially the same as general partner (doesn’t require direct contact)
o (2) direct contact between creditor and limited partner
 Holding
 In this case, some evidence suggests that G.B. Investment exercised significant control
over the business of SPLP. It is not necessary for G.B. Investment to have had any
involvement in the specific transaction with Gateway. Whether or not G.B. Investment’s
role was substantially the same as that of a general partner is a question of fact. Since a
genuine issue of material fact remains, the trial court’s grant of summary judgment in
favor of G.B. Investment is reversed, and the case is remanded to the trial court for
further proceedings.
o RULPA (1985) § 303(a) (UPDATE)
 Except as provided in subsection (d), a limited partner is not liable for the obligations of a
limited partnership unless he or she is also a general partner, or in addition to the exercise of his
or her rights and powers as limited partner, he or she participates in the control of the business
 However, if the limited partner participates in the control of the business, he or she is liable
only to person who transact business with the limited partnership reasonably believing, based
on the limited partners conduct, that the limited partner is a general partner
 Moves the standard from an either/or  a both
 If you prove control, you also have to prove that you transacted directly with the
limited partner reasonably believing, based on the limited partners conduct that the
limited partner was a general partner i.e. a reliance element always has to be met
 Gateway  would have lost because they had no direct contact with GB investment
(limited partner) until after the sale
o On the other hand, the language “based upon the limited partners conduct”,
doesn’t require direct contact between the creditor and the limited partner,
then plaintiff might win
o Gateway learned of GB investments conduct through hearsay rather than from
direct contact or observation
o Also depends on RULPA (303(b) Safe Harbors
 ‘liable only to person who transact business’  does this eliminate a class of creditors
who may be trying to attach to the limited partner for liability?
o Yes – ‘transact’ is a consensual relationship where the parties mutually agree to
be bound / it is executory in nature
o Judgement creditors, particularly tort claimants of a limited partnership, should
never be able to go after the limited partner because, although they may have
some control, there is no ‘transacting’ that is taking place (nothing consensual)
o RULPA (1985) § 303(b): Safe Harbors (activities that won’t move you to the ‘control’ camp)
 A limited partner does not participate in the control of the business within the meaning of
subsection (a) solely by doing one or more of the following
 (1) being the agent or employee of the limited or general partner, or being affiliated
with a general partner that is a corporation
 (2) consulting or advising the general partner
 (3) being financially liable for debts of the limited partner
 (4) effecting a derivative action on behalf of the limited partnership;
 (5) participating in meetings and voting of partners; and
 (6) winding up the limited partnership
 Issue: Why do we have these exclusions:
 So that we can be sure that limited partners can do certain things without implicating
themselves with respect to liability
 Only time you get in trouble as LP, is when LP exercises control and made a 3d party believe
they are a GP? The word transacting. It is a consensual relationship. If suing for a tort, can sue a
LP? No because torts are not transactions and can never have any liability, despite control for
torts of the GP because no transacting going on.
 Control of the Entity General Partner (ASK)
o Issue: assume your client does not want to be personally liable under the limited partnership but
he/she wants to participate in the management; what can you do
 You could form a corporation or LLC that your client owns, and have that LLC/Corp act as the
general partner

 Why would this allow your client, who wants no liability, but wants to be the general partner,
how is that going to work
 You now have a limited liability entity as a general partner so people behind it don’t face
unlimited liability of a general partner. Ex: make XYZ inc., the entity, a GP to prevent any
individual liability.
 General partner  some sort of limited liability entity
 Individual limited partners  serve as managers of the general partner

G) Fiduciary Duties
Limited Partnerships – Fiduciary Duties
• RULPA (1985) does not explicitly address the topic of general Partner fiduciary duties  GP law is imported to deal with
the topic.
• The fiduciary Duty material that you studied in the GP area is important in the LP area as well:
RULPA § 403
(a) Except as provided in this [Act] or in the Partnership Agreement, a GenPart of a LP has the rights and powers and is
subject to the restrictions of a Partner in a GP without limited Partners.  look to RUPA...

RUPA § 404. General Standards of Partner's Conduct.


(a) The only fiduciary duties a Partner owes to the Partnership and the other Partners are the Duty of Loyalty and the
Duty of care set forth in subsections (b) and (c).
(b) A Partner's Duty of Loyalty to the Partnership and the other Partners is limited to the following:
(1) to account to the Partnership and hold as trustee for it any property, profit, or benefit derived by the Partner
in the conduct and winding up of the Partnership business or derived from a use by the Partner of
Partnership property, including the appropriation of a Partnership opportunity;
(2) to refrain from dealing with the Partnership in the conduct or winding up of the Partnership business as or
on behalf of a party having an interest adverse to the Partnership; and
(3) to refrain from competing with the Partnership in the conduct of the Partnership business before the
dissolution of the Partnership.
(c) A Partner’s Duty of care to the Partnership and the other Partners in the conduct and winding up of the Partnership
business is limited to refraining from engaging in:
(a) grossly negligent or reckless conduct,
(b) intentional misconduct, or
(c) a knowing violation of law.
(d) A Partner shall discharge the duties to the Partnership and the other Partners under this [Act] or under the Partnership
Agreement and exercise any rights consistently with the obligation of good faith and fair dealing. (“requires a party in a K-ual
relationship to refrain from arbitrary or unreasonable conduct which has the effect of preventing the pother party to the K from
receiving the fruits of the bargain”)
** Careful in decision making = BJR defense. Dumb decision but reasonable way.
Limited Partnerships – Fiduciary Duties (only apply when the lim. part. Exercises control)
• These duties can be contractually substantially modified, particularly in Delaware.
• Can they be modified under RULPA (1985) or ULPA (2001)? YES!!

RUPA § 103. Effect of Partnership Agreement; Nonwaivable Provisions 


ULPA (2001) § 110. Effect of Partnership Agreement; Nonwaivable Provisions.
(a) Except as otherwise provided in subsection (b), relations among the Partners and between the Partners and the
Partnership are governed by the Partnership Agreement . . .
(b) The Partnership Agreement may NOT:
(3) eliminate the Duty of Loyalty under Section 404(b) or 603(b)(3), but:
(i) the Partnership Agreement may identify specific types or categories of activities that do not violate the Duty
of Loyalty, if not manifestly unreasonable; or
(ii) all of the Partners or a number or percentage specified in the Partnership Agreement may authorize or
ratify, after full disclosure of all material facts, a specific act or transaction that otherwise would violate the
Duty of Loyalty;
(4) unreasonably reduce the Duty of care under Section 404(c) or 603(b)(3);
(5) eliminate the obligation of good faith and fair dealing under Section 404(d) . . . .

In re USACafes, L.P.
Issue:
RULE: The Duty of Good Faith and Fair Dealing inheres in K’s and Partnership Agreements;
 sets the floor for conduct of gen. and lim. Partners
 requires a party in a K-ual relationship to refrain from arbitrary or unreasonable conduct which has the effect of
preventing the pother party to the K from receiving the fruits of the bargain.
 [EB] (1) General Partner
o RULPA § 403: General Powers and Liabilities
 (1) except as provided in the [Act] or in the partnership agreement, a general partner of a
limited partnership has the rights and powers and is subject to the restrictions of a partner in a
partnership without limited partners
o RULPA (1985)  does not explicitly address the topic of general partner fiduciary duties
 So, general partnership law is imported to deal with the topic
 Thus, the fiduciary duty material that you studied in the general partnership area is important
in the limited partnership area as well
o RUPA § 404: General Standards of Partners Conduct
 (a) duty of loyalty + duty of care
 (b) duty of loyalty is limited to
 Accounting for profits
 Refraining from self-dealing
 Refraining from competing/usurping of corporate opportunity
 (c) duty of care
 don’t be grossly negligent or reckless, no intentional misconduct, or violation of the law
(protected by BJR)
 (d) duty of good faith and fair dealing (which falls under loyalty)
o Issue: can we limit these duties (under RUPA) in a limited partnership to where the general partner
might not have to comply?
 K principles may allow that in some form or another
 Delaware  duties can be contractually modified (you could be allowed to compete or usurp
opportunities, you could put this in the partnership agreement that itemizes certain activities
that they would allow that would generally be limited by RUPA 404)
o Issue: can we modify the duties under RULPA?
 RULPA does not address limits on partners contractual freedom
 Linkage would presumably make the limitation (that we’re examining under RUPA) applicable to
the extent that RUPA was the governing general partnership statute
o RUPA § 103: Effect of Partnership Agreement, Nonwaivable Provisions
 You can’t
 (1) eliminate wholesale [although you can identify specifics] the duty of loyalty
 (2) unreasonably reduce the duty of care; or
 (3) eliminate the obligation of good faith and fair dealing
o ULPA (2001) § 110: Effect of Partnership Agreement, Nonwaivable Provisions
 (b) a partnership agreement may not
 (1) Eliminate the duty of loyalty, but the partnership may
o (i) identify specific types or categories of activities that do not violate the duty
of loyalty, if not manifestly unreasonable; and
o (ii) specify the number or percentage of partners which may authorize or ratify,
after full disclosure to all partners of all material facts, a specific act or
transaction that otherwise would violate the duty of loyalty
 (2) unreasonably reduce the duty of care
 (3) eliminate the obligation of good faith and fair dealing
o Delaware / Uniform Laws
 Delaware  Allow wholesale elimination
 Uniform Laws  quite limited
o General Partner (of LP) is Business Entity
 Issue; When a general partner of a limited partnership is a business entity, do managers of the
entity personally owe fiduciary duties to the limited partners and the limited partnership?

 In Re USA Cafes, LP (Delaware)


 Facts
o USACafes, L.P. (the Partnership) was a limited partnership consisting of
USACafes General Partner, Inc. (the General Partner) (defendant) and a large
group of limited partners who owned partnership units (the Unitholders)
(plaintiffs). In 1989, Metsa Acquisition Corp. (Metsa) (defendant) purchased
substantially all of the Partnership’s assets.
 GP (corporation) approved a buyout of all of the assets of the limited
partnership for $10/unit
o The unitholders sued the General Partner, the directors of the General Partner
(the Directors) (defendants) and Metsa, alleging that the asset sale price had
been unfairly low. They claimed that Metsa made side payments to the
Directors to induce the Directors to cause the General Partner to accept the low
price. The Unitholders alleged that the General Partner and its Directors had
breached their duty of loyalty to the Unitholders by failing to obtain the best
price, and that Metsa had induced the breach by offering the side payments.
The Directors moved to dismiss the charges against them for failure to state a
claim, asserting that while the General Partner owes a duty of loyalty to the
Unitholders, the Directors themselves do not.
 Unit holder  suing the GP and the general partnership for breach of
fiduciary duty / also suing Wiley brothers (who own all stock in the
general partner corporation) individually due to their status as
shareholders and directors of the corporate general partner
 Unit holders claim  Wiley brothers sold the limited partnership assets
at a low price in a transaction in which the Wiley brothers allegedly
received substantial side payments from the buyer (Metsa)
 Wiley brothers  filing motion to dismiss because they claim that the
general partner (USA inc.) clearly owes fiduciary duties to the limited
partnership, BUT they as shareholders and directors of the general
partner do not in their individual capacity

 Issue
o Does a director of a corporate general partner stand in a fiduciary relationship
to other (limited) partners and to the partnership as a whole?
 Rule
o General Rule: Partners owe each other fiduciary duties, including the duties of
care and loyalty.
 A partner thus cannot take action which would benefit himself at the
expense of the other partners.
o Rule: Where a corporation acts as a general partner, the individual directors of
the corporation–in addition to the entity itself–owe duties to the other
partners and to the partnership.
 An analogous principle applies in trust law  Directors of corporate
trustees stand in a fiduciary relationship to the beneficiaries. Trustee
controls property that belongs to the beneficiaries, the law of trust
states that one who controls property of another may not use that
property in a way that benefits the controller to the detriment of the
property or its beneficial owner
 When directors of a corporate general partner control the partnership’s
property, because of their control of the corporation that in turn
controls the limited partnership, they also owe a fiduciary duty to the
limited partnership and the limited partners as beneficial owners of the
property
 Holding
o In this case, the General Partner and its Directors were obliged to seek out the
best price for the sale of the Partnership’s assets, and they were not permitted
to use their positions to benefit themselves at the expense of the Unitholders.
o If, as the Unitholders allege, the Directors accepted side payments in exchange
for supporting Mesta’s bid, they breached their duty of loyalty to the
Unitholders. Since the Unitholders have stated a claim upon which relief may be
granted, the motion to dismiss is denied.
 (2) Limited Partner
o KE property management v. 275 Madison Management Corp (Delaware) (cases where limited partners
might owe fiduciary duties / situation where a limited partner is controlling or participating in the
control of the entity)
 Facts
 275 Madison Associates L.P. was a Delaware limited partnership. KE Property
Management Inc. (KE) (plaintiff) was one of its general partners, and KJ Capital
Management, Inc. (KJ), an affiliate of KE Property, was a limited partner. 275 Madison
Management Corp. (275 Madison) (defendant) was the limited partnership’s managing
general partner.
 Harry Skydell, 275 Madison’s president, misappropriated $2 million of the limited
partnership’s funds. 275 Madison’s partnership agreement provided that the limited
partners could expel any general partner that injured the partnership by fraud or
willful misconduct, if at least 25 percent of the units outstanding were in favor. KJ,
with 25 percent of the units outstanding in favor, sent a letter to 275 Madison removing
it as managing general partner, claiming that Skydell’s misappropriation constituted
fraud or willful misconduct on 275 Madison’s part. 275 Madison then filed a bankruptcy
petition on behalf of the limited partnership. KE Property brought suit, seeking a
declaratory judgment that the removal of 275 Madison was effective, and seeking to
enjoin 275 Madison from purporting to act as the managing general partner. KE moved
for summary judgment.
 Issue
 Whether a limited partner of a Delaware limited liability partnership owed a fiduciary
duty to the general partner when removing the general partner pursuant to its
contractual right under the partnership agreement
 Rule
 Rule: To the extent that a partnership agreement permits a limited partner’s discretion
to take actions affecting the limited partnership’s governance, the limited partner may
be subject to fiduciary obligations toward the other partners.
o i.e. Where a partnership agreement permits a limited partner to remove a
general partner, a limited partner who does so will owe a fiduciary duty to the
general partner
 Delaware ULPA  does not clarify whether a limited partner owes a fiduciary duty to
other partners, it does provide that any matter not covered by it is governed by the
Delaware Uniform Partnership Law.
o The Delaware Uniform Partnership Law states  that all partners owe each
other fiduciary obligations.
o Thus, to the extent that a partnership agreement permits a limited partner to
take actions affecting the limited partnership’s governance, then the limited
partner may be subject to fiduciary obligations when taking those actions,
such as the duty to act in good faith.
 Holding
 275 Madison (defendant)  argues that KJ (limited partner) acted in bad faith in
removing it as managing general partner.
o Because there is a presumption that KJ acted in good faith, 275 Madison bears
the burden of showing the existence of bad faith.
o 275 Madison’s claim of bad faith rests on the idea that KJ owed a fiduciary duty
to it, as the partner that removed it as managing general partner.
o However, although a general partner owes a fiduciary duty to its partners, KJ
was a limited partner.
 In this case, the partnership agreement permits a limited partner to remove a general
partner for fraud or willful conduct that injures the limited partnership.
o Thus, a limited partner removing a general partner for fraud or willful conduct
may have a fiduciary obligation to act in good faith in removing that partner.
 Allegations of bad faith that are supported by competent evidence raise a question of
fact sufficient to defeat a motion for summary judgment. Because 275 Madison would
have the burden of persuasion at trial to overcome the presumption of good faith, after
an opportunity for discovery it must introduce competent evidence to rebut the
presumption of good faith if it is to survive KE’s motion for summary judgment.
Although the parties have already conducted discovery in related proceedings regarding
275 Madison’s removal, 275 Madison has failed to present any competent evidence to
rebut the presumption of good faith. KE’s motion for summary judgment on its claim is
therefore granted.

H) Ownership interests and Transferability


Transferability
• So let’s say Kima, McNulty, and DeAngelo form TG, Inc., to act as the GP in their LP.
• TG’s only officers, directors, and shareholders are Kima, McNulty, and DeAngelo.
• What if Bubbles wants to become a Partner?
• May become a Partner if:
• the Partnership allows
• the Agreement allows
• all the Partners vote and say yes
• GET ONLY THE RIGHTS TO PROFIT, NOT CONTROL

§ 702. Assignment of Partnership Interest.


• Except as provided in the Partnership Agreement, a Partnership interest is assignable in whole or in part. An assignment
of a Partnership interest does not dissolve a limited Partnership or entitle the assignee to become or to exercise any rights
of a Partner. An assignment entitles the assignee to receive, to the extent assigned, only the distribution to which the
assignor would be entitled. Except as provided in the Partnership Agreement, a Partner ceases to be a Partner upon
assignment of all his [or her] Partnership interest.
§ 101. Definitions.
• (10) “Partnership interest” = a Partner's share of the profits and losses of a LP and the right to receive distributions
of Partnership assets.
§ 704. Right of Assignee to Become LimPart.
• (a) An assignee of a Partnership interest, including an assignee of a general Partner, may become a limited Partner if
and to the extent that
(i) the assignor gives the assignee that right in accordance with authority described in the Partnership
Agreement; OR
(ii) ALL other Partners consent.
ULPA (2001)
§ 701. Partner's Transferable Interest.
The only interest of a Partner which is transferable is the Partner's transferable interest. A transferable interest is personal
property.
§ 702. Transfer of Partner's Transferable Interest.
(a) A transfer, in whole or in part, of a Partner's transferable interest:
(1) is permissible;
(2) does NOT by itself cause the Partner's dissociation or a dissolution and winding up of the limited
Partnership's activities; and
(3) does NOT, as against the other Partners or the LP, entitle the transferee to participate in the management or
conduct of the LP's activities, to require access to information concerning the limited Partnership's
transactions except as otherwise provided in subsection (c), or to inspect or copy the required information or
the limited Partnership's other records.
 (b) A transferee has a right to receive, in accordance with the transfer:
(1) distributions to which the transferor would otherwise be entitled; and
(2) upon the dissolution and winding up of the limited Partnership's activities the net amount otherwise
distributable to the transferor.
UPLA (2001) § 102.
(22) “Transferable interest” means a Partner's right to receive distributions.

ULPA § 301. Becoming Limited Partner.


A person becomes a LimPart:
 (1) as provided in the Partnership Agreement;
 (2) as the result of a conversion or merger under [Article] 11; or
 (3) with the consent of all the Partners.

RULPA (1985)§ 401. Admission of Additional General Partners.


After the filing of a LP's original certificate of limited Partnership, additional GenParts may be admitted:
 as provided in writing in the Partnership Agreement ; OR,
 if the Partnership Agreement is silent, with the written consent of all Partners.
Note: ULPA (2001) has a similar provision.

• What if TG, Inc., is solely owned by DeAngelo, and he wishes to transfer the GP over to Bubbles, but Kima and McNulty
don’t – could DeAngelo simply sell his shares of TG to Bubbles?
• Could DeAngelo merge TG into Bubbles, Inc.? (See Star Cellular and compare In re Asian Yard Partners).

 [EB] (1) Limited Partners


o General rule: just like general partnership statutes, financial rights are transferrable in limited
partnerships while management rights are not
o RULPA (1985)
 § 702: Assignment of Partnership Interest
 A partnership interest is assignable, but does not give the assignee any partnership
rights, except to distributions to which the assignor would be entitled
 § 704: Right of Assignee to Become a Limited Partner
 An assignee of a partnership interest may become a limited partner if and to the extent
that
o (i) the assignor gives the assignee that right in accordance with authority
described in the partnership agreement; or
o (ii) all other partners consent
o ULPA (2001)
 § 701: Partner’s Transferable Interest
 The only interest of a partners which is transferable is the partner’s transferable
interest, which means a partners’ right to receive distributions
 § 702: Transfer of Partners Transferable Interest
 A transfer of a partner’s transferable interest
o Does not cause dissolution or winding up
o Does not entitle transferee to participate in management
o Only gives the transferee a right to receive distributions to which transferor
would otherwise be entitled, including the net amount owed to transferor upon
dissolution and winding up
 § 301: Becoming a Limited Partner
 A person becomes a limited partner
o As provided in the partnership agreement; or
o With the consent of all the partners
o NEVER by transferring the interest to another
o Charging order
 Comes up when a judgment creditor obtains a judgment against a partner and views the
partnership as an asset
 Analogous to a partnership, under RULPA/ULPA, you just get to sit back as the judgement
creditor and hope that the partnership declares distributions because that is all you are
entitled to
 (2) General Partners
o RULPA § 401: Admission of Additional General Partners
 After the filing of a limited partnership’s original certificate of limited partnership, additional
general partners may be admitted as provided in writing in the partnership agreement or, if the
partnership agreement does not provide in writing for the admission of additional general
partners, with the written consent of all partners
o ULPA (2001)
 Similar provision
 Control of the Entity Partner Through Sale or Merger
o Hypo

 Assume the partnership agreement doesn’t allow for a new general partner and assume you
cant obtain full consent from the other limited partners
 Issue: what if Carl is the only one that wants his friend, Larry, to be the new general partner
running the show; He can’t replace the new general partner because he cant get everyone
consent, but how could he accomplish result
 (1) by selling his shares out of XYZ Inc. (not out of ABC, LP)?
o Rule: Carl is entitled to do that because he owns XYZ Inc. and absent something
in the partnership agreement prohibiting this from happening, there is nothing
in the law that is stopping the transfer of shares of a corporate general
partner. Thus, the entire completion of the board and general partner could
change without any problem
o Transfer of ownership interests of an entity general partner is DISTINCT from
the transfer of the general partner itself
 (2) This could also be accomplished by merging XYZ with Larry’s corporation
 Issue: how do you get around Carl being able to do this (selling shares/merging)
 Drafting
 Star Cellular v. Baton Rouge (Delaware)
o Anti transfer provision  “the general partner may transfer or assign its
general partners interest only after written notice to all to the other partners
and the unanimous vote of all the other partners to permit such transfer”
 If this language were in the hypothetical above  Carl could even
transfer his interest to Larry without unanimous consent (way of getting
around it)
o Held  the word transfer could not be read to encompass a merger (would
need “transfer and/or merger” to truly protect the makeup)

I) Dissociation and Dissolution


Dissociation
• Return to our prior example – where Kima, McNulty, and DeAngelo are the limited Partners, and TG, Inc., is the general
Partner.
• What if TG or DeAngelo want to leave? TG: need a new GenPart. to continue; both get $ in proportion to their contribution
after paying their proportionate share of the LP’s obligations and any proportionate share of profits (or must contribute if net
losses @ time of dissociation!)
• How would they withdraw under RULPA?
• Upon withdrawal, what does the withdrawing Partner get?
• RULPA says “fair value.”
Note that many state have amended the dissociation statute to eliminate the default exit right.
Why? To decrease the tax liability of the person who buys the interest
To understand this, what would be the fair value of a Partnership interest if the Partnership Agreement stated that a limited Partner
were prohibited from withdrawing from the venture (i.e., from receiving fair value for his interest)? MUCH LESS
• Why would the fair market value be lower?
• FMV is lower BC the interest cannot be transferred (conditions on the alienability of the asset reduce the
FMV)
• Why would you want this decrease in value? Estate tax planning purposes
Does IRC § 2704(b) recognize these discounts? So long as the state statute does not contain a default exit right, the IRS will
recognize this “discounted” valuation
• With regard to this “fair value” problem and IRC 2704(b), what has ULPA (2001) done?

RULPA (1985)
§ 603. Withdrawal of Limited Partner.
A limited Partner may withdraw from a limited Partnership at the time or upon the happening of events specified in writing in
the Partnership Agreement. If the Agreement does NOT specify in writing the time or the events upon the happening of which a
limited Partner may withdraw or a definite time for the dissolution and winding up of the limited Partnership, a limited Partner may
withdraw upon not less than six months' prior written notice to each general Partner at his [or her] address on the books of the
limited Partnership at its office in this State.

§ 604. Distribution Upon Withdrawal.


Except as provided in this Article, upon withdrawal any withdrawing Partner is entitled to receive any distribution to which he
[or she] is entitled under the Partnership Agreement and, if not otherwise provided in the Agreement, he [or she] is entitled to
receive, within a reasonable time after withdrawal, the fair value of his [or her] interest in the limited Partnership as of the date
of withdrawal based upon his [or her] right to share in distributions from the limited Partnership.

NOTE: LimPart, unable to get out of the LP, may be “oppressed” similar to minority shareholders in a closely held Corp.

 [EB] (1) Dissociation


o Hypo

o Issue: What if LP2 wants to get out / what if Carl wants XYZ (the general partner) to get out of ABC, L.P.;
how do we accomplish this?
 RULPA (1985) § 602: Withdrawal of General Partner
 A general partner may withdraw from a limited partnership at any time by giving
written notice to the other partners, but if the withdrawal violates the partnership
agreement (because term partnership), the limited partnership may recover from the
withdrawing general partner damages for breach of the partnership agreement and
offset the damages against the amount otherwise distributable to him
 RULPA (1985) § 603: Withdrawal of Limited Partner
 A limited partner may withdraw from a limited partnership at the time or upon the
happening of events specified in writing in the partnership agreement.
 If the agreement does not specify in writing the time or events upon the happenings of
which a limited partner may withdraw for a definite time for the dissolution and winding
up on the limited partnership, a limited partner may withdraw upon not less than six
months prior written notice to each general partner at his address on the books of the
limited partnership at its office in this state
 RULPA (1985) § 604: Distributions Upon Withdrawal
 Except as provided in this article, upon withdrawal any withdrawing partner is entitled
to receive any distribution to which he is entitled under the partnership agreement
and, if not otherwise provided in the agreement, he is entitled to receive, within a
reasonable time after withdrawal, the fair value of his interest in the limited
partnership as of the date of withdrawal based upon his right to share in distributions
from the limited partnership
o Note: may states have amended their versions of RULPA to eliminate this
default exit right to allow people to build in restrictions in the partnership
agreement that get around the internal revenue code and thus can benefit from
this sort of arrangement
 ULPA
 Eliminated default buyout rights for general and limited partners
 (2) Dissolution
Dissolution
• When can the limited Partnership be dissolved? LP can be dissolved upon time specified, time horizon built in the
Agreement, upon written consent of all Partners, if the GP leaves (Under RULPA) and there’s no new GP to fill that
vacuum), OR if it is judicially decreed
• What section governs judicial dissolution?
• Upon dissolution, how should the assets be distributed? In dollars!! – “in kind” is disfavored and must be expressly provided
for in the Partnership Agreement

RULPA
§ 801. Nonjudicial Dissolution.
A limited Partnership is dissolved and its affairs shall be wound up upon the happening of the first to occur of the following:
(1) at the time specified in the certificate of limited Partnership;
(2) upon the happening of events specified in writing in the Partnership Agreement;
(3) written consent of all Partners;
(4) an event of withdrawal of a general Partner unless at the time there is at least one other general Partner and the written
provisions of the Partnership Agreement permit the business of the limited Partnership to be carried on by the
remaining general Partner and that Partner does so, but the limited Partnership is not dissolved and is not required to
be wound up by reason of any event of withdrawal if, within 90 days after the withdrawal, all Partners agree in writing
to continue the business of the limited Partnership and to the appointment of one or more additional general Partners if
necessary or desired; or
(5) entry of a decree of judicial dissolution under Section 802.
§ 802. Judicial Dissolution.
On application by or for a Partner the [designate the appropriate court] court may decree dissolution of a limited Partnership
whenever it is not reasonably practicable to carry on the business in conformity with the Partnership Agreement.
§ 804. Distribution of Assets.
Upon the winding up of a limited Partnership, the assets shall be distributed as follows:
(1) to creditors, including Partners who are creditors, to the extent permitted by law, in satisfaction of liabilities of the limited
Partnership other than liabilities for distributions to Partners under Section 601 or 604;
(2) except as provided in the Partnership Agreement, to Partners and former Partners in satisfaction of liabilities for
distributions under Section 601 or 604; and
(3) except as provided in the Partnership Agreement, to Partners first for the return of their contributions and secondly
respecting their Partnership interests, in the proportions in which the Partners share in distributions

ULPA (2001)
§ 812. Disposition of Assets; When Contributions Required.
(a) In winding up a limited Partnership's activities, the assets of the LP, including the contributions required by this section, must be
applied to satisfy the LP's obligations to creditors, including, to the extent permitted by law, Partners that are creditors.
(b) Any surplus remaining after the LP complies with subsection (a) must be paid in cash as a distribution.

o RULPA § 801: Nonjudicial Dissolution


 A limited partnership is dissolved:
 (1) at the time specified in the certificate of limited partnership (term partnership);
 (2) upon the happenings of events specified in writing in the partnership agreement
(could still be considered a term)
 (3) upon the written consent of all the partners
 (4) an event of withdrawal of a general partner under 402 (except when certain
requirements are met); or
 (5) by entry of a decree of judicial dissolution under section 802 (i.e. because of
deadlock)
o RULPA § 802: Judicial Dissolution
 On application by or for a partner the court may decree dissolution of a limited partnership
whenever it is not reasonably practicable to carry on the business in conformity with the
partnership agreement
 Means  when we have deadlock or freezeout or such poor compatibility that the
business can’t function, a limited or general partner could go to the judge and ask for it
to be dissolved judicially
o RULPA § 804: Distribution of Assets (upon dissolution)
 (1) creditors
 (2) deficient partner distributions (partners behind on distributions)
 (3) return of contributions
 (4) surplus paid as distributions
o ULPA (2001) § 801: Nonjudicial Dissolution
 A limited partnership is dissolved, and its activities must be wound up, only upon the occurrence
of any of the following
 (1) the happening of an event written in the partnership agreement
 (2) the consent of all general partners and of limited partners owning a majority of the
rights to receive distributions
 (3) after the dissociation of a person as a general partner (except when certain
requirements are met)
 Note: no unanimity requirement as under RULPA
o ULPA (2001) § 802: Judicial Dissolution
 On application by a partner, the court may order dissolution of a limited partnership if it is not
reasonably practicable to carry on the activities of the limited partnership in conformity with
the partnership agreement
o ULPA (2001) § 804: Distribution of Assets
 (1) creditors
 (2) surplus in cash as distributions

Recapitulation
Fiduciary Duties:
• GP fiduciary duties imported from Partnership law (see Duty of care and Duty of Loyalty, under RUPA)
• Fiduciaries of a limited liability Entity GenPart may be held to have fiduciary duties to LPs
• Limited Partners might owe fiduciary duties if they gain control, managerial discretion
Transferability: can’t generally transfer non-financial Partnership rights; might avoid this through transfer/merger of limited liability
Entity GP
Dissociation:
• Under RULPA (1985), LimParts are more restricted than GenParts in ability to withdraw
• Due to 2704(b), some states eliminate those default rules
Dissolution: understand the basic rules of RULPA (1985) and ULPA (2001) regarding when dissolution occurs and distribution of
assets upon dissolution

Recap
 Fiduciary duties
o General partner fiduciary duties imported form partnership law (see duty of care and duty of loyalty,
under RUPA)
o Fiduciaries of a limited liability entity GP may be held to have fiduciary duties to LP’s
o Limited partners might owe fiduciary duties if they gain control, managerial discretion
 Transferability
o Can generally transfer non-financial partnership rights; might avoid this through transfer/merger of
limited liability entity GP
 Dissociation
o Under RULPA limited partners are more restricted than general partners in ability tot withdrawal
o Due to 2704(b), some states eliminate those default rules
 Dissociation
o Understand the basic rules or RULPA and ULPA regarding when dissolution occurs and distributions of
assets upon dissolution

Chapter 19 – The Limited Liability Partnership


A) Introduction
 LLC  noncorporate business structure that provides its owners, known as “members” with a number of
benefits
o (1) limited liability for the obligations of the venture, even if a member participates in the control of the
business;
o (2) pass through tax treatment; and
o (3) tremendous freedom to contractually arrange the internal operations of the venture
 Statutes (haven’t been widely adopted)
o ULLCA (uniform limited liability company act)  promulgated in 1996, but only four states in the US
Virgin Islands have adopted the ULLCA
o In 2006, the ULLCA was substantially revised and updated, but only four states have adopted the revised
ULLCA (RULLCA)
o In 2013, an amended version of RULLCA was introduced (2013 RULLCA); 12 states and the district of
Columbia have adopted 2013 RULLCA
o Consequentially, one must look to law on state-by-state basis – see e.g. DLLCA or Texas Business
Organizations Code

B) Formation
a. (1) Certificate / articles of organization (public), (2) Operating agreement.
b. Will help client get Tax ID number (EIM)
c. Opening a bank account to avoid co-mingling. Take documents and Tax ID number.
 Section 202: Organization
o (a) one or more persons may organize a limited liability company, consisting of one or more members,
by delivering articles of organization to the office of the secretary of state for filing
o (b) unless a delayed effective date specified, the existence of an LLC begins when the articles of
organization have been filed
o (c) the filing of the articles of organization by the [secretary or state] is conclusive proof that the
organizers satisfied all conditions precedent to the creation of a limited liability company
 Section 203: Articles of Organization
o (1) name of the company
o (2) the address of the initial designated office
o (3) the name and the street address of the initial agent for service of the process
o (4) the name and address of each organizer
o (5) whether the company is to be a term company and, if so, the term specified
o (6) whether the company is to be manager-managed, and, if so, the name and address of each initial
manager; and
o (7) Whether one or more of the members of the company are to be liable for its debs and obligations
under section 303(e)
 Section 203 Cmt.: Purpose of Articles of Organization
o The articles serve primarily a notice function and generally do not reflect the substantive agreement of
the members regarding the business affairs of the company
 Note: this is simply making sure others know this is a limited liability entity
o Operating agreement  governs the business affairs of an LLC; this may be unwritten
 ULLCA § 203: conflict between articles of incorporation and operating agreement
o (c) articles of organization of an LLC may not vary the nonwaivable provisions of section 103(b). As to all
other matters, if any provision of an operating agreement is inconsistent with the articles of
organization
 (1) the operating agreement controls as to managers, members and members’ transferees; and
 (2) the articles of organization control as to persons, other than managers, members and their
transferees, who reasonably rely on the articles to their detriment
 Note: the operating agreement governs disputes among the insiders of the firm
(members, managers, transferees) while the articles of organization control disputes
involving outsiders of the firm (this is a function of notice)
o Issue: does LLC have to have operating agreement to be effective?
 Holding: no statutory requirement for an operating agreement (“LLC agreement may…”); LLC
should have operating agreement because the statutes tend to include very few default rules,
thus without an operating agreement there can be a great deal of uncertainty about basic
governance matters. Further, most statutory schemes contemplate that the members will enter
into some type of operating agreement
o Issue: is it possible, if one of your clients is already operating as a partnership or general partnership, to
then convert into an LLC?
 Holding: Yes – all limited and general partnerships can convert to an LLC; they would most like
want to do this for limited liability for everyone involved
C) Management and Operation (General Governance)
 General ways LLC can be managed
o (1) member managed
o (2) manager managed
 Section 101: Default Rule how LLC is Managed
o (10) “manager”  a person, whether or not a member of a manager-managed company, who is vested
with authority under section 301
o (11) “manager-managed company”  limited liability company which is so designated in its articles of
organization
o (12) “member-managed company”  limited liability company other than manager-managed company
 Default Rule: an LLC is a member managed (just like with partnership), unless otherwise set
forth in the articles of organization
 Manager Managed > Member Managed
o Members might not have the skill, experience, or time to participate effectively in the management of
an LLC
o Manager managed LLC with professional management may be preferred
o Note  limited liability makes the passive role of non-managers more palatable
 General partnership  passive role risks personal liability as a result of the poor decision of
others
o Just like with general partnership, the greater number of members, the harder it is to have a member
managed structure where everyone is entitled to participate in management  result is too many cooks
in the kitchen
 Member Managed > Manager Managed
o Unlike public corporation, LLC members cannot use the market to cash out the business whenever they
are dissatisfied with the direction of the LLC
o The LLC interests cannot be freely transferred and there is usually no market at all for LLC interests
o As a consequence, members may want an active management role to have some say in the direction of
the business and to help protect the value of their investments
 Voting Rights
o Depends on if the LLC is member managed or manager managed but basically follows the per capita
rule (voting based upon contributions)
 One vote per member or manager depending on the LLC you’ve elected to do
 Per capita is preferred when the contributions of the members are substantially different (2
member LLC – $10k v. $90k commitment).
o Most, if not all, will change to pro rata.
 Q. Why would we encourage them to have pro rata type voting power?
 A. Allows people with bigger stake in the company to have more of an influence.
o ULLCA 404
 (a) in a member-managed company
 (1) each member has equal rights in the management and conduct of the company’s
business; and
 (2) except as otherwise provided in subsection (c), any matter relating to the business of
the company may be decided by a majority of the members
 (b) in a manager-managed company
 (1) each manager has equal rights in the management and conduct of the company’s
business
 (2) except as otherwise provided in subsection (c), any matter relating other business of
the company may be exclusively decided by the manager or, if there is more than one
manager, by a majority of the manager; and
 (3) a manager
o (i) must be designated, appointed, elected, removed, or replaced by a vote,
approval, or consent of a majority of the member; and
o (ii) holds office until a successor has been elected and qualified unless the
manager sooner resigns or is removed
 (c) the only matters of a member or manager managed company’s business requiring the
consent of all the members are
 (1) the amendment of the operating agreement
 (2) the authorization or ratification of acts or transactions which would otherwise
violate the duty of loyalty
 (3) an amendment to the articles of organization
 (7) admission of a new member
 (9) the consent to dissolve the company
 (11) consent of the members to merge with another entity
 Authority to act and bind
o Note: Decision of either member-managed, or manager-managed form affects who has the authority to
bind the LLC
o ULLCA § 201: Agency of Members and Managers
 (a) subject to subsection (b) and (c):
 (1) each member is an agent of the LLC for the purpose of its business, and as an act of
a member including the signing of an instrument in the company name, for apparently
carrying on in the ordinary course of the company’s business or business of the kind
carried on by the company binds the company, unless the member has no authority to
act for the company in the particular matter [see articles or operating agreement] and
the purpose with whom the member was dealing knew or had notice that the member
lacked authority
 (2) an act of a member which is not apparently for carrying on in the ordinary course of
the company business or the business of the kind carried on by the company binds the
company only if the act was authorized by the other members
 (b) subject to subsection (c), in a manager managed company;
 (1) a member is not an agent of the company for the purpose of its business solely by
reason of being a member. Each manager is an agent of the company for the purpose
of its business and an active manager including the signing of an instrument in the
company’s name apparently carrying on in the ordinary course of the company’s
business or business of the kind carried on by the company binds the company unless
the member has no authority to act for the company in the particular matter [see
articles or operating agreement] and the purpose with whom the member was dealing
knew or had notice that the member lacked authority
 (2) an act of a manager which is not apparently for carrying on in the ordinary course of
the company business or the business of the kind carried on by the company binds the
company only if the act was authorized under section 404
 (c) Unless the articles of organization limit their authority, any member of a member managed
company or manager of a manager-managed company may sign and deliver any instrument
transferring or affecting the company’s interest in real property. The instrument is conclusive in
favor of a person who gives value without knowledge of the lack of authority of the person
signing and delivering the instrument
 Takeaway
o (a)/(b) Members are agents for ordinary business except in manager managed
companies where managers generally have authority and members do not
o (c) Members/managers have more explicit authority which is harder to limit
when real estate is involved
o Taghipour v Jerez (shows how difficult it can be to restrict the authority of owners and managers in the
LLC / non-uniform state)
 Facts
 Taghipour, Rahemi, and Jerez formed Jerez, Taghipour and Associates, LLC. The articles
of organization designated Jerez as manager of the limited liability company (LLC). The
operating agreement between the LLC members stated that no loans could be made on
the LLC’s behalf unless authorized by a resolution of all the members. Jerez obtained a
loan on behalf of the LLC from Mt. Olympus (with LLC real estate as collateral) without
the other members’ knowledge. Jerez failed to make loan payments, the LLC defaulted,
and Mt. Olympus foreclosed on the LLC’s property. Taghipour, Rahemi, and the LLC
(Taghipour) (plaintiffs) filed suit against Mt. Olympus and Jerez (defendants), claiming
that the loan agreement and foreclosure were invalid under the terms of the operating
agreement (b/c defendant lacked authority under the operating agreement)
 Issue
 Where a statute states that mortgage documents executed by a limited liability
company’s manager are valid and binding on the company, but the operating
agreement states that no loans can be made without authorization by all the members,
will mortgage documents executed by a manager without the other members’ consent
be valid and binding on the company? (mortgage statute v. restriction statute)
 Rule
 General Rule: When two statutory provisions cover the same subject and conflict in
their operation, the provision that is more specific in application will govern over the
more general provision.
o Where a statute states that mortgage documents executed by a limited liability
company’s manager are valid and binding on the company, but the operating
agreement states that no loans can be made without authorization by all the
members, mortgage documents executed by a manager without the other
members’ consent will be valid and binding on the company.
 Utah Code § 48-2b-127(2)  states that instruments and documents providing for the
acquisition, mortgage, or disposition of property of an LLC are valid and binding on the
LLC if they are executed by a manager.
 Utah Code § 48-2b-125(2)(b)  states that an LLC’s manager has authority to bind the
LLC, unless otherwise provided in the articles of organization or operating agreement.
 Holding
 The trial court and the court of appeals  found that the loan agreement in this case
was valid under § 48-2b-127(2), because Jerez is the LLC’s manager.
 Plaintiff argues that the loan agreement was not valid and binding on the LLC under §
48-2b-125(2)(b), because the operating agreement between the LLC members stated
that no loans could be made on the LLC’s behalf unless authorized by a resolution of all
the members.
 When two statutory provisions cover the same subject and conflict in their operation,
the provision that is more specific in application will govern over the more general
provision.
o Here, § 48-2b-127(2) is more specific, because it applies only to specific
documents, that is, instruments and documents providing for the acquisition,
mortgage, or disposition of LLC property.
o Section 48-2b-125(2)(b) is more general because it applies to every situation in
which a manager can bind an LLC.
 Plaintiff argues  that § 48-2b-125(2)(b) is more specific because it is more restrictive.
However, whether a statute is more specific in application does not depend on
whether it is more restrictive in application. In addition, a finding that § 48-2b-125(2)(b)
is more specific would render § 48-2b-127(2) superfluous.
 Holding  Under § 48-2b-127(2), the loan agreement is valid and binding on the LLC,
because it was made by the LLC’s manager, Jerez. The court of appeals’ decision
affirming the trial court’s dismissal of the claims against Mt. Olympus is therefore
affirmed.
 Notes
 Issue: how could the LLC members have protected themselves in this case
 Option 1: members could file public documents that placed creditors on notice of
restrictions on managers authority  can put this authority under the articles of
organization and the file those in real property records in the county the real estate is
located; this would maybe convey some constructing knowledge of limited authority
(this may not overcome the plain language of the Utah statute)
 Option 2: members could put something in the operating agreement requiring the
manager to indemnify the LLC for any authorize action; the problem is he is probably
bankrupt or in prison
 Under ULLCA (if uniform state) persons dealing with an LLC in real property
transaction must review the articles restrictions on authority or bare the risk
o (c) wouldn’t have helped in this case because the authority restriction is in the
operating agreement, not the articles, and it must be in the articles (to provide
notice) since it’s the public document

D) Inspection and information rights


 Note: LLC statutes often provide members and managers rights to inspect books, and records of the venture.
o Better option: build something in to the operating agreement that gives them the right.
 ULLCA § 408: Members right to information
o (a) a limited liability company shall provide members…access to its records, if any, at the company’s
principal office…the company shall provide former members…access for proper purposes to records
pertaining to the period during which they were members the right of access provides the opportunity
to inspect and copy records during the ordinary business hours. The company may impose a reasonable
charge, limited to the costs of labor and material copies to records furnished
o (b) a limited liability company shall furnish to a member
 (1) without demand, information concerning the company business or affairs reasonably
required for the proper exercise of the members rights and performance of the members duties
under the operating agreement or this act; and
 (2) on demand, other information concerning the company’s business or affairs, except to the
extent the demand or the information demanded is unreasonably or otherwise improper under
the circumstances
o (c) a member has the right upon written demand given to the limited liability company to obtain at the
company’s expense a copy of any written operating agreement
 Kasten v. Doral Dental (Record (a) v. Information (b))
o Facts
 Marie Kasten (plaintiff) was a member of Doral Dental USA, LLC (Doral) (defendant). Suspecting
that Doral’s management was engaging in actions adverse to her interests, Kasten made
numerous requests to inspect company records, including electronic files and documents
drafts. When Doral did not allow inspection of all of the documents requested, Kasten filed suit,
seeking an order requiring Doral to provide her with the documents.
 Defendant argument  request is way too broad and unreasonable

o Issue
 Where a limited liability company’s operating agreement allows any member, upon reasonable
request, to inspect any record or company document, does a member’s right of inspection
include the right to inspect emails and document drafts?
 What is the scope of an LLC member’s right of inspection, and does this right encompass the
right to inspect emails and document drafts
o Rule
 Rule: The Wisconsin Limited Liability Company Law provides that a limited liability company
member may, upon reasonable request and at the member’s own expense, inspect and copy
any company record, unless otherwise provided in the operating agreement.
 Where a limited liability company’s operating agreement allows any member, upon
reasonable request, to inspect any record or company document, a member has the
right to inspect emails and document drafts, provided the request is not so burdensome
as to be unreasonable.
 Section 183.0405(2) WLLCL  provides that a member may, upon reasonable request and at
the member’s own expense, inspect and copy any LLC record required to be kept under §
183.0405(1), and any other LLC record, unless otherwise provided in the operating agreement.
 The WLLCL, unlike Wisconsin’s Uniform Limited Partnership Act, does not restrict the
right to inspect to those records specifically enumerated by the statute. Nor does the
WLLCL, like Wisconsin’s corporation inspection statute, require that inspection requests
be made for a proper purpose and identify with reasonable particularity the records
sought, or limit inspection to only those records that are directly connected with the
requester’s purpose.
 Doral’s operating agreement  like § 183.0405(2), provides for inspection upon reasonable
request, but provides a right to inspect both records and “company documents.”
 Section 183.0405(2) WLLCL  by its plain language, gives an LLC member the right to inspect
anything that is a record, upon reasonable request. The WLLCL also permits companies to craft
their own inspection rules in an operating agreement.
o Holding
 Doral’s operating agreement  like § 183.0405(2), provides for inspection upon reasonable
request, but provides a right to inspect both records and “company documents.”
 The dictionary definition of a record is an “authentic official copy of a document
deposited with a legally designated officer,” with “document” defined as “a writing
conveying information” or “a computer file containing information.”
 This court finds that information that is stored as email, as well as a document draft, may be a
“company document” subject to a member’s reasonable inspection under Doral’s operating
agreement.
 However, the operating agreement’s grant of the right to inspect company emails and
document drafts is triggered only upon a member’s reasonable request.
 The WLLCL does not explain how to apply the reasonable request requirement. In other
states that have adopted inspection statutes, approximately half permit inspection only
upon demand for a proper purpose. Other states, such as Wisconsin, do not contain a
proper purpose requirement, and instead require only that the request be reasonable.
 The absence of a proper purpose requirement in the WLLCL indicates that the
legislature did not intend to require LLC members to demonstrate that their inspection
request was not being made with an improper motive.
 However, the purpose of the reasonable request requirement is to protect a company
from member inspection requests that would impose an undue financial burden on the
company.
 Thus, the reasonable request requirement pertains to the breadth of an inspection, as
well as the timing and form of the inspection.
 In determining whether an inspection request is so burdensome as to be unreasonable, the
WLLCL’s bias in favor of member inspection must be weighed against the costs to the company.
 Relevant factors in this balancing test may include: (1) whether the request is limited by
date or subject matter; (2) the reason for the request, and whether the request is
related to that reason; (3) the importance of the information to the member’s interest
in the company; and (4) whether the information could be obtained from another
source.
 The circuit court’s order denying inspection of company emails and documents drafts because
they were not company documents is reversed. Whether Kasten’s inspection requests were
reasonable under the WLLCL and the operating agreement is a question for the circuit court,
applying the legal standards set out in this opinion
o Notes
 Issue 1: what is broader, a document or a record?
 Holding: A “document” is much more broad than a record, meaning, the drafter of the operating
agreement actually expanded the members rights by adding ‘document’ on top of what the
statute already allowed, a ‘record’
 Issue 2: Thus, do plaintiffs requests fall under a company ‘document’ (email?)
 Holding: Email might be a company document if its related to the business (personal emails
probably should not be turned over
 Issue 3: both operating agreement and statute provide that all requests have to be reasonable;
is plaintiffs request reasonable?
 Holding: timing, format of the request, breadth of the request, whether proper purpose – don’t
want the member to bury the company in costs and time
 Reasonableness  balancing test: (1) whether the request is limited by date or subject
matter; (2) the reason for the request, and whether the request is related to that
reason; (3) the importance of the information to the member’s interest in the company;
and (4) whether the information could be obtained from another source.

E) Financial rights and obligations


 With respect to financial rights, statutory default rules tend to provide either
o (1) ownership-like equal allocation (per capita)
o (2) corporate limited partnership (pro rata) based upon contributions to the firm
 Section 405: Sharing of and Right to Distributions (default rule)
o (a) any distributions made by a limited liability company before its dissolution and winding up must be
in equal shares
o (b) a member has not right to receive and may not be required to accept, a distribution in kind
o (c) if a member becomes entitled to receive a distribution, the member has the status of, and is entitled
to all remedies available to, a creditor of the limited liability company with respect to the distribution
 Default rule  provides for per capita distribution (it retains the partnership like trait re
allocation)
 Issue 1: What if the operating agreement provides for pro rata distributions; what problems could arise from
allowing additional contributions?
o When financial and voting rights are determined on a pro rata basis, i.e. based on contributions to the
firm, additional contributions can affect the existing balance of power within the company, unless all
the members make contributions in proportion to their existing shares of the business
 Issue 2: Can a member-managed LLC compel the LLC to pay distributions; the company has a large surplus
sitting around and one of the members wants distributions?
o ULLCA § 404(c)(6)
 Indicates that in a member-managed LLC, member can receive an interim profit contribution if
all of the members consent
o RULLCA § 404(b)
 A person has the right to receive distributions before the dissolution and winding up of the LLC
only if the company decides to make an interim distribution
 Left to the LLC’s management to make the decision pursuant to the companies voting
right rules under RULLCA
 This could be a lever for oppression when some are getting salaries but not
distributions are being declared

F) Entity Status
 In every jurisdiction, an LLC is explicitly characterized as a separate entity
 ULLCA Section § 201: LLC as Legal Entity
o An LLC is a legal entity distinct from its members
o *Note*: for federal diversity jurisdiction purposes, the LLC is treated as the aggregate of its members
 Premier Van Shaack Realty v. Sieg
o Facts
 Premier Van Schaack Realty, Inc. (Premier) (plaintiff) entered into an agreement with Thomas
Sieg (defendant) regarding the sale of Sieg’s real estate. Under the agreement, Sieg would owe
Premier a commission, in the amount of 7 percent of the acquisition price, if a sale or exchange
of the property occurred. Sieg signed an operating agreement forming MJTM, a limited liability
company. Under the operating agreement, Sieg would convey the real estate to MJTM, and
would receive a 40 percent interest in MJTM, a 9 percent preferential return on its future
profits, and a beginning balance of $670,000 in his initial capital contribution account. MJTM
also agreed to assume $580,000 of Sieg’s debt, and the other LLC members agreed not to
encumber the real estate without Sieg’s approval. The operating agreement also stated that the
property was valued at $1.3 million. Sieg transferred title of the property to MJTM. Sieg
personally guaranteed a $1.413 million loan that MJTM obtained with the property as security,
which it used to pay off $300,000 of Sieg’s debt. Premier demanded a commission of 7 percent
of $1.3 million. Sieg refused to pay, and Premier brought suit.
 Sieg goes into business with buyers / forms an LLC / contributes the property as his
capital contribution to the LLC / receives a promise preferential return and a promise by
the LLC to repay Sieg the difference (since the property was worth more than his
percentage received as his interest)
 Premiere sues for breach of brokerage agreement, arguing that this was a sale or
exchange and consideration was received by Sieg so he owed them the realtors
commission fee
 Sieg claim  this was simply an investment of his property (ive neither sold nor
exchanged)
o Issue
 Does a sale or exchange occur when a property owner conveys his property to a limited liability
company in which he is a member if he retains an ownership interest in the property?
o Rule
 No. A sale or exchange does not occur when a property owner conveys his property to a limited
liability company in which he is a member if he retains an ownership interest in the property.
 in either a sale or an exchange, there must be consideration given
 One who maintained a substantial ownership interest in the property caused him to assume the
risks of an investor, not the risks of a seller
 An investor assumes the risk that the value of his investment will increase or decrease,
or be completely lost.
 In contrast, once a person sells property, he no longer has the risk associated with any
appreciation or depreciation of the property value.
o Holding
 plaintiff argues  that the transaction between Sieg and MJTM constituted a sale or exchange,
thus triggering the 7 percent brokerage fee provision in its agreement with Sieg.
 However, in either a sale or an exchange, there must be consideration given.
 plaintiff argues  that Sieg received consideration for the property in the form of a 40 percent
interest in MJTM and a 9 percent preferential return on its future profits.
 Defendant cites  to two cases in which the courts found that a sale or exchange does not
occur if a property owner conveys his property to a partnership or joint venture in which he is
a member, in exchange for an interest in the company and a preferential interest in its future
profits, if he retains an ownership interest in the property.
 In this case, the interest in MJTM and preferential interest in the company’s future profits does
not constitute valuable consideration, because Sieg maintained a substantial ownership
interest in the property, which caused him to assume the risks of an investor, not the risks of a
seller.
 An investor assumes the risk that the value of his investment will increase or decrease,
or be completely lost.
 In contrast, once a person sells property, he no longer has the risk associated with any
appreciation or depreciation of the property value.
 Here, Sieg retained an ownership interest in the property, including the potential value of its
future sale and a right to prevent MJTM from encumbering the property without his permission.
Although Premier argues that MJTM’s assumption of $580,000 of Sieg’s debt also qualifies as
consideration, the promised debt relief was merely illusory. Sieg himself personally
guaranteed the $1.413 million loan that MJTM used to pay off $300,000 of Sieg’s debt. Because
Sieg continued to have the same ownership interest in the property after transferring it to
MJTM as he had before the deed was executed, there was no consideration. A sale or exchange
therefore did not occur, and Sieg owes no commission under his agreement with Premier.
o Notes
 Sieg didn’t technically transfer the property, in fact, he retained ownership in the property
because he became an owner of the LLC i.e. he continued to have substantially the same
ownership in the property after it was transferred to the LLC
 Case law  dealing with cases involving partnerships, not LLC’s so it seems that the court is
analogizing an LLC directly to a partnership, treating LLC like a non-entity partnership meaning
that the ownership just flows through from the entity to the members
 Adopting aggregate view of partnerships but in the context of an LLC
 Issue: Does Sieg actually have any ownership property of the LLC
 No  under the statute, the LLC is a separate entity and title was actually placed in that
separate entity; he is simply a percentage member of that separate entity, not the title
owner
 Hauge  thinks this was a transfer; if the LLC became liable to a third party, that party
could sue, obtain a judgement, and actually take the property via a share of sale; it
clearly belongs to the LLC
 Takeaway
 Courts can be fooled into viewing the LLC as a non-entity type of partnership rather
than a distinct entity which it clearly is under the law

Recap
 Formation
o Formed by publicly filing articles
o Most issues will be governed by internal operating agreement
 Management and operation
o Default  member managed
o Default  per capita voting
o Authority depends on member-managed scheme v. manager-managed scheme
 Inspection and information rights
o Jurisdictions likely vary; pay attention to what can be reviewed and any constraints on review rights (i.e.
reasonableness or scope)
 Financial rights and obligations
o Default rule for distribution  per capita
 Entity status
o Distinct entity, though courts sometimes mistakenly undertake partnership-type aggregate analysis

G) Limited Liability
 Limited liability  You aren’t vicariously liable, not liable due to the actions of the business entity
o we are not talking about your own bad acts, you can never escape liability for your own personal acts
 1) The scope of Limited Liability
o Pepsi-Cola v. Handy (the intentional failure to disclose an issue with property in order to make money
and then hide behind the corporate veil in order to avoid liability)
 Facts
 Randall Handy, officer, director, and shareholder of Handy Realty, Inc., contracted to
purchase a parcel of real property for development on behalf of himself, Michael
Ginsburg, and Larry McKinley. Handy then learned that the property contained
wetlands, which negatively affected the property’s value and development potential.
The Pepsi-Cola Bottling Company (Pepsi) (plaintiff) acquired an option to purchase the
property from Handy, who did not disclose the existence of the wetlands. Handy,
Ginsburg, and McKinley then formed a limited liability company (LLC), Willow Creek.
Willow Creek purchased the property for $174,000, and four months later sold the
property to Pepsi for $455,000. After learning of the wetlands, Pepsi brought an action
for rescission and damages against Handy Realty, Inc., Willow Creek, Handy, Ginsburg,
and McKinley (defendants). Handy, Ginsburg, and McKinley (Handy) moved for dismissal
of Pepsi’s claims against them, arguing that Pepsi cannot recover against individual
members of the LLC.
o Pepsi alleges  bought property from willow Creek LLC
o Handy, who was member and manager of Willow Creek, knew the property
contained wetlands and fraudulently failed to disclose that fact when it sold the
property to Pepsi
o These wrongdoings took place before the LLC was formed and before the
property was acquired by the LLC
o Pepsi sues the individuals for recission and return of the money
o Individuals  file motion to dismiss for failure to state a claim because
protected from limited liability principles as a matter of law
 Issue
 Is a member of a limited liability company protected against liability for claims arising
from acts occurring prior to the formation of the company?
 Whether the defendants here are being sued ‘solely by reason of being a member of
willow creek’ (LLC) where the claim is based on fraudulent acts committed by the LLC
members before the LLC was formed and took title to the property
 Rule
 Rule: A member of a limited liability company is protected against liability for claims
arising solely by reason of being a member or acting as a manager of the company.
o A member of a limited liability company (LLC) is not protected against liability
for claims arising from acts occurring prior to the formation of the LLC.
 Section 18-303(a) of the Delaware Limited Liability Company Act  protects LLC
members against liability for claims arising solely by reason of being a member or acting
as a manager of the LLC.
o The language of § 18-303(a) implies that LLC members will not be shielded from
liability on claims that do not arise solely by reason of being a member or
acting as a manager of an LLC.
 Delaware law provides  to make a member liable for the acts of the business
(vicariously liable), the plaintiff must be able to
o (1) pierce the corporate veil; or
o (2) proof that there was an illegal distribution made that rendered the
corporation insolvent
 THESE ARE NOT EXCLUSIVE
o (3) individuals liable for their own acts, their own torts
 Holding
 It is clear that in this case, Handy is not being sued solely by reason of being a member
of Willow Creek. Pepsi’s claims are based on fraudulent acts committed by the LLC
members before the LLC was formed and took title to the property. Handy contracted
to purchase the property from a third party, received conclusive evidence that the
property contained wetlands, abandoned construction plans, and negotiated an option
with Pepsi to purchase the property without disclosing the existence of wetlands, all
prior to forming the LLC and acquiring title to the property.
o Handy is therefore not protected from liability under § 18-303.
 Handy also argues  that the LLC members are protected from liability based on DLLCA
§ 18-607, which states that LLC members who know, at the time a distribution is made,
that it will leave the LLC insolvent, are liable to the LLC for the amount of that
distribution.
o Handy argues that § 18-607 is the only provision allowing for an LLC member’s
individual liability, absent a piercing of the corporate veil.
o However, nothing in § 18-607 provides that LLC members are shielded from
liability against all claims except those arising under 18-607.
o In addition, under § 18-303 LLC members may be liable for any claims that do
not arise solely by reason of being a member or acting as a manager of an LLC.
 Because Pepsi’s claims are based on conduct that occurred before Willow Creek was
formed, the individual LLC members are not protected from liability, and Handy’s
motion to dismiss is therefore denied
 Notes
 Holding  because it was established that the LLC was not formed and the property
was not acquired by the LLC until after the allegedly critical wrongful acts had been
committed, it follows that defendants could not have been acting ‘solely as members of
the LLC when they committed those acts
 Issue: Result had the actions occurred before LLC formed/property acquired
o No  when a member gets sued for personally committing a tort or
wrongdoing (fraudulent concealment or failure to disclose), the member is not
being sued solely by reason of being a member; they are being sued for their
own wrongdoing
 Issue: Would the result have changed if the two other individuals that formed the LLC
did not know of the misrepresentation/
o They could not be held liable unless they committed fraud or some other act of
wrongdoing themselves
o Couldn’t be held liable simply because of their status as co-members with
defendant
 Issue: would result have been different if this were a case under ULLCA
o No – standard is the same (see below)
o ULLCA § 303: Liability of Members and managers
 (a) except as otherwise provided in subsection (c), the debts, obligations, and liabilities of a
limited liability company, whether arising in contract, tort, or otherwise, are solely the debts,
obligations, and liabilities of the company. A member or manager is not personally liable for a
debt, obligation, or liability of the company solely by reason of being or acting as a member or
manager

 2) Piercing the Corporate veil (in the LLC context)


o Kaycee Land & Livestock v. Flahive
 Facts
 Kaycee Land and Livestock (Kaycee) (plaintiff) agreed to allow Flahive Oil & Gas, LLC to
use the surface of Kaycee’s property. Flahive Oil & Gas’s use allegedly contaminated
Kaycee’s property. Kaycee brought suit against Roger Flahive, the managing member of
Flahive Oil & Gas, personally, seeking to pierce the limited liability veil of an LLC.
 Issue
 May an aggrieved party pierce the LLC veil in the same manner as it would pierce the
corporate veil?
 Whether, in the absence of fraud, the entity veil of an LLC can be pierced in the same
manner as a corporation
 Rule
 Rule: An aggrieved party may pierce the LLC veil in the same manner as it would pierce
the corporate veil (aside from corporate formalities)
o Simply because Wyoming has no statute providing for the piercing of the LLC
veil does not mean that the legislature did not intend it to be a valid remedy.
o Piercing the veil is an equitable remedy, so the lack of express statutory
authority to do so is not a bar to doing so.
o Furthermore, there is no reason, “in either law or policy,” that LLCs should be
treated differently than corporations in this context.
o Accordingly, piercing the LLC veil can be an appropriate remedy if the facts of
the case so warrant
 Notes
 LLC v. Corporation (re piercing analysis)
o (1) LLC’s share traits of partnership law and general partners are always
vicariously liable for partnership debts
o (2) LLC’s are much more flexible than corporations, so the absence of minutes,
meetings, and corporate formalities are never going to be dispositive
 These organizational formalities necessary to make corporations
function do not apply to LLC’s
o ULLCA § 303: Liability of Members and Managers
 (b) the failure of a limited liability company to observe the usual company formalities or
requirements relating to the exercise of its company powers or management of its business is
not a grounds for imposing personal liability of the members or managers for liabilities of the
company
 Piercing the veil  based on equity and fairness; certain unjust circumstances can arise
because the immunity from liability shelters those who have failed to operate the
corporation as a separate entity; consequently when LLC’s co-mingle funds or ignore
restrictions in its articles of organization or intentionally remain undercapitalized or
taking advantage of others, we may pierce that veil

H) Fiduciary Duties
 1) The Basic Duties
o Number of LLC statutes address the concept of fiduciary duties, many of these indicate that fiduciary
duties are owed to the LLC as well as to the individual members.
o Itemize these in the contract (things that are / aren’t a breach).
o **Always remember: there remains the covenant of good faith and fair dealing.
 What the parties would have reasonably thought at the time of enterting.
 Even if we have itemization or wholesale elimination, still have to focus on reasonable
expectations of time of K.

 VGS v. Castille (Delaware / fiduciary duty is largely judge-made law when it comes to LLC’s)
o Facts
 David Castiel (plaintiff) formed Virtual Geosatellite LLC (the LLC) to pursue an FCC license. The
only member of the LLC was Virtual Geosatellite Holdings, Inc. (Holdings). Subsequently, Ellipso,
Inc. (Ellipso) and Sahagen Satellite Technology Group LLC (Sahagen Satellite) joined the LLC as
members. Castiel controlled Holdings and Ellipso. Peter Sahagen controlled Sahagen Satellite.
The LLC agreement provided that Holdings received 660 units (63 percent of the equity in the
LLC), Sahagen received 260 units (25 percent), and Ellipso received 120 units (12 percent). The
LLC was controlled by a board of three managers. Castiel had the power to appoint and remove
two of the managers, and Sahagen had the powers with regards to the third manager. Castiel
named himself and Tom Quinn as managers and Sahagen named himself as the third manager.
Soon after, the relationship between Castiel and Sahagen soured as the two had different ideas
as to how the LLC should be run. Sahagen convinced Quinn that Castiel could not run the LLC
properly and Sahagen and Quinn, without notice to Castiel, merged the LLC into VGS, Inc.
(VGS) (defendant). The LLC thus ceased to exist and Sahagen took control of VGS. VGS issued
stock in such a way that reduced Holdings’s and Ellipso’s combined interest from 75 percent in
the LLC to 37.5 percent in VGS. Meanwhile, Sahagen went from owning 25 percent of the LLC to
62.5 percent of VGS. The only reason that Sahagen and Quinn did not give Castiel notice of the
merger was because Castiel had the ability to remove Quinn as manager and certainly would
have if he had been given notice of the merger. Castiel brought suit.

o Issue
 Is it always permissible for members of an LLC to neglect to give notice to other members before
acting by written consent on behalf of the LLC?
 Under LLC operating agreement or Delaware law
o Rule
 Rule: Members of an LLC violate their duty of loyalty to a fellow member if they do not give him
notice of an LLC action that is adverse to him and that he would be able to prevent given his
controlling role in the LLC.
 No. Although the LLC Act does not require members of an LLC to give notice to another
member before acting by written consent on behalf of the LLC, the acting members
violate their duty of loyalty to the fellow member if they do not give him notice of the
action when the action is adverse to him and he would be able to prevent the action
given his controlling role in the LLC
 Technical compliance with statutes and operating agreements may not protect parties from
conduct that a court deems inequitable
o Holding
 In the case at bar, Sahagen and Quinn knew that if Castiel got notice of their merger plan, he
would remove Quinn as manager and appoint someone who would not approve the merger.
Because of that knowledge, their failure to notify Castiel was a failure to discharge their duty
of loyalty to Castiel in good faith.
 The purpose of allowing LLC action by written consent without notice is to allow managers to
take action quickly without going through a minority of managers for approval when those
managers could not block the action anyway. That was not Sahagen and Quinn’s purpose as
they knew that Castiel would have blocked the merger and secretly excluded him from the
chance of protecting his majority interest.
 This exclusion was a violation of their Sahagen and Quinn’s duty of loyalty. Accordingly, the LLC
action of merging into VGS was invalid and the merger is rescinded.
o Notes
 Issue: What if defendants truly believed that their secret course of action was the only feasible
way of protecting the LLC from financial ruin?
 Hold: Court implies this wouldn’t matter,
 Issue: What if other members were involved
 Hold: Issue, because now you are trying to lookout for other members, this is a duty to
castile v. duty to several other members (not entirely clear)
 Issue: What provision of Delaware LLCA does the duty of loyalty come from?
 It has no provision on the fiduciary duties of LLC members and managers; the operating
agreement (in this case) does not seem to state anything on point so this is purely a
matter of judicial law making just as fiduciary duty has largely been a matter of
common law in the corporate setting
 ULLCA § 409: General Standards of Members and Manager Conduct
o (a) the only fiduciary duties a member owes to a member-managed company and its other members
are the duty of loyalty and the duty of care imposed by subsection (b) and (c)
o (b) a member’s duty of loyalty to a member-managed company and its other members is limited to the
following
 (1) to account to the company and to hold as trustee for it any property, profit, or benefit
derived by the member in the conduct or winding up of the company business or derived from a
use by the member of the company property, including the appropriation of a company
opportunity;
 (2) to refrain from self-dealing with the company in the conduct or winding up of the company’s
business as or on behalf of a party having an interest adverse to the company; and
 (3) to refrain from competing with the company in the conduct of the company’s business
before the dissolution of the company
o (c) a member’s duty of care to a member-managed company and its other members in the conduct of
and winding up of the company’s business is limited to refraining from engaging in grossly negligent or
reckless conduct, intentional misconduct, or a knowing violation of law
o (d) a member shall discharge the duty to a member-managed company and its other members under
blank or under [this act] or under the operating agreement and exercise any right consistent with the
obligations of good faith and fair dealing
o (h) in a manager-managed LLC
 (1) a member who is not also a manager owes no duties to the company or to the other
members solely by reason of being a member
 (2) a manager is held to the same standards of conduct prescribed for members in subsection
(b) through (f)

 2) The Role of Contract


o Manager managed company  member owes no duties solely by reason of being a member
o Fisk Ventures v. Segal (Delaware doesn’t use ULLCA and doesn’t impose fiduciary duties/based on
operating agreement)
 Facts
 Dr. Andrew Segal (defendant) founded Genitrix, LLC, a Delaware limited liability
company, and retained 55 percent of Genitrix’s Class A membership interest. H. Fisk
Johnson held much of the Class B membership interest, along with Fisk Ventures, LLC
(Fisk) (plaintiff), Stephen Rose, and William Freund. Fisk was owned and controlled by
Johnson. Under Genitrix’s LLC agreement, the board of member representatives, which
managed the company’s business and affairs, consisted of five members: two appointed
by Johnson, one by Fisk, and two by Segal. Johnson appointed Rose and Freund, his
employees, to the board.
 Section 9.1 of the LLC agreement stated that no member has any duty to any other
member, except as expressly set forth in the LLC agreement itself, and that no member
is liable for damage to the company unless it is the result of gross negligence, fraud, or
intentional misconduct. The LLC agreement also required approval of 75 percent of the
board for most actions, thus requiring the cooperation of both the Class A and the
Class B members. The Class B members had a “Put Right” clause in their contracts,
which stated that at any time, they could sell back to Genitrix any or all of their Class B
interests for a price determined by an independent entity. Since it was formed, Genitrix
continually had financial difficulty and trouble attracting investors. Segal felt that the
Put Right contractual clauses scared off potential investors, but despite his multiple
requests, the Class B members refused to suspend their Put Rights. Segal then drafted a
private-placement memorandum he planned to use to attract investors, but the Class B
members refused to approve it. Throughout this time, the plaintiffs infused Genitrix with
operating cash, but not enough to allow Genitrix to succeed. Genitrix soon ran out of
operating cash and was eventually left with Segal as the only employee, no office, no
funds, and no revenue. Fisk brought suit in the Delaware Court of Chancery, seeking
dissolution of Genitrix. Segal filed counterclaims, charging that Fisk breached the LLC
agreement and the covenant of good faith and fair dealing implied in the agreement
by allegedly blocking Genitrix’s chances at funding. Fisk filed a motion to dismiss
Segal’s counterclaims.
o Class b members had to agree with class a member in order to govern the
company (in order to make things happen at the board level)
o Genitrix was trying to raise money but was unable to do so because of a put
right for which class b members had contracted
 Put right allowed class B members to force the company to purchase
their interests at any time at a price determined by an independent
appraisal
 This put right would subordinate the senior claims of new investors
 Problem  this put right scared off the needed investors to keep the
company running
o Segal asked class b to suspend their put right so he could raise funds, but they
refused and the result was that the company fell
o Fisk ventures and other class b members file a suit for dissolution of the LLC
o Segal (on behalf of genitrix) responds to the lawsuit by filing a counter claim and
a third-party complaint claiming that the plaintiffs and third-party defendants
breached duties to Segal and to the company by frustrating efforts to raise the
necessary capital to keep the company going
 Issue
 (1) May a court insert itself into an LLC agreement to decide which member’s business
judgment was more in line with the LLC’s best interests?
 (2) May the fiduciary duties of an LLC member or manager be expanded, restricted, or
eliminated by the LLC agreement?
 Rule
 (1) Rule: An LLC agreement regulates the terms by which members control the LLC, and
a court will not insert itself into the agreement to decide which member’s business
judgment was more in line with the LLC’s best interests.
o No. Delaware's Limited Liability Act provides for freedom of contract for LLCs.
An LLC agreement regulates the terms by which members control the LLC, and a
court will not insert itself into the agreement to decide which member’s
business judgment was more in line with the LLC’s best interests.
 (2) Rule: The fiduciary duties of an LLC member or manager may be expanded,
restricted, or eliminated by the LLC agreement.
o Yes. Delaware’s Limited Liability Act allows a member or manager’s fiduciary
duties to be expanded, restricted, or eliminated by the LLC agreement.
Accordingly, LLC members cannot be liable for breach of fiduciary duty if the
LLC agreement states that members have no duties other than those
articulated in the agreement, and the agreement does not articulate any
fiduciary duties.
 You cant breach an implied covenant when you’re abiding by the
contract
 DLLCA § 1101(e): Fiduciary duties and the role of contract
o An LLC agreement may provide for the limitation or elimination of any and all
liabilities for breach of contract and breach of duties (including fiduciary duties)
provided, that a limited liability company agreement may not limit or eliminate
liability for any act or omission that constitutes a good faith violation of the
implied contractual covenant of good faith and fair dealing
o
 Holding
 (1) In this case, the LLC agreement splits the power in Genitrix between Class A and
Class B members. The agreement by no means requires Fisk or other Class B members
to go along with Segal’s wishes simply because Segal feels strongly about how Genitrix
should be funded. It is entirely possible that Segal was correct in his assessments of
Genitrix’s financial future and need for funding. But it is not a place for the court to
insert itself into the agreement and determine whether Segal or Fisk’s business
judgment was better for Genitrix at the time the decisions were made. Fisk did not
violate the agreement by voting against Segal; thus, Segal’s breach of contract
counterclaim fails. In addition, Segal’s implied covenant of good faith and fair dealing
claim must fail, because this implied covenant covers only matters that are actually
bargained for within the contract. Nowhere in the agreement did Segal bargain for
unilateral power over the direction of Genitrix’s finances. Similarly, a party cannot use
the implied covenant of good faith and fair dealing if a matter is expressly provided for
in the contract. Here, the covenant does not apply to Fisk’s voting actions, because the
agreement grants Fisk the right to vote on how Genitrix should acquire funding. Fisk is
merely acting within his bargained-for rights.
 (2) In this case, Segal argues that Fisk, Johnson, Rose, and Freund breached their
fiduciary duty to Genitrix. However, the LLC agreement states that members have no
duties other than those articulated in the agreement. Because the agreement does not
articulate any fiduciary duties, they are eliminated. Furthermore, even if the agreement
expressed a fiduciary duty not to act in bad faith or with gross negligence, Segal fails to
allege facts supporting a breach of such a duty. Because Segal fails to state a claim for
breach of contract, the implied covenant of good faith and faith dealing, or fiduciary
duty, his counterclaims and third-party claims must be dismissed.
 Notes
 Court found  members that are being countersued by Segal actually eliminated their
fiduciary duties
o Holding  the members only have the duties set forth in the operating
agreement because Delaware hasn’t adopted the ULLCA and doesn’t impose
fiduciary duties and because the operating agreement doesn’t articulate specific
fiduciary obligations they are eliminated
 Takeaway
o Courts in Delaware are not very willing to give protections to LLC members that
are not explicitly set forth in the operating agreement
o ULLCA § 103: Fiduciary Duties and the Role of Contract
 (a) except as otherwise provided in subsection (b), all members of an LLC may enter into an
operating agreement to regulate the affairs of the company and the conduct of its business, and
to govern relations among the members, managers, and company
 (b) the operating agreement may not
 (2) eliminate the duty of loyalty, but the agreement may
o (i) identify specific types of categories of activities that do not violate the duty
of loyalty, manifestly unreasonable; and
o (ii) specify the number or percentage of members of disinterested managers
that may authorize or ratify, after full disclosure of all the material facts, a
specific act or transaction that otherwise would violate the duty of loyalty
 (3) unreasonably reduce the duty of care
 (4) eliminate the obligations of good faith and fair dealing, but the operating
agreement may determine the standards by which the performance of the obligation is
to be measured, if the standards are not manifestly unreasonably
o Covenant good faith/fair dealing  Focuses on what the parties would have
reasonably thought (reasonable expectations) at the time of the contract i.e.
being faithful to the purpose and spirit of the contract you made / requires a
court to have guessed what you would have liked to have had in the contract
but that you didn’t necessarily put down
 Courts use this as a gap filler  not to make the contract better than
what the parties bargained for or to take things away but to fill in gaps
that the parties probably thought they should have had in there
o Anderson v. Wilder (defendant follows language of operating agreement because fails good faith/fair
dealings)
 Facts
 FuturePoint Administrative Services, LLC is a member-managed limited liability
company. Pursuant to the terms of the operating agreement, the members of
FuturePoint who owned 53 percent of the company (defendant-members) expelled the
members who owned a minority interest in the company (plaintiff-members),
purchased the ownership interests of the expelled members at $150 per membership
unit, and then sold the membership units to a third party for $250 per unit. Several of
the members who were expelled had refused to go along with the majority’s plan to
expel other minority members, acquire their membership interests, and sell them at a
higher value. After the plaintiff-members were expelled, the management committee
made cash distributions to the remaining members. The plaintiff-members brought
suit, alleging a breach of fiduciary duty. The defendant-members moved for summary
judgment, arguing that their actions were expressly permitted under the operating
agreement, and that they acted in good faith because the plaintiff-members were
planning to vote to distribute the company’s excess cash of $60,000 to the members, to
the company’s detriment.
o Problem  plaintiffs know that their shares are $100 more valuable and the
only reason they were expelled was to make a quick $50k profit
o Operating agreement  did allow for this, the ownership interests were
purchased and the members expelled pursuant to the operating agreement / all
they needed was majority votes of members and they could expel with or
without cause (and they have this 53%)
o Plaintiffs file  breach of fiduciary duty and breach of duty good faith/fair
dealing
 Issue
 Do majority shareholders in a member-managed, closely-held limited liability company
owe fiduciary duties to the minority shareholders?
 Rule
 Rule: Under Tennessee law every member of a member-managed limited liability
company must discharge his or her duties in good faith, with the care an ordinarily
prudent person in a like position would exercise under similar circumstances, and in a
manner he or she reasonably believes to be in the company’s best interests.
 Holding
 The plaintiff-members argue  that majority shareholders in a member-managed,
closely-held LLC owe fiduciary duties to the minority shareholders.
o It is well established that there is a fiduciary relationship between members of
a partnership or a closely-held corporation.
o This court finds that a majority shareholder of an LLC has a similar fiduciary
duty to deal fairly and honestly with the minority shareholders, and to act in
good faith toward them.
o Contrary to the defendant-members’ arguments, this finding does not conflict
with Tennessee’s LLC law. Tennessee Code § 48-240-102 states that every
member of a member-managed LLC must discharge his or her duties in good
faith, with the care an ordinarily prudent person in a like position would
exercise under similar circumstances, and in a manner he or she reasonably
believes to be in the LLC’s best interests.
 The court also finds that there is a genuine issue of material fact as to whether the
defendant-members’ actions were taken in good faith, as required under Tennessee’s
LLC Act, or whether the defendant-members expelled the plaintiff-members solely to
acquire their membership units and sell them at a higher price, in violation of the
majority shareholders’ fiduciary duty.
 The defendant-members argue  that they expelled the plaintiff-members in good faith
because they were planning to vote to distribute the company’s excess cash of $60,000
to the members, to the company’s detriment.
o However, the plaintiff-members note that several of the expelled members
were not on the management committee and were not in a position to vote on
whether to make a cash distribution.
o In addition, the record shows that, after the plaintiff-members were expelled,
the management committee made cash distributions to the remaining
members.
o Finally, certain plaintiff-members have alleged first hand knowledge that they
were expelled because they refused to go along with the majority’s plan to
expel other minority members, acquire their membership interests, and sell
them at a higher value..
 Notes
 Issue: If operating agreement allowed for this very thing (expulsion of minority
members)  why did the appellate court side with plaintiff
o Even actions that are permitted by an agreement, they still have to be
performed in good faith and in compliance with fiduciary duties (equity and
fairness component)
 Issue: What to dhow to prevail against a party that is simply following the language of
the operating agreement
o An ulterior bad faith motive  termination for purpose of stealing value from
minority members ($100/unit) = bad faith action and violation of duties
 Issue: Are there limitations on the freedom to contract even in a very flexible LLC?
o Rule: Even a contract that says an action can be performed, it still cannot be
performed for a purpose that is unfair or in bad faith so even something that is
technically permitted, a court won’t let you defeat the other sides expectations
 Issue: Would the court have reached the same result under the ULLCA?
o ULLCA § 409: General Standards of Members and Managers conduct
 (c) a member duty of care to a member-managed company and its
other members in the conduct of an winding p of the company’s
business is limited to refraining from engaging in grossly negligent or
reckless conduct, intentional misconduct, or a knowing violation of
law
 (d) a member shall discharge any duties to a member managed
company and its other members under this act or under the operating
agreement and exercise any rights consistently with the obligation of
good faith and fair dealing

I) Ownership interests and Transferability


 Issue: can a member of an LLC unilaterally transfer his full ownership interests to another person?
o No – LLC interests have the same default restrictions as partnership interests.
o Financial rights can be assigned but not full membership rights
o Becoming a member and receiving management rights requires the consent of all the non-transferring
members
 “Bundle of Rights” – Member of LLC cannot unilaterally transfer membership interest to another person.
o Can assign the right to receive a dividend.
o Becoming a member can only happen by unanimous consent of other members.
 ULLCA: Ownership interests and Transferability
o § 501: Membership Distributional Interest
 A member’s membership in an LLC is personal property
o § 502: Transfer of Distributional Interest
 However, the assignee/transferee of that interest only receives entitlement to distributions to
which transferor would be entitled
o § 601: Events Causing Member Dissociation
 Once a member transfers all of his membership interest, he’s dissociated
 Issue: Why do we strip members tatus from a member that has left voluntarily or
involuntarily all of his /her LLC interest?
 Rule: The remaining members won’t want someone who doesn’t have some financial
stake in the business to have the rights of the member (management and inspection
rights) / they now are not incentivized to benefit or look after the best interest of the
LLC
o § 503: Rights of transferee
 (a) transferee can only become a member pursuant to operating agreement or unanimous
consent
 (d) if transferee doesn’t become a member, he cannot participate in management or conduct of
the LLC’s business.
 (e) Instead, he is entitled to distributions and, If the LL is dissolved, his share of the surplus
(depends on if set up as per capita or pro rata contributions)
o § 504: Rights of Creditor (charging order)
 One who has a judgement against a member of the LLC, may charge the distributional interest
of that member to satisfy the judgment. This charging order constitutes a lien on the judgment
debt distributional interest and is the exclusive remedy to the members interest
 But a judgment creditor is created like any other transferee, which means no participation in
management rights
 Single Member LLC’s
o Olmstead v. Federal trade commission
 The general rules may not apply, in the case of a single member LLC
 Holding: A charging order might not be the exclusive remedy if the LLC only has a single
member (such that the creditor might execute on that membership interest and succeed to the
rights of a ‘full member’)
 The pick your partner rule no longer exists (no longer operative in this instance).
 **Do not ever set up this way!
 some states have harsh remedies that say if you have a single member LLC and the
judgement creditor has a judgement against that member, they may be able to get
more than just he financial rights, they could step into that members shoes completely
and have management and financial rights because there is no one else in the LLC to
protect
 Note: not all states follow this line of thinking
 Note: this would be easy to avoid (99% and 1% to someone you trust).
 Achaian Inc. v. Leemon Family LLC (Delaware / membership transfer to person already member)
o Facts
 Omniglow, LLC was a Delaware limited liability company (LLC) with three members. Leemon
Family LLC (Leemon) (defendant) had a 50 percent member interest, the Randye and Stanley
Holland Trust (Holland) had a 30 percent member interest, and Achaian, Inc. (Achaian) (plaintiff)
had a 20 percent member interest. Omniglow’s LLC agreement defined a membership interest
as the member’s “entire ownership interest.” Section 7.1 of the agreement allowed a member
to transfer all or part of its interest in Omniglow to any person at any time. Under § 7.2 of the
agreement, no person could be admitted as a member after the date of the agreement
without the written consent of the existing LLC members. Holland transferred and assigned its
entire 30 percent interest to Achaian.
 Achaian sued Leemon, claiming that because they each had an equal 50 percent ownership
interests in the LLC, they were deadlocked as to the management of Omniglow, and seeking a
declaratory judgment and order of dissolution.
 Leemon moved for dismissal, arguing that Holland’s assignment of its interest only gave
Achaian an additional economic interest in the profits and losses of the company, and that
Achaian could not have received an additional membership interest, including voting rights,
without Leemon’s consent to the assignment.
o Issue
 Can the default rule that an assignment of an LLC interest only assigns an economic interest, and
not voting rights, be displaced by the terms of the LLC agreement?
 Whether Achains status as a member, does that affect the courts analysis and interpretation of
the operating agreement?
 Yes  operating agreement permitted ownership rights to be transferred / once you’re
admitted, you don’t have to be voted on or admitted again
o Rule
 DLLCA  the default rule is that an assignment of an LLC interest does not entitle the assignee
to become a member of the LLC, but instead simply gives the assignee an economic interest in
the LLC, allowing the assignee to share in the LLC’s profits and losses.
 However, this default rule may be displaced by the terms of the LLC agreement.
o Holding
 Here, Omniglow’s LLC agreement, when taken as a whole, allows all of the rights that make up
an interest in the company, including voting rights, to be transferred to an existing member
without the written consent of the other members.
 The definition of membership interest and § 7.1 of the agreement indicates that a member, like
Holland, may freely transfer all or part of its “entire ownership interest,” which would include
a member’s voting rights.
 The freely transferable nature of a membership interest is subject only to § 7.2, which requires
the consent of the members before a person may be admitted as a member. There is nothing to
indicate that a party that has already been admitted as a member, such as Achaian, must be
readmitted in order to acquire additional voting rights after acquiring additional interests in
Omniglow. Leeman’s motion to dismiss is denied. Achaian’s request for a declaratory judgment
that Omniglow has two members, Leemon and Achaian, each holding a 50 percent membership
interest, is granted.
o Notes
 Transfer to outside third party
 If Achain wasn’t already a member, Leemons argument would work
 Pick your partner rule  you cant just transfer your membership rights to a third party
stranger, only economic interests, unless everyone agrees

J) Dissociation and Dissolution


 Statutes
o (1) Some have easy liquidity provisions usually involving dissociation, triggered buyouts, and dissolution
o (2) Some have eliminated easy liquidity provisions
 The impetus for this elimination is the same concern over estate and gift tax issues
 If the default rule provides for an easy exit liquidation, then the IRS will disallow restrictions that
help get values down for estate tax purposes
 While elimination of default exit rights may accomplish estate tax goals, such elimination also
leaves minority members vulnerable to oppressive majority actions and can also lead to the
unintended results
 Lieberman v. Wyoming
o Facts
 Wyoming.com LLC (defendant) was a Wyoming limited liability company (LLC). E. Michael
Lieberman (plaintiff) was a member of the LLC. Lieberman served Wyoming.com with a notice
of withdrawal from the company, which demanded the return of his share of the current value
of the company, which he estimated at $400,000. The members of Wyoming.com accepted
Lieberman’s withdrawal, elected to continue, rather than dissolve, Wyoming.com, but refused
to return Lieberman’s requested share of the company’s current value. Wyoming.com filed an
action for declaratory judgment to define its rights against Lieberman. Lieberman filed suit
requesting dissolution of Wyoming.com.
 Disagreement over what Lieberman should be paid upon his withdrawal (financial
consequences)
 Lieberman argument t his fair share of the current value of the company was $400k,
the remaning members didn’t want to give him that, they wanted to give him $20k (the
amount of his original capital contribution)
o Issue
 May a court liquidate a limited liability company member’s equity interest in the company upon
his dissociation, if he intended to withdraw his non-economic membership, but not his
economic, or equity, interests in the company?
o Rule
 Rule: A limited liability company member’s non-economic and economic, or equity, interests in
the company are distinct.
 A court may not liquidate a limited liability company member’s equity interest in the
company upon his dissociation, if he did not intend to withdraw his economic, or
equity, interests in the company.
o Holding
 The Wyoming LLC Act  does not explain what happens to a member’s equity interest upon his
dissociation, nor is there any provision in Wyoming.com’s operating agreement regarding the
equity interest of a dissociating member.
 While the operating agreement provides a method for distributing capital upon
liquidation, it does not explain when liquidation must occur, or mandate a buyout or
liquidation of a member’s equity interest.
 While this alone is sufficient grounds to reverse the district court’s order, there is still a question
as to what actually happened to Lieberman’s equity interest.
 Lieberman did not voluntarily forfeit his equity interest upon his withdrawal, because
his notice of withdrawal did not indicate an intent to do so. Instead, the notice
specifically demanded Lieberman’s share of the current value of the company,
estimated at $400,000.
 Wyoming.com argues  that Lieberman’s withdrawal mandates his withdrawal as an equity
owner and liquidation of that interest.
 However, an LLC member’s non-economic interests in an LLC are distinct from his
economic, or equity, interest. In this case, Lieberman’s notice of withdrawal only related
to his non-economic membership, and made clear that he did not intend to forfeit his
equity interest. In addition, while there are no provisions in Wyoming.com’s operating
agreement regarding the equity interest of a dissociating member, the operating
agreement does provide that a person who buys into the company can maintain the
rights of equity ownership without becoming a member, indicating that the rights of
equity ownership are distinct from membership. There is no reason to treat a
withdrawing member any differently, absent any contractual provision to the contrary.
Because there is no buy-out provision in the operating agreement, Lieberman cannot
force Wyoming.com to buy his ownership interest, and Wyoming.com cannot force
Lieberman to sell his interest. Lieberman therefore retains his equity interest in
Wyoming.com, and the district court’s order liquidating Lieberman’s interest is
reversed.
o Notes
 Court looks to two places when allaying the financial consequences of a member withdrawal
 (1) LLC statue
 (2) Operating agreement
o Operating agreement contains no provision regarding the equity interest of a
dissociating member
o Since there is no provision mandating a different result, Lieberman maintains his
equity interest (he doesn’t have to sell and the company doesn’t have to buy his
interest)
o Court is saying, you could have prevented this but we are not going to fix it for
you by rewriting your operating agreement and there are no gap fillers in the
statute so can’t use that
o Dissent
 Under the terms of the LLC’s articles of organization and operating agreement, and under the
LLC statute, Lieberman’s withdrawal as a member should have resulted in a forced buyout of
his entire interest.
 Wyoming.com’s articles of organization give members the right to withdraw from
membership, and the remaining members the right to continue the business. If the
remaining members elect to continue, they will impliedly need to compensate the
withdrawing member for his interest. Because the provisions do not provide for a
forfeiture of the interest, or the option to become a non-member equity owner, the
remedy must be a buy-out.
 In addition, LLC statutes typically apply partnership principles, rather than corporate principles,
to exiting LLC members. Partnership rules allow a partner to dissolve the firm at any time,
demand liquidation, and be paid for his equity interest.
 Similarly, an LLC member who terminates his membership must be paid for this
interest, unless otherwise provided for in the operating agreement. Moreover,
Lieberman himself intended to withdraw his entire interest from the LLC, demanding in
his notice of withdrawal his share of the current value of the company, estimated at
$400,000.
 Finally, under the majority’s holding, LLC members could expel a member and then refuse to
negotiate with him for a buyout of his equity interest, while retaining earnings and forcing the
former member, as an equity owner, to pay taxes on those earnings.
 ULLCA § 409: General Standards of Members and Managers conduct
o (d) a member shall discharge the duties to a member managed company and its other members under
this [act] or under the operating agreement and exercise any rights consistently with the obligation of
good faith and fair dealing
 Lieberman  Arguable that failing to buyout an equity interest is a violation of good faith and
fair dealing
 ULLCA § 101: Dissociation and Dissolution Definitions
o (2) “At-will company”  a limited liability company other than a term company
o (19) “Term company”  an LLC in which its members have agreed to remain members until the
expiration of a term specified in the articles of organization (or specified project)
 Lieberman  Lieberman’s withdrawal would constitute an event of dissociation whether it is
term or at will
 ULLCA § 601: Events Causing Members Dissociation
o A member is dissociated from a limited liability company upon the occurrence of any of the following
events
 (1) the company’s having notice of the members express will to withdraw upon the date of
notice or on a later date specified by the member
 (2) an event agreed to in the operating agreement as causing the members dissociation
 (3) upon transfer of all of a members distributional interest, other than a transfer for security
purposes or a court order charging the members distributional interest which has not been
foreclosed
 (4) the members expulsion pursuant to the operating agreement
 ULLCA § 602: Member’s Power to Dissociate; Wrongful Dissociation
o (a) unless otherwise provided in the operating agreement, a member has the power to dissociate from a
limited liability company at any time, rightfully or wrongfully, by express will pursuant to 601(1)
o (b) if the operating agreement has not eliminated a member’s power to dissociate, the member’s
dissociation from an LLC is wrongful only if
 (1) it is in breach of an express provision of the agreement; or
 (2) before the expiration of the specified term of a term company
 (i) the member withdraws from express will
o *NOTE: if company is at will  dissociation can’t be wrongful
 ULLCA: Dissociating from an “at-will” company
o Section 601(1)  a member can dissociate by giving notice of withdrawal
o Section 602(b)  a member’s withdrawal would not be wrongful if the LLC is an at will company
o Section 301(a)(1) / 701(a)(1)  the LLC would have to purchase that members interest at “fair value
determined as of the date of the member’s dissociation”
o Section 801  the member’s withdrawal would not result in dissolution of the company
 ULLCA: Dissociating from an “Term” company
o Section 602(b)  if an LLC is a term company, under 602(b), a members withdrawal would be wrongful
o Section 602(c)  the member is liable to the company and other members for damages caused by the
wrongful dissolution
o Section 603(a)  If the company decides not to dissolve and wind up its business, the member would
be entitled under 603(a) to have his interest purchased “on the date of the expiration of the term
specified at the time of the members dissociation”
o Section 701(a)(2)  the purchase price would be the fair value determined as of the date of the
expiration of the specified term that existed on the date of the members dissociation if the expiration of
the specified term does not result in a dissolution or winding up of the company’s business under 801
 At will v. term
 In contrast to a buyout at an at will company, a term company buyout occurs only at
the expiration of the term
 Under the partnership concepts (ULPA)  a membership interest will always be bought
out regardless of whether the LLC is at will or term
o The difference is buyout may be delayed for some time, however, in term LLC.
 DLLCA (Delaware): Dissociation and Dissolution
o DLLCA 18-603  a member may not resign from an LLC prior to dissolution unless an LLC agreement
provides otherwise. Other than this section, DLLCA seems to have no dissociation provision. Thus, a
member may not be able to withdraw if such a right is not provided in the operating agreement
 Other than this section, Delaware seems to have no dissociation provision
 Lieberman  may not have been able to withdraw if such a right wasn’t provided in the
operating agreement
o DLLCA 18-604  if a member has the right to resign from the LLC as set forth in the LLC agreement, 18-
604 would allow that member to receive his FMV interest in the LLC

 IE Test, LLC v. Carroll (focuses on the judicial dissolution provisiosn that exist in many LLC statutes)
o Facts
 Kenneth Carroll (defendant), Patrick Cupo, and Byron James were the members of IE Test, LLC
(IE) (plaintiff). The members discussed signing an operating agreement, but Cupo and James
rejected Carroll’s proposed agreement (carol was trying to recover compensation that was lost
from their previous venture/proposed operating agreement that would allow for such
compensation). An impasse ensued. IE filed suit seeking authorization to dissociate Carroll
from IE on two statutory grounds (wrongful conduct that aversely effected the LLC business
AND member engaged in conduct relating to the LLC which made it not reasonably practicable
to carry on the business with the member as a member of the LLC). IE claimed that despite
increasing revenues, it could not obtain a line of credit without an operating agreement.
 New Jersey’s Limited Liability Company Act  authorized a member’s dissociation from an LLC
if the member engaged in conduct making it reasonably impracticable to continue the business
with the member. 
o Issue
 Does a disagreement among LLC members over the terms of the LLC’s operating agreement
necessarily compel the dissociation of a dissenting LLC member?
o Rule
 Rule: A disagreement among LLC members over the terms of the operating agreement does not
necessarily compel the expulsion of a dissenting LLC member.
 If an LLC’s member can manage the LLC without an operating agreement, invoking is
necessary the default majority rule provision of the LLC act, then a conflict among LLC
members may not warrant a members expulsion under the act
 New Jersey law  allows an LLC to operate by majority rule without an operating agreement.
More importantly for this case, however, the law authorizes dissociation if a member engages
in conduct making it not reasonably practicable to continue the business.
 Disputes among members that have no relation to the LLC’s business will not warrant
dissociation under this provision.
 Disputes that make it merely more difficult, but not impracticable, to run the business
likewise will not warrant dissociation.
 Rather, it must be unfeasible, despite reasonable efforts, to continue the business.
 To assist in this determination of whether it is not reasonably practicable to operate an LLC in
light of an LLC members conduct, courts use the following factors:
 (1) the nature of LLC members conduct relating to the LLCs business
 (2) whether with the LLC member remaining a member the entity may be managed so
as to promote the purpose for which it was formed,
 (3) whether the dispute among the LLC members precludes them from working with
one another to pursue the LLC’s goals
 (4) whether there is a deadlock among the members,
 (5) whether, despite that deadlock, members can make decisions about the
management of the company pursuant to the operating agreement or accordance with
the applicable statutory provisions
 (6) whether, due to the LLC’s financial position, they is still a business to operate; and
 (7) whether it is financially feasible to continue the operation of the LLC absent
dissociation.
o Both provisions require the court to evaluate the LLC’s conduct relating to the
LLC and assess whether the LLC can be managed not withstanding that conduct
in accordance with the terms of an operating agreement or the default
provision of the statute / in that inquiry the court should consider the following
factors…
o Holding
 In this case, the trial court erred in authorizing Carroll’s dissociation.
 (1) Despite the impasse with respect to the operating agreement, there is no evidence
that Carroll actively interfered with or tried to undermine the operation of IE.
 (2) (3) There is a genuine issue of material fact as to whether the purposes and goals of
IE can be furthered absent dissociation. While IE revenues have increased despite the
disagreement, IE claims that the lack of operating agreement has prohibited it from
obtaining financing. These factors cannot be judged on the current record.
 (4) (5) There is no evidence that there is a deadlock among the members that has
prevented the LLC from making decisions. Although there is an impasse with respect to
an operating agreement, the LLC can be managed without an operating agreement. IE
has not established that these factors support dissociation.
 (6) (7) Finally, there is no evidence that IE is not in a financial position to continue its
operations. IE revenues have in fact increased.
 Accordingly, these factors also weigh against dissociation. In sum, IE has not established that it is
not reasonably practicable to continue its business with Carroll. The judgment of the appellate
court is reversed, and the case is remanded.
 Dunbar Group v. Tignor
o Facts
 The Dunbar Group, LLC (Dunbar) (plaintiff) and Archie Tignor (defendant) formed and were the
sole managers of XpertCTI, LLC (Xpert). Tignor was also the president of X-tel, Inc. Edward
Robertson was the sole member and manager of Dunbar. Xpert entered into a contract with
Samsung Telecommunications America, Inc. (Samsung). Disputes arose between Robertson and
Tignor over Xpert’s management. Dunbar, Xpert, and Robertson brought suit, requesting an
order expelling and dissociating Tignor as a member of Xpert. Tignor brought a separate action
against Dunbar and Xpert, seeking dissolution of Xpert under Virginia Code § 13.1-1047, which
provides for judicial dissolution of an LLC if it is not reasonably practicable to carry on the
business according to the articles of organization and any operating agreement. The two actions
were consolidated. The record showed that Tignor had placed checks payable to Xpert into X-
tel’s bank account, caused checks written from Xpert’s checking account to bounce, evicted
Robertson from the office space Dunbar was renting from X-tel, and terminated Robertson’s
email account with Xpert.
 Dunbar (plaintiff)  wants Tignor expelled and wants to keep the LLC running
 Tignor (defendant)  wants dissolution of the LLC and payout
o Issue
 May a court order dissolution of a limited liability company if the record does not show that it is
not reasonably practicable to carry on the business according to the articles of organization and
operating agreement?
o Rule
 General rule: A court may order dissolution of a limited liability company if it is not reasonably
practicable to carry on the business according to the articles of organization and operating
agreement.
 A court may not order dissolution of a limited liability company (LLC) unless the record
shows that it is not reasonably practicable to carry on the business according to the
articles of organization and operating agreement. 
o Holding
 Dunbar (defendant) argues  that the evidence was not sufficient to support an order for
dissolution, because the record fails to show that it would not be reasonably practicable to carry
on Xpert’s business after Tignor’s expulsion as a member. Because the chancellor resolved the
dissolution issue in Tignor’s favor, his court must consider the evidence regarding the order for
dissolution in the light most favorable to Tignor.
 Virginia Code § 13.1-1047  states that, upon a member’s application, a court may order
dissolution of an LLC if it is no longer reasonably practicable to carry on the business according
to the articles of organization and any operating agreement.
 Here, the record fails to show that, after Tignor’s expulsion and the change of his role from
active participant in Xpert’s management to one of a passive investor, it would no longer be
reasonably practicable for Xpert to carry on its business according to its operating authority. In
fact, the chancellor’s dissolution order required Xpert to continue operating while the Samsung
contract was in effect, which indicates that the chancellor found that Tignor’s expulsion would
make it reasonably practicable for Xpert to continue to operate. Because the evidence does not
support the portion of the chancellor’s order providing for the dissolution of Xpert, that portion
of the order is reversed.
o Notes
 Trial court  Court says it will (1) expel Tignor, and (2) dissolve the LLC when the contract with
Samsung came to an end
 Issue: what did this expulsion mean for Tignor, while the LLC was still operating?
 Holding: he would no longer have any management rights but he would have his
economic interest in the LLC. As such, the court also ordered that dunbar would have to
give him an accounting each month, because he still has rights to distributions
 Problem (pg. 1035)
o A, B, and C decide to forma Delaware manager managed LLC to engage in the manufacture and
distribution of T-shirts. They each invest $25k in the company in return for a 33% ownership stake. They
quit their prior employment to work full time at the company and they all participate actively as
managers of the venture. The LLC’s operating agreement is skimpy, but it does specify manager
managed governance, it names A, B, and C as the initial managers, and it states that business decision
shall be made by the managers on a majority rule basis
o After a few years of operation, personal animosity develops between the founders. A and B vote to
terminate C’s employment an to remove him as a manager of the venture. C’s salary ceases, and the
company continues its prior policy of reinvesting profits and avoiding distributions. C is upset by these
decision as he considers himself to be ‘frozen out” of the business
 (a) Do you agree with C’s assessment?
 He appears to be frozen out. Due to his founders status and the quitting of his prior
employment, he likely had a reasonable expectation of employment that was frustrated
by his termination
 As a founder and manager since inception, he likely had a reasonable expectation of
management participation as well. His removal as manager frustrates this expectation
 C is earning no return for the company while A and B at a minimum are drawing a salary.
Whether that salary includes a component of deface distributions is unclear but C can at
least argue that he has been totally cut off from any business returns
 Indeed the company has a policy of avoiding explicit distributions
 In short, C has a good case for claiming that he has been frozen out (participatory and
financial rights have been eliminated)
 (b) Does C have an options in this situation? (DLLCA)
 A and B, since they make up a majority, probably had the right to remove C as a
manager just by majority rule
 They also probably have the right to terminate his employment by majority rule
 C also has not right to compel dividends unless that right is specified in the LLC which
most likely it is not
 C could attempt to sell his ownership interest but that would only give the transferee
the financial right to receive distributions
 Unless dissolution is specified in the LLC agreement, dissolution is not an option
 Nor does he have the right to resign unless specified in the LLC agreement
 He could move for judicial dissolution but A and B could presumably continue to operate
the business in conforming with the LLC agreement
o Oppression of C doesn’t affect operation of the business
 He could try to assert that A and B’s actions have breached a common law fiduciary duty
to him  problem is that this company is a Delaware company i.e. no special common
law duties for shareholders of closely held corporations
 Delaware courts  would likely say that C could have protected himself via the contract
and he did not do so so he loses
 (c) Would anything change if ULLCA governed
 Under ULLCA  if the articles of organization did not specify a term, we know the LLC is
at at will company.
 C could still be terminated from employment, removed as a manager, and frozen out
when it comes to distributions
 Further, C could try to sell his ownership interest but given the closely held nature of the
business and current climate of oppression, it is hard to imagine a buyer being found
o Moreover, the buyer wouldn’t receive any management rights as a result of the
purchase or assignment and there are no distributions to potentially attract a
purchaser
 BUT C’s dissolution rights are much stronger under ULLCA
 Section 603  if at will and if dissolution rightful, the LLC would have to purchase C’s
interest at fair value determined as of the date of members dissociation
 Issue: can C force a dissolution?
o 804(a)  C’s withdrawal by itself would not result in dissolution of the company
o 801(4)(v)  C would have a good case for asking for judicial dissolution on the
grounds of oppression
 Uniform state  better chance of forcing a buyout or judicial dissolution
 (d) Would anything change if RULLCA or RULLCA governed?

K) The Nature of the LLC: Regulatory Issues


 LLC: Partnership v. Corporation
o Question is important because of the applicability of various regulatory statutes may turn on the answer
 Many of these statutes were enacted when partnerships and corporations were the only options
to form business organizations with more that one owner
 As a result, the statutes tend to refer only to partnerships and corporations leaving considerable
uncertainty about whether newer business forms, such as the LLC, fall within the statutory
coverage
 Poore v. Fox Hollow
o Facts
 Tammy Poore (plaintiff) filed a motion to strike Fox Hollow Enterprise’s (defendant) answering
brief, because the attorney who drafted it admitted he did not have a Delaware law license. Fox
Hollow argued that the attorney could represent it without a Delaware license because it is a
limited liability company.
 LLC gets sued and the member (non-lawyer) files an answer on the LLC’s behalf
o Individuals can represent themselves (pro se)
o Partnership can represent itself (aggregate of partners)
o Corporation cannot represent itself (separate entity who needs legal
representation)
o Issue
 (1) whether a limited liability company more closely resembles a partnership or a corporation.
 (2) May a limited liability company appear in a Delaware court without a Delaware licensed
attorney?
o Rule
 (1) Resembles a corporation  Based on the limited liability of an LLC, and its contractual
nature, the court finds that the purpose of the rule prohibiting a corporation’s appearance in
court without a Delaware licensed attorney also applies to LLCs. An LLC member or manager
appearing in court to represent the company must do so with Delaware legal counsel.
 (2) Rule: A corporation cannot appear in a Delaware court without representation by a
Delaware licensed attorney.
 A limited liability company (LLC) cannot appear in a Delaware court without a Delaware
licensed attorney
o Holding
 Corporation  That court has reasoned that, while a natural person may represent himself in
court, a corporation is an artificial entity that acts only through its agents and may therefore
only act before the court through an agent that is duly licensed to practice law in the state.
 Partnership  A partnership, in contrast, may represent itself in court.
 The Delaware Limited Liability Company Act (DLLCA) was intended to combine the best
features of partnerships and corporations. For example, the DLLCA treats an LLC as a
partnership for federal income tax purposes. On the other had, it gives members and
managers limited liability, similar to the shareholders and directors of a Delaware
corporation. Like a corporation, an LLC is also largely contract-based, with the LLC
agreement specifying management, economic, voting, and other rights and obligations.
An LLC formed under the DLLCA is also a legal entity separate from its members. The
interest of an LLC member is analogous to a shareholder in a corporation. A member
contributes personal property and has no interest in the LLC’s specific assets. An LLC
member also cannot be held liable for the company’s debts above his own contribution
to the company. Based on the limited liability of an LLC, and its contractual nature, the
court finds that the purpose of the rule prohibiting a corporation’s appearance in court
without a Delaware licensed attorney also applies to LLCs. An LLC member or manager
appearing in court to represent the company must do so with Delaware legal counsel.
Because Fox Hollow did not obtain Delaware legal counsel, Poore’s motion to strike its
answering brief is granted.

 Recap
o Limited liability
 Generally not liable due solely to membership (vicarious v. individual liability)
 Piercing the LLC veil is similar to corporations, except for focus on corporate formalities
o Fiduciary duties
 Generally, comes from common law and can be found in some statutes; duties look roughly
similar to what we’ve seen (duty of care/duty of loyalty/BJR)
 Can limit duties by contract, though there are generally limits – see governing statutes,
precedents
o Ownership/Transferability
 Keep in mind difference between financial rights and full membership right; recall conversations
regarding charging orders

o Dissociation and Dissolution
 This should be provided for in parties’ contractual agreement; if not, look to statute; see ULLCA
for an example of a statute that grants rights on dissolution (not all statutes do)
o Regulatory issues
 LLCs can be treated as either corporations or partnership, which may affect regulatory regime
Hague K’s
(1) Right to Inspect:

(2) Dissociation / Redemption Price:

 *Prevents us from litigating those issues (how it is that a member can be bought out).
 Article VI: Profit, Loss, and Distributions.
(Defined):

(3) Limitation on Managers’ Power:

You might also like